hkmo 2 차시험대비 i 엠제곱 (M 2 ) 실전수학올림피아드 1400 중학생중 / 고급풀이편 고봉균 http://cafe.naver.com/xmo Rev: 2009 년 4 월 2 일 KAIST 수학문제연구회
2 머리말 KAIST 수학문제연구회는한국수학올림피아드가처음탄생할무렵인 1988년부터지금까지다양한영재교육프로그램을운영하거나적극적으로기여해왔습니다. 한국수학올림피아드통신강좌와계절학교등의교육프로그램, 대전 충남지역중학생영재수학교실, KAIST Cyber영재교육등이그예들입니다. 이런활동의결과들을수학잡지 MathLetter를통해담아내어왔으며, 수학경시준비를위한교재를따로만들어달라는요청에부응하여 2003년에엠제곱 (M 2 ) 수학올림피아드셈본중학생초급 / 중급 / 고급을펴내었습니다. 그후셈이의문제해결기법, 셈본고등학생초급,BalticWay 팀경시대회등출판활동이꾸준히이어졌고, 이번에는셈본으로기본학습을마쳤으나보다다양한실전문제들로 KMO 2차시험을준비하는중학생들을위해또한권의책을펴내게되었습니다. 이책에는수론 / 대수 / 기하 / 조합의네분야별로, 또중급 / 고급의난이도로나뉘어, 총 1400문항에이르는매우방대한분량의문제가마련되어있습니다. 각분야의고급은 KMO 중등부 2차시험정도의난이도로볼수있습니다. 각분야의중급은 1차시험정도의난이도로볼수는있으나, 이책의실전연습은2차시험을향해맞추고있으므로문제의유형은단답식인 1차시험과는다를것입니다. 많은문제를스스로풀어보며접하여본학생일수록안정적인실력을갖추게되고또한응용력도성장하게됩니다. 끈기있게도전하여힘든여정을겪으면서자신의능력에차차자신감을갖게될것입니다. 그러나, 모든문제를다풀고넘어가겠다고하는것은시간이아까운면이있으니, 어떤수준의문제가 70% 정도가풀리고있다면그수준은그것으로끝내고다음단계의수준으로넘어갈것을권합니다. 모든문제에는최대한원래의출전을밝히려고노력하였습니다. 원래의출전대신에그것을인용한중간출전을적어둔경우도있습니다. 이책에는문제의풀이와해답이실려있지않습니다. 책이지나치게두꺼워질것을우려하여과감히생략하였고, 대신에 xmo 까페 (http://cafe.naver.com/xmo) 의 `실전MO 풀이 ' 게시판에문제의풀이를게시하고있으니, 거기서찾아보시기바랍니다. 기본학습단계가아닌실전단계에서는, 몇몇문제가잘풀리지않더라도, 가능하면풀이를참고하지않고스스로풀어내겠다고마음먹고승부욕을불태우는것이꼭필요하다고생각합니다. 게시판에올려진풀이보다더멋진여러분만의풀이를찾아내어까페에서함께논의하기를기대해봅니다. 2008 년 3 월, 책임편집자고봉균
차례 1 수론 5 1.1 수론중급문제... 5 1.2 수론고급문제... 31 2 대수 61 2.1 대수중급문제... 61 2.2 대수고급문제... 88 3 기하 123 3.1 기하중급문제...123 3.2 기하고급문제...155 4 조합 197 4.1 조합중급문제...197 4.2 조합고급문제...225 5 종합문제 265 5.1 종합중급문제...265 5.2 종합고급문제...283
4 차례 문제의풀이는 xmo 까페 (http://cafe.naver.com/xmo) 의 ` 실전 MO 풀이 ' 게시판에게시하고있습니다. xmo 까페로오시면그밖에도많은학습자료를찾을수있고, 여러회원들과온라인에서만나수학문제에대한의견 교환및질의응답도할수있습니다.
제 1 장 수론 1.1 수론중급문제 1. 정수계수의이차방정식이정수해를가지면판별식은제곱수가됨을보여라. 증명 ax 2 + bx + c =0에서정수해 x는근의공식으로 x = b p D 2a 를만족하므로, 이것을풀면 2ax = b p D,2ax + b = p D, D =(2ax + b) 2 으로제곱수가된다. 2. n진법의수 100 (n) 과 21 (n) 은 n의값과관계없이항상서로소임을보여라. (KAIST Cyber영재교육 2003 겨울캠프평가시험 ) 증명다음과같이여러방법으로증명이가능하다. (1) 100 (n) = n 2,21 (n) =2n +1 이다. 만일두수가서로소가아니라면공통소인수 p가존재한다. p는 n 2 의소인수이므로 n의소인수이기도하다. 그럼 p는 (2n +1) 2(n) =1의소인수이기도하다. 이것은모순. 따라서두수는서로소이다. (2) 4(n 2 ) (2n 1)(2n +1)=1이므로 n 2 과 2n +1은항상서로소이다. (3) 유클리드호제법의원리에의해 gcd(a; b) =gcd(a + b; b) 이므로 (gcd는최대공약수를나타냄 ), gcd(n 2 ; 2n +1)=gcd(n 2 ;n 2 +2n +1) =gcd(n 2 ; (n +1) 2 ) =(gcd(n; n +1)) 2 =1 이된다. 3. a와 b를최대공약수 (gcd) 가 d인두자연수라하자. 다음수들 a; 2a; 3a; ::: ; (b 1)a; ba 중에서정확히 d개가 b로나누어짐을증명하여라. ( 헝가리 1901-3) 힌트 d j A, d j B이면 A j BC, A=d j (B=d)C, 또 1=(A; B) 이면 A j BC, A j C이다. 4. 정수나유한소수가아닌유리수는순환소수임을보여라.
6 수론 증명정수나유한소수도끝에 0이계속이어지는것으로생각하면순환소수에포함되는것으로생각할수있다. 즉, 유리수는항상순환마디가생긴다는것을보이면된다. 임의의유리수를 q p 라하고, 나눗셈을통해소수전개를구하는과정을생각해보자. 소수점이하의자리를계산할때는항상 p보다작은수가나머지로남게된다. p보다작은나머지는음이아닌정수이므로유한개 (p개) 뿐이고, 따라서나눗셈을계속하다보면같은나머지가반드시생긴다. 즉, 소수점이하 i번째와 j번째를계산할때나머지가같아지는i<j가반드시있다. 그럼 i번째자리를계산할때부터와 j번째자리를계산할때부터의과정이완전히일치하게되므로, i번째부터의소수전개와 j번째부터의소수전개가같다. 따라서,( 길이가 j i 혹은그약수인 ) 순환마디가생긴다. 별증유리수를 s r 이라할때, s; 10s; 102 s; : : : 중에 r로나눈나머지가같은것이있다 ( 비둘기집 ). 그럼그두수는r로나누었을때의소수부가같고, 하나가다른것의 10 k 배이므로하나가다른수의소수전개를 k자리만큼올린것이되어 k자리의순환마디가생긴다. 5. 다음중어떤수도완전제곱수가될수없음을보여라. ( 플란더즈 2002-J2) 11, 111, 1111, ::: 22, 222, 2222, ::: 33, 333, 3333, :::.. 88, 888, 8888, ::: 99, 999, 9999, ::: 증명완전제곱수는 1의자리수로0, 1, 4, 5, 6, 9만을가질수있으므로 2, 3, 7, 8로끝나는것은완전제곱수가될수없다. 또, 완전제곱수는 4로나눈나머지가 0 또는 1이므로 11, 22, 55, 66, 99로끝나는것은완전제곱수가될수없다. 이제 4 4 만확인하면되는데,4 4=4 (1 1) 이고 1 1은완전제곱수가아니므로이것도역시완전제곱수가아니다. 6. 자연수 A, B, C, D 는 A 5 = B 4, C 3 = D 2, C = A +19 를만족한다. D B 는얼마인가? (AIME 1985-7) 풀이 We must have A = m^4, B = m^5 for some m, and C = n^2, D = n^3 for some n. Hence n^2 = m^4 + 19. The difference between n^2 and the next smallest square (n-1)^2 is 2n-1, so n is at most 10. Checking n = 1, 2,..., 10, we find that only n = 10, m = 3 works. 답 757 7. 다음의두조건을만족하는정수 n 이존재함을보여라. (i) n을이진법으로전개하면정확히 2004개의 0과 2004개의 1이나타난다. (ii) n은 2004의배수이다. ( 영국 2004 2차-2) 증명이문제는2004를 N번곱하여 2진수가 1이 2004개,0이2004개인수를만들수있음을밝히면된다. 그것을풀기위하여 2004를 2진법으로고치면 11111010100이다. 이수에서 1은 7개,0은4개임을알수있다.0은마지막에얼마든지추가해줘도되기때문에 (2를계속곱하는작용 )1의개수만생각하겠다. 11111010100의패턴이여러번반복해서이어붙여도 2004의배수임을알수있다 (2004의배수2개를더한것이므로 ). 그럼 1을 7개씩늘릴수있고, 이렇게이패턴을285개이어붙이면모두 1995개의 1이생긴다 (0은현재1140개). 한편 11111010100에 3을곱하면 1011101111100이되고, 여기에는 1이 9개 (0은 4개 ) 있다. 이패턴을앞에서만든것에이어붙이면 1의개수는2004개로맞춰지고, 여기에 2를계속곱하면 0의개수도맞게된다. 8. 1987 은 b 진법으로세자리수 xyz 로쓸수있다. x + y + z =1+9+8+7 일때, x, y, z, b 의모든가능한값을구하여라. ( 캐나다 1987-2) 풀이 1000 (12) =12 3 < 1987 < 45 2 =100 (45) 이므로 13 b 44 이다. xb 2 + yb + z = 1987; x+ y + z =25 에서두식을빼면 x(b 2 1) + y(b 1) = 1962, 따라서 (b 1) j 1962 = 2 3 2 109. 여기서 b의범 위를만족하는약수는 b 1=18뿐이다. 즉, b =19. 그리고, 1987 = 5 19 2 +9 19 + 11. 답 (x; y; z; b) =(5; 9; 11; 19)
1.1 수론중급문제 7 9. 어떤자연수를받으면그수를 2 진법으로쓴후이것을 3 진법으로쓴것으로간주하여계산하는조작을 A 라하자. 예를들어 11 은 2 진법으로 1011 이고이것을 3 진법으로읽으면 31 이되므로 A(11) = 31 이다. A(A(A(n))) > 2006 이되는가장작은 n 은얼마인가? (IT 꿈나무올림피아드 2006 1 차 ) 풀이 x<y이면 x를이진법으로쓴것보다 y를이진법으로쓴것이더크고, 그것을삼진법으로읽은것도 y쪽이더크다. 즉, A(x) <A(y) 이므로, n이증가함에따라 A(n) 도증가함을알수있다. A(A(A(12))) = A(A(A(1100 (2) ))) = A(A(1100 (3) )) = A(A(36)) >A(A(32)) = A(A(100000 (2) )) = A(100000 (3) )=A(243) >A(128) = A(10000000 (2) ) = 10000000 (3) = 2187 > 2006 A(A(A(11))) = A(A(A(1011 (2) ))) = A(A(1011 (3) )) = A(A(31)) = A(A(11111 (2) )) = A(11111 (3) )=A(121) <A(127) = A(1111111 (2) ) = 1111111 (3) < 2 729 < 2006 이므로 A(A(A(n))) > 2006 인가장작은자연수 n 은 12 이다. 10. 음이아닌어떠한정수 n에대하여도,2 2n+1 은네개의양의정수의제곱의합으로표현될수없음을보여라. ( 한국 2003-J1) 증명 (1) n =0: 2는네양의정수의제곱의합이될수없음 ( 성립 ) (2) n = k: 2 2k+1 이네양의정수의제곱의합이될수없다고가정 (3) n = k +1: 2 2(k+1)+1 이네양의정수의제곱의합이된다면, A =2 2(k+1)+1 = a 2 + b 2 + c 2 + d 2 (a; b; c; d 2 N) 그런데,8j A 이고, n 2 N 에대해 n 2 0; 1or4(mod8) 이므로, a 2 + b 2 + c 2 + d 2 = A 가되려면, a; b; c; d 모두짝수이어야한다. ) a =2a 0 ;b=2b 0 ;c=2c 0 ;d=2d 0 ) A =2 2(k+1)+1 =4(a 2 0 + b 2 0 + c 2 0 + d 2 0) ) 2 2k+1 = a 2 0 + b 2 0 + c 2 0 + d 2 0 ) 가정에모순 ) 원명제는참임 (1){(3) 에의해문제는성립한다. 11. 5 보다큰임의의소수 p 에대하여, p 2 1 은 24 로나누어떨어짐을증명하여라. (1997 교육청경시 ) 증명먼저, p는 3의배수가아니므로, p =3k 1꼴이고, 그럼 p +1 또는 p 1 이 3의배수이다. 따라서, p 2 1=(p +1)(p 1) 은 3의배수이다. 또, p는홀수이므로 p +1과 p 1 은모두짝수이다. 그리고, p +1과 p 1 은차이가 2이므로둘중에하나는 4의배수이다. 따라서, p 2 1=(p +1)(p 1) 은 4 2=8의배수이다. 즉,3의배수이고 8의배수이므로 24의배수이다. 12. 10000을연속되는두개이상의자연수의합으로나타낼수있는경우의수는얼마인가? ( 단, 더하는순서는무시한다.) ( 한국 2004 1차-J2) 풀이 10000 = m +(m +1)+ +(m + n) (m; n 1) 으로두자. 즉양변에 2 를곱하면 (n +2m)(n + 1) = 20000
8 수론 이다. n +2m, n +1 은홀짝이다르고각각 3 이상,2 이상이다. 20000 = 2 5 5 4 이므로위와같은두수의곱으로나타내는방법은 (2 5 ; 5 4 ); (2 5 5 1 ; 5 3 ); (2 5 5 2 ; 5 2 ); (2 5 5 3 ; 5 1 ) 의네가지뿐이다 ( 순서쌍의둘중큰쪽이 n +2m). 13. 다음과같이주어진수열 fa ng, fb ng 이있다 a n =3a n 1 +2b n 1 ; a 0 =1 b n = a n 1 +2b n 1 ; b 0 =2 이두수열에공통적으로나타나는항을모두찾아라. (KAIST Cyber 영재교육 2002 겨울캠프평가시험 ) 풀이 1 두수열모두자연수로만나온다는것은자명하다. 두점화식을직접비교하면 a n >b n (n 1), 또 b n 의점화식에서 b n >a n 1 (n 1) 임을알수있다. 즉, (a 0 <b 0 < )b 1 <a 1 <b 2 <a 2 <b 3 <a 3 < 와같이번갈아가며커지는수열이된다. 따라서, 두수열에나타나는항은모두다르다. 풀이 2 (a n 1 ;b n 1 ) (1; 2) (mod 3) 이면 a n =3a n 1 +2b n 1 0+2 2 1 (mod 3) b n = a n 1 +2b n 1 1+2 2 2 (mod 3) 으로 (a n ;b n ) (1; 2) (mod 3) 도된다. 즉, n =0 일때에도참이므로, 수학적귀납법으로항상이렇다. 따라서, 수열 (a n ) 에나타나는항들과수열 (b n ) 에나타나는항들은 3 으로나눈나머지가서로다르므로원래도서로다르다. 풀이 3 a n + pb n = q(a n 1 + pb n 1 ) 에문제의점화식의 a n, b n 을대입하여 p, q 의값을구하면 (p; q) =(1; 4) or ( 2; 1) 이된다. 즉, a n + b n =4(a n 1 + b n 1 )= =4 n (a 0 + b 0 )=3 4 n a n 2b n = a n 1 2b n 1 = = a 0 2b 0 = 3 이두식을연립하여 a n =2 4 n 1; b n =4 n +1 와같이 a n, b n 의일반항을구할수있다. 이런두수열에공통항이없음은 mod 4 에의한비교나크기비교, 혹은 a i = b j 라두고풀어보기등여러가지방법으로알수있다. 14. 가장높은자리의숫자를제거했을때원래수의 1/35 가되는정수는존재하지않음을보여라. ( 캐나다 1970-4b) 풀이 가장높은 10 n 자리수를 t 라하고이것을제외한나머지수를 a 라하자. 그럼 t 10 n + a =35a t 10 n =34a 우변은 17 의배수인데,1 t 9 이므로 t 는 17 을소인수로가질수없고 10 n 도 17 을소인수로갖지않는다. 이것은모순이고, 따라서문제의성질을만족하는정수는없다.
1.1 수론중급문제 9 15. 분수 n3 +2n n 4 +3n 2 은모든n에대하여기약분수임을보여라. ( 소련 ) +1 증명 gcd(n 3 +2n; n 4 +3n 2 +1) 즉분자분모의최대공약수가 1임을보인다. a = bk + r (0 r<b) 일때gcd(a; b) =gcd(b; r) 임을증명없이이용한다 ( 유클리드호제법 ). gcd(n 3 +2n; n 4 +3n 2 +1)=gcd(n 3 +2n; n 2 +1)=gcd(n; n 2 +1)=gcd(n; 1) = 1 모든 n 에대하여주어진분수의분자분모의최대공약수는 1 이므로항상기약분수이다. 16. n 2 을십진법으로썼더니 0 이아닌어떤숫자가자릿수에 m 번연속되어나타나는것으로끝났다. m 의최대값은얼마인가? ( 아일랜드 1989-4) 풀이 0을제외하고제곱수의일의자리는 1, 4, 5, 6, 9만될수있다. 이제제곱수의마지막두자리가 11, 44, 55, 66, 99가가능한지살펴보자. 제곱수는 4로나눈나머지가 0 또는 1인성질이있으므로, 11, 55, 66, 99는제곱수의마지막두자리가될수없다 ( 이것을마지막두자리로갖는수는 4로나눈나머지가각각3, 3, 2, 3이된다 ). 44는 12 2 = 144 에서보듯이가능하다.444도예를들어 462 2 = 213444 에서보듯이가능하다. 4444는불가능함을보이자. 모든정수는 8로나눈나머지에따라 8k + r (r =0; 1; 2; 3; 4) 꼴로분류할수있다. 그럼 (8k + r) 2 은 mod 16으로 (8k + r) 2 =64k 2 +16kr + r 2 r 2 0; 1; 4; 9 (mod 16) 만이가능하다. 그런데마지막네자리가 4444 로끝나는수는 10000n +4444 0 + 12 = 12 (mod 16) 이다. 따라서, 제곱수가될수없다. 이상으로부터,0 이아닌같은수가반복되는제곱수의꼬리의길이 m 은최대 3 임을알수있다. 답 3 17. 자신을제외한양의약수들의곱이자기자신과같은수를 ` 멋진수 ' 라고말하기로하자. 가장작은 10 개의멋진수의합을구하여라. (AIME 1987-3) 풀이 p^n is not unless n = 3, pq is, p^2q is not, p3q is not, pqr is not proper. So 6, 8, 10, 14, 15, 21, 22, 26, 27, 33. 답 182 18. 어떤수도다른수로나누어지지않는데각각의제곱수는다른모든수로나누어지는 10개의자연수가존재하는가? (Towns 1998봄 JA1) 풀이존재함. n = p 1 p 10 일때, n i = p i n 으로두면됨 (i =1; 2;:::;10). 19. 여섯자리수 abcabc 는 2 4 개의 ( 양의 ) 약수를갖고 a, b, c 는소수이다. abc 의최대값을구하여라. (1998 교육청경시 ) 풀이 abcabc =1001abc =7 11 13 abc 인데, 소인수가 3개이상이므로 ( 양의 ) 약수의개수는 3개이상의 (1보다큰 ) 자연수의곱이다. 그럼 2 4 은 (1) 4 2 2 또는 (2) 2 2 2 2 로분해되어야한다.(1) 의경우는더이상의소인수가없고 p 3 qr꼴이어야하므로 abc가 7 2,11 2,13 2 중하나여야하는데, 그럼세자리수가될수없어곤란하다.(2) 의경우는 abc가소수라야한다. a, b, c도모두소수임으로부터가능한수를큰수부터차례로써보면 777, 775, 773, ::: 이다. 이중777, 775는소수가아니고,773은소수이다. 답 773 20. 수열 fa n g 을 a 0 = a 1 =1,a n+1 = a n + a n 1 (n 1) 이라하자. 이때에연속한두항이서로소임을증명하여라. ( 통신강좌 1991-3-4) 21. 임의의정수 n>1 에대해,3 n +1 은 2 n 으로나누어지지않음을증명하여라. ( 헝가리 1911-3) 증명 3 2 1(mod8) 이므로, n이홀짝임에따라 3 n +1 4; 2(mod8) 이된다.2 3 으로나누어지지않으므로 n 3에대해서는모두되었고, n =2일때는8로나눈나머지가 2이므로 2 2 으로나눈나머지도 2가되어역시나누어지지않는다. 22. 자신의양의약수의개수의제곱과같은자연수를모두구하여라. ( 셈본중등초급도전문제 2.1.1)
10 수론 풀이우선 1은문제의조건을만족하고, 이제부터는소인수를하나이상갖는수만을생각하자. 그런자연수를 n이라하고, n의소인수분해를 n = p e 1 1 per r 이라하자. n이제곱수이므로, e 1 ;:::;e r 은모두짝수이다. 그럼 n의약수의개수 (e 1 +1) (e r +1) 은홀수가되고, 그제곱수인n도홀수. 따라서, p 1 ;:::;p r 3 이다. p e 1 1 pe r r =(e 1 +1) 2 (e r +1) 2 ( ) p i 3 이므로 p e i i 3 e i ( 등호는 p i = 3 일때에만성립 ) 이고, e i 2 (e i 는짝수이므로 ) 에서 3 e i (e i +1) 2 ( 등호는 e i =2일때에만성립 ) 가됨을알수있다. 따라서, p e i i (e i +1) 2 ( 등호는 p i =3,e i =2 일때만성립 ) 이다. 즉,( ) 의등호가성립하려면이식에서등호가아닌경우가있어서는안되고, 따라서, 만족하는 n 은 3 2 뿐이다. 답 1, 9 23. 분모가짝수인기약분수는분모가홀수인기약분수들의합으로나타낼수없음을보여라. 증명홀수인기약분수들의합은분모를통분했을때여전히분모가홀수이다. 따라서, 분자를계산하고나서약분을해도여전히분모는홀수이다. 즉, 분모가짝수인기약분수는될수없다. 24. 임의의정수 n 에대해, n 2 +2n +12 는 121 의배수가아님을보여라. ( 캐나다 1971-6) 증명 121 의배수이면 11 의배수이기도하다. n 2 +2n +12=(n +1) 2 +11 이 11 의배수이려면 (n +1) 2 이 11 의배수이고, 즉 n +1 도 11 의배수. 그럼 (n +1) 2 은 121 의배수이므로이식은 121 로나누어 11 이남는다. 따라서,121 의배수일수없다. 25. k 개의연속한자연수의합이 3 11 이될수있을때, k 의최대값은얼마인가? (AIME 1987-11) 풀이 Take numbers as N+1, N+2,..., N+k. So k(2n+k+1) = 2 3^11. Since N >= 0, we have k < 2N+k+1, and hence k < (2 3^11)^1/2. Also k divides 2 3^11. Largest such k is obviously 2 3^5. 답 486 26. 어떤자연수의제곱근이유리수이면이유리수는정수임을증명하여라. 증명 p n = q p 라하자 (p와 q는서로소 ). 이때, p =1이됨을보이면된다. p를이항하고양변을제곱하면 p 2 n = q 2. 그럼 q 2 j p 2 n 인데, p와 q는서로소이므로 q 2 j n 이어야한다. n = q 2 r 이라두고대입하면 p 2 r =1. 따라서, p = r =1이어야한다. 27. 11p +9 가완전제곱수가되는소수 p 를모두구하여라. (2000 교육청경시 - 대전 ) 풀이 11p +9=n 2 으로놓으면 11p = n 2 9=(n +3)(n 3). 11과 p는모두소수이므로, 가능성은 (1) 11 = n +3, p = n 3, (2) 11 = n 3, p = n +3, (3) 11p = n +3, 1=n 3 의세가지뿐이다. 각각풀어보면 (1) 은 n =8,p =5,(2) 는 n = 14, p =17,(3) 은 n =4,p =7=11 이되므로가능한 p의값은 답 5, 17 28. 임의의양의정수 n 에대해, 다음수가 8 의배수임을증명하여라. ( 헝가리 1912-2) A n =5 n +2 3 n 1 +1 풀이 5 2 3 2 1(mod8) 이므로 A n 을 8 로나눈나머지는 2 를주기로반복하게되며, 따라서 n =1; 2 일때만확인하면된다. n =1, 즉홀수일때, A n 5+2+1 0 으로잘되고, n =2, 즉짝수일때, A n 1+2 3+1 0 으로역시잘된다.
1.1 수론중급문제 11 29. 다음방정식을만족시키는자연수 m, n 이무수히많을때, 정수 a 의값을구하여라. m 2 n 2 6n 14 m + n +2 = a ( 한국 2004 1 차 -J13) 풀이 분모를이항하여정리하면 () m 2 am n 2 (a +6)n 2a 14 = 0 ³ m a µ 2 n + a +6 2 =5 a 2 2 A =2(m a 2 ), B =2(n +3+ a ) 2 로치환하면, 위식은 (A + B)(A B) =4(5 a) 가된다. A, B, 그리고우변이모두정수이므로, 우변이 0 이아니면우변을두인수의곱으로나타내는경우를따지는방법으로이부정방정식을풀수있고, 그럼이식을만족하는해가유한개밖에없게된다. 따라서, 해가무한히많기위해서는 a =5 이다.( 그리고이때 A = B 이면되므로이를만족하는해는실제로무한히많다.) 30. 각자릿수의합이원래의수와같은자연수는 1, 2, 3,..., 9 뿐임을증명하여라. ( 셈본중등초급도전문제 2.1.4a) 증명그런수 n이 k자리의수라고하자. n은 10 k 1 이상의수이고, 그자릿수의합은 10k보다작으므로, k가 3 이상이면 n 10 k 1 > 10k >n이되어해가없다. k =2, 즉두자리의수는그자릿수의합이 20보다작으므로 10,..., 19 만살피면되고, 이중에는해가없다. 이제 k =1일때만남았는데, 한자리수인이들은모두해가된다. 별증 그런자연수를 a n a 1 a 0 이라하자. 그럼 10 n a n + +10a 1 + a 0 = a n + + a 1 + a 0 () (10 n 1)a n + +(10 1)a 1 =0 좌변의각항이 0 이상이므로이식이 0 이되려면좌변의모든항이 0 이어야한다. 따라서, a n = = a 1 =0 이다. 즉, 한자리수들만답이될수있다. 31. 10201은 3 이상의몇진법으로읽어도항상합성수임을증명하여라. ( 캐나다 1972-3a) 증명다음과같이인수분해되므로합성수이다. 10201 (n) = n 4 +2n 2 +1=(n 2 +1) 2 32. n =1; 2; 3;::: 에대해,100+n 2 과 100 + (n +1) 2 의최대공약수를 f(n) 으로정의하자. f(n) 의최대값은얼마인가? (AIME 1985-13) 풀이 Let gcd be d. d must divide difference 2n+1. Hence also 2n^2+n and 2(n^2+100), hence n-200. Hence also (2n+1) - 2(n-200) = 401. So certainly d 401. But taking n = 601 gives 401 divides 100+n2 and 2n+1 and hence d 401. 답 401 33. a, b, c 는서로다른세자연수이고, abc + ab + bc + ca + a + b + c = 1000 이다. a + b + c 의값을구하여라. ( 아벨콘테스트 1993 예선 )
12 수론 풀이양변에 1 을더하면 (a +1)(b +1)(c + 1) = 1001 로인수분해된다.1001=7 11 13 이고 a +1, b +1, c +1 은모두 2 이상의자연수이므로 fa +1;b +1;c+1g = f7; 11; 13g 일수밖에없다. 즉, (a +1)+(b +1)+(c + 1) = 7 + 11 + 13 = 31 이고 a + b + c =28 답 34. 1 a + 1 b = 1 를만족하는양의정수쌍 (a; b) 는모두몇개인가? ( 플란더즈 2004-J4) 2004 풀이 a, b가양수이므로문제의식에서 a; b > 2004 임을쉽게알수있다. 양변에 2004ab를곱하여정리하면 (a 2004)(b 2004) = 2004 2 따라서,(a 2004)(b 2004) 는 2004 2 =2 4 3 2 167 2 을두자연수의곱으로나타내는것과같고, 그경우의수는양의약수의개수와같으므로 (4 + 1)(2 + 1)(2 + 1) = 45 개이다. 35. 연속하는두자연수 n, n +1 의합이다른자연수 m 의제곱과같을때, 다음을증명하여라. (1) n은 4의배수이다. (2) n, n +1,m을세변으로하는삼각형은직각삼각형이다. (1991 도대회 ) 증명 2n +1 = m 2 임에서 m 은홀수이므로 m =2r +1 로두면 2n +1 = 4r 2 +4r +1, 즉 n = 2r 2 +2r =2r(r+1) 이다. r(r+1) 은짝수이므로 (1) 이확인된다. 또, n 2 +m 2 = n 2 +(2n+1) = (n+1) 2 이므로피타고라스의정리가성립하여,(2) 도확인된다. 36. p 4 5p 2 +9 가소수가되는소수 p 의개수는얼마인가? ( 한국 2004 1 차 -S2) 풀이 p>5 인경우소수들은 5k 1, 5k 2 꼴이고, 그럼언제나 p 4 5p 2 +9=(p 2 1)(p 2 4) + 5 = (p +2)(p +1)(p 1)(p 2) + 5 로 5 의배수가된다. 이것은 5 보다크므로합성수이다. 따라서, p 5 인 2, 3, 5 만생각하면되고, 이중준식을소수가되게하는것은 2, 5 의두개뿐이다. 주페르마의작은정리를안다면 5 - p 일때 p 4 1(mod5) 를이용해도되겠다. 37. m, n 이 3 으로도 5 로도나누어떨어지지않을때, m 4 n 4 은 15 로나누어떨어진다는것을증명하여라. ( 셈본중등초급도전문제 2.3.1 변형 ) 증명 15 = 3 5 이므로 m 4 n 4 이 3으로도 5로도나누어떨어짐을보이면된다. 먼저 m 4 을 3으로나눈나머지는일정하게 1이됨을보이자. m 4 1=(m 2 +1)(m 2 1) = (m 2 +1)(m +1)(m 1) 인데, m 은 3 의배수가아니므로 m +1, m 1 둘중에하나가 3 의배수가되고, 따라서위의식은 3 의배수이다. 따라서, m 4 을 3 으로나눈나머지는항상 1 이다. 또 m 4 을 5 로나눈나머지도일정하게 1 이됨을보이자. m 4 1=(m 2 +1)(m 2 1) (m 2 4)(m 2 1) (mod 5) =(m +2)(m 2)(m +1)(m 1) 이므로 5 - m 일때언제나 m 4 1 은 5 의배수가된다. 따라서, m, n 이 3 의배수가아니고 5 의배수도아니면언제나 m 4 n 4 은 3 의배수이면서 5 의배수가되므로문제가성립한다. 주 일반적으로, 자연수 n 이소수 p 의배수가아닐때 n p 1 1 (mod p) 가된다. 이것은페르마의작은정리라하는것이고언젠가는공부할기회가있을것이다. 이것을이용하면위의문제에서필요한 m 2 1(mod3),m 4 1(mod5) 가바로얻어진다.
1.1 수론중급문제 13 38. n 이 3 의배수가아닐때, n 7 n 은 2 7 2 의배수임을보여라. (1998 겨울학교모의고사중등부 ) 증명된다. 2 7 2=126=2 3 2 7 이므로 n 7 n 이항상 2 의배수,9 의배수, 또 7 의배수가됨을보이면 n 7 n = n(n 6 1) = n(n 1)(n +1)(n 2 n +1)(n 2 + n +1) 로인수분해된다. (1) n(n +1) 은연속한정수의곱이므로 2 의배수이다. (2) n(n 1)(n+1) 은연속한세정수의곱이므로 3 의배수이다. 또, n 이 3 의배수가아니므로 n =3k 1 로둘수있고, 이때 n 2 n +1=(3k 1) 2 (3k 1) + 1 = 3( )+1+1+1 이 3 의배수가된다. 따라서,9 의배수이다. (3) n =7k 혹은 7k 1 일때 n(n 1)(n +1) 이 7 의배수가되고, n =7k 2 일때는 n 2 n +1= 7( )+4+2+1 이 7 의배수가된다. n =7k 3 일때는 n 2 n +1=7( )+9 3+1 이 7 의배수가된다. 39. p 와 p +2 가모두 3 보다큰소수이면 6 이 p +1 의약수임을보여라. ( 캐나다 1973-3) 증명 p를 6으로나눈나머지에따라분류하자. p =6k, 6k 2, 6k +3꼴일때는 p가 3보다큰소수일수없다. p =6k +1꼴일때는 p +2=6k +3이 3보다큰소수일수없다. 따라서, p =6k 1꼴이고, 그럼 6 j p +1. 별해 p와 p +2가둘다홀수이므로p +1은짝수. 연속한세정수에는반드시 3의배수가하나있어야하는데 p와 p +2는둘다3의배수가아니므로 p +1이 3의배수. 따라서, p +1은 6의배수이다. 40. n 은 a n 1(modm) 을만족하는가장작은자연수이다. a k 1(modm) 이면 n j k 임을증명하여라. 증명 k를 n으로나눗셈을하여 k = qn+ r (0 r<n) 이라하자. 그럼 a k =(a n ) q a r a r (mod m) 이므로 a r 1(modm) 이다. 만일 r 6= 0이면 r은 n보다더작은자연수이므로 n의최소성에모순이다. 따라서, r =0이고, 즉 n j k 이다. 41. a + b + c =1998 와 0 <a<b c<2a 를만족하는정수 a, b, c 중에서 a, b, c 의최대공약수 g 가최대가되는해를하나구하여라. 그리고, 이런해가유일한지그렇지않은지확인하여라. ( 플란더즈 1998-1) 풀이 g =111(a; b; c) =(555; 666; 777), (444; 777; 777) 등으로유일하지않은듯. 42. n =1; 2; 3;:::;1992 에대해, m! 이정확히 n 개의 0 으로끝나는자연수 m 이존재하지않는 n 은모두몇개인가? (AIME 1992-15) 풀이 There are always enough powers of 2, we are constrained by powers of 5. We jump powers of 10, at multiples of 25. So 24! ends in 4 zeros (one 5 each from 5, 10, 15, 20), but 25! ends in 6 zeros. Similarly, multiples of 125 skip two, multiples of 625 skip three etc. So consider 7980! It has 1596 multiples of 5, including 319 multiples of 25, 63 multiples of 125, 12 multiples of 625, and 2 multiples of 3125. Hence total of 1596+319+63+12+2 = 1992. It has skipped 319+63+12+2 = 396. 답 43. 9997 보다큰 9997 의배수중에서각자릿수가모두홀수인가장작은정수를구하여라. (IMTS R2-1) 풀이면 구하는수를 9997n 이라하자. 우선일의자리가홀수라야하므로 n 은홀수. 만일 3n 10000 이 9997n = 10000n 3n = 10000(n 1) + (10000 3n) 에서만의자리는 n 1 의일의자리이므로짝수가된다. 따라서,3n>10000, 즉 n>3333 이고, n 은홀수이므로 n 3335. n =3335 일때 9997n = 33339995 답 44. 연속한네자연수의곱은어떤정수의제곱이될수없음을증명하여라. ( 헝가리 1926-2)
14 수론 증명 연속한네자연수를 n 1, n, n +1,n +2 로두자. 이들의곱 A 는 A =(n 1)n(n +1)(n +2) = f(n 1)(n +2)gfn(n +1)g = fn 2 + n 2gfn 2 + ng = f(n 2 + n 1) 1gf(n 2 + n 1) + 1g =(n 2 + n 1) 2 1 이므로어떤제곱수보다 1 이작다. A 도제곱수라면두제곱수의차가 1 이되어야하는상황이다. 그런데, 제곱수들은 0, 1, 4, 9, 16, 25, ::: 등으로차가 1 이되는것은 0 과 1 뿐이다. 즉, A =0 이어야한다. 그러나, A 는네자연수의곱이므로자연수이고, 이것은모순이다. 따라서, A 는제곱수가될수없다. 45. (m 2 + n)(m + n 2 )=(m + n) 3 을만족하는모든정수해 (m; n) 을구하여라. ( 아일랜드 2003-1) 풀이 ( 대원외국어고 2 학년최일규 ) mn + m 2 n 2 =3m 2 n +3mn 2 mn(mn 3m 3n +1)=0 mn =0 혹은 (m 3)(n 3) = 8 이된다. 오른쪽부정방정식에서 (m 3;n 3) 은 ³ m 3 ³ 8 = ; n 3 1 ³ 8 ³ 1 ; ; 1 8 가가능하고, 그럼모든정수해 (m; n) 은다음과같다. ³ 1 ³ 4 ; ; 8 2 ³ 4 ³ 2 ; ; 2 4 ³ 2 4 ³ m ³ k ³ 0 ³ 11 = ; ; ; n 0 k 4 ³ 5 ; 2 ³ 4 ³ 2 ³ 7 ; ; ; 11 5 5 ³ 1 ; 1 ³ 5 ³ 1 ; 7 1 46. 수열 a 1 ;a 2 ;a 3 ;::: 은 a 1 =2, 그리고 n 2 일때 a n 은 a 1 a 2 a n 1 +1 의가장큰소인수로정의된다. 이수열에 5 가나타나지않음을보여라. ( 호주 1982-5) 증명 a 1 =2,a 2 =3이므로이후 2의배수나 3의배수는나타나지않는다. 그런데 5가나타난다면, 가장큰소인수가 5인데 2나 3은소인수가아니므로 a n = a 1 a 2 a n 1 +1=5 m 꼴이다.1넘기면우변은 4의배수가되는데좌변은짝수가오직a 2 =2한번만나타나므로모순. 따라서 5는나타나지않는다. 47. 1; 2; 3;:::;n 의최소공배수를 M n 이라하자. 예를들어, M 1 =1,M 2 =2,M 3 =6,M 4 =12,M 5 = 60, M 6 =60 등이다. M n 1 = M n 이되는 n 은어떤수들인가? 자신의주장을증명하여라. ( 호주 1991-2) 풀이 M n 1 M n 임은자명. 등호가성립할때는 n이이미m n 1 의약수일때이다. n이소수의거듭제곱꼴일때에는 1; 2;:::;n 1 중에어느것도 n만큼그소수의지수이상을포함하는경우가없으므로 n - M n 1 로성립하지않는다. n이소수의거듭제곱꼴이아닐때에는 n = ab꼴 (a, b는서로소이고 1보다큼 ) 로나타낼수있고, a와 b는 1; 2;:::;n 1 중에있으므로 a; b j M n 1, 즉 n j M n 1 이다. 답소수의거듭제곱꼴이아닌수 48. f(x) =x 2 + x 라하자. a, b 가양의정수일때 4f(a) =f(b) 가성립할수없음을증명하여라. ( 캐나다 1977-1)
1.1 수론중급문제 15 증명 4a(a +1)=b(b +1) 의양의정수해를묻는문제이다. b 2a =) b(b +1) 2a(2a +1)< 4a(a +1) b 2a +1 =) b(b +1) (2a +1)(2a +2)> 4a(a +1) 로각각성립하지않는다. 따라서, 이식을만족하는정수해는없다. 49. P 는 5 보다큰모든소수들의집합이다. 모든 p 2 P 에대해 p 8 1 꼴의정수들의최대공약수를구하여라. ( 플란더즈 1996-2) 풀이 p>5 에대해선 p 8 1 6 0(modp) 이므로공약수가될수없고,2,3,5의지수는p =11을대입하면 2 5 3 5 가최대임이확인됨. p 8 1 을인수분해하여항상 2 5,3,5의배수가된다는것도증명할수있음. 50. k 개의연속한정수들의합으로나타나는수들의집합을 A k 라하자. A 9 \ A 10 \ A 11 의원소중에가장작은자연수를구하여라. (AIME 1993-6) 풀이구하는수를 a라하면a =(l 4)+ +(l+4) = (n 4)+ +(n+5) = (m 5)+ +(m+5) 꼴이다. 정리하면, a =9k =10n +5=11m. 99의배수중에서일의자리가 5인것이므로, 최소의자연수는 답 495 51. 분모가 2005 이하인기약분수중에서 9 와가장가까운분수를구하여라. ( 한국 2005-J1) 25 풀이구하려는분수를 q p 라하면 ( 단,gcd(p; q) =1) 9 25 q j9p 25qj p = 25p 가최소일때를구하면된다. (i) 9p 25q = 1 일때 ;2q 1(mod9) 이므로 q 5(mod9). 따라서, 이방정식의일반해는 q =9t +5 p =25t +14 or q =9t +4 p =25t +11 이각각의경우에대해 25p 가최대일때를구해보면 p = 1989, q = 716. 즉, 이때최소이다. (ii) j9p 25qj 2 일때 ; 이때 25p 25 2005 에서 j9p 25qj 25p 2 25p 2 25 2005 > 1 25 1989 이므로,(i) 에서구한최소의경우보다항상크다. 이상에서구하는분수는 q p = 716 1989 이다. 52. 세정수 a, b, c 가 ab = c 2 을만족하면, 적당한정수 t, u, v 에대해 a = tu 2, b = tv 2 이됨을보여라. 증명소인수분해로생각하자. 자연수 n을소인수분해했을때, 지수가홀수인소인수들만모아곱한것을 n 0 이라하자. 즉, n =(p 2e 1 1 p 2er r )(q 2f 1+1 1 qs 2fs+1 ) 이 n의소인수분해라하면 ( 지수가짝수인것들과홀수인것들을분류하였음 ), n 0 = q 1 q s 이다. 그럼 n n 0 = p2e 1 1 p e r r q 2f 1 1 q 2f s s =(p e 1 1 pe r r q f 1 1 qf s s ) 2 으로제곱수이므로 n = n 0 m 2 꼴이된다. 즉, 모든자연수 n 은 n = n 0 m 2 꼴로나타낼수있다. 여기서 n 0 은어떤소인수도두번곱해져있지않으므로 ( 소인수에지수가붙지않으므로 ) n 의무승파트라부르기로하자.
16 수론 ab = c 2 을만족하는자연수 a 와 b 도각각 a = a 0 u 2, b = b 0 v 2 꼴이되도록하는무승파트 a 0, b 0 이있다. 그럼 a 0 b 0 (uv) 2 = c 2 이된다. 양변의소인수분해를생각하면 (uv) 2 과 c 2 으로부터모든소인수에지수가짝수가되는것들만생기므로, a 0 b 0 의소인수분해에도모든소인수의지수는짝수가되어야한다. 그런데, a 0 과 b 0 은무승파트이므로각각소인수는한번씩만나타나고, 그럼지수가짝수가되려면 a 0 의소인수는 b 0 에도있고, 또 b 0 의소인수는 a 0 에도있어야한다. 즉, a 0 과 b 0 의소인수는완전히일치해야하고, 그럼 a 0 = b 0 이다. 즉, t = a 0 = b 0 이라하면 a = tu 2, b = tv 2 꼴이된다. 53. x +1 y, y +1 가둘다자연수가되도록하는두자연수 x, y는서로소임을보여라. x ( 폴란드 1994/1995 1차-1 변형 ) 증명 d 가공약수라면 d j x j y +1,d j y 이므로 d j 1. 주이런조건을만족하는해도사실몇개없다.WLOGx y 라하면y = x 또는 x +1뿐. 각경우를풀어보면 (x; y) =(1; 1), (1; 2), (2; 3) 뿐이다. 54. p(q r) =q + r 을만족하는모든소수해 (p; q; r) 을찾아라. ( 호주 2003-1) 풀이 1 p(q r) =q + r () q(p 1) = r(p +1). ) q : r = p +1:p 1 만약 p =2 이면 q : r =3:1 이므로 q =3r. 이것은모순이므로, ) p =2k +1 꼴이다. ) q : r =2k +2:2k = k +1:k q, r 모두소수이므로 q = k +1, r = k, p =2k +1. p, q, r 이모두소수가되게하는자연수 k =2 뿐이다. ) p =5,q =3,r =2. 풀이 2 (q r) j (q + r) =) (q r) j 2r. ) q r =1; 2; r; or 2r (1) q r = r 혹은 2r 일때 : q =2r 또는 3r. ) 모순. (2) q r =2 일때 :2p =2r +2. ) p = r +1. ) 연속한세수가소수 ) 모순. (3) q r =1 일때 : q = r +1. ) r =2,q =3) p =5. ) 답은 (5; 3; 2) 뿐. 55. 3k +1 꼴의모든자연수들을모은집합을 A 라하자. n 2 A 일때 n 이 1 과자기자신이외에는 A 에속하는약수를갖지않으면 n 을 A- 소수라고한다. 그리고, A 의수들을 A- 소수들의곱으로나타내는것을 A- 소인수분해라고한다. 곱하는순서를무시할때, A- 소인수분해는보통의소인수분해처럼항상유일한가? 풀이유일하지않다. 100 = 10 10 = 25 4 이고,4,10,25 는모두 A- 소수들이다. 56. m n 크기의방에 1 1 크기의타일을겹치거나빈곳이없이말끔히깔았더니 (n >m), 방의가장자리에깔린타일의개수가방에깔린전체타일의개수의절반이었다. 가능한방의크기를구하여라. ( 플란더즈예선 1995/1996 1 차 ) 풀이방의가장자리를제외한곳에깔린타일의개수도방에깔린전체타일개수의절반이다. (m 2)(n 2) = 1 mn ) mn 4m 4n +8=0 2 ) (m 4)(n 4) = 8 ) (m 4;n 4) = (1; 8); (2; 4) ) (m; n) =(5; 12); (6; 8): 답 5 12 or 6 8 57. n 2 이 n! 을나누지않는자연수 n 을모두구하여라. ( 소련 1964-8) 풀이모든소수들과 4가답이다. 우선 n이소수이면 n! 에는소인수 n이한번밖에들어있지않으므로 n 2 은 n! 을나누지않는다. n이합성수일때, n = ab, 1<a;b<n인두자연수 a, b가있고, 그럼 n! 에는 a; b; n이들어있으므로 a 6= b 라면 n 2 j n! 이다. n = ab 이면 a = b 일수밖에없는수는 p 2 꼴의수들이므로이제이것들만확인하면된다. p 3 이면 n! 에 p; 2p; p 2 이들어있으므로 n 2 j n! 이고, p =2일때는 4 2 은 4! 을나누지않는다.
1.1 수론중급문제 17 58. m, n 은 m 2 +3m 2 n 2 =30n 2 +517 을만족하는정수이다.3m 2 n 2 은얼마인가? (AIME 1987-5) 풀이 (3x^2 + 1)(y^2-10) = 507 = 3 13^2. The first factor is not divisible by 3, so the second must be. It cannot be 3, since 13 is not square, or 507 (nor is 517). So it must be 39, hence 3x^2 = 12, y^2 = 49. 답 588 59. 두제곱수의합이또다른두제곱수의합의절반이되는경우가무한히많음을보여라. 증명 ( m+n 2 ) 2 +( m n 2 ) 2 =(m 2 + n 2 )=2 60. 두유리수의합과곱이정수이면본래의두수는모두정수임을증명하여라. (1999 KAIST 대전. 충남영재수학교실 2 차평가 ) 증명 두유리수를 q p, s r 이라고하자. p와 q, 그리고 r과 s는서로소이다. 두수의합 qr + ps pr 에 s 를곱하고두수의곱 에 r 을곱하여빼면, qs pr qrs + ps 2 qrs pr = ps2 pr = s2 r 은정수가되므로 r 과 s 가서로소임에서 r =1 일수밖에없다. p 도마찬가지로 1 이다. 61. 6 m +2 n +2 가완전제곱수가되는음이아닌정수쌍 (m; n) 을모두구하여라. ( 호주 2004-5) 풀이 m; n 2 이면 4k +2꼴이므로완전제곱수가될수없다. m =0,n 2 일때와m 2, n =0 일때는4k +3꼴이므로역시완전제곱수가될수없다. 남은경우는다음과같다. (i) m; n 1 일때 :(m; n) =(0; 0), (1; 0) 만해가된다. (ii) m =1,n 2 일때 : 준식은 2 n +8 이다. n 4 이면 16k +8꼴이어서완전제곱수가아니고, n =2; 3 일때검토하면n =3일때만해가된다. (iii) m 2, n =1일때 : 준식은 6 m +4이다. m =2; 3 일때는직접검토하면해가안되고, m 4 일때는6 m +4=4(2k +1) 2 꼴이므로정리하면 2 m 4 3 m = k(k +1). 여기서 k와 k +1은홀짝이다르며서로소이고 k +1이더크므로 (k; k +1)=(2 m 4 ; 3 m ) 이될수밖에없는데,2 m 4 보다 3 m 이훨씬커서차이가1일수없으므로해가없다. 이로부터해는 (m; n) =(0; 0), (1; 0), (1; 3) 뿐이다. 62. abc 는세자리의수이다. acb + bca + bac + cab + cba =3194 일때, abc 를구하여라. (AIME 1986-10) 풀이 Put n = abc. Then N + n = 222(a+b+c). We have 222 14 = 3108 which is too small. 222 15-136 = 3194, which does not work (sum of digits of 136 is not 15), 222 16-358 = 3194, which does work, 222 17-580 = 3194, which does not work, 222 18-802 = 3194, which does not work and 222 19 is too big. 답 358 63. abcdef 는 a 6= 0,d 6= 0 인 6 자리의자연수이고 abcdef =(def) 2 을만족한다. 이런수를모두구하여라. ( 플란더즈 1999-1) 풀이 def(def 1) = abc000 에서 def와 def 1 둘중하나는일의자리가 5, 즉 5 3 의배수라야함. n =125; 126; 250; 251; 375; 376; 500; 501; 625; 626; 750; 751; 875; 876 이후보가되고, 그중실제로가능한것은 답 376, 625
18 수론 64. 각자릿수의합을자기자신과곱하면각자릿수의세제곱의합과같아지는두자리의수를모두찾아라. ( 플란더즈예선 1999/2000 2 차 ) 풀이두자리수를 10a + b 라고하면 (a + b)(10a + b) =a 3 + b 3 =(a + b)(a 2 ab + b 2 ). 즉, 10a + b = a 2 ab + b 2 이것은정계수이차방정식이므로정수해를가지려면판별식이완전제곱수라야한다. a에대한식으로생각했을때 m 2 = D =(b +10) 2 4(b 2 b) (m 0) 정리하면 m 2 +3(b 4) 2 =148 이것을만족하는해는 148 = 121 + 3 9=100+3 16 = 1 + 3 49 뿐이고, 각경우를풀어주면만족하는답은37, 48뿐이다. 주 b =0; 1;:::;9 일때로나눠각각막노동으로풀어주는게더마음이편할지도...; 65. 1,2,3,4,5,6,7,8,9의숫자를각각 2번씩사용하여합이가장작게되는서로다른소수들을만들려고한다. 그가장작은합은얼마인가? (IMTS R4-1) 풀이 2를제외한짝수는일의자리에올수없으므로십의자리에있어야한다.2와 5는적어도하나씩은십의자리에쓰여야한다. 따라서, 이합은최소2(40+60+80)+(2+20)+(5+50)+2(1+3+7+9) = 477 이상이다. 실제로 2, 5, 23, 41, 47, 59, 61, 67, 83, 89로하면딱477이가능하다. 답 477 66. 1 부터 n 까지의자연수를모두곱한것을 n! 로나타낸다. abc = a!+b!+c! 을만족하는세자리의자연수 abc 를모두구하여라. 풀이 7!, 8!, 9! 은네자리이상이므로 7, 8, 9는나타날수없다.6!=720이므로 6이나타난다면백의자리가 7 이상이되어곤란하다. 따라서,6도나타나지않는다. 만일 5도나타나지않는다면최대의수는 4! + 4! + 4! = 72 로세자리의수가아니므로곤란하다. 따라서,5는반드시나타난다. 이때, 최대의수는 5! + 5! + 5! = 360 이므로백의자리는 3 이하이다. 또그때, 최대의수는 3! + 5! + 5! = 246 이므로백의자리는 2 이하이다. 만일 5가 2개나타난다면백의자리는반드시 2이고, 그때2! + 5! + 5! = 242 6= 255 이므로성립하지않는다. 따라서,5는1개이다. 그때, 최대의수는 2! + 4! + 5! = 146 이므로백의자리는 1이다.1과5를제외한나머지한자리를 d라하자 (1 d 4). 그럼 1! + d! +5!=d! +121=1d5 또는 15d 이다. 즉, d!+21=d5 또는 5d. d =1; 2; 3; 4 일때, d!+21=22; 23; 27; 45 이므로만족하는것은 d =4일때뿐이고, abc =145 답 67. k 는 1 보다큰주어진자연수이다. 다음방정식의모든정수해를찾아라. y k = x 2 + x (Towns 1981-J1) 풀이 x = (n +1) 일때는x = n 일때와같으므로x 0 일때만보자. x와 x +1은서로소이므로 y k = x(x +1) 에서 x = a k, x +1=b k 꼴이되어야한다. k 2 이므로 k제곱수 0; 1; 2 k ; 3 k ; 4 k ;::: 중에서차이가 1이나는쌍은0과 1뿐이다. 따라서, x =0일때뿐이고, 짝이되는음수일때도고려하면 x = 1 일때도같다. 답 (x; y) =(0; 0), ( 1; 0) 68. 각자리의숫자의합이 9 이상인두자연수들의합으로표시할수없는최대의자연수를구하여라. ( 통신강좌 1992-5-4) 풀이 90 이상의자연수들은 81; 82;:::;90에일의자리가9인수를더하여모두만들어낼수있으므로문제의조건을만족하지않는다.89가문제의조건을만족하며실제로해보면된다. 따라서 89가답이다. 별해 9; 19; 29;:::;89는문제의조건을만족하는 ( 자릿수의합 9 이상인두수의합으로나타낼수없는 ) 수이면서또한그자신은자릿수의합이 9 이상인수이고,9로나눈나머지가각각 0; 1; 2;:::;8이다. 9의배수는자릿수의합이 9의배수이므로자릿수의합이 9 이상이다. 따라서,9; 19; 29;:::;89에적당한 9의배수를더하면 90 이상의수는모두문제의조건을만족하지않음을알수있다. 즉,89가최대이다.
1.1 수론중급문제 19 69. 자연수 n 에대하여 1 n +2 n +3 n +4 n 을십진법으로표현했을때, 일의자리에서부터연속하여나타나는 0 의개수를 f(n) 이라하자. f(n) 의최대값은얼마인가? (KMC 예선 2002 전기고 1) 풀이 1 n +2 n +3 n +4 n 를나누는가장큰 2 k 에대해생각해보자.1 n 은항상1이고,3 n 을 8로나눈나머지는 1; 3; 1; 3;::: 으로반복되므로 1 n +2 n +3 n +4 n 는 8의배수가될수없고 n이 3 이상홀수일때 4의배수이다. 즉 f(n) 2이므로, f(n) =2인 n을찾아낸다. n =3일때1 n +2 n +3 n +4 n =1+8+27+64=100. 답 2 70. n +10 이 n 3 +100 의약수가되는가장큰자연수 n 을구하여라. (AIME 1986-5) 풀이 n+10 divides n^3+10n^2, so 10n^2-100. Also 10n2+100n, so 100n+100. Also 100n+1000, so 900. Hence largest n+10 is 900. Check (900-10)^3 + 100 = -10^3 + 100 mod 900. 답 890 71. 세제곱했을때마지막세자리가 888 로끝나는가장작은자연수를구하여라. (AIME 1988-9) 풀이 Let number be N. Only single digit cube ending in 8 is 2^3. So must be (10a + 2)^3 = 120a + 8 mod 100. Only single digit multiples of 12 ending in 8 are 4 8 = 48 and 9 12 = 108, so a = 10b + 4 or 10b + 9, and N = 100b + 42 or 100b + 92. Hence N^3 = 3 42^2 100b + 42^3 or 3 42^2 100b + 92^3 mod 1000 = 200b + 88 or 200b + 688 mod 1000. So smallest is 192. 답 192 72. ( )( )( )( ) 의 ()()% 는 2000 이다. 괄호안을 1 에서 9 까지의서로다른숫자로채워넣어라. ( 플란더즈예선 1999/2000 1 차 ) 풀이 abcd ef = 200000 = 2 6 5 5 이다. 여기서 abcd와 ef가모두10의배수일수없으므로둘중어느한쪽이2 6 또는 5 5 을다갖는다. 어느한쪽이 100의배수일수없다는것과각각두자리수, 네자리수라는것등도참고하면, 가능한경우는 (2 6 ; 5 5 ) 과 (2 5 ; 2 5 5 ) 뿐이다. 전개해보면 (64; 3125), (32; 6250) 에서전자만가능하다. 73. n! 은 1 부터 n 까지의자연수들의곱을나타낸다. 예를들어, 5! =5 4 3 2 1 =120 이다. 1!+2!+3!+ +n! 이완전제곱수가되는 n 을모두구하여라. ( 플란더즈예선 2000/2001 2 차 ) 풀이 n 4 이면 mod 5 로말이안되어서... n =1; 3 뿐. 74. M = ab 은두자리의수이고 N = cde 는세자리의수이다.9MN = abcde 일때, M, N 을구하여라. (AIME 1997-3) 풀이 Put f(m, N) for the number abcde = 1000M + N. We have 9 M 111 < f(m, 111). Also if N >= 123, then 9 M N >= 9 M 123 = 1000M + 107M >= 1000M + 1070 > f(m,n), so N must be 112, 113,..., or 122. Now bootstrap. If N >= 113, then 9 M N >= 1000M + 17M >= 1000M + 170 > f(m,n). So the only possibility is N = 112. Hence 1000M + N = 1008 M, so M = N/8 = 14. 답 14, 112 75. 어떤정수 A가있다. 이때f(x) = x 1, g(x) = x 이다. 만약 A에 f를두번, g를세번을적용했더 2 2 니 ( 적용시킨순서는모른다 )1이되었고변환과정에서항상정수였다. A가될수있는정수들전부의합은얼마인가? (2007 IT꿈나무올림피아드 ) 풀이 답 444 이진법으로생각하면좋다. 과정을거꾸로생각해보면 f를적용하여 1이되는것은11 (2), g를적용하여 1이되는것은10 (2) 이다. 즉, f를거꾸로적용하는것은이진법으로쓴마지막에 1을추가하는것이고, g를거꾸로적용하는것은 0을추가하는것이다. 따라서, A가될수있는수는1xxxxx 꼴들이다 (x들중에 1은딱2개 ). 이런꼴은 10개있고, 각자릿수별로합하면 10 2 5 +4 (2 4 +2 3 +2 2 +2+1) = 10 32+4 31 이된다.
20 수론 76. 자연수 n 이두제곱수의합으로나타내어질수있다는것과 2n 이두제곱수의합으로나타내어질수있다는것이동치임을증명하여라. ( 헝가리 1938-1) 증명 n = a 2 + b 2 이면 2n =(a + b) 2 +(a b) 2 이다. 역으로 2n = c 2 + d 2 이면, c와 d는홀짝이같고, µ c + d 2 µ c d 2 n = + 2 2 임에서 c + d 2 와 c d 2 가모두정수임을확인할수있다. 77. 연속한 39 개의자연수는그중자릿수의합이 11 로나누어지는수를적어도하나포함함을증명하여라. ( 소련 1961-3) 증명끝자리 0 :::9 를최소한세번포함한다. 이세덩어리를옮겨갈때자리올림이발생하는데, 두번중적어도한번은십의자리에서는자리올림이발생하지않는다. 그때의두덩어리를 n 0; n 1; :::; n 9; n+1 0; n+1 1; :::; n+1 9 이라하자. 앞쪽덩어리가 11의배수가아니려면 10개의수가 11로나눈나머지가각각차례대로 1, 2, :::; 10이되어야한다. 뒷쪽덩어리도마찬가지이다. 즉, n 9와 n+1 0은 11로나눈나머지가각각 10, 1이다 (1). 그런데, 십의자리에서는자리올림이발생하지않으므로, n보다 n +1이자릿수의합이1 크고, 따라서, n 9보다 n+1 0이자릿수의합이 8 작다. 이것은 (1) 과맞지않으므로모순. 따라서, 이두덩어리에 11의배수가반드시있다. 주 38 개의자연수중에는 11 의배수가없는것이있다 ( 예 : 999981, :::; 1000018). 78. 한아이가한변의길이가 1 cm 인 42 개의정육면체꼴블록들을쌓아커다란직육면체꼴을만들었다. 이직육면체의밑면의둘레의길이가 18 cm 라고한다. 그럼높이는얼마인가? ( 영재교육원선발시험기출문제 ) 풀이 가로, 세로, 높이를각각 a, b, c (cm) 라하자. 그럼 abc =42; a+ b =9 이다. 그럼 fa; bg = f1; 8g; f2; 7g; f3; 6g; f4; 5g 가후보가되는데, 이중에서ab가 42의약수가되는것은 f2; 7g뿐이다. 그럼높이는 3 cm 답 79. 양의정수열 a 1 ;a 2 ;:::;a 2006 은 1; 2;:::;2006을적당히재배열한수열이다. 이때, 정수 j(a 2 1 1)(a2 2 2) (a 2 2006 2006)j 이항상3의배수임을보여라. ( 한국 2006-J1) 증명 ( 고양대화중 2 학년한석원 ) 1; 2; 3;:::;2006 의수들중 3k +1 꼴의수는 669 개,3k 꼴의수는 668 개이므로, a 3k+1 꼴의항 669 개가모두 3 의배수일수없다. 그항을 a m 이라하자. 그럼 a 2 m 1(mod3),m 1(mod3) 이므로 a2 m m 은 3 의배수이고, 따라서준식도 3 의배수이다. 80. 아래시계의시계바늘은 1 부터 12 까지의숫자를 f5; 6; 7; 8g 과 f9; 10; 11; 12; 1; 2; 3; 4g 의두집합으로나누고있는데각집합의원소의합은한쪽이다른쪽의두배이다.
1.1 수론중급문제 21 이와같이시계는시계바늘에의해두집합으로나뉘어진다. 두부분으로나뉘어진두집합의원소의합이한쪽이다른쪽의배수가되는경우는하루에몇분동안나타나는가?( 단, 시계바늘이정확히숫자를가리키는경우는배제한다.) (IT 꿈나무올림피아드 2006 1 차 ) 풀이합이작은집합의합 m은 1+2+3+ +12 = 78 = 2 3 13 의약수이어야한다. 즉, m =1; 2; 3; 6; 13; 26; 39 만이후보가된다. (1) m =1일때 : f1g (2) m =2일때 : f2g (3) m =3일때 : f3g, f1; 2g (4) m =6일때 : f6g, f1; 2; 3g (5) m =13일때 : f12; 1g, f6; 7g. (6) m =26일때 : f5; 6; 7; 8g, f11; 12; 1; 2g. (7) m =39일때 : f4; 5; 6; 7; 8; 9g[f10; 11; 12; 1; 2; 3g. 대칭적이므로 1가지로봐야한다. 따라서, 모두 11가지가가능하다. 위의각경우마다분침이 5분간움직일수있고, 시침과분침이서로바뀌는경우가있으며, 하루에 12시간을두번도니까, 각경우마다하루에 20분씩해당된다. 따라서, 11 20 = 220( 분 ). 81. 0 <a<b<c<d<500, a + d = b + c, bc ad =93을만족하는정수해 (a; b; c; d) 는모두몇개나있는가? (AIME 1993-4) 풀이 Put b = a + n, c = a + n + m. Then d = a + 2n + m. So 93 = bc - ad = n(n + m). So n = 1, n+m = 93 or n = 3, n+m = 31. There are 405 tuples (a, a+1, a+93, a+94) and 465 tuples (a, a+3, a+31, a+34) 답 870 82. 한원위에 2000 개의점이있는데, 다음과같이 1, 2, 3,..., 1993 의수를각점에부여한다 : 먼저아무점에나 1 을부여하고, 거기서시계방향으로두번째점에 2 를부여한다. 거기서다시시계방향으로 3 번째점에 3 을부여하고, 이런식으로계속한다. 어떤점들에는둘이상의수가중복되어부여되었을수있다. 1993 이부여된점에함께부여된수들중에서가장작은것은무엇인가? (AIME 1993-9) 풀이 n(n+1) 1993 1994 (mod 2000), 2 2 즉 (1993 n)(1994 + n) 0 (mod 4000) 이다.1993 n 과 1994 + n 둘중에하나만짝수이고또한하나만 5의배수이므로,soeither1993 n or 1994 + n is a multiple of 125 and the other is a multiple of 16(32로수정해야할듯?). If 1993 n is a multiple of 125, then n 118 (mod 125) and 6 (mod 16), and smallest is obviously 118. If 1993 n is a multiple of 16, then n 9 (mod 16) and 6 (mod 125), so smallest is > 118. 답 118 83. 10 진법의자연수 n 의각자릿수의합을 s(n) 으로쓰기로하자. 다음각각의경우에대해 n=s(n) 의최소값을구하여라 :(i)10 n 99 (ii) 100 n 999 (iii) 1000 n 9999 (iv) 10000 n 99999 (IMTS R1-1) 풀이 (1) 10 n 99 (solution) 모든 n에대하여 n>s(n) > 1이므로 n 1 s(n) 1 > n s(n) 이다. 따라서 n=s(n) 을최소로하는 (10이상인 ) n의 1의자리수는 9이다. 일반적으로 n의 10 k 의자리가 1씩바뀔때n값은 10 k, s(n) 은 1씩바뀐다. n 10 k s(n) 1 > n s(n) 이라고가정하면아래식과동치가된다. ) n s(n) 10 k s(n) >n s(n) n ) n>10 k s(n): 즉, n 10 k s(n) 이양수라면10 k 자리수가클수록, 음수라면 10 k 자리수가작을수록 n=s(n) 값이작아진다. n =10a +9(1 a 9) 라고하면 n 10s(n) =10a +9 10(a +9)< 0 ) n 10s(n) < 0이므로 10 n 99인경우n =19이다. 답 :19:
22 수론 (2) 100 n 999 (solution) n 의 10 의자리를구한다. n 100 일때 n =100k +10a +9 라고표현하면 (k 1; 1 a 9) 이때 s(n) =s(k)+a +9 이다. n 10s(n) =100k +10a +9 10(s(k)+a +9) = 100k +9 10s(k) 90 = 10(10k s(k)) 81 10(10k k) 81 90 81 > 0 따라서 n 100 일때 n 의 10 의자리가클수록 n=s(n) 이작아진다. n =100k +99 를대입한다. n 100s(n) =100k +99 100(s(k) + 18) = 100(k s(k)) 1701: 100 이상의 n =100k +99 에서 k s(k) > 17 이면 n 의 100 의자리는 9, 아니면 0 또는 1 인데, k 20 일때 k s(k) > 17 임을쉽게알수있다. n 이세자리수일때 k<10 이므로세자리수 n =199: 답 :199: (3) 1000 n 9999 (solution) 앞선결과에서 n =1000a +100b +99(a; b 는한자리자연수 ) 의꼴임을알수있다. n 의 1000 자리수에따라 100 자리수가달라진다. n 1000s(n) = 1000s +100b +99 1000s(a) 1000b 1800 = 1000 900b 1701 < 0: 즉 a =1,b =0 이다. 답 :1099: (4) 10000 n 99999 (solution) n = 10000a +1000b +100c +99 라고할때, n 100s(n) = 10000a + 1000b +100c +99 100(a + b + c +18)> 10000a 100(9+9+9+18)> 0 이므로 c =9 이다. n 1000s(n) = 10000a + 1000b +999 1000(a + b + 27) = 9000a 18001 이므로 a 3 일때 b =9, 아니라면 b =0 이다. n 10000s(n) = 10000a +1000b +999 10000(a + b +27)< 0 이므로 a =1: 답 : 10999: 84. 6 개의숫자 1, 2, 3, 4, 5, 6 을어떤순서로써도그결과를십진법으로읽었을때 11 의배수가될수없음을보여라. 증명 11 의배수판정법을이용하자.6 개의숫자를배열한 6 자리수를 abcdef 라하면 j(a + c + e) (b + d + f)j = j(a + b + c + d + e + f) 2(b + d + f)j = j21 2(b + d + f)j = X 가 11 의배수가되어야한다.1+2+3 b + d + f 4+5+6 이므로 12 2(b + d + f) 30, 즉 X 는 0 X 9 범위의수이다. 여기서 11 의배수인것은 0 뿐이고, 그럼 2(b + d + f) =11 로홀짝이맞지않으므로모순. 따라서, 11 의배수가될수없다. 85. 2006 2005 2004 1004 를계산하여팔진법으로쓰면마지막에 0 이연속하여여러개가나타난다. 이연속한 0 은모두몇개인가? (IT 꿈나무올림피아드 2006 1 차 ) 풀이이수가8 n 의배수일때 n이최대얼마인지를묻는문제이다. 소인수분해했을때 2의지수를파 2006 2005 1 악하면된다. 1003 1002 1 = 2006 1003 2004 1002 2 1 (2005 2003 2001 1) = 21003 ( 홀수 ) 가된다. 따라서, 소인수분해했을때 2의지수는1003이고, 그럼 8 n =2 3n 임에서 3n 1003. 즉, n 334 이다. 답 334개 86. 7 진법으로쓴수를 10 진법으로읽으면원래수의두배가되는자연수를더블이라부르기로하자. 예를들어,51 은 7 진법으로쓰면 102 이므로더블이다. 가장큰더블을구하여라. (AIME 2001 1 차 -8) 풀이 a_0+10a_1+10^2a_2 +... = 2(a_0+7a_1+7^2a_2+...) or a_0 + 4a_1 = 2a_2 + 314a_3 + 7599a_4 +.... But a_i 6, so lhs 30, so a_3, a_4,... = 0 and a_0 + 4a_1 = 2a_2. Largest soln is evidently a_2 = 6, a_1 = 3, a_0 = 0, giving 630_7 = 315_10 (note that a_i must be 6). 답 315
1.1 수론중급문제 23 87. 다음과같은성질을갖는두개의 1999자리의수 A, B가존재하는가?: `B의자릿수들은 A의자릿수들의순서를재배열한것이고, A + B 의자릿수들은모두 9이다.' 또, 이런성질을갖는한쌍의 2000자리의수는존재하는가? ( 소련 1967-2a) 풀이우선이런두수의덧셈에서자리올림은생기지않음을보이자. 일의자리의합이 19가될수는없으므로그냥 9이고, 따라서일의자리의합에서는자리올림이생기지않는다. 다음십의자리의합을보면역시 9가되려면자리올림이생기지않는다. 이런식으로계속생각하면모든자리에서자리올림이발생하지않음을알수있다. 자리올림이없으므로 A의모든자릿수는자신안에 9에대한보수를갖는다. 즉, 예를들어 A에 3이 n개있으면 6도 n개있다. 그렇게보수끼리짝지어질수있으므로 A는짝수자리의수라야한다. 따라서, 1999자리의수에서는만족하지않는다. A =1818 18 B =8181 81 2000 자리의수에는위와같은예가있다. 88. 연속한 n 개의자연수가모두, 서로다른소인수가 2 개이하인수들이라고한다. n 의최대값을구하여라. (IT 꿈나무올림피아드 2006 1 차 ) 풀이 30 = 2 3 5 임에주목하면 30개이상의연속한자연수에는반드시 30의배수가있으므로 3개이상의소인수를갖는수가있다. 또, 1, 2, 3,..., 29는모두서로다른소인수가2개이하이므로 29가 n의최대값이다. 89. 양의정수쌍 (a; b) 에대하여유리수 Q(a; b) = a2 b +2ab 2 5 을생각하자. 유리수 Q(a; b) 를정수가되 ab +1 게하는모든양의정수쌍 (a; b) 를 (a 1 ;b 1 ), (a 2 ;b 2 ), (a 3 ;b 3 ), :::; (a n;b n) 이라할때, a 1 + a 2 + + a n 의값을구하여라. ( 한국 2006 1차-J14) 풀이 이므로 K = a+2b+5 ab+1 고, 인수분해하면 Q(a; b) =a +2b a +2b +5 ab +1 이정수가되어야한다. K 의분모분자가모두양수이므로 a +2b +5 ab +1 이 (a 2)(b 1) 6 a 5, b 4 이면좌변이 9 이상이되어성립하지않으므로, a 4 또는 b 3 이다. 먼저 a 4 일때를보면다음과같다. a =1일때 : K = 2b+6 b+1 =2+ 4 b+1 이정수이므로 b =1또는 3. a =2일때 : K = 2b+7 2b+1 =1+ 6 2b+1 이므로 b =1. a =3일때 : K = 2b+8 6b+24 22,3K = =2+ 3b+1 3b+1 3b+1 이정수이므로 b =7. a =4일때 : K = 2b+9 4b+18 17,2K = =1+ 4b+1 4b+1 4b+1 이므로 b =4. 다음 b 3 일때를보는데, a 5 인경우만생각하면된다. b =1일때 : K = a+7 a+1 =1+ 6 a+1 이고 a 5 이므로 a =5. b =2일때 : K = a+9 2a+18 17,2K = =1+ 2a+1 2a+1 2a+1 이정수이므로 a =8. b =3일때 : K = a+11 3a+33 32,3K = =1+ 3a+1 3a+1 3a+1 이고 a 5 이므로 a =5. 이상을종합하면해는 (1,1), (1,3), (2,1), (3,7), (4,4), (5,1), (8,2), (5,3) 이전부이고, 구하는답은 1+1+2+3+4+5+5+8=29 답 90. 다섯개의자연수 1, 1, 1, 1, 2 는이중어느세개의수를택해도그수들의합이나머지두수의합으로나누어떨어지는성질이있다. 이런성질을갖는서로다른다섯개의자연수들이있는가? ( 뉴질랜드 2003-8)
24 수론 풀이서로다른다섯개의자연수를 0 < a < b < c < d < e 라하자. d + e j a + b + c 인데, a + b + c<d+ e + e<2(d + e) 로 2 배가안되므로 a + b + c = d + e 이다. 비슷하게 c + e j a + b + d 인데 a + b + d<c+ e + e<2(c + e) 이므로역시 a + b + d = c + e 이다. 여기서연립하면 c = d 가되고이것은모순이다. 따라서, 그런서로다른다섯개의자연수는존재하지않는다. 주 `서로다르다 ' 는조건을없앴을때문제를만족하는해가 (a; a; a; a; 2a) 꼴뿐임을보일수도있다. 해를 0 <a b c d e 라하자. 그럼위의풀이에서와비슷하게 d + e j a + b + c : a + b + c d + e + e<2(d + e) ) a + b + c = d + e c + e j a + b + d : a + b + d c + e + e<2(c + e) ) a + b + d = c + e b + e j a + c + d : a + c + d b + e + e<2(b + e) ) a + c + d = b + e 가된다. 여기서 b = c = d 임을알수있고, 또 e = a + b 이다. 즉, 다섯개의수는 (a; b; b; b; a + b) 이다. 다시 b + b j a + b +(a + b) 인데, 그럼 2b j 2a, b j a 이므로 a = b 가된다. 따라서, 다섯개의수는 (a; a; a; a; 2a) 가된다. 91. 적당한정수 m, n에대하여 m(5m +4n) 꼴로쓸수있는양의정수들중 2006 이하인것은모두몇개인가? ( 한국 2006 1차-J18) 풀이 (1) m이홀수일때 : 5m 2 +4mn 은 4k +1꼴이고, 또한 m =1을대입하면4k +1꼴은모두다됨을알수있다. (2) m이짝수이고 4의배수일때 : m(5m +4n) 은 16의배수이고, 또한 m =4를대입하면 16n +80 이므로 16의배수는모두다된다. (3) m이짝수이지만 4의배수는아닐때 : m(5m+4n) 은 4의배수이지만 8의배수는아니므로 8k +4꼴이고, 또한 m =2를대입하면 8n +20이므로 8k +4꼴의수는다된다. 총정리하면, 원하는꼴로쓸수있는것은 4k +1, 8k +4, 16k꼴들이다. 이중2006 이하인것은각각 502, 251, 125개다. 따라서총 878개 답 92. 1,2,3,...,1000 중에서두완전제곱수의차가될수없는것은모두몇개인가? (AIME 1997-1) 풀이 2n+1 = (n+1)^2 - n^2 and 4n = (n+1)^2 - (n-1)^2. Odd squares = 1 mod 4 and even squares = 0 mod 4, so we cannot get 2 mod 4 by the difference of two squares. Thus we can get all except 2 = 1 4-2, 6=2 4-2, 10=3 4-2,..., 998=250 4-2. 답 750 93. (2m + n)!(m +2n)! (m!) 2 (n!) 2 (m + n)! 이정수임을보여라. 증명자연수 a; b가있을때 (a+b)! a!b! 가정수임을보인다. 어떤소수 p와자연수 k가 p k jq! 과 p k+1 6 jq! 을만족할때 p k jjq라고표기한다. (a + b)! 을소인수분해했을때어떤소수 p에대해p k jj(a + b)! 이고 p j jja!b! 이라면 k j임을증명한다. (a + b) 와 p가주어져있을때 k = b a+b p c + b a+b p 2 b x+y z c bx z c + b y c z 이므로다음부등식이성립한다. k = b a+b p c + b a+b p 3 c + b a+b p 2 c + b a+b p 3 임의의소수에대해위의부등식이성립하므로 (a+b)! a!b! 는정수이다. 이를이용하여주어진식을증명한다. (2m+n)!(m+2n)! (m!) 2 (n!) 2 (m+n)! = (2m+n)! (m!) 2 n! (2m+n)! (2m)!n!, (2m)! (m!) 2, c + b a+b p 4 c + 로구한다. c + b a+b p 4 c + (b a p c + b b p c)+(b a p 2 c + b b p 2 c)+(b a p 3 c + b b p 3 c)+ = j (m+2n)! n!(m+n)! = (2m+n)! (2m)!n! (2m)! (m!) 2 (m+2n)! n!(m+n)! (m+2n)! (2m+n)!(m+2n)! n!(m+n)! 각항이정수이므로원식 (m!) 2 (n!) 2 (m+n)! 이정수이다. 별증 준식은 ³ 2m + n m + n ³ m + n m ³ 2n + m m + n 과같다.
1.1 수론중급문제 25 94. 선생님이칠판에 50000보다작은자연수를적었다. 첫번째학생은이수가2의배수라고말했다. 두번째학생은이수가 3의배수라고말했다. 이와같이차례로,12번째학생은이수가 13의배수라고말했다. 선생님은잘관찰을하다가, 두명의학생을제외하고모두가옳은주장을했고, 그두명은연이어말을했다고하였다. 칠판의수는무엇인가? (IMTS R12-1) 풀이 a =2; 3; 4; 5; 6 각각에대해, a의배수라고말한학생이틀렸다면 2a의배수라고말한학생도틀렸고그럼그두학생이연이어있지않아서모순. 따라서, 이런 a에대한언급은모두참이다. 그럼 10 = 2 5, 12 = 3 4 의배수임도참이고, 그럼남은수중에서연이어있는것은 (7; 8) 일때와 (8; 9) 일때뿐이다.(7; 8) 일때는칠판의수가 4 9 5 11 13 = 25740의배수일때가능하고 ( 그중50000 이하인것은25740뿐 ), (8; 9) 일때는칠판의수가최소한 4 3 5 7 11 13 = 60060 이상이어야해서모순. 답 25740 95. 10 진법으로 1000 자리의자연수가있는데,5 가아닌자릿수가많아야한자리뿐이다. 이자연수가완전제곱수가될수없음을보여라. ( 독일 BW 1973 1 차 -1) 증명 ( 한석원 ) 이자연수가완전제곱수 k 2 이라하자. k 2 0; 1(mod4) 만가능하므로, k 2 의마지막두자리가 55일수없다. 따라서,5가아닌자릿수는십의자리또는일의자리이다. 그자릿수를 a라하고, k의마지막두자리를mn이라하자.(1)a가일의자리에있을때 : k 2 (10m + n) 2 20mn + n 2 (mod 100) 이므로십의자리가홀수이려면 n 2 의십이자리가홀수여야하고그럼 n =4또는 6이다. 둘모두제곱했을때일의자리는6이되므로 a =6. 이때k 2 5 999 + 6 6(mod9). 그럼 k 2 는 3의배수인데 9의배수는아니므로모순. (2) a가십의자리에있을때 : n =5인상황이므로 k 2 25 (mod 100) 이고 a =2. 그럼 k 2 2(mod3) 이되어역시모순. 96. a, b, c, d는 ab = cd 를만족하는자연수들이다. 이때, a 2 + b 2 + c 2 + d 2 은절대소수가될수없음을보여라. ( 독일BW 1971 2차-1a) 증명1 gcd(a; c) =g 라하고a = ga, c = gc 라하면Ab = Cd 이고 d = ha꼴, 그럼 b = hc 가된다. 이것을대입하면 a 2 + b 2 + c 2 + d 2 =(A 2 + C 2 )(g 2 + h 2 ) 으로 2 이상의두자연수의곱이므로합성수. 증명2 a 2 b 2 = c 2 d 2 이므로 a 2 + b 2 + c 2 + d 2 = (a2 + c 2 )(a 2 + d 2 ) a 2 인데, 이수는자연수이므로완전히약분되어져야해서, 적당한두자연수의곱 a 2 = mn 에대해 a2 + c 2 m, a2 + d 2 이자연수가된다. n 그런데 m; n a 2 <a 2 + c 2 ;a 2 + d 2 이므로이두수는 1보다큰자연수여서이곱은합성수. 97. 3 의거듭제곱은십의자리가항상짝수임을보여라. (Towns 1987 가을 J1) 증명 3의거듭제곱은일의자리가 1, 3, 7, 9만가능하다. 이들에대해각각 3배를하면 3 1 = 03, 3 3=09,3 7=21,3 9=27로일의자리의3배가십의자리의홀짝에영향을미치지않음을알수있다. 따라서, 십의자리가짝수로출발했으면아무리 3을여러번곱해도계속짝수. 98. a, b, c 는 a = b + c 인임의의세자연수이다. a 4 + b 4 + c 4 이어떤완전제곱수의 2 배임을보여라. (Towns 1988 봄 JA1) 증명 (b + c) 4 + b 4 + c 4 =2(b 4 +2b 3 c +3b 2 c 2 +2bc 3 + c 4 )=2(b 2 + bc + c 2 ) 2. 99. 다음을만족하는두개의여섯자리수를모두찾아라 : 두수를이어쓴12자리수는원래두수의곱의배수이다. (Towns 1989봄 SA1) 풀이문제의의미대로 mn j 10 6 m + n 라하자. 그럼 m j 10 6 m + n 이므로 m j n. n = mk 라하면 m, n은둘다여섯자리수이므로1 k<10. 그리고, mk j 10 6 + k. 여기서 k j 10 6 이되는데, 1 k<10 에서만족하는것은 k =1; 2; 5뿐.(1)k =5일때 : m j 200001, 즉 200001 = md 이고 d는홀수. m, n =5m이여섯자리수이려면100000 m<200000 이어야하는데 1 <d<3 이되어만족하는 d 없음.(2)k =2일때 : 500001 = md 이고 100000 m<5000000. 1 <d<5 이므로만족하는홀수 d는 d =3뿐. 그때m = 166667, n = 333334 실제로가능.(3)k =1일때 : 1000001 = md 이고 100000 m<1000000. 1 <d<10 인데, 1000001은한자리의소인수를갖지않음을확인할수있어서모순. 답 166667과 333334
26 수론 100. p + q =198 을만족하는방정식 x 2 + px + q =0 의가능한모든정수해를구하여라. ( 러시아 1989 4 차 -y8-2) 풀이 ( + ) = 198, 즉 ( 1)( 1) = 199 인수분해법.199 는소수이므로해는 2 쌍뿐. 101. 1 2 3 2001 + 2002 2003 4002 가 4003 으로나누어떨어짐을보여라. ( 러시아 1989 4 차 -y9-1) 증명뒷항을 ( 2001)( 2000) ( 1) 로보면끝. 102. xy = 1995 1996 인자연수 x, y 에대해, x + y 는 1996 의배수가될수없음을증명하여라. ( 몰도바 1996 최종 -y8-5) 증명 x, y 는홀수이고 x + y 는 4 의배수이므로 xy 3(mod4) 라야한다. 그런데 1995 1996 1 (mod 4) 임. 103. 양의유리수들이다음과같은규칙으로나열되어각각무한번나타난다. 예를들어 2 3 는 9; 42;::: 번째위치에나타난다. 1 1 ; 2 1 ; 1 2 ; 3 1 ; 2 2 ; 1 3 ; 4 1 ; 3 2 ; 2 3 ; 1 4 ; 5 1 ; 4 2 ; 3 3 ; 2 4 ; 1 5 ;::: (a) 1 2 이나타나는처음다섯위치를찾아라. (b) 2 3 가 n번째로나타나는위치를식으로표현하여라. (c) p, q가서로소이고 p<q일때, p q 가처음으로나타나는곳의위치를식으로표현하여라. ( 호주 1983-1) 풀이분모와분자의합이 k +1이되는항들을k번째그룹이라고말하자. 약분을하지않을상태의 a b 는 a + b 1번째그룹의 b번째항이다. k번째그룹은 k개의항을가지므로 a b 는맨앞에서 1+2+ + (a + b 2) + b = (a+b 1)(a+b 2) + b 2 번째항이다. (a) 1 2 이나타나는처음다섯위치는 n =1; 2; 3; 4; 5 일때의 n (3n 1)(3n 2) 2n 이나타나는위치를말하므로 + 2 2n 으로계산하여각각 3, 14, 34, 63, 101번째이다. (b) 2 2n (5n 1)(5n 2) 3 가 n번째로나타나는위치는 3n 이나타나는위치이므로 +3n = 25n2 9n+2. 2 2 (c) (p+q 1)(p+q 2) + q 2 104. n 2 인자연수 n에대해, n의양의약수들의곱 (1과 n 포함 ) 을 P (n) 이라하자. P (n) =n 10 인가장작은 n을구하여라. (IMTS R18-2) 풀이 n의약수들을순서대로 1=d 1 < <d k = n 라하면n = n > > n =1 d 1 d 들도 n의약 k 수들을역순으로쓴것. 따라서, P (n) 2 =(d 1 d k )( n n )=n d 1 d k. 즉, P (n) =n k=2 이 k 다 (k는 n의양의약수의개수 ). 즉, 문제는약의약수가20개인수중에가장작은것을묻는것이다. 약수가 20개인수는 p 19, p 9 q, p 4 q 3, p 4 qr 등의꼴이전부이고, 각꼴마다가장작은수는 2 19,2 9 3, 2 4 3 3, 2 4 3 5 이며, 이들중에서가장작은것은 2 4 3 5=240 답 105. 2 n! 을소인수분해했을때의 2 의지수를구하여라. ( 헝가리 1939-2) 풀이 m! 을소인수분해했을때 p의지수는 b m p c + b m p 2 c + b m p 3 c + 의공식으로구할수있으므로, 구하는값은 2 n 1 +2 n 2 + +2+1=2 n 1 답 106. n 1 ;n 2 ;:::;n 1998 은다음을만족하는자연수들이다. n 2 1 + n 2 2 + + n 2 1997 = n 2 1998 적어도두개의항이짝수임을보여라. ( 주니어발칸 1997-5)
1.1 수론중급문제 27 증명홀수는짝수개이므로, 모두홀수일수없음만확인하면됨. 모두홀수이면 mod 8로 5 1 이되어모순. 107. n 이자연수일때, n(n + 1)(2n +1) 꼴의모든수를나누는가장큰자연수 d 를구하여라. ( 몰도바 1997 최종 -y7-1) 풀이 2 j n(n +1) 이고 3 j 2 n(n +1)(n +2) 3n(n +1)=n(n +1)(2n +1)=:f(n) 이므로준식은항상6의배수. 또한, d j f(1) = 6 이므로더큰수는없음. 답 6 108. 좌표평면에서점 ( p 2; p 3) 을중심으로하는원은격자점 ( 정수좌표의점 ) 을많아야하나만지남을증명하여라. ( 이탈리아 1986-4) 증명둘있다고하고무리수의상등으로둘이같을수밖에없음을증명. 109. x 가정수이면 3x 5 +5x 3 8x 는항상 120 의배수임을보여라. ( 이탈리아 1987-1) 증명 8, 3, 5 의배수임을보이면.. 뭐쉽.. 110. 9 진법으로나타낸수열 1; 11; 111; 1111;::: 의모든항이삼각수임을, 즉적당한정수 m 에대해 m(m+1) 2 꼴의수임을증명하여라. ( 이탈리아 1988-4) 증명 9n 1 8 = 3 n +1 2 3n 1 2. 혹은 9 2 m(m +1) 2 +1= (3m +1)(3m +2) 2 의점화법으로도. 111. 정수계수의두삼차다항식이공통의무리수근을가지면, 공통의무리수근이하나더있음을증명하여라. ( 폴란드 1966 3 차 -1) 증명켤레로뻔한거지만, 중등중급수준에서증명연습을해볼필요는있... 112. p는 2보다큰소수이다. 2 p 는 x>y인두자연수x, y에대해 1 x + 1 y 의꼴로나타내는방법이딱한가지가있음을증명하여라. ( 헝가리 1931-1) 증명 2 p = x+y xy. 분모넘기고 2 곱하고인수분해법으로정리하면 (2x p)(2y p) =p2. (x; y) = ( p+p2 ; 1+p 2 2 ). 113. m! n! =k! 은 1 보다큰정수해를무한히많이가짐을증명하여라. (Towns 1982 가을 J3) 증명 (m; n; k) =(n! 1;n;n!) 114. a, b, c가 1 이외의공약수를가지지않는자연수로, a, b, c를세변의길이로가지는삼각형의넓이도정수일때, 우리는 (a; b; c) 를원시적헤론세짝이라고한다. a =96일때, b와 c는모두홀수여야함을증명하여라. (IMTS R19-2) 증명 최대공약수가 1이므로둘다짝수일순없고, 하나만홀수이면헤론의공식에서 S가정수가될수 없음. 115. 다음세등차수열에서모두등장하는가장작은자연수를찾아라. 그리고, 이세등차수열에서모두등장하는자연수는무한히많음을증명하여라. (IMTS R21-2) 5, 16, 27, 38, 49, 60, 71, ::: 7, 20, 33, 46, 59, 72, 85, ::: 8, 22, 36, 50, 64, 78, 92, ::: 증명각각 11k 6, 13k 6, 14k 6 꼴의수열.11 13 14k 6 꼴은늘나타남.
28 수론 116. 각각의수가다른두수의합을나누는세자연수의순서쌍 (a 1 ;a 2 ;a 3 ) 을모두구하여라 ( 단, a 1 a 2 a 3 ). ( 호주 1993-3) 풀이 a 2 + a 3 =2a 1 일때와a 2 + a 3 = a 1 일때로구분. 전자일때는 (n; n; n) 뿐. 후자일땐다시 2a 3 = a 2 일때와2a 3 =2a 2 일때로구분하여각각 (3n; 2n; n), (2n; n; n). 117. 2n +1 과 3n +1 이모두완전제곱수인자연수 n 이있다.5n +3 이소수일수있는가? ( 러시아 1993 최종 -y9-1) 풀이 2n +1=a 2,3n +1=b 2 이라하면 5n +3=4a 2 b 2. 인수분해법 118. 자연수 x, y, z가 (x y)(y z)(z x) =x + y + z 를만족한다. x + y + z 가 27로나누어떨어짐을보여라. ( 러시아 1993 최종-y9-5) 증명 x, y, z가 mod 3으로모두서로다를수없음 ( 우변만 3의배수 ). 둘은같고하나만다를수도없음 ( 좌변만 3의배수 ). 즉, 모두 mod 3으로같음. 119. x 2 +3x +24 가완전제곱수가되는정수 x 를모두구하여라. ( 유고슬라비아 1980 고 2-1) 풀이 x 2 +3x +24=k 2 이라두면정수해를가지므로판별식이제곱수, 즉 D =9 4(24 k 2 )= 4k 2 87 = m 2. k; m 0 이고 (2k + m)(2k m) =87=3 29 이므로 2k + m 2k m; 0 임에서 (2k + m; 2k m) =(87; 1), (29; 3) 만가능. 각각 (k; m) =(22; 43), (8; 13) 이되고그때이차방정식을풀면 x = 1 ( 3 m) 2 임에서 x =20; 23; 5; 8 답 120. 합과곱이모두 n 이되는 n 개의정수 ( 양수든음수든상관없고, 서로다를필요도없다 ) 가존재하면 n 을좋은자연수라고부른다.4k +1 꼴과 4l 꼴 (l 2) 의자연수는모두좋은자연수임을보여라. ( 인도지역예선 1995-2) 증명 4k +1: 4k +1; 1; 1; 1; 1; :::. 4l :2l; 2; 1;:::;1; 1; 1; 1; 1; :::. 121. 모든자연수 n 에대해 a n =20+n 2 이고 d n =gcd(a n;a n+1 ) 이라하자. d n 이취할수있는모든값들의집합을구하고, 각각의값에대한예를하나씩찾아라. ( 인도지역예선 1997-2) 풀이 d j 2n +1, d j n(2n +1) 2(n 2 +20) = n 40, d j (2n +1) 2(n 40) = 81. d =1; 3; 9; 27; 81 각각예를찾아주면. 122. n 은자연수이고 p 1 ;p 2 ;p 3 ;:::;p n 은 n 개의 5 보다큰소수들이며 p 2 1 + p2 2 + p2 3 + + p2 n 은 6 으로나누어떨어진다. n 도 6 으로나누어떨어짐을증명하여라. ( 인도지역예선 1998-2) 증명다 6k 1 꼴이니까뻔함. 123. 방정식 y 3 = x 3 +8x 2 6x +8 을만족하는자연수해 x, y 를구하여라. ( 인도지역예선 2000-2) 풀이 부등식적부정방정식.(x +1) 3 <y 3 < (x +3) 3 이므로 y 3 =(x +2) 3 이라야함. 답 (9,11) 124. 다음방정식의모든정수해 (m; n) 을구하여라 : ( 인도지역예선 1999-6) (m n) 2 = 4mn m + n 1 풀이분모이항하고정리하면 (m n) 2 (m+n) =(m+n) 2. m+n =0일때는항상성립. m+n 6= 0 일때는m = n + k 대입해서풀면... 답 (m; n) =(t; t), ( k2 +k ; k2 k ). 2 2 단, k 6= 1. 125. 한정육면체를 99 개의작은정육면체로분할했는데, 이 99 개의정육면체들은하나만빼고한변의길이가 1 인정육면체들이라고한다. 원래정육면체의부피를구하여라. ( 몰도바 1997 최종 -y11-1)
1.1 수론중급문제 29 풀이 m 3 n 3 =98. m 과 n 은홀짝이같아야한다. m = n +2k 라두고대입하여정리하면 k(4k 2 + 6nk +3n 2 )=49. k =1,n =3,m =5. 126. 갑과을은 13환 ( 통화단위 ) 짜리은행수표여러장으로봉급을받았다. 한식당에서, 갑은빵 9조각, 주스 10잔, 소시지 7개를주문했고, 을은빵 5조각, 주스 7잔, 소시지한개를주문했다. 빵, 주스, 소시지의낱개가격은각각정수환이다. 갑이거스름돈없이몇장의은행수표로지불할수있으면을도그렇게할수있음을보여라. ( 몰도바 1998 최종-y7/8-3) 증명둘다 17 로나누어떨어진다는옛날헝가리문제랑비슷한... 13 j 2(9x +10y +7z) (5x +7y + z) 127. 1보다큰자연수 a에대해, a보다작은모든자연수로 a를나누었을때의나머지들을모은집합을 M이라하자. M의 ( 서로다른 ) 모든원소들의합이 a일때, a를구하여라. ( 몰도바 1997 최종-y9-2) 풀이 a 11 이면 S(M) 1+2+ + b a 1 c >a 2 임을금방알수있음. a 10 만각각검사해보면... a =10만가능함 ( 집합의원소의합은같은수는한번만더하게됨을유의할것 ). 답 10 128. 소수 0:a 1 a 2 a 3 이있는데, a 1 과 a 2 는임의의숫자이고그다음자릿수들은바로앞의두자릿수의합을 10으로나눈나머지라고하자. 이소수가순환소수임을보여라. ( 몰도바 1997 최종-y9-6) 증명 (a n ;a n+1 ) 의패턴이유한가지뿐이므로. 유한 - 무한의비둘기집. 순환잉여수열 129. 10 과 10 29 사이의수들중, 십진법으로나타내었을때각자릿수가 1 만으로이루어지는소수는많아야 9 개임을보여라. ( 호주 1995-1) 증명 1의개수가소수개라야하고, 그런게 2, 3, 5, 7, 11, 13, 17, 19, 23, 29로 10개인데,111은제외할 수있음 130. 각자연수 n 에대해, n!+1 과 (n +1)! 의최대공약수 f(n) 의공식을구하여라. ( 아일랜드 1996-1) 풀이유클리드호제법으로 g j n +1. n +1 이 prime 이면 n +1 이최대공약수. 그렇지않으면 1. 131. 1997 1997 의각자릿수의합의각자릿수의합의각자릿수의합을구하여라. ( 몰도바 1997 최종 -y10-2) 풀이자릿수의합은 mod 9 로불변. a! b! c! d 가되었다면, d 1(mod9). a<10 10000, b<90000, c<45, d 12. 답 8 132. 1 이아닌서로소인두자연수의합으로나타낼수있는모든자연수를찾아라. (Towns 1983 가을 J2) 풀이 5 이상의홀수는 2k +1=k +(k +1). 8 이상의짝수는 4k =(2k 1) + (2k +1) 과 4k +2= (2k 1) + (2k +3). (by 김규완 )1,2,3,4,6 만불가능. 133. a 1 ;a 2 ;a 3 ;::: 는자연수들의단조증가수열이다. 임의의 k 에대해 a ak =3k 가항상성립한다고할때, a 100 을구하여라. (Towns 1983 가을 J4) 풀이 a 81 = a a54 =162 이고 a 162 = a a81 =243. 162 81 = 243 162 이므로 a 81 부터 a 162 까지 162...243 의연속한자연수로가득. a 100 =181 답 134. 자연수 n 의 ( 양의 ) 약수들 (1 과 n 을모두포함한 ) 의합이 2n 일때 n 을완전수라한다. n 1 과 n +1 이모두소수인완전수 n 을모두구하여라. ( 주니어발칸 2006-3) 풀이쌍둥이소수는 (3,5) 를제외하곤6k 1꼴이고,4는완전수가아니므로 n =6k꼴. k 2 이면 ¾(n) 1+k +2k +3k +6k =2n +1로불가능. n =6뿐. 135. 선생님이사탕이많이가져와서 13명의남학생과 10명의여학생들에게나눠주려고한다. 남학생들끼리같은개수의사탕을받고여학생들끼리도같은개수의사탕을받도록나누어주길원한다 ( 각자적어도하나씩은받는다 ). 이렇게할수있는방법이딱한가지뿐이라면, 선생님이갖고있는사탕은최대몇개인가? ( 몰도바 1997 최종-y10-6)
30 수론 풀이 260 개. 그보다많으면방법이두가지이상이고,260 개일때방법이유일함을말하면됨. 136. 무게가각각 1; 2;:::;1998인 1998개의추가있다. 이추들을총무게가각각같은 n개의집합으로분할하려고한다. n =2; 3; 4; 5 중에서가능한것을모두찾아라. ( 몰도바 1998 최종-y9-6) 풀이 1 에서 1998 까지의합은 1999 999 로 2, 4, 5 의배수가아니므로 n =2; 4; 5 는불가능하고, n =3 일때는합이 1999 인쌍이 999 개있으므로이것을 333 쌍씩분할하면됨. 137. n 은자연수이다. A 는숫자 4 로만이루어진 2n 자리의수이고, B 는숫자 8 로만이루어진 n 자리의수이다. A +2B +4 는완전제곱수임을증명하여라. ( 주니어발칸 2003-1) 증명 4 102n 1 9 +16 10n 1 9 +4=( 2 10n +4 3 ) 2 138. 다음을만족하는자연수 x, y 를모두구하여라. ( 주니어발칸 2005-1) 9(x 2 + y 2 +1)+2(3xy + 2) = 2005 풀이판별식이제곱수임을이용. fx; yg = f7; 11g. 139. a, b, c 는자연수로 a + c, b + c 가연속한두자연수의제곱수가된다. 그러면 ab + c 와 ab + a + b + c 도연속한두자연수의제곱수가됨을보여라. ( 유고슬라비아 1981 고 1-1) 증명 a + c = n 2, b + c =(n +1) 2 이라하면 b = a +2n +1, c = n 2 a 를대입하여그냥정리해보면 ab + c =(a + n) 2, ab + a + b + c =(a + n +1) 2. 140. 다음식의모든해를구하여라. 단, n 과 x 는둘다정수이다. 2 n +7=x 2 (Towns 1984 가을 JO3) 풀이 n 1 이면정수가아니라서, n 2 이면 mod 4로 3이라서곤란. n =0; 1 일때만체크하면됨. 답 n =1,x = 3 141. 각각의 n 2 에대해 n 번째항이 n 1 번째항의수와그수의각자릿수를모두합한것으로결정되는수열이있다. 이수열의첫항이 1 일때, 이수열에 123456 이라는항이나타나는가? (Towns 1985 봄 JO5) 풀이 3 으로나눈나머지가계속 2 배씩되므로,3 의배수는나타날수없음. 142. n은 3으로나누었을때 2가남는5 이상의자연수이다. n의제곱이어떤소수와어떤완전제곱수의합이될수없음을증명하여라. ( 몰도바 2000 최종-y7-6) 증명 p = n 2 m 2 =(n + m)(n m). n m =1 이라야하는데그럼 3 j n + m. 143. a 2000 1 이 10의배수가되는자연수 a를모두구하여라. ( 몰도바 2000 최종-y8-1) 풀이 a는일단10과서로소, 즉 5의배수가아닌홀수. 2 j a k 1 은자명하고,5ja 4k 1 이므로이런 a는항상됨. 144. m; n 2 은정수이고 m + n 1 이 m 2 + n 2 1 의약수라고한다. m + n 1 은소수가아님을증명하여라. ( 몰도바 2000 최종-y8-3) 증명귀류법으로, p := m+n 1 3 이소수라면, p j m 2 +n 2 1=(p+1) 2 2mn 1 =p 2 +2p 2mn. 즉, p j mn. 그런데, p = m + n 1 >m;n이므로 p - m; n 이되어모순. 145. 번계승 N!! 은 N 이홀수일때 N(N 2)(N 4) 5 3 1 로, N 이짝수일때는 N(N 2)(N 4) 6 4 2 로정의된다. 예를들어 8!! = 8 6 4 2 이고 9!! = 9 7 5 3 1 이다. 1986!! + 1985!! 이 1987 의배수임을보여라. (Towns 1986 가을 S4)
1.2 수론고급문제 31 증명 1986!! ( 1)( 3)( 5) ( 1985) 1985!! (mod 1987) 146. n은다음을만족하는서로다른네소수 a, b, c, d의곱이다. (i) a + c = d (ii) a(a + b + c + d) =c(d b) (iii) 1 + bc + d = bd n을구하여라. ( 아일랜드 2002-7) 풀이 (i) 에서 a =2또는 c =2. 두경우에대해각각 (iii) 먼저고려하고 (ii) 를풀면금방풀림. 답 (2,7,11,13) 147. 1, 2, 5, 10, 20, 50 센트및 1 달러짜리동전들이많이있다. B 개의동전으로 A 센트를만들었다. A 개의동전으로 B 달러를만들수있음을보여라. ( 레닌그라드 1987-20) 증명 x 2 M 이면 100 100 2 M x 임에주목. x센트짜리 b개로 a센트를만들었다면 x 센트짜리 a개로 b달러를만들수있음. 이것들을선형으로조립하면됨. 148. x 2 + xy + y 2 =2의정수해 (x; y) 를모두구하여라. ( 이탈리아 1989-1) 풀이1 x가홀수이면 x 2 +(x + y)y 는홀수가되어모순. x, y가모두짝수이면 4의배수가되어모순. 해없음. 풀이 2 mod 3 으로 (x y) 2 2 가되어모순. 149. 어떤정수 x 에대해서도 x 2 +5x +16 은 169 로나누어떨어지지않음을증명하여라. ( 이탈리아 1990-5) 증명 13 j (x 4) 2. 캐나다의 121 의배수가아니더란문제와비슷. 150. 46 2n+1 +296 13 2n+1 이 1947 의배수임을증명하여라. ( 헝가리 1947-1) 증명 46 2 13 2 (mod 1947) 이므로 13 2n (46 + 296 13) 0 (mod 1947) 1.2 수론고급문제 1. 10101 은몇진법으로읽어도항상합성수임을증명하여라. ( 캐나다 1972-3b) 증명다음과같이인수분해되므로합성수이다. 10101 (n) = n 4 + n 2 +1=(n 2 +1) 2 n 2 =(n 2 + n +1)(n 2 n +1) n 2 이므로 n 2 n +1> 1 은틀림없이성립한다. 2. 네개의홀수인양의정수 a, b, c, d (a b c d) 중에서임의로세수를뽑아서, 그합을남아있는한수로나누면그나머지가언제나 1 이라고한다. 이러한네홀수의순서쌍 (a; b; c; d) 를모두구하여라. ( 한국 2003-J5)
32 수론 풀이 a + b + c + d = S 라하면, 적당한정수 k 1 ;k 2 ;k 3 ;k 4 에대해 S a = ak 1 +1; S b = bk 2 +1; S c = ck 3 +1; S d = dk 4 +1 S a = ak 1 +1, S b = bk 2 +1 에서다음이성립한다 : a(k 1 +1)=b(k 2 +1). 이와같이풀면 a(k 1 +1)=b(k 2 +1)=c(k 3 +1)=d(k 4 +1) 1 식 1 의값을 k 라고하자.4d S = d(k 4 +1)+1, d(3 k 4 ) 1(* d c b a 1). 3 k 4 1, 2 k 4. 그런데 S 는짝수이므로 d(k 4 +1)+1 은짝수이다. 즉, k 4 6=1, k 4 =2 ) d(k 4 +1)=k 에서 k =3d. c(k 3 +1)=d(k 4 +1)=k 에서다음이성립한다. 3c a + b + c = S d =2d +1= 2 3 c(k 3 +1)+1 3:5 3 2c k 3, k 3 은정수이므로 3 k 3. 그런데 3d = c(k 3 +1) 에서 k 3 은짝수이므로 k 3 =2 즉, c = d 이다. b(k 2 +1)=c(k 3 +1) 에서다음이성립한다. 2b a + b = S 2c =3c +1 2c = c +1= 1 3 b(k 2 +1)+1; 4 k 2 그런데 3c = b(k 2 +1) 에서 k 2 는짝수이므로 k 2 =2또는 4이다. k 2 =2이면 b = c = d에서 S = 3d +1=a +3d 이므로 a =1이다. 그러나이것은 S a를 a로나눠나머지가 1인것에모순이다. 따라서, k 2 =4 즉, a(k 1 +1)=5b =3c =3d 이고, a + b = c +1에서 2c =5a 5, 15(a 1) = 6c =2a(k 1 +1), 15a 15 = 2ak 1 +2a. (13 2k 1 )a =15=15 =5 3 앞에서 a =1이면모순이므로, a =15또는 5또는 3이다. a =15! b =21; c =35; d =35 ( 성립 ) a =5! b =6; c =10; d =10 (b; c; d가짝수 ) a =3! b =3; c =5; d =5 ( 성립 ) ) (a; b; c; d) =(3; 3; 5; 5), (15; 21; 35; 35). 3. 다음등식을만족하는양의정수해 (a; b; c) 는존재하지않음을보여라. (2a + b)(2b + a) =2 c ( 뉴질랜드 2003-9) 증명귀류법으로, 양의정수해 (a; b; c) 가존재한다고가정하자. 그러면좌변에서 2a + b; 2b + a 3 이므로둘다 2 k 꼴, 즉짝수이고, b, a 모두짝수여야한다. a =2 n a 0 ; b =2 n b 0 인최대의 n 을잡으면 a 0 과 b 0 중적어도하나는홀수이다. 그러면, 준식은다음과같다. 2 2n (2a 0 + b 0 )(2b 0 + a 0 )=2 c 여기서 a 0, b 0 중적어도하나는홀수이므로,2a 0 + b 0,2b 0 + a 0 중적어도하나는 3 이상의홀수이다. 그것은우변 2 c 이 3 이상의홀수를약수로갖는다는것이므로모순. 따라서, 양의정수해 (a; b; c) 는존재하지않는다.
1.2 수론고급문제 33 4. f1; 16; 27g 중에서어떤두수의곱에 9를합한것은항상제곱수임에주목하자. n +9,16n +9,27n +9 가모두제곱수가되도록하는유일한자연수 n을구하여라. (IMTS R3-1) 풀이 n +9=a 2,16n +9=b 2 이라하면 16(a 2 9) = b 2 9, 16a 2 b 2 =135, 즉 (4a + b)(4a b) =3 3 5 이렇게인수분해법으로풀고그중에서 27n +9 도완전제곱수가되는것을찾으면... n =280. 그런데문제에힌트처럼제시한것은무슨도움이되는걸까... 5. 17 명의해적이금화한상자를훔쳐서공평하게나누다보니 3 개가남았다. 그남은것을서로차지하려고싸우다가한명이죽었다. 그래서다시나누어보니이번에는 10 개가남았다. 또싸움이벌어져서세명이죽고나서야비로소공평하게나누어가질수있었다. 각자 1000 개이상씩의금화를가졌다면훔친금화는모두몇개인가? 그최소값을구하여라. ( 통신강좌 1990-1-6) 풀이 x 3 (mod 17), x 10 (mod 16), x 0(mod13) 을동시에만족시키며 x 13000 인것들중최소인x를찾으면된다. 으로놓고중국인의나머지정리를이용하자. m 1 =17; m 2 =16; m 3 =13; a 1 =3; a 2 =10; a 3 =0 m =17 16 13 = 3536; M 1 = m m 1 =16 13; M 2 = m m 2 =17 13; M 3 = m m 3 =17 16 k =1; 2; 3 에대하여 M k X k 1(modm k ) 를계산하여 X 1 4 (mod 17); X 2 5 (mod 16); X 3 1 (mod 13) 을얻는다. 따라서 x = a 1 M 1 X 1 + a 2 M 2 X 2 + a 3 M 3 X 3 3 16 13 ( 4) + 10 17 13 5+0 17 16 ( 1) 8554 (mod m): 따라서구하는 x는 15626이다. 6. 1보다큰모든자연수n에대해,(n 1) 2 이 n n 1 1 의약수임을증명하여라. ( 플란더즈 2001-1) 풀이 n 대신 n +1 로치환하면 n 2 j (n +1) n 1 임을보이는문제가되고, 우변을인수분해하거나낮은차수의항을간단히이항전개해보면금방확인된다. 7. 임의의정수 n>1 에대해, n 의가장큰소인수를 p(n) 으로나타내자. 다음을만족하는서로다른양의정수 x, y, z 의순서쌍을모두구하여라. (i) x, y, z는등차수열을이룬다. (ii) p(xyz) 3. ( 영국 2003 2차-1) 풀이 gcd(x; y; z) =g 라하고 (x; y; z) =(ga; gb; gc) 라놓자. 그럼,gcd(a; b; c) =1이고, g =2 r 3 s 꼴이며, a, b, c는여전히등차수열이다. a + c =2b 이므로 a와 c는홀짝이같다. (1) a, c가모두홀수일때 ; a>1이라면 a와 c는모두3의거듭제곱수이고, 그럼 b =(a+c)=2도3의배수가되어 gcd(a; b; c) = 1 이라는데에모순이다. 따라서, a =1. c = 3 n 이라놓으면공차 d가 3의배수일수없으므로 b도 3의배수가아니다. 즉, b는짝수. b =(a + c)=2 이므로 a + c(= 1 + c) 가 4의배수이고, 여야하므로, n 은홀수. n =2m +1 로두면 c =3 n 3 (mod 4) c =3 9 m ; a+ c 4 (mod 8); b 2 (mod 4) 가된다. 그러므로, b =2,c =3. (x; y; z) =(g; 2g; 3g).
34 수론 (2) a, c 가모두짝수일때 ; b 도짝수이면안되므로, b 는 3 의거듭제곱수이다. a 나 c 가 3 으로나누어진다면모두 3 의배수가되어모순이다. 따라서, a, c 는짝수, 즉 2 의거듭제곱수이다. a 와 c 가모두 4 이상이면 b =(a + c)=2 가짝수가되어곤란하므로, a =2. c =2 n+1 이라놓으면 b =(a + c)=2 =2 n +1=3 m 꼴이다.(a; b; c) =(2; 3; 4), (2; 9; 16) 은해가되고, m 3 이면더이상해가없음을보이자. n 4 이므로 b =3 m 1(mod16) 이다. m =4k + r 로두면 b =3 r 81 k 3 r (mod 16). 따라서 r =0, 즉 m =4k 꼴일때에만가능하다. 그런데,2 n =81 k 1 0(mod5) 이므로 5 j 2 n. 이것은모순. (1), (2) 에의해답은 (x; y; z) =(g; 2g; 3g); (2g; 3g; 4g); (2g; 9g; 16g) 가전부이다. 단, g =2 r 3 s 꼴의수이다. 8. 어떤양의정수가연속한둘이상의양의정수들의합이된다는것과그수가 2 의거듭제곱수가아니라는것이필요충분임을증명하여라. ( 캐나다 1976-5) 증명 임의의연속한둘이상의양의정수들의합은 n; k 1 에대해 n +(n +1)+ +(n + k) = (k +1)(2n + k) 2 로나타낼수있다. 분자의 (k +1) 과 (2n + k) 는서로홀짝이다르고둘다 2 이상이므로이합은홀수인소인수를갖는다. 따라서 2 의거듭제곱일수없다. 한편, N 이 2 의거듭제곱이아닌임의의양의정수라하자. 그럼 2N 도 2 의거듭제곱이아니고, 2N = ab (a 는 2 의거듭제곱수, b 는홀수 ) 와같이분해할수있다. a 와 b 모두 2 이상이고홀짝이다르므로, 둘중작은것을 k +1 로두고둘중큰것을 k +2n 으로둘수있다 (n; k 1). 그럼 N = (k +1)(2n + k) 2 = n +(n +1)+ +(n + k) 로연속한둘이상의양의정수들의합으로나타낼수있다. 9. 1990 은오른쪽에자릿수하나를붙이고왼쪽에몇개의자릿수를붙임으로써 ` 제곱수로만들수있다 '. 예를들면,419904 =648 2 이다. 같은과정을 1991 에적용시켰을때, 1991 은제곱수로만들수없음을보여라. 즉, yx1991d 가제곱수가되도록하는자릿수 d;x;y;::: 는존재하지않음을보여라. (IMTS R3-2) 증명십의자리가홀수임에서일의자리는 6. 그리고나서 mod 16으로체크하면 12가되어제곱수가아님을확인. 10. p 와 p 2 +8 이모두소수이면 p 3 +4 도소수임을증명하여라. ( 통신강좌 1993-7-20) 풀이 p =3이면, p 2 +8 = 17, p 3 +4 = 31 로모두소수이다. p 6= 3이면 3 - p 이므로 p 1 (mod 3), 결국 p 2 +8 0(mod3) 이되고,3의배수. 따라서, p와 p 2 +8이모두소수이면 p 3 +4 도소수이고, 이런경우는 p =3일때뿐이다. 11. m 1 ;m 2 ;:::;m s 가임의의자연수라고하자.[m i ;m j ] 를 m i, m j 의최소공배수,(m i ;m j ) 를 m i, m j 의최대공약수라할때다음을증명하여라. ( 통신강좌 1991-3-6) [(m 1 ;m s); (m 2 ;m s);:::;(m s 1 ;m s)] = ([m 1 ;m 2 ;:::;m s 1 ];m s) 풀이 a = Q p e i i, b = Q p f i i 라하자. 단, p i는 i번째소수이다. 그러면 [a; b] = Y p max(e i;f i ) i ; (a; b) = Y p min(e i;f i ) i (1)
1.2 수론고급문제 35 이된다. 준식의좌변과우변을각각 Q p i i, Q p i i 라하고, m j = Y i p n j;i i (j =1; 2; ;s) 라하자. 그러면 (1) 로부터 i = max[min(n 1;i ;n s;i ); min(n 2;i ;n s;i ); ; min(n s 1;i ;n s;i )]; i = min[max(n 1;i ;n 2;i ; ;n s 1;i );n s;i ] 이성립하며, 임의의 i 에대해 i = i 임을보이면된다. 편의를위해마지막첨자 i 를생략하기로하고, max(n 1 ;n 2 ; ;n s 1 )=n k 라하자. (i) n s n k 일때 ; =max(n 1 ;n 2 ; ;n s 1 )=n k 이고, =min(n k ;n s)=n k 이므로성립한다. (ii) n s <n k 일때 ; =max(t 1 ;t 2 ; ;t s 1 ) 라할때, t j =min(n j ;n s) n s 이고 t k = n s 이므로 = n s 이다. 또 = n s = 이므로이경우에도성립한다. (i), (ii) 에의해임의의 i 에대해 i = i 가성립하므로문제의등식이성립한다. 12. x 2 + y 와 x + y 2 이모두완전제곱수가되게하는자연수 x 와 y 는존재하지않음을보여라. ( 소련 1966-3) 증명 x 2 + y 는 x 2 보다큰완전제곱수이므로 x 2 + y (x +1) 2, 따라서 y 2x +1; 마찬가지로 x 2y +1 그럼 y>x, x>y 이므로이것은모순이고, 이런자연수해는존재하지않는다. 13. 임의의양의정수 n 에대해항상 n 3 (n 2 1)(n 2 4) 의약수가되는가장큰정수를구하여라. (1989 뉴욕주수학리그 ) 풀이 n 3 (n 2 1)(n 2 4) = n 2 (n 2)(n 1)n(n +1)(n +2)=A 가된다. 이때연속한 n 개의수는 n! 임의배수임을이미알고있다. 이값을실제 n =3,4 일때계산해보면 9*5!, 32*3*5! 이된다. 이둘의공약수는 3*5! 이므로우리는 A 가항상 3*5! 을약수로가진다고추측할수있다. 만약 n =3k 라면 A = n 2 *(5! 의배수 ) 이므로 A 는 3*5! 의배수이다. 또한 n 6= 3k 라면 n 2, n 1, n +1,n +2 중적어도두개는 3 의배수이고, A 는 9 의배수가된다. 5! 의배수이며동시에 9 의배수이므로 3*5! 의배수가된다. 따라서항상 A 는 3*5! 을약수로가진다. 만약 A 가 3*5! 보다큰수를약수로가진다고하면그수는 k 3 5! 로표현가능할것이나 (k 6= 1)n =3, 4 일때에대입해보면 k 는 3 의약수이면서 32 의약수여야하는데이는불가능하다. 따라서 3*5! 가항상 A 의약수가되는가장큰정수이다. 14. w! =x!+y!+z! 의모든양의정수해w, x, y, z를구하여라. ( 캐나다 1983-1) 풀이 x; y; z < w 이다. ² w 4 이면 w! > 3(w 1)! x!+y!+z! 이므로해가없다. ² w 2 이면 w! 2 < 3 x!+y!+z! 이므로해가없다. ² w =3이면 3! = 3 2! x!+y!+z! 이고등호는 x = y = z =2일때성립. 따라서, 해는유일하다. 답 (w; x; y; z) =(3; 2; 2; 2) 15. 다음과같은꼴로나타낼수있는양의정수를모두구하여라. b a + c a + c b + a b + a c + b c 단, a, b, c 는쌍마다서로소 ( 즉,gcd(a; b) =gcd(b; c) =gcd(c; a) =1) 인양의정수이다. ( 한국 2006-J2)
36 수론 풀이 a b c 라해도되겠다 1. 이고이것이양의정수이려면 준식 = (a + b)ab +(b + c)bc +(c + a)ca abc abc j (a + b)ab +(b + c)bc +(c + a)ca 여야한다.(b + c)bc +(c + a)ca 는 c 의배수이므로나머지항 (a + b)ab 도 c 의배수이다. ab 와 c 는서로소이므로 c j (a + b) 이다. 1에의해 a + b 2c 이므로 a + b = c 또는 2c이다. (1) a + b =2c 일때 : 이때a = b = c 이며, 쌍마다서로소이므로그값들은모두 1이다. 그럼준식 =1+1+1+1+1+1=6. (2) a + b = c 일때 : c j (a + b) 를이끌어낸과정과같은방법으로 b j (a + c) 가된다. 즉, b j (2a + b) 이고, b j 2a 이다. 1에의해 2a 2b 이므로 2a = b 또는 2b이다. (2a) b =2a 일때 : a =1,b =2,c =3이고준식 = 1+2 + 2+3 + 3+1 =8. 3 1 2 (2b) b = a 일때 : a =1,b =1,c =2이고준식 = 1+1 + 1+2 + 2+1 =7. 2 1 1 답은 6, 7, 8이다. 16. a, b, c, d 는고정된정수들이고 d 는 5 로나누어지지않는다고하자. m 을 가 5 로나누어지도록하는정수라고가정하자. 그럼 am 3 + bm 2 + cm + d dn 3 + cn 2 + bn + a 도 5 로나누어지도록하는정수 n 이존재함을증명하여라. ( 헝가리 1900-1) 증명 m이 5의배수이면 am 3 + bm 2 + cm + d은 5의배수가아니므로,5-m. 그럼 m은 5와서로소이므로 mod 5로 m의잉여역수 n이존재한다 (mn 1(mod5) 가되는수를말함 ). 그럼 n 3 (am 3 + bm 2 + cm + d) a + bn + cn 2 + dn 3 0(mod5). 17. 어떤자연수의자릿수근이란각자리의수들을모두곱하는것을계속반복하여얻은한자리의수를말한다. 예를들어 24378! 1344! 48! 32! 6 이므로 24378 의자릿수근은 6 이다. 자릿수근이 1 인수는모든자릿수가 1 인수뿐임을보여라. ( 아일랜드 1992-7) 증명 a! 111 1! 1 에서 111 1 은 10 보다작은소인수만갖는다. 그중 2 나 5 는있으면안되고, 3 과 7 뿐인데,3 은 111 의약수이므로 3 이있으려면 37 이있어야해서안된다. 고로 7 뿐.111 1=7 라하면,7 n 들은 mod 100 으로 01! 7! 49! 343! 2401 와같이순환함. 따라서, =0 뿐. 18. a n 이 1 3 +2 3 + + n 3 의일의자리숫자이다. 이때,0:a 1 a 2 a n 이유리수임을보여라. ( 셈본중등초급도전문제 2.3.4) 증명 0:a 1 a 2 a n 이순환소수임을확인하면된다.(10k + r) 3 r 3 (mod 10) 이므로,0 3 +1 3 + +9 3 a (mod 10) 이라하면 a n+10 a n + a, 즉 a n+100 a n +10a a n (mod 10) 이다. 즉, 순환마디가 100( 혹은 100의약수 ) 인순환소수가된다. 주 r 3 +(10 r) 3 r 3 r 3 =0(mod10) 임을관찰하면 a 5 임을알수있고순환마디도보다짧은 20임을관찰할수도있다.
1.2 수론고급문제 37 19. (1) x 2 + y 2 +1=z 2 이무한히많은정수해를가짐을보여라. (2) 임의로주어진정수 a 에대해 x 2 + y 2 + a = z 2 이무한히많은정수해를가짐을보여라. ( 셈본중등고급도전문제 2.3.2 및확장, 스웨덴 1974-5 유사 ) 증명 (1) 임의의정수 m 에대해 (x; y; z) =(2m 2 ; 2m; 2m 2 +1) 이면항상해가된다. (2) 주어진 a 의홀짝에따라 x 2 +a 가항상홀수가되도록 x 를무한히많이택할수있다. x 2 +a =2k +1 로두면 y = k, z = k +1 이면항상해가된다. 20. p p+1 + q q+1 이소수가되는소수 p, q 를모두구하여라. (2000 대전. 충남영재수학교실 2 차평가 ) 풀이우선 p =2,q =3( 또는 p =3,q =2) 일때p p+1 + q q+1 =89로성립한다. p, q가둘다홀수인소수이면p p+1 + q q+1 은 2보다큰짝수가되어소수가될수없다. 둘다 2일때도마찬가지. 따라서둘중에하나는2가되고나머지하나는홀수인소수가되어야한다. 홀수인소수 (q라하자) 가 3이아닐경우 2 2+1 =8 2 (mod 3) q q+1 =(3k 1) 짝 1 (mod 3) 즉,2 2+1 + q q+1 0(mod3) 이되어소수가아니다. 따라서구하려는쌍은 (2; 3), (3; 2) 둘뿐이다. 21. n 은음이아닌정수이고, d 0 ;d 1 ;:::;d n 은 0, 1, 2 로만이루어진수열이다. d 0 +3d 1 + +3 k d k + +3 n d n 이어떤자연수의제곱이라면, d i =1 이되는 i (1 i n) 가적어도하나존재함을증명하여라. ( 호주 1989-5) 개요귀류법.3 m (3k +2) 꼴이라서제곱수가아님. 22. 연속한 1984 개의양의정수들의제곱의합은정수의제곱이될수없음을보여라. ( 캐나다 1984-1) 증명 연속한 1984 개의제곱의합은 1984 X S = (n + i) 2 = 1984 X (n 2 +2ni + i 2 ) i=1 i=1 = 1984n 2 1984 1985 1984 1985 3969 +2n + 2 6 = 992(2n 2 +2 1985n +1985 1323) 이식은 2 5 ( 홀수 ) 꼴이므로 2 의지수가짝수가아니어서제곱수가될수없다. 23. x + y 2 + p xy =40을만족하는서로다른양의정수쌍 fx; yg는꼭하나있음을증명하여라. ( 루마니아지역예선 2005-y7-3b) 증명 x+y 2 + p xy =40,2 배하면 ( p x + p y ) 2 =80 와같은식이다. 즉, p x + p y = p 80 ( ) 한항을넘긴식 p y = p 80 p x 를제곱하면 y =80+x 2 p 80x. 따라서 2 p 80x 은정수.2 p 80x = m 으로두고제곱하면 8 2 5x = m 2. 따라서 x =5a 2 마찬가지로 y =5b 2 꼴이다 (a, b 는적당한양의정수 ). ( ) 에대입하면 a + b =4. x 6= y 라했으므로 a = b =2 일수없고, 따라서순서를무시하면 (a; b) =(3; 1) 뿐이다. 즉, 서로다른양의정수쌍 (x; y) 는순서를무시하면 (45; 5) 뿐이다.
38 수론 24. 다음과같은꼴로나타낼수있는양의정수를모두구하여라. m 2 +20mn + n 2 m 3 + n 3 단, m, n 은서로소인양의정수이다. ( 한국 2006-J5) 풀이 ( 서울당산서중 2 학년박민재 ) 준식 = m 2 +20mn + n 2 (m + n)(m 2 mn + n 2 ) 이므로 (m + n) j (m 2 +20mn + n 2 ), 즉 (m + n) j ((m 2 +20mn + n 2 ) (m + n) 2 )=18mn 이어야한다. 그런데 gcd(m; n) =1 이므로 gcd(mn; m+n) =1(* gcd(m; m+n) =gcd(n; m+n) =1) 이고결국 m + n j 18 을얻는다. (i) m + n =9or18 일때 : 준식 = m 2 +20mn + n 2 (m + n)(m 2 mn + n 2 ) = (m + n) 2 +18mn (m + n)((m + n) 2 3mn) 이다. m + n 이 9 의배수이면분모는 27 의배수이므로분자역시 27 의배수이어야한다. 그러기위해서 m 또는 n 이 3 의배수이어야한다. 이것은 gcd(m; n) =1 이라는것에모순이다. 즉, 이경우에해는없다. (ii) m + n =2or3or6 일때 :gcd(m; n) =1 이되는 (m; n) 은 (1,1), (1,2), (1,5) 뿐이다 ( 순서상관없음 ). 세경우모두대입해보면해가된다. m, n 은양의정수이므로 m + n =1 이될수없고 (i), (ii) 에의해서는해는세가지뿐이다. 각각의경우원래식의값을구해보면 11, 5, 1 이나온다. 25. 2005 2 을 2 개의 ( 양의 ) 완전제곱수의합으로나타내는방법이네가지이상있음을증명하여라. ( 플란더즈 2005-J3/S3) 증명 2005 = 5 401 = (2 2 + 1)(20 2 +1) 이다.Lagrange 항등식 (a 2 + b 2 )(c 2 + d 2 )=(ac bd) 2 +(ad + bc) 2 에의해 (c 2 + d 2 이 d 2 + c 2 으로순서가바뀔수도있으므로 ) 2005 = (2 2 +1)(20 2 +1)=39 2 +22 2 =41 2 +18 2 이된다. 다시이로부터 2005 2 =(41 2 +18 2 )(41 2 +18 2 )=(41 2 18 2 ) 2 +(2 41 18) 2 2005 2 =(39 2 +22 2 )(39 2 +22 2 )=(39 2 22 2 ) 2 +(2 39 22) 2 2005 2 =(41 2 +18 2 )(39 2 +22 2 )=(41 39 18 22) 2 +(41 22 + 18 39) 2 2005 2 =(41 2 +18 2 )(39 2 +22 2 )=(41 39 + 18 22) 2 +(41 22 18 39) 2 등네가지방법으로나타낼수있다 ( 물론위의네가지계산에서나타나는수들이모두다르다는것은확인할필요가있다 ). 26. 서로다른임의의세정수 a, b, c 가주어졌을때, a + n, b + n, c + n 이둘씩서로소가되도록하는정수 n 을항상찾을수있을까? ( 셈본중등고급도전문제 2.2.2 변형 ) 증명임의의서로다른양의정수 a, b 에대해 n, n + a, n + b 가서로소가되도록 n 을잡아보자. a = da 0, b = db 0,gcd(a 0 ;b 0 )=1 이라할때, a 0 b 0 은 a 0, b 0 과모두서로소이다. 따라서, kb 0 1 (mod a 0 b 0 ) 인 k 를잡을수있다. 이제 n =1 da 0 b 0 k 라두면세수가모두서로소가된다.
1.2 수론고급문제 39 27. 임의의자연수 n에대해 (2n)! 이항상정수가됨을보여라. (n +1)!n! 증명임의의소수 p에대해, p가분자에곱해져있는횟수가분모에곱해져있는횟수이상임을보이면된다. m! 을소인수분해했을때소수 p의지수는 b m p c + b m p 2 c + b m p 3 c + 의공식으로구할수있음을이용하자. 즉, 임의의소수 p와임의의자연수 k에대해 ¹ º 2n p k ¹ n +1 p k º ¹ º n + p k 임을보이면충분하다 ( 이런식을모든 k에대해변변더하면됨 ). a가 b의배수가아닐때는 b a b c = b a 1 b c 가성립한다. 즉, n이나 n +1 중에 p k 의배수가아닌것이있으면둘중하나를 1만큼줄여도우변이변하지않고, 그럼 ba + bc bac + bbc 의당연한부등식에의해성립한다. n과 n +1 모두 p k 의배수가되는경우만염려하면되는데, 그럼 p k j (n +1) n =1이므로이런경우는없다. 주이항계수를안다면 2n n 2n n+1 을정리했을때준식이된다는것을확인할수도있다. 그럼정수의차이므로당연히정수. 한편, 준식은 n개의개괄호와 n개의폐괄호를잘정렬하는경우의수인 Catalan 수로알려져있는식이기도하다. 28. 1 4 9 ::: n 2 꼴의합을생각하자. 임의의정수를이꼴로표현할수있음을증명하여라. ( 예를들어,3= 1+4,8=1 4 9+16+25 36 49 + 64 이다.) ( 이탈리아 1991-3) 증명 k 2 (k +1) 2 (k +2) 2 +(k +3) 2 =4임을이용.0,1,2(= 1 4 9 + 16), 3= 1+4가가능하므로,4씩증가시켜서모두다가능함. 홀수는 (a +1) 2 a 2 =2a +1로도할수있음. 29. a, b, c는 1 a + 1 b = 1 를만족하는최대공약수가 1인양의정수들이다.(a + b) 가완전제곱수임을증명하 c 여라. ( 인도지역예선 1992-2) 풀이 a와 b의최대공약수를 d라하자. a = dx; b = dy라하자. 그러면 x와 y는서로소이다. 양변을통분하고 d를오른쪽으로넘기면, x + y = d xy c 이된다. x 와 y 가서로소이므로 x + y 와 xy 도서로소이고, c 와 d 도서로소이므로양변이모두기약분수이다. 즉, x + y = d, xy = c 이다. 따라서 a + b = dx + dy = d 2 으로 a + b 는완전제곱수이다. 30. 맨앞의자릿수를맨마지막으로이동시키면 2 배가되는정수가존재하는지그렇지않은지밝혀라. ( 캐나다 1985-2) 증명주어진조건을만족하는수를 a 10 n + b 라하자 (a 는맨앞의자릿수이고, b 는 n 1 자리의수 ). 10b + a =2 (a 10 n + b) 8b =(2 10 n 1)a 좌변은 8 의배수이고 (2 10 n 1) 은홀수이므로 8 j a, 즉 a =8 이어야한다. 그럼 b =2 10 n 1 이고, 이것은 n 자리의수이므로자리수가넘어모순. 따라서, 이런정수는존재하지않는다. 31. p n + p n +2005 가정수가되도록하는정수 n 을모두구하여라. ( 플란더즈 2005-4) 풀이 p n이유리수이면 p n은정수이다 ( ). 즉, p n은정수이거나무리수이다. (i) p n, p n + 2005 중하나만정수인경우 : 정수 ( 유리수 ) 와무리수의합은무리수이므로불가능. (ii) p n, p n +2005 가둘다무리수인경우 : p n = a p b, p n +2005 = c p d 라하자 ( 단, b, d는무승수, 즉제곱수를약수로갖지않는수 ). p p p p n + n +2005=a b + c d = k( 정수 )
40 수론 라하면 a 2 b + c 2 d +2ac p bd = k 2 이므로 p bd는유리수, 즉 ( ) 에의해정수이다. 그럼 bd는완전제곱수인데, b와 d는무승수이므로소인수분해를생각하면모든소인수를공통으로갖고있어야해서 b = d 임을알수있다. 이때 p p p n + n +2005=(a + c) b 6= 정수 이므로모순. (iii) p n, p n + 2005 가둘다정수인경우 : p n = m, p n + 2005 = k 라하면 m 2 = n, k 2 = n + 2005, 즉 k 2 m 2 =2005 이므로 (k + m)(k m) = 2005 = 5 401 여기서가능한경우는 (k m;k + m) = (1; 2005) 혹은 (5; 401) 뿐이다. 각각풀면 (k;m) = (1003; 1002), (203; 198), 즉 n = 198 2 과 1002 2 만가능하다. (i){(iii) 에서답은 n =198 2,1002 2. 32. 옛날옛날에서코나라가있었다. 그나라의임금님은이름을남기는것을좋아했다. 어느날자신의이름을어디다남길까고민하다가 \ 그래나만의화폐를만드는거야!" 하고결심했다. 그래서그다음날그나라의화폐는모두서코임금의초상화가그려진화폐로모두바뀌었다. 발행된동전은 5원짜리,9원짜리,12원짜리의 3가지였다. 어느날심심한서코임금은신하들에게이런문제를냈다. \ 내가만든 3 가지금화로거스름을받지않고는정확하게지불할수없는가장큰금액은얼마이겠는가?" 과연얼마일까? ( 셈본중등초급도전문제 2.2.2) 풀이먼저 12원짜리는고려하지말고 5원짜리와 9원짜리동전만생각해보자.9원짜리 5개를쓰는것은 5원짜리 9개를쓰는것으로바꿀수있으므로 9원짜리 5개이상을쓰는것은고려하지않아도된다. 9원짜리를 0개,1개,2개,3개,4개쓸때를각각생각해보면두동전으로만들수없는금액은 5n꼴 9원짜리 0개 : 없음 5n +1꼴 9원짜리 4개 : 161116212631 5n +2꼴 9원짜리 3개 : 27121722 5n +3꼴 9원짜리 2개 : 3 8 13 5n +4꼴 9원짜리 1개 : 4 들이다. 이외의수는 5 원짜리와 9 원짜리두동전으로다만들수있다. 이제 12 원짜리를함께고려하자. 12 원짜리를 1 개쓰면위의목록중 12, 17, 21, 22, 26, 31 원은만들수있게된다.12 원짜리를 2 개써서더만들수있는금액은없다. 따라서, 세가지금화로정확하게지불할수없는금액은 1, 2, 3, 4, 6, 7, 8, 11, 13, 16 원. 답 16 원 33. 성식이는각각 1; 2; 4; 8;::: 의 2 의거듭제곱수가차례로한장에하나씩적혀있는카드를갖고있었다. 성식이가갖고있는이카드의수는 30 장에서 35 장사이였는데, 며칠전에그중몇장을잃어버렸다. 남은카드의수의합은원래전체카드의수의합의정확히 1=5 이라고한다. 잃어버린카드는모두몇장인가? (1994 미국지역별수학리그 ) 풀이원래카드의수가 n장이었다면카드의수의합은 1+2+ +2 n 1 =2 n 1. 이것이일부카드를잃어버린후의 5배이므로 5의배수.2 n 은 mod 5로 2, 4, 3, 1을반복하므로, n은 4의배수.30 n 35 라했으므로 n =32. 11111111111111111111111111111111 (2) =101 (2) = 110011001100110011001100110011 (2) 이므로나중의카드개수는 16 개, 잃어버린카드의개수도 16 개이다. 답
1.2 수론고급문제 41 34. 7 j 3 n + n 3 은 7 j 3 n n 3 +1 이되기위한필요충분조건임을보여라. ( 통신강좌 1997-14-2, 몰도바 1996 최종 -y9-5 변형 ) 증명 7 j (3 n 1)(3 n +1)(n 3 1)(n 3 +1) 만보이면됨. 35. 잉여식 x a (mod n), x b (mod m) 이해를가질필요충분조건은 gcd(n; m) j (a b) 임을보여라. 또, 해가존재하는경우, 그해는 lcm(n; m) 에대한잉여계로유일함을보여라. ( 통신강좌 1993-7-3) 풀이 x a(mod n), x b(mod m) 의해가존재한다는것은 x = pn + a = qm+ b를만족하는 p; q가존재한다는것이다. a b = qm pn을만족하는 p; q가존재할필요충분조건은 gcd(m; n)j(a b) 이다또다른정수y가 y a(mod n), y (mod m) 을만족한다면, x y(mod n), x y(mod m), 즉 nj(x y), mj(x y) 이므로 lcm(n; m)j(x y) 이다. 따라서 x y(mod lcm(n; m)) 이므로해의유일성이증명되었다. 36. 자연수 n 의막자리합 D(n) 은다음과같이반복적으로정의된다 : D(n) = ( n; 1 n 9 일때 D(a 0 + a 1 + + a m); n > 9 일때 단, a 0 ;a 1 ;:::;a m 은 n을십진법으로썼을때의각자릿수들이다. 예를들어, D(989) = D(26) = D(8) = 8 과같이되는것이다. 임의의 n =1; 2; 3;::: 에대해 D(1234 n) =D(n) 이성립함을증명하여라. ( 셈본중등초급도전문제 2.4.3) 증명 n을십진법으로썼을때의각자릿수의합을 f(n) 이라하자. 그럼 D(n) 은결과가한자리수가될때까지f(f( f(n) )) 와같이f를반복적용하는것이다. D(n) n (mod 9) 임을보이자. f(n) n (mod 9) 임만보이면충분하다. n 을십진법으로쓴것이 n = a m a 1 a 0 이라면, n f(n) =(10 m a m + 10a 1 + a 0 ) (a m + + a 1 + a 0 ) =9 99 a {z m + +9a 1 +0a 0 m 개 로 9 j n f(n) 임을알수있다. 따라서, f(n) n (mod 9) 이다. 따라서 D(n) n (mod 9) 이고, ( n을 9로나눈나머지 (9 - n 일때 ) D(n) = 9 (9 j n 일때 ) 가된다. 이로부터 A B (mod 9) 이면 D(A) =D(B) 가됨을알수있다. 이제 1234n n (mod 9) 이므로문제가증명되었다. 37. a 2 + b 2 = n! 의모든양의정수해 a, b, n 을구하여라. 단, a b 이고 n<14 이다. ( 캐나다 1987-1) 풀이우선 n 7 일때를보자.7jn! 이므로 7 j a 2 + b 2. 완전제곱수는 n 2 0; 1; 2; 4(mod7) 만이가능하므로, a 2 + b 2 0(mod7) 이될수있는쌍은 (a 2 ;b 2 ) (0; 0) (mod 7) 뿐이다. 따라서,7ja; b 이고,7 2 j a 2 + b 2 = n!. 그러나, n<14 이므로 7 2 - n! 이고, 그럼이경우에는해가없다. 다음으로 3 n 6 일때를보자. 이때3jn! =a 2 + b 2 이되고, n 2 0; 1(mod3) 만이가능하므로, 앞에서와비슷하게 3 j a; b, 즉 3 2 j a 2 + b 2 = n! 이다. n 5 이면 3 2 - n! 이므로 n =6이어야한다. mod 4로생각하면 a, b 모두짝수, 즉 6 j a; b 임도알수있다.(6a 0 ) 2 +(6b 0 ) 2 =6!=720의해를구하면, a 02 + b 02 =20에서 (a 0 ;b 0 )=(2; 4) 뿐이므로 (a; b) =(12; 24) 이다. 마지막으로 n =1; 2 일때는그냥구해보면 (a; b; n) =(1; 1; 2) 만이가능하다. 답 (a; b; n) =(1; 1; 2), (12; 24; 6)
42 수론 38. 자연수 m, n, k 가주어져있다. 이때, rm + sn 이 k 의배수가되도록하는서로소인두수 r, s 를항상찾을수있음을증명하여라. ( 소련 1961-9) 증명 Care is needed. Although easy, this is more awkward than it looks. Let d =(m; n), the greatest common divisor of m and n. Let r = n=d, s = nhk m=d, where h is any integer su±ciently large to ensure that s>0. Now rm + sn = mn=d + nnhk mn=d = nnhk, whichisa multiple of k. If e divides r, then it also divides rdhk = nhk. So if e divides r and s, thenitalso divides s nhk = m=d. Butn=d and m=d are relatively prime, so e must be 1. Hence r and s are relatively prime. 39. 다음방정식을만족하는자연수 a, b, c를모두구하여라. ( 인도지역예선 1996-2) µ 1+ 1 µ 1+ 1 µ 1+ 1 =3 a b c 관찰좌변에서 a; b; c 등이충분히크면 1+ 1 a,1+1 b, 1 c 등이 1보다얼마크지않은수가되어그곱이 3보다작을것이다. 즉, a; b; c가충분히작을때만해가존재할가능성이있다. 따라서, 이부정방정식은부등식법으로접근하는것이좋겠다. 풀이 일반성을잃지않고 a b c 라하자. a 3 이면준식은 µ 3= 1+ 1 µ 1+ 1 µ 1+ 1 µ 1+ 1 3 = 64 a b c 3 27 < 3 으로성립하지않는다. 따라서, a 2. (i) a =2일때 : 준식은다음과같이바뀐다. µ 1+ 1 µ 1+ 1 =2 b c 양변에 bc 를곱하고정리하면 (b +1)(c +1)=2bc bc b c 1=0 (b 1)(c 1) = 2 따라서,2의약수를생각하면 1 b 1 c 1 임에서 (b 1;c 1) = (1; 2) 뿐이고, 즉 (a; b; c) = (2; 2; 3) 이다. (ii) a =1일때 : 준식은다음과같이바뀐다. µ 1+ 1 µ 1+ 1 = 3 b c 2 양변에 2bc 를곱하고정리하면 2(b +1)(c +1)=3bc bc 2b 2c 2=0 (b 2)(c 2) = 6 따라서,6의약수를생각하면 0 b 1 c 1 임에서 (b 2;c 2) = (1; 6), (2,3) 뿐이고, 즉 (a; b; c) =(1; 3; 8) 또는 (1; 4; 5) 이다. (i), (ii) 에서구하는해는 답 fa; b; cg = f1; 3; 8g, f1; 4; 5g, f2; 2; 3g 주위의 (i), (ii) 에서도인수분해법대신에부등식법을계속쓸수도있다. 예를들어 (i) 에서 b 3 이면 (1 + 1 b )(1 + 1 c ) 16 < 2 9 이므로해가없고, 따라서 b =2, 이것을대입하면 c =3이된다. (ii) 에서도 b 5 이면 (1 + 1 b )(1 + 1 c ) 36 25 < 3 2 이므로해가없고 b 2 일때도 (1 + 1 b )(1 + 1 c ) > 3 2 로해가없어서 b =3또는 4이고, 이것을대입하면각각 c =8,5가된다.
1.2 수론고급문제 43 40. 어떤 2000 자리의자연수의모든자리의숫자의합이 3 이다. 또한이자연수는두완전제곱수의합이다. 이러한자연수를모두구하여라. ( 셈본중등중급도전문제 2.2.2) 풀이 2000자리자연수의모든자릿수숫자의합이 3이므로이수는 3의배수이며,9의배수가아니다. 이때이수 (A라고하자 ) 가다른홀수b, c의제곱의합이된다고하면 A = b 2 + c 2 인데 b, c가각각3으로나눈나머지가 1, 2 중하나라고할때A의3으로나눈나머지는 1or2가되어A가3의배수라는데모순이다. 따라서 b, c 모두 3의배수여야하는데그럴경우 A가 9의제곱이되어서모순이다. 따라서조건을만족하는 A는존재하지않는다. 41. m =3; 4; 5 혹은 6 일때, 연속한 m 개의완전제곱수의합은완전제곱수가될수없음을보여라. ( 아일랜드 1991-6a) 증명연속된세수는0; 1; 2( mod 3) 에하나씩대응되므로연속된세완전제곱수의합은항상 0 2 + 1 2 +2 2 2( mod 3) 이다. 그런데임의의완전제곱수는 0; 1( mod 3) 의값밖에가질수없으므로, 연속한 3개의완전제곱수의합은완전제곱수가될수없다. 한편,( mod4) 에대해연속된완전제곱수들을살펴보면 1; 0; 1; 0; ( mod 4) 의반복된수열이된다. 이때연속된네완전제곱수의합은 2( mod 4) 이고, 연속된다섯완전제곱수의합은 2 또는 3( mod 4) 이고, 연속된여섯완전제곱수의합은 3( mod 4) 이된다. 그런데임의의완전제곱수는 0; 1( mod 4) 의값밖에가질수없으므로 m =4; 5; 6일때연속한 m개의완전제곱수의합은완전제곱수가될수없다. 42. 네정수a 1, a 2, b 1, b 2 가 a 1 b 2 a 2 b 1 = 1 을만족하면 a 1 + a 2 가기약분수임을증명하여라. b 1 + b 2 ( 통신강좌 1995-11-1) 증명 (a 1 + a 2 )b 2 (b 1 + b 2 )a 2 = 1 ) (a 1 + a 2 ;b 1 + b 2 ) j 1 ) (a 1 + a 2 ;b 1 + b 2 )=1 즉, a 1 + a 2 와 b 1 + b 2 는서로소이고, a 1 + a 2 는기약분수이다. b 1 + b 2 43. 1 p + 1 q + 1 r = 1 을만족하는소수 p, q, r과자연수 n을모두구하여라. ( 아벨콘테스트 1994 결선 2a) n 풀이 p; q; r 이모두다르다고하자. 이때통분하여정리하면, pqr = n(pq + qr + rp) 가된다.(p; pq + qr + rp) =1 이므로 pjn 이고, 같은방법으로 qjn; rjn 이다. 즉, n = pqr 이어야하고, pq + qr + rp =1 인데 p; q; r 2 이므로모순이다. 그러므로서로다른해는없다. p = q 6= r 이라해보자. 통분하여정리하면, pr = n(2r + p) 이다. 그런데 p 가 2r + p 를나누지않으므로 pjn 이고, 같은방법으로 rjn 이다. 이때 n = pr; 2r + p =1 이어야하는데 p; r 2 이므로모순이다. 즉, p = q = r 인해만존재할수있다. p = q = r 이라고하면,3n = p 에서 p 가소수이므로 n =1;p =3 이다. 결론적으로 n =1;p = q = r = 3 의해가유일하다. 44. 자연수 m, n 이서로소이다. m + n 과 m 2 + n 2 의최대공약수는 1 이거나 2 임을증명하여라. ( 소련 1963-4) 증명 m + n 과 m 2 + n 2 이소수 p>2 를공약수로갖는다고하자 ( 앞으로사용할 는 mod p 에대한것이라고약속하자 ). 그러면, m + n 0 이다. 즉, m n 이고, 양변을제곱하면 m 2 n 2 이다. 한편, m 2 + n 2 0 이므로 2m 2 0 이고따라서 m 0, 즉 m 은 p 의배수이다. m n 이므로 n 도 p 의배수가되고이것은 m; n 이서로소라는것에모순이다. 즉, m + n 과 m 2 + n 2 의최대공약수는 1 또는 2 이다. 45. m = pq 이고 p 와 q 는소수이다.(a; m) =1 을만족하는모든 a 에대해 a m 1 1(modm) 이라고하자. p 6= q 임을보여라. ( 통신강좌 1995-10-3 변형 )
44 수론 증명 p = q, 즉 m = p 2 이라하자. 그러면 Á(p 2 )=p 2 p이고오일러의정리에의해 a p2 p 1( mod p 2 ) 이므로 a p2 1 a p2 p a p 1 a p 1 1( mod p 2 ) 이성립한다. 이제 (a; p) =1 이면서 a p 1 6 1( mod p 2 ) 인 a 가존재함을보이면충분하다. 어떤 b 가 (b; p) =1 이며 b p 1 1( mod p 2 ) 를만족시킨다고하자. 이때 a = b + p 라하면 (b + p) p 1 b p 1 +(p 1)pb p 2 ( mod p 2 ) 1 pb p 2 ( mod p 2 ) 이다.(b; p) =1 이므로 1 pb p 2 6 1( mod p 2 ) 이고따라서 (a; p) =1 이며 a p 1 6 1( mod p 2 ) 인 a 가존재한다. 즉, p 6= q 이다. 46. 상수다항식이아닌정수계수다항식 P (x) 가있다. 모든정수 n 에대하여 P (n) 의값이항상소수를취하는것은불가능함을증명하여라. ( 통신강좌 1995-11-3) 증명문제의조건을만족시키는다항식 P (x) 가존재한다고가정하자. 조건에의해서 P (0) 는소수이어야하므로그값을 P 0 라하자. 이때 P (kp 0 ) P (0) 0 (mod P 0 ) 이므로, P (kp 0 ) 도 P 0 의배수이다. 그런데 P (x) 는항상소수이므로, P (kp 0 ) 도소수다. 즉 P 0 가소수이므로 P (kp 0 )=P 0 이다.(k =1; 2; ) F (x) =P (x) P 0 라하자. x 가 P 0,2P 0,3P 0, 일때 F (x) 는 0 이고 P 0 < 2P 0 < 3P 0 < 이므로,identitytheorem 에서 F (x) =0 즉, P (x) =P 0 인데이는 P (x) 가상수다항식이아님에모순. ) 모든정수 n 에대해 P (n) 이항상소수를취하는것은불가능하다. 47. 완전수란자기자신을제외한양의약수의합이자기자신과같은수를말한다. 예를들어,6=1+2+3, 28=1+2+4+7+14 이므로 6 과 28 은완전수이다.2 p 1 이소수일때 2 p 1 (2 p 1) 은완전수이고, 짝수인완전수는이런꼴들뿐임을증명하여라. ( 플란더즈예선 1990/1991) 증명 x =2 p 1 (2 p 1), 2 p 1 은소수라고하자. 그러면 x 의약수의합은 ¾(x) = (1+2+ +2 p 1 )(1 + 2 p 1) = (2 p 1)2 p =2x 이된다. 즉, x 는완전수이다. x 가짝수인완전수라고하자. 그러면어떤홀수 m 과 k 2 에대해 x =2 k 1 m 꼴로나타낼수있다. 이때, ¾(x) =(2 k 1)¾(m) =2 k m 이되고,(2 k 1; 2 k )=1 이므로 (2 k 1)jm 이된다. m =(2 k 1)m 0 이라하자. 그러면, 위의식에서 m 의약수의합 ¾(m) =2 k m 0 =(2 k 1)m 0 + m 0 = m + m 0 이고, 따라서 m 의약수는 m 과 m 0 밖에없음을알수있다. 곧 m 0 =1 이고 m =2 k 1 은소수이다. 48. 두자리의자연수 n = ab 가있다. n = a 2 + b 3 을만족하는모든 n 을구하여라. (1999 교육청경시 )
1.2 수론고급문제 45 풀이 a 2 + b 3 =10a + b에서, a 2 10a +(b 3 b) =0이된다. 근의공식에의해 q a =5 5 2 (b 3 b) 가된다. 여기서근호안의내용이 0 이상이어야하므로 0 b 3 이된다. 이때 5 2 (b 3 b) 가제곱수가되려면 b =0; 1; 3 의경우밖에없다. b 가 0 또는 1 일때, a 는 10 또는 0 이어야하는데, a 는 0 이아닌한자리수의정수이므로불가능하다. 즉, b =3 이어야하고, 그러면 a 는 4 또는 6 이된다. 그러므로가능한 n 은 43 과 63 밖에없다. 49. k 는 133 5 +110 5 +84 5 +27 5 = k 5 을만족하는정수이다. k 는얼마인가? (AIME 1989-9) 풀이 The last digits of n and n^5 are the same. Hence last digit of lhs is same as that of 3 + 0 + 4 + 7 = 4. Hence last digit of k is 4. Also 133 = 1 mod 3, 110 = -1 mod 3, 84 = 0 mod 3, 27 = 0 mod 3, so lhs = 0 mod 3. Obviously k > 133. So smallest possibility is 144, next is 174. Now 11^5 = (10 + 1)^5 = 10^5 + 5 10^4 + 10 10^3 + 10 10^2 + 5 10 + 1 = 161051, so 110^5 < 2 10^10. Obviously, 27 and 84 < 100, so 27^5 and 84^5 < 10^10. Similarly, 133^5 < (1331/10)^5 = 11^15/10^5 < 5 10^10. Hence lhs < 10^11. But 170^2 = 28900 > 28000, 170^4 > 780000000 > 7 10^8 and 170^5 > 10^11. So only possibility is 144. 답 144 50. x 3 +5y 3 =9z 3 을만족하는정수해 (x; y; z) 를모두구하여라. ( 아벨콘테스트 1994 결선 2b) 풀이만일해 x; y; z가 d>1를공약수로갖는다면 x=d; y=d; z=d또한해가되기때문에,(x; y; z) =1인해가존재함을알수있다. 자명하지않은해 x; y; z가존재하여서로소라고가정하자. 정수는 3k; 3k +1; 3k 1꼴의세가지로나눌수있는데이를세제곱하면각각 27k 3 ; 27k 3 +27k 2 +9k + 1; 27k 3 27k 2 +9k 1가되므로모든정수의세제곱은 mod 9에대해0; 1; 1의값만을가질수있다. 따라서 x 3 +5y 3 0( mod 9) 이려면 x; y가모두3의배수일수밖에없다. 이때 x 3 +5y 3 이 27의배수이므로 9z 3 도 27의배수이고, 따라서 3jz 3 이다. 결국 3jz이고 x; y; z가서로소라는가정에위배된다. 즉, x 3 +5y 3 =9z 3 의정수해는 x = y = z =0의자명한해밖에없다. 51. x 3 + y 3 + z 3 =2002 를만족하는정수들의순서쌍 (x; y; z) 를모두구하여라. ( 통신강좌 1995-11-20) 풀이 ( 서울과학고 1 학년우지철 ) x 3 + y 3 + z 3 =2002 4(mod9) a 0 (mod 3) 일때a 3 0 (mod 9) a 1 (mod 3) 일때a 3 1 (mod 9) a 2 (mod 3) 일때a 3 1 (mod 9) ) x 3 + y 3 + z 3 0; 1; 2; 3; 1; 2; 3 이므로해는존재하지않는다. 52. 1 + 1 2 + 1 3 + + 1 은정수가아님을보여라.(n 2) n 증명 n 2이므로 2 k n 2 k+1 인자연수 k를찾을수있다. 이때 1; 2; ; 2 k 1; 2 k +1; ;n의최소공배수는 2 k 1 m(m은홀수 ) 꼴이된다. 1+ 1 2 + + 1 n = H 라하자. 양변에 2 k 1 m을곱하면 µ m 2 k 1 1+ 1 2 + + 1 = Hm 2 k 1 n 이다. 이를정리하면 m 2 k 1 1 2 k = m 2 를제외한좌변의모든항이정수가된다. m이홀수이므로이항은정수가될수없고, 따라서좌변은정수가아니다. 즉, H는정수가아니다.
46 수론 53. 순순환소수란소수점직후의 k개의숫자마디가계속반복되는소수이다. 예를들면다음과같은것이다. 0:243243243 = 9 37 혼순환소수란순환마디가있긴하지만순순환은아닌소수이다. 예를들면다음과같은것이다. 0:011363636 = 1 88 혼순환소수를기약분수 p q 꼴로나타냈을때, 분모 q는 2나 5, 혹은둘다로나누어떨어짐을증명하여라. ( 미국 1988-1) 풀이 혼순환소수 0:a 1 a 2 a k _ b 1 b 2 _ b l 를기약분수 p=q 로나타냈다고하자. 이때 x = b 1 b 2 b l a 1 a 2 a n 라고하면, p q 10l p q = x 10 k 가된다. 이를정리하면 p(10 l 1)10 k = qx 가된다. p, q 가서로소이므로 p j x 이고, 따라서정수 x 0 = x=p 에대해 (10 l 1)10 k = qx 0 라할수있다. q 가 2 나 5 로나누어떨어지지않는다고가정하자. 그러면 q 와 10 k 는서로소이다. 그러면 q j (10 l 1) 이고,10 k j x 0 이된다. 곧 x=10 k 가정수라는것이고주기가 a 1 a 2 a k b 1 b 2 b l k 인순순환소수가되어가정에위배된다. 그러므로 q 는 2 나 5 로나누어떨어진다. 54. 이차방정식 x 2 + ax 66a =0 의두근이모두정수일때, 두근을구하여라. 단, a 는소수이다. (1995 서울시 ) 풀이 x 2 ax 66a =0의근이모두정수이므로판별식 D = a 2 +264a가완전제곱수여야만한다. a가소수이므로 a(a +264) 가제곱수이려면 aj(a +264) 이고 aj264이다.264=2 3 3 11이므로 a =2; 3; 11이고각각에대해 a(a +264) 를계산하면 a =11일때만 a(a +264)=55 2 으로제곱수가된다. 따라서 a =11이고두근은 33과 22이다. 55. 모든양의실수는십진법으로전개했을때모든자리의수가 0 또는 7 뿐인 9 개의수의합으로나타낼수있음을증명하여라. (Towns 1981 S3) 증명먼저모든양의실수는십진법으로전개했을때모든자리의수가 0 또는 1뿐인 9개의수의합으로나타낼수있음을확인하자 ( ). 십진법으로전개한임의의양의실수를 a = a 3 a 2 a 1 a 0 :a 1 a 2 a 3 라하자. 이제 0 과 1 로만이루어진 9 개의수 b 1 ;b 2 ;:::;b 9 을정하는데 ( 지수가아니고첨자임 ), b k 는십진법전개로 a 와마찬가지로 b k = b k 3b k 2b k 1b k 0:b k 1b k 2b k 3 인것으로보자 (k =1; 2;:::;9). 이때, b k i 들을정하는방법은다음과같다 : b1 i ;b2 i ;:::;b9 i a i 개를 1로한다 ( 좀더구체적으로한다면, b 1 1 부터 b1 a i 까지를 1로한다 ). 그럼 중에서정확히 a i = b 1 i + b2 i + b9 i 가되므로, a = b 1 + + b 9 도성립함을알수있다.( 예를들어, a = 320:1 이라면 b 1 = 110:1, b 2 = 110:0, b 3 =100:0, b 4 = = b 9 =000:0 과같이하는것이다.) 따라서,( ) 이확인되었다. 이제 x를임의의양의실수라하면 x=7도양의실수이므로,( ) 에의해 x 7 = b1 + + b 9 를만족하는각자리가 0 과 1 로만이루어진 b 1 ;:::;b 9 가존재한다. 이제양변에 7 을곱하면 x 를각자리가 0 과 7 로만이루어진 9 개의수 7b 1 ;:::;7b 9 의합으로표현한것이된다.
1.2 수론고급문제 47 56. 9 4000 은 3817 자리의수이고최고자리의숫자가 9 라는것이알려져있다.0 n 4000 에대해모든 9 n 꼴의수들중에최고자리의숫자가 9 인것은모두몇개인가? (AIME 1990-13) 풀이 If 9^n starts with 9, then 9^{n-1 must start with 1 and have the same number of digits. If 9^n starts with any other digit, then it has one more digit than 9^{n-1. Now 9^4000 has 3816 more digits than 9^1. So in 3816 cases from 2, 3,..., 4000, the first digit of 9^n is not 9 and in the other 183 cases it is 9. Add in the case 9^1 (but not 9^4000) and we get 184. 답 184 57. 2 진법으로쓴자연수에서 1 을모두 0 으로,0 은모두 1 로바꿔쓰는조작을반전이라고하자. 또,2 진법으로쓴자연수를뒤에서부터거꾸로읽는조작을뒤집기라고하자. 예를들어,2 진수 11010010 을반전시킨것과뒤집은것은각각 101101 과 1001011 이다. 우리에게허락된조작이이두가지뿐일때, (1) 387은최소몇번의조작으로0으로만들수있는가? 그리고, (2) 5번이상의조작을해야만 0으로만들수있는자연수중에서가장작은것은무엇인가? (IT꿈나무올림피아드 2006 1차 ) 풀이 0이나 1이연속하여나온부분을 `덩어리 ' 라고부르기로하자. 반전은덩어리의수를 1개감소시키고, 뒤집기는덩어리의수를 1개감소시키거나혹은똑같이유지시킨다. 따라서, 한번의조작으로최대 1개의덩어리를없앨수있으므로, 덩어리가 n개인수는최소 n번의조작이있어야 0으로만들수있다.387은 2진법으로쓰면 110000011이므로덩어리가 3개이다. 그리고, 덩어리가 5개인자연수중에서가장작은것은 10101, 즉 21이다. 답 (1) 3번 (2) 21 58. 자릿수의합이 7 의배수가되는수를행운의수라고부르기로하자.25 바로다음에나타나는행운의수는 34 이고, 이럴때두행운의수 25 와 34 가이웃해있다고말하기로하자.25 와 34 의차는 9 이다. 이렇게, 이웃한두행운의수의차가될수있는수를모두구하여라. (KAIST Cyber 영재교육 2005 가을 ) 풀이우선 13 이하의모든자연수는다음과같이모두이웃한두행운의수의차가될수있다 :1= 70000 69999, 2 = 61 59, 3 = 500002 499999, 4 = 403 399, 5 = 7000000 6999995, 6 = 2005 1999, 7 = 59 52, 8 = 10006 9998, 9 = 16 7, 10 = 100006 99996, 11 = 106 95, 12 = 1000006 999994, 13 = 1006 993. 이제 14 이상의자연수는이웃한두행운의수의차가될수없음을보이자. 즉, M 을임의의행운의수라하면 M +14 이전에다른행운의수가존재함을보이면된다 ( ). 십의자리이상과일의자리로구분하여 M =10a + b 라하자. b 2 일때는 M +7 또한행운의수이다. b 3 일때에는 10(a +1); 10(a +1)+1;:::;10(a +1)+6 중에행운의수가반드시있으므로그수와 M 과의차이는기껏해야 16 b 13 이하이다. 59. 자연수 n 의서로다른양의약수중홀수인것들의개수를 N(n) 으로쓰기로하자. 예를들어,24 의양의약수중홀수는 1, 3 뿐이므로 N(24) = 2 이다. 합 N(1) + N(2) + + N(1989) 가홀수인지짝수인지알아내어라. ( 호주 1989-1 변형 ) 풀이 N(n) = 홀수, N =2 m ( = 홀수부 ) 일때 = 제곱수. =1; 3 2 ; 5 2 ; 7 2 ; 9 2 ;::: 에대해좀막노동을뛰면... 60. 1 + p + p 2 + p 3 + p 4 이완전제곱수가되는소수 p 를모두구하여라. ( 호주 1995-4) 풀이부등식법 : p 5 이면 (p 2 + p 1 2 )2 < 준식 < (p 2 + p+1 2 )2 임을이용. 61. n 개의자릿수 a 1 ;a 2 ;:::;a n 이임의로주어져있다. 제곱근의소숫점이하십진전개가이 n 개의숫자로 ( 주어진순서대로 ) 시작하는자연수가항상존재하는가? ( 독일 BW 1973 1 차 -3) 풀이항상존재함을보이겠다.0:a 1 a 2 a n = h 라할때 k + h< p m<k+ h + 1 10 n 을만족하는자연수 m, k 가존재함을보이면된다. 각변을제곱하면 (k + h) 2 <m<(k + h) 2 +2(k + h) 1 10 n + 1 10 2n 1 2(k + h) 10 n 이 1보다커지도록 k를충분히크게잡자 (k =10n 쯤으로잡으면충분 ). 그럼부등식의좌변과우변의차이가 1보다커지므로그사이에정수 m이존재함은분명하다. 굳이구체적으로써준자면 m = b(10 n + h) 2 +1c 로잡으면된다.
48 수론 62. 2 1917 +1 에서 2 1991 1 까지의모든정수의곱이제곱수가아님을증명하여라. (Towns 1992 봄 SA1) 증명 n! 의소인수분해에서소수 p의지수는f(n!;p)=b n p c + b n p 2 c + b n p 3 c + 로구할수있음이잘알려져있다. 특히 f(2 n!; 2) = 2 n 1 +2 n 2 + =2 n 1 의간단한형태로얻을수있다. 그럼문제에서제시한수 m =(2 1917 +1) (2 1991 1) 의 2의지수는 f(m; 2) = f(2 1991!; 2) f(2 1917!; 2) 1991 = (2 1991 1) (2 1917 1) 1991 로홀수가된다. 따라서, m 은제곱수가아니다. 주 n과 2n 사이에는항상소수가적어도하나있음이베르트랑의공준 (Bertrand's Postulate) 이라하여잘알려져있다. 만일이것을사용할수있다면, m에곱해진수들중가장큰소수의지수가 1임을금방알수있어서문제가너무쉬워진다. 이것이 `어려운정리 ' 를함부로쓰지말라는경고를한이유이다. 베르트랑의공준은페르마의작은정리를아주교묘하게적용하여고등경시수준에서도증명을할수있지만, 그아이디어는결코쉽다고할수없다. 따라서, 비교적쉬운수준에서다른방법으로풀어낼수있는시험문제를굳이베르트랑의공준으로풀어낸다면, 별다른경고문구가없더라도충분히상당한감점이적용되거나심지어점수를거의받지못할수있으니주의하여야한다. 그것은마치빈대한마리잡으려다초가삼간불태우는격이다. 삼각형의넓이를구하기위해미적분까지사용한다거나, 아폴로니우스의중선정리를증명하라는데더어려운스튜어트의정리를사용한다거나, 체바의정리를증명하라는데동급의메넬라우스의정리를사용한다거나하는것은엉뚱한일이아니겠는가? 63. n 이 1 보다큰자연수이고 1+2 n +4 n 이소수이면, n 은 3 의배수임을보여라. ( 불가리아 1981 3 차 -4 변형 ) 증명 mod 7 로하면됨. 64. 2의거듭제곱수중에자릿수를재배열하여또다른 2의거듭제곱수를만들수있는것이있는가? 단, 재배열할때맨앞자리에 0이나올수없다. (Towns 1988가을 SO1) 풀이맨앞에 0이나올수없다는조건에의해하나가다른것의 10배이상이될수없으므로, 두수는 (k; 2k), (k; 4k), 아니면 (k; 8k) 가된다. 그런데, 자릿수의재배열로는 9로나눈나머지가변하지않는데, k는 2k, 4k, 8k 어느것과도 9로나눈나머지가같을수없으므로모순 (k는 2의거듭제곱수이므로 9와서로소, 따라서차인 k, 3k, 7k는어느것도9의배수가될수없다 ). 따라서, 불가능. 65. 다음을증명하여라. (a) n =4k +1꼴이면합과곱이같은 n개의양의홀수들이존재한다. (b) n이이런꼴이아니면그런홀수들도존재하지않는다. (Towns 1990 봄 SO2) 증명 (a) k = 0 일때는자명. k 1 일때는 2k +1; 3; 1;:::;1 (1 이 4k 1 개 ) 로하면곱은 3(2k +1)=6k +3, 합도 (2k +1)+3+(4k 1) = 6k +3 이된다. 따라서, 모든음아닌정수 k 에대해합과곱이같은 4k +1 개의양의홀수들이존재한다. (b) 합과곱이같은 n 개의홀수들중 4t 1 꼴의수가 b 개라하자. 그럼 mod 4 로곱 ( 1) b 이고합 b ( 1) + (n b) 1 이므로 n 2b +( 1) b (mod 4) 이다. 이식은 b 가짝수일때나홀수일때나둘다 n 1(mod4) 가됨을확인할수있다. 66. 2 1989 과 5 1989 을 10 진법으로이어썼다. 그럼전체는몇자리수가되는가? (Towns 1989 가을 SO4) 풀이두수를각각a자리수, b자리수라하자. 그럼 10 a 1 < 2 1989 < 10 a 이고 10 b 1 < 5 1989 < 10 b (10의거듭제곱수가아니므로등호는성립불가 ). 따라서, 변변곱하면 10 a+b 2 < 10 1989 < 10 a+b 이고 a + b 2 < 1989 <a+ b. a + b는정수이므로 1989 = a + b 1 만가능하다. 그럼 a + b = 1990 답 67. N개의정수가주어져있다. 이중임의의 N 1개의곱에서나머지하나를뺀것이항상N의배수이면, N개전체의제곱의합도 N의배수임을증명하여라. (Towns 1991봄 JO1)
1.2 수론고급문제 49 증명 N의배수인항이있으면다른항이나머지하나가될때그다른항도 N의배수. 즉, 모두 N의배수이므로 OK. N의배수인항이없으면 0도일단없고, 그럼전체곱을 P 라하면P a 2 i 이늘N의배수이므로 a 2 i P (mod N) 으로일정. 따라서, 다더하면N의배수. 68. 다음의방정식을만족하는모든자연수 n 과정수 x, y (x 6= y) 를찾아라. x + x 2 + x 4 + + x 2n = y + y 2 + y 4 + + y 2n (Towns 1991 봄 SO1) 풀이넘기고 x y로나눠주면 1+(x + y)+(x + y)(x 2 + y 2 )+ +(x + y)(x 2 + y 2 ) (x 2n 1 + y 2n 1 ) = 0. 여기서 x + y > 0 일수없고x + y j 1 이므로 x + y = 1. 대입하고 1 곱하면 (x 2 + y 2 )+(x 2 + y 2 )(x 4 + y 4 )+ =0. n 2 이면이식에항이남아있고x6= y 라서 x 2 + y 2 등이모두양수이므로좌변 > 0 으로모순. 즉, n =1이어야하고, 그때x + y = 1 인모든정수쌍에대해성립. 69. 자릿수가오름차순으로나타나고 ( 꼭모두다를필요는없고 )5 로끝나는수들을찾고있다. 이수들의제곱도역시같은성질을가져야한다. (a) 이런수를 4개만찾아라. (b) 이런수가무한히많음을증명하여라. (Towns 1991 봄 JA3) 풀이법. 35 2 =1225를확장하면됨. 335 2 = 90000 + 21000 + 1225 = 112225 등의꼴을관찰하여귀납 70. 4 545 +545 4 가합성수임을보여라. ( 러시아 1989 4 차 -y8-5) 증명 m 4 +4n 4 은인수분해되므로합성수. 71. 2005 2005 는두완전제곱수의합이지만두완전세제곱수의합은아님을보여라. ( 아일랜드 2005-1) 증명완전제곱수의합임은 Lagrange 항등식으로부터. 세제곱수의합이아님은 mod 7 로. 72. m, k, n은자연수이고 n>1 이다. 이때m(m +1)=k n 이성립할수없음을보여라. ( 소련 1964-2) 증명 m과 m +1은서로소이므로각각이 n제곱수. 같은 n제곱수끼리차이가 1인경우는자연수범위에서는없음. 73. 방정식 x 4 +131=3y 4 을만족하는정수해 (x; y) 가존재하지않음을보여라. ( 호주 1984-1) 증명 mod 5 로해없음. 74. 각각의자연수 n 마다음이아닌정수 f(n) 을배정하여다음조건이만족되도록하였다. (i) 임의의자연수 m, n에대하여f(mn) =f(m) +f(n). (ii) n의마지막자릿수가 3이면 f(n) =0. (iii) f(10) = 0. f 를구하여라. ( 호주 1984-6 변형 ) 풀이 (iii) 에서 f(2) = f(5) = 0, 즉소인수2와5는무시됨. 즉,10과서로소인수들만생각하면됨. 그럼일의자리가1, 3, 7, 9 중의하나이고, 이런수는적당한홀수 (3,1,9,7) 를곱해일의자리를3으로만들수있음. 그럼 (ii) 에서 0. 75. 자연수로이루어진세무한등차수열이있다.1,2,3,4,5,6,7,8 각각에대해그수가나타나는수열이적어도하나씩은있을때, 1980 이나타나는수열이있음을보여라. ( 오스트리아 - 폴란드 1980-1)
50 수론 증명 2, 4, 6, 8 을포함하는집합을대상으로경우를조금만나누면 ; 76. 세제곱의합이 1991 이되는두정수가존재하는가? ( 러시아 1991 4 차 -y9-5) 풀이 mod 7 77. 2 x+1 + y 2 = z 2 을만족하는소수 (x; y; z) 를모두구하여라. ( 러시아 1992 4차-y9-6) 풀이1 2 x+1 =(z + y)(z y). y; z 5 일때는소수가6k 1꼴뿐이므로 6 j (y + z)(y z). 모순. 나머지만검토하면 z =5,y =3,x =3. 풀이2 z + y =2 a, z y =2 b (a>b) 라하면2z =2 b (2 a b +1), 즉 z =2 b 1 (2 a b +1). 소수이므로 b =1,z =5,y =3. 풀이 3 mod 3 으로그만. 78. x 3 + y 5 = z 7 을만족하는세자연수의순서쌍 (x; y; z) 가무한히많음을증명하여라. (IMTS R13-4) 증명 (2 30 m 35 ; 2 18 m 21 ; 2 13 m 15 ) 79. 칠판에 2 와 3 이라는수가적혀있다. 그리고이칠판에새로운자연수를보충해서쓰는데, 단이미 a 와 b 의수 (a = b 라도상관없음 ) 가적혀있을때 ab a b 의수를쓰는것을규칙으로한다. 이칠판에쓰여질수있는수를모두찾아라. (1999 KAIST 대전. 충남영재수학교실 2 차평가변형 ) 풀이 b =2넣으면 a! a 2 를쓸수있으므로,2와 3으로부터시작하여 3 이하의모든정수를쓸수있고 b =0넣으면 a에의해 a를항상쓸수있으므로모든자연수를쓸수있다. 주만일문제의의미가칠판에오직자연수만써야하고 0이나음수는쓸수없는거라면1, 2, 3만적은후더이상쓸수없다는것이답이된다. 80. c 가어떤값이든, 방정식 x(x 2 1)(x 2 10) = c 가다섯개의정수해를가질수없음을보여라. ( 러시아 1989 4 차 -y10-5) 증명 y = x(x 2 1)(x 2 10) 의그래프와 y = c 의그래프의교점을관찰하자고따지면, c>0 이면 (0; 1) 에, c>0 이면 ( 1; 0) 에근을가지게됨. 별해 ( 김대현 ) 근과계수와의관계에서다섯해의합은 0, 둘씩곱한합은 11 이므로, 제곱의합은 22. c =0 일땐안되고, c 6= 0 이면일단 0, 1 은해가아니므로모든해는절대값이 2 이상. 즉제곱이 4 이상. 모두 2 2 일때도안되고, 하나라도 3 2 이상이있어도안되니까끝. 81. 방정식 x 3 + y 3 =4(x 2 y + xy 2 +1) 이정수해를갖지않음을보여라. ( 러시아 1993 4 차 -y9-5) 증명 (x + y) 3 4(mod7) 이됨. 82. 음이아닌정수들 e 1 >e 2 > >e r 에대해 n =2 e 1 +2 e 2 + +2 e r 이라두자. n! =n(n 1) 2 1 이라할때, n!=2 n r 은홀수임을보여라. ( 호주 1988-3) 증명그냥 n! 의소인수분해 83. 자연수 n 을십진법으로썼을때나타나는 0 이아닌모든자릿수들의곱을 P (n) 이라하자. P (n) 이 n 을나눌때, n 을 ` 굉자리한 (prodigitious)' 수라한다. 14 개의연속한굉자리한자연수들은없음을증명하여라. (IMTS R16-2) 증명일의자릿수 k 가두번나오면 k j 10 이거나 k =0. 이런 k 들중에연속된 4 개의숫자는없어서. (0 1 2 5 이므로 012 의 3 개연속이최대라서아마 13 개가한계 )
1.2 수론고급문제 51 84. 십진법으로모든자릿수가 1인자연수n에대해서 p(n) 이다시모든자릿수가 1인자연수가되는이차정계수다항식 p가존재하는가? ( 러시아 1994 4차-y9-3) 풀이 1- 반복수다항식.9x 2 2x +1 85. a는임의의양의정수이다. x + 1 (x + y 1)(x + y 2) = a 2 를만족하는양의정수 x, y가유일하게존재함을증명하여라. ( 헝가리 1936-3) 증명유사진법. x + y 1=n( 1) 으로두면 1부터 n 1까지의합을 f(n) 이라할때f(n) <a f(n +1) 을만족하는 n이유일하게존재할테고 x = a f(n) n 은그편차. 86. (a 1 ;a 2 ;:::;a n ) 과 (b 1 ;b 2 ;:::;b n ) 는각각 1; 2;:::;n 을재배열한것이다. n 이짝수이면, a i + b i 들 (i = 1; 2;:::;n) 중에 n 으로나눈나머지가같은것이있음을보여라. ( 몰도바 1997 최종 -y8-5) 증명나머지가모두다르다면, 모두합했을때 2(1+2+ + n) 0+1+ +(n 1) (mod n), 즉 n(n 1) n(n +1) 0(modn). 2 그럼 n(n 1) = nk, 2 즉 n =2k +1이므로모순. 87. n! 의자릿수의합이 9 인자연수 n 을모두구하여라. ( 유고슬라비아 1979 고 1-4) 풀이 9의배수여야하므로n 6. 만일 11의배수라면, 홀수번째자릿수의합을 A, 짝수번째자릿수의합을B라할때A B (mod 11). A = B 라면 A + B가짝수이므로 9일수없고, A 6= B 이면큰쪽이11보다크므로 A + B도 11보다커서불가능. 즉, n<11 일때만보면되고, 실제계산해보면 n =6; 7; 8 만답이됨. 88. 다음방정식의정수해를모두구하여라. ( 폴란드 1966 3 차 -2) x 4 +4y 4 =2(z 4 +4u 4 ) 풀이 x; z; y; u 가차례로짝수가되어무한내림으로모두 0 뿐. 89. x y = y x y 을만족하는자연수순서쌍 (x; y) 를모두구하여라. ( 주니어발칸 1998-3) 풀이 y =1이면 x =1. x = y 일때도x =1. 따라서, x>y 2 일때만보자. y = m n x 대입하고 (m <n은서로소 ) 양변을 n x 제곱하면 xm =( m n x)n m. 즉 x =( m n ) n m 2m n 이고 y =( m n ) m 2m n. ( 단,2m n =0일때는 (m; n) =(1; 2) 이고그럼 1= 1 2 로성립하지않음 )(i)2m n>0 이면 y쪽의지수가 0보다크므로정수가되려면 n =1. 그럼 m<n임에서모순. (ii) 2m n<0 이면 y쪽의분자와분모를바꿔생각할때 m =1. x = n n 1 n 2, y = n n 2 1 이고 n 3. n = y n 2 2 n 2 에서 n 5 이면이부등식은성립하지않으므로 n =3; 4 일때만확인하면되고, 각각 (x; y) =(9; 3), (8; 2). 답 (x; y) =(1; 1), (8; 2), (9; 3). 90. 음이아닌정수 n 에대하여 A n =2 3n +3 6n+2 +5 6n+2 으로정의하자. A 0 ;A 1 ;:::;A 1999 의최대공약수를구하여라. ( 주니어발칸 1999-2) 풀이 A 0 =1+9+25=35 이므로 5 와 7 에대해서만살피면됨.5- A 1 이므로 5 는아님.2 3 ; 3 6 ; 5 6 1 (mod 7) 이므로 A n 1+9+25 0(mod7) 으로 7 은항상됨. 답 7 91. n 2 +3 n 이어떤정수의제곱이되는자연수 n 을모두구하여라. ( 주니어발칸 2000-2) 풀이 3 n =(m + n)(m n) 에서 m + n =3 a, m n =3 b 꼴일수밖에없고 (a + b = n, a>b), 2p n 2p +1일때2n =3 a 3 b 3 p+1 3 p =2 3 p. 즉,2p +1 3 p 인데, 이것은 p>1 일때 3 p =(1+2) p > 1+2p 이므로모순. p =0; 1 일때만확인하면 n =1; 3 만답이됨. 92. x, y 가자연수이고 3x +4y, 4x +3y 가모두완전제곱수이면 x 와 y 는모두 7 의배수임을증명하여라. ( 주니어발칸 2004-3)
52 수론 증명 m 2 +n 2 0(mod7) 인데, 제곱수는 mod 7로 0, 1, 2, 4뿐이므로만족하는경우는 m, n 모두 7의배수일때뿐. 그럼 7 2 j m 2 + n 2 =7(x +y) 이므로 7 j x + y 이고, 한편 7 j 2(4x +3y) 7(x+y) =x y. 따라서,7j2x; 2y, 즉 7 j x; y. 93. m 과 n 이서로다른자연수일때,2 2m +1 과 2 2n +1 은서로소임을증명하여라. ( 헝가리 1940-2) 증명 m > n 이라하고공통소인수 p 가존재한다고하자. p j 2 2n +1 이므로 p j 2 2n +1 1 = (2 2n +1)(2 2n 1), 마찬가지로 p j 2 2n +2 1=(2 2n +1 1)(2 2n +1 +1), 이렇게계속하면 p j 2 2m 1 이되고, p j 2 2m +1 과빼면 p j 2, 즉 p =2. 이것은모순. 94. 서로다른두자연수 x, y 의산술평균은두자리의수이다. 이산술평균의십의자릿수와일의자릿수의자리를서로바꾸면 x, y 의기하평균이된다. (a) x, y를구하여라. (b) 10진법에서해는유일함을보이고,12진법에서는해가없음을보여라. (c) 해가있는진법과해가없는진법의예를하나씩더구해보아라. ( 독일BW 1972 2차-3) 풀이 1 (a) 1 a 9, 1 b 9, a와 b는자연수, x > y 라가정한다. (x + y) = 10a + b, 2 p xy = 10b + a. 산술평균은항상기하평균이상이고 a 6= b 이므로 a > b. 산술평균에서 x = 2(10a + b) y. 이를기하평균에대입하여정리하면 y =(10a + b) 3 p 11(a + b)(a b). 똑같은방법으로 x =(10a + b) 3 p 11(a + b)(a b). x 6= y 이고 x>y이므로 x =(10a + b)+3 p 11(a + b)(a b)y =(10a + b) 3 p 11(a + b)(a b) 1 a; b 9 이므로 a b 의최대값은 8, a + b 의최대값은 17, 그리고 p 11(a + b)(a b) 는자연수여야하므로 a + b =11. 이런순서쌍 (a; b) 를구하여비교하면오직 a =6,b =5 만이식을만족. 계산하면 x =98,y = 32. (b) 10 진법에서는오직 (6,5) 만이 (a; b) 를만족하므로이미끝.12 진법의수 x, y, a, b 에서는 1 a; b 11, 1 2 (x + y) =12a + b, p xy =12b + a 이므로 (a) 에서와똑같은과정을거칠때 x =(12a + b)+ p 13 11(a + b)(a b)y =(12a + b) p 13 11(a + b)(a b) 1 a; b 11에서 (a + b)(a b) 는 13 11 을인수로가져야하는데 (a + b)(a b) =a 2 b 2 11 2 1 < 13 11 로불가능하므로위의식을만족시키는순서쌍 (a; b) 는존재하지않음. (c) 일반적인 n진법의수 a, b, x, y에서 1 a; b n 1, 1 2 (x + y) =na + b, p xy = nb + a 이므로 (a) 와똑같은과정을거칠때 x =(na + b)+ p (n +1)(n 1)(a + b)(a b)y =(na + b) p (n +1)(n 1)(a + b)(a b) (a + b)(a b) a 2 b 2 (n 1) 2 1 은무조건 (n +1)(n 1) 보다작으므로 (n +1)(n 1) 은 1 을제외한완전제곱수를인수로가져야함. 이에맞는조건의 n 값은 8 등이있음 ( 그리고,8 일때 a =4,b =3, x =56,y =14 의해가실제존재 ). 이에맞는않는조건의 n 값은 7 등이있음. 95. p가소수이면 7p +3 p 4 는완전제곱수가아님을증명하여라. ( 주니어발칸 2007-4) 증명 p 5 일때먼저mod p로 1임을보여 p =4k +1꼴임을확인하고, 그것을이용해 mod 4로 2가됨을확인. 96. 자연수 n에대하여, 다음을만족하는실수 x (1 x<n) 의개수를구하여라. x 3 bx 3 c =(x bxc) 3 ( 단, bxc는 x를넘지않는최대의정수이다.) ( 호주 1992-2) 풀이 x = n + h 라하고대입해서정리하면 3nh(n + h) 가정수일때.
1.2 수론고급문제 53 97. K와 L은자연수이다. 적당한자연수 M이존재하여, M보다큰자연수 n에대해서는 µ K + 1 n µ + L + 1 n 2 2 이정수가될수없음을보여라. ( 호주 1992-4) 증명 2 n 으로통분하고분자만생각. n이짝수일때는 mod 4로, n이홀수일땐인수분해했을때복잡한항이홀수... 98. 다음을만족하는모든정수해 (x; y) 를구하여라. ( 호주 1993-5) (x +2) 4 x 4 = y 3 풀이 y는짝수이고인수분해하면 (x +1)((x +1) 2 +1)=y1 3. 좌변두항이서로소임을이용. mod 8도도움이될수도. 99. 자연수 a, b 가주어졌을때, 방정식 x a+b + y = x a y b 을만족하는자연수 x, y 를모두찾아라. ( 오스트리아 1983-5) 풀이 y = kx a 대입하고정리. 다시 k = mx b 대입하고정리. 1 = m(x ab 1). m =1,x =2, a = b =1 일때만해있음. 100. 1; 12; 123; 1234;:::; 1234567890; 12345678901;::: 의수들중에적어도하나는 1981 로나누어떨어짐을보여라. ( 유고슬라비아 1980 고 1-2) 증명 1234567890(1 + 10 10 + +10 10k ) 중에 1981의배수가있음을보이면충분. 즉,1+10 10 + +10 10k 중에 1981의배수가있음을보이면충분.10 10,10 10 1 이모두1981과서로소임을확인했으면이건금방... 101. 다음방정식을만족하는음이아닌정수들의순서쌍 (a; b; c; d) 를모두구하여라. a 2 + b 2 + c 2 + d 2 = a 2 b 2 c 2 ( 오스트리아 1986-1) 풀이모두홀수면안되고, d만짝수여도 mod 8로안되니까 a, b, c 중에짝수가존재. 그럼우변은 4의배수. 좌변이 4의배수이려면모두다짝수. 그럼무한내림... 102. q 가정수일때, 이차방정식 x 2 +7x 14(q 2 +1)=0 은정수해를갖지않음을보여라. ( 인도지역예선 1995-4) 증명 mod 7, 근과계수 103. 다음식을만족하는자연수순서쌍 (n; m) 을모두찾아라. ( 오스트리아 - 폴란드 1982-1) gcd((n +1) m n; (n +1) m+3 n) > 1 풀이 p를최대공약수의한소인수라하자. n +1, n 모두 p의배수가아님. p j (n +1) 3 1 을얻을수있음. 그담엔 m을 mod 3으로분류해서풀면 m =3k 일때 (m; n) =(3k; 7l +1) 꼴만답이됨. 104. a 2 + b 2 = c 2 을만족하고, a, b, c의최대공약수가 1인세정수의쌍 (a; b; c) 가무한히많음이알려져있다. (rs) 2 +(st) 2 =(tr) 2 을만족하고, r, s, t의최대공약수가 1인세정수의쌍 (r; s; t) 도무한히많음을증명하여라. (IMTS R41-2) 증명 (r; s; t) =(bc; ca; ab) 의치환이면끝. 105. n 은자연수이다.2n +1 과 3n +1 모두완전제곱수일필요충분조건은 n +1 이연속된두제곱수의합인동시에, 한제곱수에다그바로다음제곱수의 2 배를더한수임을보여라. ( 호주 1994-6)
54 수론 증명 (1) 동치변형 : 2n +1 = (2m +1) 2 () n +1 = m 2 +(m +1) 2. (2) 동치변형 : 3n +1 = (3k 1) 2 =( 3k +1) 2 () n +1=(k 1) 2 +2k 2. 106. p 2 +7pq + q 2 이완전제곱수가되는소수 p, q 를모두구하여라. ( 인도지역예선 2001-2) 풀이 ( 류길현 ) p 2 +7pq + q 2 = k 2 (p + q) 2 +5pq = k 2 k 2 (p + q) 2 =5pq (k + p + q)(k p q) =5pq k + p + q>p;q;5임이자명하므로다음의경우들만생각하면된다. (1) k p q =1,k + p + q =5pq k = p + q +1 2p +2q +1=5pq (5p 2)(5q 2) = 9 (p; q) =(1; 1) Ã소수가아니라서안됨. (2) k p q =5,k + p + q = pq k = p + q +5 2p +2q +5=pq (p 2)(q 2) = 9 (p; q) =(3; 11); (5; 5); (11; 3) (3) k p q = p, k + p + q =5q k =2p + q 3p +2q =5q! p = q일때가능 (k p q = q일때는p, q를바꾸면되니까이것과결과가같다.) 답 :(p; q) =(3; 11); (11; 3); (r; r) (r은소수) 107. 합이소수이고, p 2 + qs 와 p 2 + qr 이서로다른완전제곱수인소수 p, q, r, s 를모두구하여라. ( 러시아 1994 4 차 -y9-7) 풀이합은 2보다큰데소수라했으므로홀수이고, 그럼 p, q, r, s 중에 2가홀수개있음.2가3개인경우에는, p 2 + qs 와 p 2 + qr 이서로다르므로 r 6= s. WLOG, 2가아닌것을s라하면, p 2 + qs =8이므로모순. 따라서,2가1개인경우만보면됨. p 6= 2라면 p 2 +2a꼴의완전제곱수가있는데 (a는 q, r, s 중하나이고 2가아님 ), 2a는홀제곱수의차이므로 4의배수, 즉 a가짝수이고이것은모순. 따라서, p =2일때만보면됨.4+qs = a 2,4+qr = b 2 이라하면 qs =(a 2)(a +2), qr =(b +2)(b 2). q, s는소수이므로 qs =1 qs 또는 q s꼴로만분해됨. 전자의경우에는 qs =5가되어모순이므로후자의경우임. qs도마찬가지. 따라서, jq sj =4=jq rj. r 6= s 라했으므로 WLOG: r<q<s는공차 4의등차수열. 그럼셋중하나는 3의배수이므로 r =3일수밖에없고그럼 q =7,s =11. 이경우조건모두만족함. 답 (p; q; r; s) =(2; 7; 3; 11) 또는 (2; 7; 11; 3) 108. 방정식 x 2 + x = y 4 + y 3 + y 2 + y 의정수해 x, y를모두구하여라. ( 소련 1967-6) 풀이 4배하고 1더하면좌변은 (2x +1) 2 이고우변은 (2y 2 + y) 2 과 (2y 2 + y +2) 2 의사이. 즉부등식법으로 (2x +1) 2 =(2y 2 + y +1) 2 이유일한가능성. 109. 모든자연수 x 에대하여, 자연수 x 를십진법으로나타내었을때각자리의숫자들을모두합한것을 Q(x) 라고하고모두곱한것을 P (x) 라고하자. 이때, 모든자연수 n 에대하여 Q(Q(x)) + P (Q(x)) + Q(P (x)) + P (P (x)) = n 를만족하는 x 는무한히많음을보여라. ( 오스트리아 1983-1) 증명 r n = 1 9 (10n 1) = 11 1(1이 n 1개 ) 로정의하고, x = 1 9 (10r n 1) 10 m =11 100 0 (1이 r n 개,0이임의의 m개 ) 로두자. 그럼 Q(x) =r n 이고 P (x) =0. Q(Q(x))+P (Q(x))+Q(P (x))+ P (P (x)) = (n 1)+1+0+0=n 이다.
1.2 수론고급문제 55 110. 다음은 9에서시작하여자릿수를두배씩불려나가며곱한수이다. 9 99 9999 (10 2n 1) 이계산결과의각자릿수의합을 (n에대한식으로) 구하여라. ( 미국 1992-1) 풀이점화식. 답 9 2 n 111. 다음과같은성질을갖는자연수 a, b, c, d 가존재하도록하는자연수순서쌍 (x; y; z) 중에서 z 가최소인것을찾아라. (i) x y = a b = c d 이고 x>a>c. (ii) z = ab = cd. (iii) x + y = a + b. ( 오스트리아-폴란드 1982-7) 풀이 x = an, a = cm (m; n 2). 그럼 d = bm. c는 1일수없으므로 c 2. (i) 에대입해서풀면 c m 1 2 m 1 m 이되어등호는c =2,m =2일때만성립. a =4. x =2 k (k 3). k =3일때체크해보면 b =12일때z가최소값48을가짐. k =4; 5 일때체크해보면 z값이더커짐. 112. 십진법에서 xyxy 꼴의수는거듭제곱수 ( 제곱수, 세제곱수, 네제곱수,...) 가될수없음을보여라. 한편, b 진법에서 xyxy 꼴의수가거듭제곱수가되는경우가있는가장작은 b>1 를구하여라. ( 아일랜드 1998-3 변형 ) 풀이 101 j xyxy 이므로 101 2 j xyxy < 10 4. 이것은모순. b진법에서는 b 2 +1이무승수 (square-free number) 이면똑같은모순이발생. b =2; 3; 4; ::: 을차례로확인해보면무승수가아닌것은 b =7이최초. 이때, 2626 (7) = 1000 = 10 3 으로성립. 113. 정확히 16 개의약수 1=d 1 <d 2 < <d 15 <d 16 = n 을갖고, d 6 =18,d 9 d 8 =17 인자연수 n 을모두구하여라. ( 아일랜드 1998-6) 풀이 18 j n. 18의약수가벌써1, 2, 3, 6, 9, 18로 6개이므로 6번째약수가 18이려면 n의 18 이하의약수는이것이전부. 그럼 4 - n 이므로 n =2 1 3 2+b p r q s 꼴이고, 약수의개수는 16 = 2(3+b)(1+r)(1+s) 꼴. 가능한것은 2 4 2 =2 8이전부.2 8은 n =2 1 3 7 인데 1, 2를제외하고는약수가모두 3의배수이므로 3 j d 9 d 8 =17에서곤란. 즉 2 4 2 뿐이고, n =2 3 3 p꼴. p 19. fd 7 ;d 8 ;d 9 ;d 10 g = f27; 54;p;2pg 이고여기서경우를잘따져보면 p =37. 답 2 3 3 37 114. 7 x 3 2 y =1 을만족하는자연수 x, y 를모두구하여라. ( 러시아 1990 4 차 -y11-5) 풀이 x =1 일때는 y =1. x 2 이면 y 3 이고그럼 mod 8 로 x 는짝수.(7 X +1)(7 X 1) = 3 2 y 에서 7 X +1=2 a+1,7 X 1=3 2 b+1 꼴.2 a 3 2 b =1. 홀수이므로 b =0. 그럼 a =2,X =1, y =4,x =2. h n i h n i h n i 115. 자연수 n에대해a n = + + + 이라고정의하자. n이소수일때, 또그때만a n =2+a n 1 1 2 n 임을증명하여라. ( 몰도바 1997 최종 -y10-8) 증명가우스항중에서새로다음정수가되는것들의개수만큼추가되므로 a n = a n 1 + d n. 116. 다음방정식을만족하는정수쌍 (x; y) 를모두구하여라. ( 유고슬라비아 1981 고 2-3) y 4 x(x +1)(x +2)(x +3)=1 풀이 x =0; 1; 2; 3 일때는y = 1. y 4 =(x 2 +3x +1) 2 이므로 x 4 이거나 x 1 일때는 (x +1) 4 과 (x +2) 4 사이에낀다는부등식법.
56 수론 117. a 는정수이다. 방정식 x 2 + axy + y 2 =1 이무한히많은서로다른정수해 (x; y) 를갖도록하는 a 를모두구하고그것을증명하여라. ( 아일랜드 1995-2) 증명 jaj 1 일때는각각특수해를구해주고, jaj 2 일때를풀자. a 대신 a를대입한식과원래의식의해가일대일대응 (y! y) 하므로 a = b>0 일때만보아도충분.(x; y) =(1;b) 의특수해로부터 (x; y)! (y; ay x) 의점화법으로새로운해를계속만들수있다. 118. 자연수 n 을십진법으로쓴각자릿수의합을 S(n) 으로나타내자. 모든 n 에대해다음을증명하여라. S(2n) 2S(n) 10S(2n) 또한, S(n) = 1996S(3n) 을만족하는자연수 n 이있음을증명하여라. ( 아일랜드 1996-2) 증명 m + n 을계산할때자리올림이한번도없으면 S(m + n) =S(m)+S(n). 각자릿수는 2배하면앞자리에많아야 1이올라가고, 앞자리는 2배가되어짝수가되었으니 1을받아도자리올림이발생하지않는다. 즉, 예를들어세자리의수 n =100a +10b + c 라하면200a, 20b, 2c를합할때자리올림이생기지않으니 S(2n) =S(200a +20b +2c) =S(200a)+S(20b)+S(2c) =S(2a)+S(2b)+S(2c). S(2a) =2a 또는 2a 9 이므로 S(2a) 2a 이다. 이로부터 S(2n) 2a +2b +2c =2S(n). 한편, a i (0 i k) 를 n의각자릿수라하고, a i 가 5 이상일때 i =1, 그렇지않으면 i =0라할때, 5S(2n) =5 P i S(2a i)=5 P i (2a i 9 i )= P i a i + P i 9(a i 5 i ) S(n) 이다. 119. 양의약수의개수의네제곱이자기자신이되는모든자연수를구하여라. ( 아일랜드 1999-9) 풀이소인수분해에서지수가모두 4의배수. 네제곱근한등식의양변을비교. 소인수가 7 이상이면 p e > 4e +1. 따라서, 소인수 3, 5의지수들을살펴경우를좀따져주면됨. 120. 피보나치수열은 F 0 = F 1 =1, 그리고 n 0 에대해F n+2 = F n+1 + F n 으로정의된다. 피보나치수열에서연속한 2000개의항의합은피보나치수가될수없음을증명하여라. ( 몰도바 2000 최종-y7-7) 증명 F n +F n+1 + +F n+1999 = F n+2 +F n+4 + +F n+1998 +F n+2000 = X 라하면 X +F n+1 = F n+2001 이됨을귀납적으로확인할수있고, 그럼 F n+2000 <X<F n+2001 이라서. 121. 다음을만족하는모든자연수해 (a; b; c; n) 을찾아라. ( 아일랜드 2001-1) 2 n = a!+b!+c! 풀이 WLOG a b c 라하면a! j 2 n 이므로 a 2. (i) a =1이면우변이짝수가되기위해 b =1 이고, 그럼 c! =2 n 2=2(2 n 1 1) 은 2 홀수이므로 c =2; 3. 해는 (1; 1; 2; 2), (1; 1; 3; 3). (ii) a =2 이면우변이 4의배수가되기위해 b =2; 3. b =2이면 2 n 2 =1+ c! c! 4 이고, 4 가홀수가될수없어서모순. b =3이면 2 n 3 =1+ c! c! 8 이고, 8 이홀수인경우는 c =4; 5뿐. 해는 (2; 3; 4; 5), (2; 3; 5; 7). 답 (1; 1; 2; 2), (1; 1; 3; 3), (2; 3; 4; 5), (2; 3; 5; 7) 122. 0 이아닌자릿수가딱둘뿐이고그중하나는 3 인완전제곱수를모두찾아라. ( 중미 2001-3) 풀이마지막 0은짝수개.10 2n 으로나누어주면, 일의자리가 3일수는없으므로,300 0x꼴. 문제조건에의해 x는 0은안되고,mod10에서 2, 3, 7, 8도안됨. 현재 x =1; 4; 5; 6; 9가남았는데,mod25에서 ( 혹은 mod 3에서 )5도안됨.mod9에서 9도안됨. x =6이면유명한명제에의해십의자리가홀수, 즉 36만가능하고이것은실제로됨. x =1; 4의경우만남았다.(i)x =1일때 :3 10 n =(k +1)(k 1), 차가 2인정수이므로둘다 5의배수일수없어서5 n 은한쪽에몰림. 또둘다짝수. 그럼한쪽의최소값 2 5 n 은반대쪽의최대값 3 2 n 1 보다훨씬 ( 차이가 2 초과 ) 크므로해가없음. (ii) x =4일때 : 역시 3 10 n =(k +2)(k 2) 임에서 2 5 n 은 3 2 n 1 보다훨씬큼. 답 36 123. a 1 = A, a n+1 = a n + d(a n) 으로주어지는수열 a 1 ;a 2 ;a 3 ;::: 이있다. 단, d(m) 은 m 의자신을제외한가장큰약수를나타낸다. 이수열에 2002 가나타나도록하는정수 A>1 은어떤값인가? ( 중미 2002-3)
1.2 수론고급문제 57 풀이 a n+1 =2002라하고a n=d(a n)=p 라하면2002 = (p +1)d(a n) 이므로 p +1j 2002. 2002의양의약수들중에서 1를뺐을때소수가되는것을찾으면 14와 182뿐, 즉 p =13또는 181. 그런데, 이때둘다11 j a n 이라서 p가 a n 의가장작은소인수임에모순. 답 2002 124. 홀수인소수 p 가적당한정수 x, y 에대해 x 5 y 5 꼴로나타내어질때, 적당한홀수 v 에대해다음이성립함을증명하여라. ( 아일랜드 2001-3) r 4p +1 5 = v2 +1 2 증명인수분해하여 (x y)(x 4 + :::) 꼴로보면 (x 4 + :::) q 부분이 1보다크다는걸금방확인한후그럼소수이려면 x y =1. x = y +1 4p+1 을대입하여정리하면 =2y 5 2 +2y +1= (2y+1)2 +1. 2 125. 적당한홀수 n 에대해 55 n + a 32 n 이 2001 의배수가되게할수있는최소의자연수 a 를찾아라. ( 아일랜드 2001-6) 풀이 2001 = 3 23 29 이고, a 1; 1; 1(mod3; 23; 29). 그럼나머지정리나정수분류로 a 436 (mod 2001). 126. 다음식의값이정수가되도록하는음아닌실수 x를모두구하여라. 3 q13 + p q x + 3 13 p x ( 아일랜드 2001-9) 풀이준식을 t라하면0 <t<2 3p 13( < 5) 임을금방확인할수있음. 세제곱하여 3p 169 x = f(t) 꼴로정리한후 t =1; 2; 3; 4 를각각대입하여풀면끝. 127. 다음을만족하는자연수해 (p; q; n) 을모두구하여라. 단, p 와 q 는소수이다. p(p +3)+q(q +3)=n(n +3) ( 아일랜드 2002-3) 풀이 mod 3 으로보면 p, q 둘중하나는 3 의배수, 즉 3. 그후인수분해법. 답 (2,3,4), (3,7,8) 128. 방정식 a 2 b 2 + b 2 c 2 +3b 2 c 2 a 2 =2005 는정수해를갖지않음을보여라. ( 중미 2005-2) 증명 b 가짝수일때, 홀수일때, 각각 mod 4 로생각하면끝. 별증양변에서 3 을빼면좌변이 b 2 1 로인수분해되어인수분해법으로풀수도있지만노동이됨. 129. n 은자연수이고, d 는 2n 2 의양의약수이다. n 2 + d 는제곱수가될수없음을증명하여라. ( 헝가리 1953-2) 증명 dk =2n 2 이라두면, A 2 = n 2 + d = n 2 k+2. k 여기서 k와 k +2는최대공약수가 1 또는 2이다. 최대공약수가 1이면 k +2와 k 둘다제곱수라야하고이것은불가능. 최대공약수가 2이면 k+2 2 와 k 2 둘다제곱수라야하고이것도불가능. 130. y 2 = x 3 +16 의정수해를모두찾아라. ( 이탈리아 1994-2) 풀이 (y +4)(y 4) = x 3. y가홀수일땐 gcd(y +4;y 4) = 1 이므로 y +4,y 4 둘다세제곱수이고, 차이가 8인두세제곱수는0과 8뿐이므로 y = 4, x =0. y가짝수일땐 y 2(mod4) 이면좌변은 4의배수지만 8의배수는아니라서모순. 즉 y =4Y꼴이고, 그럼 x도 4X꼴.(Y +1)(Y 1) = 4X 3 에서 Y 는홀수, 즉 Y =2Z +1꼴이고, Z(Z +1)=X 3 에서차이가 1인두세제곱수는 0과 1뿐. 정리하면됨. 131. x 2 +615=2 y 의자연수해 (x; y) 를모두구하여라. ( 이탈리아 1995-6)
58 수론 풀이 x 2 ( 1) y (mod 3) 이므로 y 는짝수. y =2z 라하면 (2 z + x)(2 z x) =615=3 5 41. (2 z + x; 2 z x) =(41; 15), (123; 5), (205; 3), (615; 1) 을각각풀면됨. 답 59, 12? 132. 방정식 2x 2 3x =3y 2 의자연수해는무한히많은지, 아니면없는지, 있기는있는데유한개뿐인지를판정하여라. (ymo 2008-1-42) 풀이 3x, 3y 2 이모두3의배수이므로 3 j 2x 2, 즉 3 j x. x =3z 를대입하고양변을 3으로나눠주면 6z 2 z = y 2. 즉 z(6z 1) = y 2. z와 6z 1 은서로소이고 z는자연수이므로둘다완전제곱수. 그런데 6z 1 꼴의완전제곱수는없다. 따라서, 이식은자연수해를전혀갖지않는다. 133. 임의의서로다른정수 x, y, z 에대하여 (x y) 5 +(y z) 5 +(z x) 5 이 5(x y)(y z)(z x) 로나누어짐을증명하여라. ( 소련 1962-12) 증명 x y = r, y z = s 로두면 z x = (r + s) 이고 (x y) 5 +(y z) 5 +(z x) 5 은 r 5 + s 5 (r + s) 5 = 5r 4 s 10r 3 s 2 10r 2 s 3 5rs 4 = 5rs(r + s)(r 2 + rs + s 2 ) 134. 자연수수열 (A n ) 은 A 1 < 1987 이고임의의자연수 i 에대해 A i + A i+1 = A i+2 를만족한다. A 1 A n 과 A 2 + A n 1 이모두 1987 의배수이면, n 은홀수임을증명하여라. ( 레닌그라드 1987-48) 증명 1987 j f(k)+( 1) k f(n k +1) 임을점화적으로알수있음. 여기서 k = n 일때n이짝수라하면 1987 j f(n)+f(1). 이걸 1987 j f(1) f(n) 과변변더하면1987 j 2f(1), 즉 1987 j f(1) 로모순. 135. 3x 2 px + q =0 이서로다른두유리근을가지는모든자연수쌍 (p; q) 를찾아라. ( 이탈리아 1993-2) 풀이판별식 D = p 2 12q 가양의완전제곱수 (d 2 ) 면됨. p 2 d 2 (mod 12) 인경우들.WLOGd의부호를적절히결정하면 p d (mod 6) 이라할수있음. 즉, d = p 6n 꼴이고, 그럼 12q = p 2 d 2 = 12pn 36n 2, 즉 q = pn 3n 2 = n(p 3n). 단, p>3n >0. 136. n 은주어진정수이다. 연립방정식 xy + yz + zx =3n 2 1, x + y + z =3n 을만족하는정수해 x y z 는 (x; y; z) =(n +1;n;n 1) 이유일함을보여라. ( 스웨덴 1977-3) 증명 (x y) 2 +(y z) 2 +(z x) 2 =2(x + y + z) 2 6(xy + yz + zx) =6. 137. a 3 +3a 2 + a 가완전제곱수인자연수 a 는존재하지않음을증명하여라. ( 이탈리아 1991-2) 증명 a(a 2 +3a +1) 이제곱수임에서 a와 a 2 +3a +1은서로소이므로각각제곱수. a = m 2 을대입하면 m 4 +3m 2 +1=n 2. 양변 4 곱해서정리하면 (2m 2 +3) 2 (2n) 2 =5. 두양의제곱수의차가 5가되는경우는9 4 뿐인데, 그럼 m =0, 즉 a =0이므로모순. 138. p 는소수이다. p j n 2 n +3 을만족하는정수 n 이존재하는것과 p j m 2 m +25 을만족하는정수 m 이존재하는것은필요충분임을보여라. ( 폴란드 1994 2 차 -6) 증명 p =2일때는둘다해가없어서ok. p =3일때는둘다해가있어서ok. 이제 p는 5 이상의소수. p j (2n 1) 2 +11 과 p j (2m 1) 2 +99 에해당하므로 ( 그리고, 홀수들의제곱은모든제곱수와 mod p로같은집합을이루므로 ) x 2 11 (mod p) 인 x의존재여부와 y 2 99 (mod p) 인 y의존재여부에해당. 이것은 y =3x 또는 x =3 y ( 잉여역수 ) 의대응으로끝. 139. x + y + z, 1 x + 1 y + 1, xyz가모두정수가되도록하는양의유리수들의순서쌍 (x; y; z) 을모두구하여 z 라. ( 폴란드 1994 3차-1) 풀이 x = q p 대입해서각각정수 a, b, c라두고첫식과막식의결과를중간식에대입하면 p j q3 임을얻어낼수있음. 그럼 p =1. 대칭적으므로 y, z도마찬가지.
1.2 수론고급문제 59 주유리해정리를참조할수도있음. 140. 각자연수 n 에대해 ( 십진법에서의 ) n 의자릿수들을재배열하여만들수있는자연수들의집합을 A n 이라하자. 그리고, A n 의원소들의최대공약수를 d n 이라하자예를들어, A 1120 = f112; 121;:::;2101; 2110g 이고 d 1120 =1 이다. n 이십진수로나타냈을때모든자릿수가다같지는않은자연수일때, d n 의가능한최대값을구하여라. ( 이탈리아 1997-5) 풀이 a와 b가 n의서로다른자릿수일때 d n j ab ba =9(a b). 따라서, d n 9 9. 실제로 n = 9999999990 일때는어떻게재배열해도 9 2 의배수이므로 d n =81이가능함. 141. 십진법에서모든자릿수가같은자연수들중에서세연속한홀수의제곱수의합이되는수를특별한수라고하자. (a) 4자리의특별한수를모두찾아라. (b) 2000자리의특별한수가존재하는가? ( 이탈리아 2000-1) 풀이세연속한홀수의제곱수의합은 3(2m +1) 2 +8꼴. (a) 5555. (b) mod 3, mod 10, mod 8 등을적절히사용해보면없음. 142. 자연수 n 에대해, m =5 n +3 n +1 이소수이면 n 은 12 의배수임을증명하여라. ( 이탈리아 2002-5) 증명 n 이홀수이면 6 j 5 n +1 이므로 3 j m 이되어모순. n 이짝수인데 4 의배수는아니면 10 j 3 n +1=9 m +1 로 5 j m 이되어모순. 고로, n 은 4 의배수. n =4k 로두면 m 25 2k +9 2k +1 4 2k +2 2k +1 2 k +4 k +1 (mod7) 인데,2 3 4 3 1(mod7) 이므로 k =1; 2 일때만체크하면 3 - k 일때는 7 j m 이되어모순임을알수있음. 143. a, b 는합이 1 인양의실수이다. a 3 과 b 3 이유리수이면, a 와 b 도유리수임을보여라. ( 폴란드 1994/1995 1 차 -9) 증명 a 3, b 3 의분모에따라적당히 a, b에몇배해주면 a + b는자연수, a 3, b 3 도자연수. ab는유리수인데그세제곱이자연수이므로역시자연수. 그럼둘은각각자연수가아니라면켤레무리수. 다시세제곱싸바싸바하면좀노동해서끝. 144. P 는정계수다항식인데, P (5) 는 2 의배수이고 P (2) 는 5 의배수이다. P (7) 는 10 의배수임을증명하여라. ( 폴란드 1995 2 차 -1) 증명 x y j p(x) p(y) 임을이용.7 5 j P (7) P (5), 7 2 j P (7) P (2) 에서각각 2; 5 j P (7). 145. m n 크기의체스판에서각격자점들과각칸의중심마다졸을하나씩놓았다. (a) 정확히 500개의졸이놓이는격자의크기 (m; n) 을모두찾아라. (b) 정확히 k개의졸이놓이는크기의격자가존재하지않는자연수 k가무한히많음을증명하여라. ( 이탈리아 2003-5) 증명 (2m +1)(2n +1)=2k 1. (a) 는그냥적당히풀고,(b) 는 2k 1 이소수이면해가없으니까. 146. 2004 2005 를두제곱수의합으로표현할수있는가? ( 이탈리아 2004-3b) 풀이 3 j a 2 + b 2 이면 a, b 모두 3 의배수임에서계속 3 2 으로나눠주면 3 의홀수승이라서곤란해짐. 147. p n +144=m 2 인자연수해 (m; n; p) 를모두찾아라. 단, p 는소수여야한다. ( 이탈리아 2006-2) 풀이 p n =(m +12)(m 12) = p a p b 꼴. 만일 b =0이면 m =13이고 p n =5 2. 이제 b>0 일때를보자.24=p a p b = p b (p a b 1) 에서두인수는서로소이므로 p b =2 3 또는 3 1, 즉 m =20또는 15. 각각 p n =2 8 또는 3 4.
60 수론 148. a, b, c, d는실수이다. a = b = c =0이고 d가유리수일때만, 임의의실수 x에대해ax 3 + bx 2 + cx + d 가항상유리수가됨을증명하여라. ( 몰도바 2001 최종-y7-5) 증명준식을 f(x) 라할때f(0) = d 는틀림없이유리수. f(x)+f( x) 를생각하면 bx 2 도항상유리수. 즉, b =0. f(x) f( x) 를생각하면 A(x) =x(ax 2 + c) 이항상유리수. f(2x) f( 2x) 로하면 B(x) =x(4ax 2 + c) 도항상유리수. B A 나 4A B 를생각하면 ax 3, cx도항상유리수이므로 a = c =0. 149. 임의의정수 n 에대해,2+2 p 28n 2 +1 이정수이면, 이수는제곱수임을보여라. ( 헝가리 1969-1) 증명준식을 m이라하면 m은짝수, 또좀더살펴보면 m은 4의배수. m =4r이라두면 7n 2 = r(r 1) 로정리됨. r과 r 1은서로소이므로다음두가지뿐.(1)r =7a 2 = b 2 +1꼴일때 :mod4또는 mod 7로따져보면불가능.(2)r =7a 2 +1=b 2 꼴일때 : 이때는m이제곱수이므로 ok. 150. 방정식 x x = y 3 + z 3 이무한히많은자연수해 x; y; z를가짐을보여라. ( 폴란드 1995/1996 1차-10) 증명 x =2 n, y = z =2 k 꼴만생각. 2 n 2n =2 3k+1 이므로 n 2 n =3k +1꼴이되는 n이무한히많음을보이면충분. n =6r 2꼴이면됨.
제 2 장 대수 2.1 대수중급문제 1. 함수 f 는임의의 m; n 2 N 에대해 f(mn) =mf(n)+nf(m) 을만족한다고한다. f(12) = f(15) = f(20) = 60 일때f(8) 의값을구하여라. (2002 플란더즈예선 ) 풀이주어진조건을 f의성질에대입하면다음과같다. f(12) = 3f(4) + 4f(3) = 60 f(15) = 5f(3) + 3f(5) = 60 f(20) = 4f(5) + 5f(4) = 60 f(3), f(4), f(5) 를각각미지수로보고연립방정식을풀면 f(4) = 8 를얻는다. 로부터다시 f(2) = 2 를얻고, 그럼 f(4) = 2f(2) + 2f(2) = 8 f(8) = 4f(2) + 2f(4) = 24 답 이다. 2. 양수 x, y 의산술평균과기하평균을각각 m a, m g 로나타내자. m a + m g = y x 일때 x y 의값을구하 여라. 단, 두수의산술평균은두수의합을 2 로나눈것이고, 기하평균은두수의곱의제곱근이다. ( 루마니아 2005 지역예선 y7-3a) 풀이 x+y + p xy = y x, 2 2 배하여정리하면 2 p xy = y 3x, 양변제곱하면 4xy = y 2 6xy +9x 2, 즉 y 2 10xy +9x 2 =0이고, 인수분해하면 (y x)(y 9x) =0이다. y = x 일때는준식의좌변은 2x로양수, 우변은 0이므로성립하지않고, y =9x 일때는8x =8x 로잘성립한다. 따라서, 답 x y = 1 9 3. 합 1 1! + 2 2! + 3 3! + +(n 1) (n 1)! + n n! 의값을구하여라. 단, n! =n(n 1)(n 2) 2 1 이다. ( 캐나다 1969-6)
62 대수 풀이 k k! =[(k +1) 1] k! =(k +1)! k! 이다. 즉, 1 1! + 2 2! + 3 3! + +(n 1) (n 1)! + n n! =(2! 1!) + (3! 2!) + (4! 3!) + +(n! (n 1)!) + ((n +1)! n!) 그럼 2! 과 2!, 3! 과 3!, :::; n! 과 n! 들이상호소거되고, 처음과마지막의 1! 과 (n +1)! 만남는다. 답 (n +1)! 1 4. 다음연립방정식을만족하는모든실수해순서쌍 (x; y; z) 를찾아라. ( 캐나다 2004-1) 8 >< xy = z x y xz = y x z >: yz = x y z 풀이우선 x = y = z 이면 (1) 에서 x 2 + x =0, 즉 (x; y; z) =(0; 0; 0) 이거나 ( 1; 1; 1) 이고, 이들은모두해가된다. 이제 x, y, z가모두같지는않다고하자.(1),(2),(3) 세식을둘씩더하거나 (7,8,9) 빼면 (4,5,6) 다음을얻는다. (x +2)(y z) =(y +2)(z x) =(z +2)(x y) =0 (4); (5); (6) x(y + z +2)=y(z + x +2)=z(x + y +2)=0 (7); (8); (9) x; y; z 중에는서로같지않은쌍이최소한둘은있으므로, (4){(6) 에서 x; y; z 중적어도둘은 2 이다. 일반성을잃지않고 y = z = 2 라하면 (7) 에서 x =0. 따라서, 대칭적으로 (x; y; z) =(0; 2; 2), ( 2; 0; 2), ( 2; 2; 0) 만이가능하고이들은모두해가된다. 따라서, 답은 (0; 0; 0), ( 1; 1; 1), (0; 2; 2), ( 2; 0; 2), ( 2; 2; 0) 답 5. 임의의실수 x, y 에대해다음을만족하는함수 f : R! R 를모두구하여라. 풀이 y =0 을대입하면 f(x)f(y) f(xy) =x + y ( 통신강좌 1997-15-10) f(0) f(x) f(0) = x f(0) 6= 0임은자명하다. f(x) = x f(0) +1 f(x) =kx +1을대입해풀어보면 k =1 고로 f(x) =x +1 p p p 6. y = 2+ 2, z = 2 p 2 일때x = y + z 의값을구하여라. ( 아벨콘테스트 1994 예선 ) 풀이 x 2 =(y + z) 2 =(2+ p 2) + (2 p 2) + 2 p 4 2=4+2 p 2=2 (2 + p 2) ) x = p p 2 2+ p 2= p 2y: 7. 다음조건을만족하는순서쌍 (x; y; z) 를모두찾아라 : 이중임의의한수를다른두수의곱에더하면그결과는언제나 2 가된다. ( 캐나다 1970-1) 풀이식으로옮기면다음과같은연립방정식이다. (1) (2), (2) (3), (3) (1) 하여 (1) x + yz =2; (2) y + zx =2; (3) z + xy =2 (x y)(z 1) = (y z)(x 1) = (z x)(y 1) = 0 (4) 을얻는다. x =1 이면 (1), (2) 에서 yz =1,y + z =2 이고이것은 t 2 2t +1=0 의두근이므로 y = z =1. 대칭적으로 y =1 이거나 z =1 일때에도 x = y = z =1 을얻는다. x, y, z 모두 1 이아니면 (4) 에서 x = y = z. 그럼 (1) 은 x 2 + x 2=0 이되고, x 6= 1 이므로 x = 2. 즉,(x; y; z) =(1; 1; 1) 또는 ( 2; 2; 2) 답
2.1 대수중급문제 63 8. a + b =1을만족시키는양수 a, b에대하여 1 1+a + a 의최소값은얼마인가? ( 한국 2004 1차-J4) 1+b 풀이준식을 x라하면a, b 모두양수이므로우선 x>0 이다. b =1 a 를대입하여 b를소거하여통분하자. x = 1 1+a + a 2 a = 2+a2 2+a a 2 분모를이항하여 a 에대한내림차순으로정리하면 a 에대한이이차방정식이해를가지므로판별식 f(a) =(1+x)a 2 xa +2(1 x) =0 (1) D = x 2 4 2(1 x) (1 + x) 0 즉,9x 2 8, 그럼 x>0 이므로 x 2p 2 3. 따라서, x의최소값은 2p 2 3 일것으로기대할수있는데, 다만이때 (1) 의해가0 <a<1 범위에있어야진짜원하는최소값이된다. x = 2p 2 3 일때는판별식이 0일때이고 (1) 이중근을가져완전제곱식이될때이다. 이때a = x 2(1+x) 로 0 <a<1 을틀림없이만족함을알수있다. 별해 x 이차방정식 (1) 의그래프를생각해보자. 축 a = 2(1+x) 는 0 < a < 1 2 의범위에있으므로 f(0) <f(1) 이다. 따라서,(1) 이 0 <a<1 에서해를가질필요충분조건은다음과같다. f( 축 ) 0; f(1) > 0 이것을풀면 2p 2 3 x< 3 2 이된다. 9. 두개의가방에몇개의수가들어있고, 전체수의개수는어떤소수이다. 가방 1 에서가방 2 로수 170 을옮겼더니각가방의수들의평균값이둘다 1 씩증가하였다. 전체수의합이 2004 라할때, 전체수의개수를구하여라. ( 플란더즈 2004-2) 풀이가방 1 속의수의개수를 m, 가방 2 속의수의개수를 n이라하고, 가방 1 속의수의합을 a, 가방 2 속의수의합을 b라고하자. 조건에맞게식을써보면, 두식을각각정리하면 a 170 m 1 = a m +1; b +170 n +1 = b n +1 m 2 +169m a =0 n 2 169n + b =0 두식을빼면 m 2 n 2 + 169(m + n) (a + b) =0 (m n)(m + n) + 169(m + n) 2004 = 0 (m n +169)(m + n) = 2004 = 2 2 3 167 m + n 이소수이고 m + n =2,3,167 중에하나이다. 그런데, m n +169 가 m + n 보다 169 이상클수없으므로 m + n =167 만가능하다. 10. x, y, z 는적당한수 a, b 에대해 a + b + x + y + z =2004 와 0 x a y b z 를만족시킨다. x + y + z 의최대값과최소값을구하여라. ( 호주 2004-2)
64 대수 풀이 (1) x + y + z a + b + x + y + z 2004 이고, 등호가 a + b =0 일때, 즉 (x; a; y; b; z) = (0; 0; 0; 0; 2004) 일때성립하므로최대값은 2004. (2) a + b y + z x + y + z 이므로 2(x + y + z) a + b + x + y + z = 2004, 즉 x + y + z 1002 이고, 등호가 a + b = x + y + z 일때, 즉 (x; a; y; b; z) =(0;a;a;b;b)(a + b = 1002) 일때성립하므로최소값은 1002. 11. x 4 + x 3 + x 2 + x +1=0 는몇개의실근을갖는가? ( 플란더즈예선 1991/1992) 풀이양변에 (x 1) 을곱하면 x 5 1=0이된다. x>1 이면 x 5 > 1 이고 x<1 이면 x 5 < 1 이므로 x =1만가능한데, 이것은원래의식은성립시키지않는다. 즉, x 4 + x 3 + x 2 + x +1=0의실근은 0개. 12. 다음연립방정식을보자. x 1 +4x 2 +9x 3 +16x 4 +25x 5 +36x 6 +49x 7 =1 4x 1 +9x 2 +16x 3 +25x 4 +36x 5 +49x 6 +64x 7 =12 9x 1 +16x 2 +25x 3 +36x 4 +49x 5 +64x 6 +81x 7 =123 이때,16x 1 +25x 2 +36x 3 +49x 4 +64x 5 +81x 6 +100x 7 의값을구하여라. (AIME 1989-8) 풀이 Label equations given (1), (2), (3). Then (1) - 2 (2) + (3) = 2(x_1 +... + x_7), (3) - (2) = 5x_1 + 7x_2 + 9x_3 +.... Hence 7x_1 + 9x_2 + 11x_3 +... = (1) - 3 (2) + 2 (3) and required value = (1) - 3 (2) + 3(3). 답 334 13. 자연수함수 f가다음과같이정의된다. ( n 3 (n 1000 일때 ) f(n) = f(f(n +6)) (n < 1000 일때 ) f(1992) f(1) 은얼마인가? ( 플란더즈예선 1991/1992) 풀이 n =999부터몇개의값을구해보면 f(999) = f(996) = 999, f(998) = f(995) = 998, f(997) = f(994) = 997로일정한규칙성을짐작할수있다. 수학적귀납법으로 n =997 6k (1 n 1000) 일때 f(n) 값이같음을증명한다. ² k =0일때f(997) = 997. ² k =0:::n 1일때f(997 6k) =997이라면, f(997 6n) =f(f(997 6n +6))=f(f(997 6(n 1)) = f(997) = 997. 따라서 f(1) = f(997 6 166) = 997이고, f(1992) f(1) = 1989 997 = 992: 14. x와 y는 x + y =1을만족하는양의실수라한다. 이때다음부등식이성립함을보여라. ( 캐나다 1971-2) µ 1+ 1 µ 1+ 1 9 x y 증명좌변을전개해보자. µ 1+ 1 µ 1+ 1 =1+ 1 x y x + 1 y + 1 + y +1 =1+x =1+ 2 xy xy xy 산술-기하평균부등식에서 x + y 2 p xy, 즉 xy 1 4 이므로 1+ 2 xy 1+ 2 =9 1 4 등호는 x = y = 1 2 일때성립한다.
2.1 대수중급문제 65 15. x =(1+ 1 n )n 이고 y =(1+ 1 n )n+1 일때, y x = x y 임을보여라. ( 캐나다 1974-1a) 증명다음과같이서로같다. µ x y = 1+ 1 n( n+1 n )n+1 µ = 1+ 1 (n+1) n+1 n n n n µ y x = 1+ 1 (n+1)( n+1 n )n µ = 1+ 1 (n+1) n+1 n n n n 16. 방정식 [x]+ 2004 = x 2 + 2004 [x] x 2 는가장큰정수이다.) 의 1 이아닌해를 a 라할때, a 2 의값을구하여라.( 단,[x] 는 x 를넘지않 ( 한국 2004 1 차 -J15) 풀이우변이양수이므로좌변도양수, 즉 [x] > 0 이라야하고, 또 1이아닌해라했으므로, a>1 이다. 이때, a 2 [a] a 2 a>0 이므로 [a]+ 2004 [a] = a 2 + 2004 a 2 () 2004 2004 [a] a 2 = a 2 [a] () 2004 a2 [a] [a]a 2 () a 2 [a] = 2004 = a 2 [a] a = p 167 일때이식이성립하고, 좌변은 a 가증가함에따라단조증가하므로해는이것하나뿐이다. 따라서, a 2 =167. 17. S(n) =1!+2!+ + n! 을나타내는것으로할때,3 1! + 4 2! + 5 3! + 6 4! + +(n +2) n! 의값을 S(n) 을이용하여나타내어라. 풀이 (k +2) k! =(k +1)!+k! 이므로준식은 S(n +1)+S(n) 1=2S(n)+(n +1)! 1 답 18. 음이아닌세실수 x 1, x 2, x 3 의합은 1=2 보다크지않다. 다음을증명하여라. (1 x 1 )(1 x 2 )(1 x 3 ) 1 2 ( 뉴질랜드 2000-2) 풀이전개하면 1 (x 1 + x 2 + x 3 ) 1 2, x 1x 2 x 1 x 2 x 3 = x 1 x 2 (1 x 3 ) 0, x 2 x 3 + x 3 x 1 0. 등호조건은하나가 1, 2 나머지둘이 0일때. 19. 실수 x는 [x +0:19] + [x +0:20] + [x +0:21] + +[x +0:91] = 546 을만족한다.[100x] 의값을구하여라. (AIME 1991-6) 풀이 %EEE 546 = 7 73 + 35 = 38 7 + 35 8. Hence [x + 0.19] = [x + 0.20] =... = [x + 0.56] = 7, [x + 0.57] =... = [x + 0.91] = 8. So 7.43 <= x < 7.44. 답 743 20. x = p 91 19 + p 91 을만족하는실수해 x의절대값을모두합한값을 k라하 19 + p 91 19 + p 91 19 + p 91 19 + x 자. k 2 을구하여라. (AIME 1991-7)
66 대수 풀이 %EEE It is clear that simplifying will give a quadratic in x. One can slog through to find it, but it is easier to note that if x = 19 + 91/x, then x will satisfy the equation, so the quadratic must be simply x2-19 x - 91. A minor trap is that one root is negative: x = ( 383 + 19)/2 or -( 383-19)/2, so sum of abs values is 383. 답 383 21. 모든양의정수 n 에대해, 다음을증명하여라. ( 캐나다 1974-1b) 1 2 2 2 +3 2 4 2 + +( 1) n (n 1) 2 +( 1) n+1 n 2 =( 1) n+1 (1 + 2 + + n) 증명 n 이짝수일때 n 이홀수일때 (1 2 2 2 )+(3 2 4 2 )+ +((n 1) 2 n 2 ) =(1 2)(1 + 2) + (3 4)(3 + 4) + ((n 1) n)((n 1) + n) = (1+2+3+4+ +(n 1) + n) 1 2 (2 2 3 2 ) ((n 1) 2 n 2 ) =1 (2 3)(2 + 3) ((n 1) n)((n 1) + n) =1+2+3+4+ +(n 1) + n 로항상성립한다. 별해수학적귀납법을이용한다. n =1일때는자명. n = k 일때성립한다고가정하면 1 2 2 2 + +( 1) k+1 k 2 k+1 k(k +1) =( 1) 2 1 2 2 2 + +( 1) k+2 (k +1) 2 k+1 k(k +1) =( 1) +( 1) k+2 (k +1) 2 2 k+2 k +1 =( 1) f2(k +1) kg 2 k+2 (k +1)(k +2) =( 1) 2 이므로 n = k +1일때도성립한다. 그러므로모든양수 n에대해준식은성립. 22. x, y, z 가모두실수일때, x + y + z =5 xy + yz + zx =3 을만족하는 z 의최대값을구하여라. ( 캐나다 1978-3) 풀이 x + y =5 z 이고, xy =3 z(x + y) =3 z(5 z) =z 2 5z +3 따라서, x, y를두실근으로갖는방정식은 t 2 (5 z)t +(z 2 5z +3)=0 이며, 판별식 D =(5 z) 2 4(z 2 5z +3)= 3z 2 +10z +13 0 임에서 3z 2 10z 13 = (z + 1)(3z 13) 0, 즉 1 z 13 3 이다. z = 13 3 일때 x = y = 1 3 임을확인할수있으므로 z 의최대값은 13 3 답
2.1 대수중급문제 67 23. 방정식 (x 2 x +1)(x 2 x +2)=12 의실근은모두몇개인가? ( 플란더즈예선 1995/1996 2 차 ) 풀이 x 2 x = t라고하면 x가실수인범위에서 t 1 4 이다. (t +1)(t +2)=12) t 2 +3t 10 = 0 ) (t 2)(t +5): 이때t 1 4 인해는t =2이고이때x2 x = t를만족하는 x는두개있다. 답 2개 24. 함수 f(x) 는임의의실수 x 에대하여 f(1 + x)+xf(1 x) =x 2 + x 를만족시킨다. f( 6) 의값을구하여라. ( 한국 2003 1 차 -J6) 풀이 (1 + x) 와 (1 x) 는 1에대칭적인값이다. x = 7 을대입하면f( 6) 과 f(8) 의값을함께구할수있다. (1) x =7대입 : f(8) + 7f( 6) = 7 2 +7=56. (2) x = 7 대입 : f( 6) 7f(8) = ( 7) 2 7=42. 아래식의 f(8) 에위식을대입한다. f( 6) 7f56 7f( 6)g =42 50f( 6) = 42 + 392 = 434 f( 6) = 8:68 답 8.68 25. x가양의실수일때, 방정식 3x 3 [x] =3을풀어라. (IMO-LL 1989, 포르투갈출제 ) 풀이 x 2 이면 3x 3 = x 3 +2x 3 >x+3 [x]+3 이므로성립하지않는다.[x] =0일때도3x 3 =3 q 에서 x =1이되어성립하지않는다. 따라서,1 x<2 이고 [x] =1. 이때x 3 = 4 3, 즉 x = 3 4 3 답 26. 임의의네양의실수 a, b, c, d 에대해, 다음을증명하여라. ( 인도지역예선 1991-2) a b + b c + c d + d a 4 풀이 A.M G.M 으로 a b + b c + c d + d a 4 4 q a b b c c d d a =4. 27. 이차방정식 x 2 + bx +6b =0 이정수해만을갖도록하는실수 b 는모두몇개인가? (AIME 1991-8) 풀이두정수근을 m n 이라하면 mn =6b, m+n = b. mn+6(m+n) =0이므로 (m+6)(n+6) = 36. 36의양의약수가9개임에서 36을두정수의곱으로표현하는방법은 ( 양수, 음수모두세었을때 ) 2d 9 e =10 2 가지이고실제로각경우에 m + n의값이모두다르므로 b는모두10개 답 28. 1000, n, 1000 n, n (1000 n), ::: 는 n 번째항에서 n +1 번째항을뺀것을 n +2 번째항으로하는수열이다. 이수열은처음으로음수가나타나면종료되는것으로한다. 이수열의길이를최대로하는자연수 n 을구하여라. (AIME 1998-8) 풀이항을계속구하면 1000, n, 1000 n, 2n 1000, 2000 3n, 5n 3000, 5000 8n, 13n 8000, 13000 21n, 34n 21000, 34000 55n, 89n 55000, ::: 등이다. 제3항부터각각이 0 이상임을부등식으로풀면 n 1000, n 500, n 666:6, n 600, n 625, n 615:3, n 619:0, n 617:6, n 618:1, n 617:9 등이된다. 답 618 29. A B C 크기의상자의부피가 (A +2) (B +2) (C +2) 크기의상자의부피의절반이된다고한다. A B C 가자연수들일때, C 의최대값은얼마인가? (AIME 1998-14)
68 대수 풀이 %EEE c = 2(a+2)(b+2)/(2ab-(a+2)(b+2)) = 2(a+2)(b+2)/(ab-2a-2b-4) = 2 + (8a+8b+16)/(ab-2a-2b-4). So (c-2)/8 = (a+b+2)/(ab-2a-2b-4). To make progress we need to realise that it helps to put c'=c-2, b'=b-2, a'=a-2, so c'/8 = (a'+b'+6)/(a'b'-8). Now we cannot have a or b = 1 or 2, because (1+2)/1 and (2+2)/2 are already 2 (so multiplying by (c+2)/c gives > 2). So a', b' 1, so a'+b' a'b'+1. Hence c'/8 (a'b'+7)/(a'b'-8) = 1 + 15/(a'b'-8) 1 + 15. Hence c 130 and it is easy to check this is realised. 답 130 (3,11) 30. (3x 2 2x 1) 4 = a 8 x 8 + a 7 x 7 + + a 1 x + a 0 일때, a 8 + a 6 + a 4 + a 2 는얼마인가? ( 플란더즈예선 1995/1996 2 차 ) 풀이 x =0 을대입하여 a 0 를구한다.( 1) 4 =0+0+ + a 0 = a 0 =1: x =1 을대입하면 :0 4 =0=a 8 + a 7 + + a 1 + a 0 (a) x = 1 을대입하면 :4 4 =256=a 8 a 7 + a 1 + a 0 (b) (a)+(b) = 256 = 2(a 8 + a 6 + a +4+a 2 )+2a 0 ) a 8 + a 6 + a 4 + a 2 = 256 2 2 =127 답 127 31. a 와 b 는서로다른실수이고, 두점 (a; a 3 ) 과 (b; b 3 ) 을지나는직선이 y = x 3 의그래프와또다른점에서다시만난다. 이점의 y- 좌표는얼마인가? ( 플란더즈예선 1996/1997 1 차 ) 풀이두점 (a; a 3 ) 과 (b; b 3 ) 을지나는식은다음과같다. y a 3 = b3 a 3 a b (x a) =(a2 + ab + b 2 )(x a) 이직선이y = x 3 과만나는또다른점 (p; p 3 ) 을대입하고 a 6= p이므로양변을 (p a) 로나눈다. p 3 a 3 =(a 2 + ab + b 2 )(p a) ) p 2 + ap + a 2 = a 2 + ab + b 2 ) p 2 + ap = b 2 + ab ) (p + a 2 )2 =(b + a 2 )2 ) p = b; (a + b) 이때p6= b이므로 p = (a + b) 이다. 답 (a + b) 3 32. 함수 f : R! R 가 f(x) +f(1 x) =7과 x + f( x 3 )= 1 2 f(x) 를만족시킬때, f( 1 9 ) 의값은? ( 단, R은실수전체의집합이다.) ( 한국 2005 1차-J9) 풀이뒷식에 x =0을대입하면 f(0) = 0. 다시앞식에 x =0을대입하면 f(1) = 7 이된다. 이제뒷식에 x =1,x = 1 3 을차례로대입하면 f( 1 3 )= 1 2 f(1) 1= 5 2, f( 1 9 )= 1 2 f( 1 3 ) 1 3 = 11 답 12 33. 다음연립방정식을주어진상수 A 에관해풀어라. ( 인도지역예선 1992-7) (x + y)(x + y + z) =18 (y + z)(x + y + z) =30 (z + x)(x + y + z) =2A 풀이세식을더하면 (2x +2y +2z)(x + y + z) =48+2A이고정리하면 (x + y + z) 2 =24+A이다. t = x + y + z = p 24 + A 라고두고 z를 A에대한식으로표현해보자.(x + y)(x + y + x) = (t z)t = t 2 zt =(24+A) zt =18, 즉 z = 6+A. t 같은방법으로 x = 6+A, y = 24 A. t t t가 0일때는해가없고, t가양일때나음일때나모두준식을잘만족함을확인할수있다. 답 (x; y; z) = p pm1 24+A ( 6+A; 24 A; 6+A) ( 단, A> 24 일때만 ) 34. 함수 f 는모든 x 에대해 f(x)+f(x 1) = x 2 을만족한다. f(19) = 94 일때, f(94) 를구하여라. (AIME 1994-3) 풀이 %EEE f(94)=94^2-f(93)=94^2-93^2+f(92)=94^2-93^2+92^2-f(91)=... = (94^2-93^2) + (92^2-91^2) +...+ (22^2-21^2)+ 20^2-f(19) = 94+93+...+21+400-94 = 115 37+306 = 4561. 답 4561
2.1 대수중급문제 69 35. 음이아닌임의의실수 x, y 에대해함수 f 가 f(xy) =f(x)+f(y) 를만족한다. f(0:5) + f(1) + f(2) 의값은얼마인가? ( 플란더즈예선 1999/2000 2 차 ) 풀이 x = y =1 대입하면 f(1) = 0 준식은 f(0:5 1 2) = f(1) = 0 36. 0 <b<10 6 일때, 다음중 p 1 b에가장가까운근사값은어느것인가?(1)1 b (2) 1 b (3) 1 b2 2 (4) 1 b2 (5) 1 ( 2 플란더즈예선 2000/2001 1차 ) 풀이제곱해서비교하면 (1) 1 2b + b 2 =1 b +( b + b 2 ) < 1 b, (2)1 b + b2 4 > 1 b 이다. b의범위에의해 b 2 >b2 > b2 > 0 2 이므로 (2) < (3) < (4) < (5) 이다. 따라서,(1) 과 (2) 중에서 p 1 b에더가까운것을찾으면된다.(1) 과 (2) 의평균인1 3 4 b와 p 1 b를비교하면,(1 3 4 b)2 = 1 3 2 b + 9 16 b2 =(1 b) b( 1 2 9 16 b) < 1 b 로 p 1 b쪽이더크다. 따라서, 더큰쪽인 (2) 가더가까운근사값이다. 답 (2) 37. 실수 x에대하여, x 2 + 16 x 2 의최소값을구하여라. +1 ( 한국 2005 1차-J11) 풀이 ( 대전대덕중 3 학년고기혁 ) x 2 +1> 0 이므로산술 - 기하평균부등식에서 ( 준식 )=(x 2 +1)+ 16 r x 2 +1 1 2 (x 2 16 +1) x 2 1=2 4 1=7 +1 x = p 3 일때실제등호가성립하므로, 최소값은 7 답 38. 4p 1 k에가장가까운정수를 f(k) 로쓰기로하자. f(1) + 1 f(2) + 1 f(3) + + 1 f(1995) 를계산하여라. (AIME 1995-13) 풀이 %EEE f(k) must take one of the values 1, 2,..., 7. The boundary values are (n + ½)^4 = n^4 + 2n^3 + ½(3n^2 + n) + 1/16, giving 5 1/16, 39 1/16, 150 1/16, 410 1/16, 915 1/16, 1785 1/16. So the sum is 5 + 34/2 + 111/3 + 260/4 + 505/5 + 870/6 + 210/7 = 400. 답 400 39. (x 1)(x 2) (x 99) 를전개했을때, x 98 의계수를구하여라. (2000 대전. 충남영재수학교실 1 차평가 ) 풀이 (1+2+3+ +99)= 4950 답 40. a 와 b 는 a 6= b 인주어진두실수이다. 상수 k > 0 가어떻게주어지느냐에따라 x 에대한방정식 jx aj + jx bj = k 의해가달라질수있다. 이에대해다음중맞는진술을모두골라라. (1) 해가딱하나있도록하는 k값은딱하나있다. (2) 해가딱하나있도록하는 k값은무한히많다. (3) 해가무한히많이있도록하는 k값은딱하나있다. (4) 해가무한히많이있도록하는 k값은여러개있다. (5) 어떤 k값에대해서도해는항상딱두개이다. ( 플란더즈예선 1999/2000 2차 ) 풀이 x의범위를a; b보다클때, a; b 사이에있을때, a; b보다작을때로나누어생각하면쉽게다음과같은사실을알아낼수있다. k>ja bj일때해는x = a+b+k ; a+b k 2 2 의두개이다. k = ja bj일때해는a, b 사이에있는모든 x로무한히많다. k<ja bj일때해는없다. 그러므로 (3) 만옳은답이다. 답 (3)
70 대수 41. 임의의 x 2 R 0 에대해 x f µ µ x 2 f =1 2 x 를만족하는함수 f : R 0! R 을모두찾아라. 단, R 0 은 0 을제외한실수전체의집합이다. ( 플란더즈 2002-2) 풀이 x 에 4 x 를대입하면 4 x f µ 2 f x µ x =1 2 이식에x를곱하고준식과더하면 3f( 2 x )=1+x. 여기에 x 대신 2 x+2 x 를대입하면 f(x) = 3x 임을알 수있다.( 이함수는준식에대입하면만족한다.) 42. 다음부등식을증명하여라. (KAIST Cyber 2002 겨울캠프평가시험 ) 1 1 + 1 p 2 + 1 p 3 + + 1 p n pn 증명분모가커질수록분수는작아지므로, 1 1 + p 1 + + p 1 p 1 + p 1 + + 1 1 p = n p = p n 2 n n n n n 등호는 n =1 일때에만성립. 43. 모든양의실수 a, b, c 에대해다음의부등식이성립함을보여라. a bc + b ca + c ab 2 a + 2 b 2 c 그리고, 이식에서등호가성립할 a, b, c 의조건을구하여라. ( 플란더즈 2002-1/J1) 증명 양변에 abc 를곱하여정리하면 a 2 + b 2 + c 2 2bc +2ca 2ab (a + b c) 2 0 따라서부등식이성립하고, 등호가성립할조건은 a + b = c 이다. 44. 임의의실수 t 에대해,[t] 은 t 보다작거나같은가장큰정수를나타낸다. 예를들어,[8]=8,[¼] =3, [ 5=2] = 3 등이다. 다음방정식이실근을갖지않음을보여라. [x]+[2x]+[4x]+[8x]+[16x]+[32x] = 12345 ( 캐나다 1981-1) 증명좌변의식을 S(x) 라하자. S(x) 는증가함수이다. ² x 196 이면 S(x) S(196) = 63 196 = 12348 ² x<196 이면좌변의각항에서최소 1씩줄어들어 S(x) S(196) 6 = 12342 이되므로, 주어진방정식의실수해는존재하지않는다. 45. 다음방정식의실근 a의개수를구하여라. 1 1 1 2 a + 3 a + 5 a = a 단,[x] 는 x 보다작거나같은가장큰정수를나타낸다. ( 캐나다 1998-1)
2.1 대수중급문제 71 풀이좌변은정수이므로우변의 a도정수. a =30n + r, 0 r<29 (n; r은정수 ) 로두자. 이것을대입하면 1 1 1 15n + 2 r +10n + 3 r +6n + 5 r =30n + r 1 1 1 n + 2 r + 3 r + 5 r = r 따라서, 각각의 r =0; 1;:::;29 이주어지면 n 이유일하게결정된다. 즉, 각각의 r =0; 1;:::;29 마다서로다른해가하나씩있으므로, 해는모두 30 개이다. 46. 2 <x 2 < 3 인음아닌실수x가있는데, 1 x 과 x2 의소수부가같다고한다. x 12 144 x 를구하여라. (AIME 1997-9) 풀이 %EEE Since 2 < x^2 < 3, we have 1 < x < 2, so 1/2 < 1/x < 1 and the fractional part of 1/x is just 1/x. The fractional part of x^2 is x^2-2, so x^3-2x - 1 = 0. Taking out the factor x + 1, and solving x^2 - x - 1 = 0, we get x = ( 5 + 1)/2. Now using x^2 = 1+x, we get successively x^4 = 2+3x, x^8 = 13+21x, x^12 = 89+144x, 1/x = x-1, so x^12-144/x = 233. 답 233 47. a, b, c, d 는 0 이아닌실수이고 f(x) = ax + b cx + d 이다. f(19) = 19, f(97) = 97 이고, x 6= d c 인모든 x 에대해 f(f(x)) = x 일때, f(x) =y 인 x 가존재하지않는유일한 y 를구하여라. (AIME 1997-12) 풀이 %EEE f(f(x)) = x simplifies to (a+d)(cx^2 + (d-a)x - b) = 0, so d = -a. There is no loss of generality in taking c = 1. Then f(19) = 19, f(97) = 97 give 38a + b = 361, 194a + b = 9409. Solving, a = 58, b = -1843. The unattained value is a/c. 답 58 48. a 1 =211,a 2 = 375, a 3 =420,a 4 = 523, 그리고 a n = a n 1 a n 2 + a n 3 a n 4 로정의된수열 fa n g 이있다. a 531 + a 753 + a 975 의값을구하여라. (AIME 2001 2 차 -3) 풀이 %EEE We find a_5 = 267, a_6 = -a_1, a_7 = -a_2, a_8 = -a_3, a_9 = -a_4, a_10 = -a_5, a_11 = a_1, a_12 = a_2, a_13 = a_3, a_14 = a_4 etc. So a_531 = a_1, a_753 = a_3, a_975 = a_5 and a_1 + a_3 + a_5 = 211 + 420 + 267 = 898. 답 898 49. 모든실수 x 에대해 f( x) =f(x) 를만족하는함수 f 를우함수, g( x) = g(x) 를만족하는함수 g 를기함수라고한다. f 가우함수, g 가기함수이고, f 와 g 는실수값을가지며항등적으로 0 이되지는않는함수들이다. 다음중기함수인것을모두골라라. f ± g, g ± f, f ± f, g ± g, f + g. ( 플란더즈예선 2002 2 차 ) 풀이각함수에 x값을넣어본다. f ± g( x) =f(g( x)) = f( g(x)) = f(g(x)) = f ± g(x): 우함수. g ± f( x) =g(f( x)) = g(f(x)) = g ± f(x): 우함수. f ± f( x) =f(f( x)) = f(f(x)) = f ± f(x): 우함수. g ± g( x) =g(g( x)) = g( g(x)) = g(g(x)) = g ± g(x): 기함수. (f + g)( x) = f( x)+ g( x) = f(x) g(x): f(x) 6= 0인 x 값을대입하면기함수가아님을알수있다. 50. 함수 f : Z! Z 는 n>999 일때 f(n) =n 3 을만족하고 n<1000 일때는 f(n) =f(f(n +5)) 를만족한다. f(84) 의값을구하여라. (AIME 1984-7)
72 대수 풀이 %EEE For n < 1002, f(n) = 997 for n even, 998 for n odd. 답 997 51. n>0 에대해 a n+2 = a n+1 a n 을만족하는정수들의수열 a 1 ;a 2 ;a 3 ;::: 이있다. 이수열의처음 1492 개항의합은 1985 이고, 처음 1985 개항의합은 1492 라고한다. 처음 2001 개항의합을구하여라. (AIME 1985-5) 풀이 %EEE The sequence is a_1, a_2, a_2-a_1, -a_1, -a_2, a_1-a_2, a_1, a_2,..., so it is periodic with period 6. Also the sum of a complete period of terms is 0. 1492 = 4 mod 6, 1985 = 5 mod 6, 2001 = 3 mod 6, so the sum of the first 1492 terms is a_1 + a_2 + a_3 + a_4 = 2a_2 -a_1, the sum of the first 1985 is a_1 +... + a_5 = a_2 - a_1. Hence a_2 = (2a_2 -a_1) - (a_2 - a_1) = 493, and the sum of the first 2001 = 2a_2 = 986. 답 986 52. n 이자연수일때, 다음식을간단히하여라. ( 플란더즈예선 1998/1999 1 차 ) 1+3+5+7+ +(2n 1) (2n +1)+(2n +3)+ +(4n 1) 풀이 n 2 =(4n 2 n 2 )=1=3 답 53. a는 a 3 = a +1 을만족하는양의실수이다. 다음중맞는식을모두골라라. (1) a 4 = a 2 + a (2) a 4 = a 5 1(3)a 4 = a 3 + a 2 1(4)a 4 = 1 ( 플란더즈예선 2000/2001 1차 ) a 1 풀이 (1) a 3 = a +2 식의양변에 a 를곱하면 a 4 = a 2 + a 를얻을수있다. (2) a 4 = a 2 + a 의양변에 a 를곱한다. 얻어진 a 5 = a 3 + a 2 의식에 a 3 = a +1,a 2 = a 4 a((1) 의식에서얻어진것 ) 을대입하면 a 5 = a +1+a 4 a = a 4 +1) a 4 = a 5 1 을얻을수있다. (3) (1) 에서얻은식 a 4 = a 2 + a 에서 a = a 3 1 을대입한다. a 4 = a 3 + a 1. (4) a 는 1 이아니므로 (2) 의식을정리하고양변에 (a 1) 을나눈다. a 4 = a 5 1 ) a 5 a 4 =1) (a 1)a 4 =1) a 4 = 1 a 1 : 답 (1), (2), (3), (4) 54. 방정식 x 3 3x 2 +3x +1=0 의모든실근의합을구하여라. ( 플란더즈예선 2002 2 차 ) 풀이 (x 1) 3 = 2. x 1= 3p 2. 근과계수와의관계에대한문제로혼동할수있음. 답 1 3p 2 55. f(x) 를 4, 9, 7, 5, x 의평균값이라정의하고 g(x) 를 4, 9, 7, 5, x 를순서대로늘어놓았을때가운데놓이는수라하자. f(x) =g(x) 가되는실수 x 를모두구하여라. (IT 꿈나무올림피아드 2006 1 차 ) 풀이평균 f(x) =5+ x 5 이다. x<5 이면 g(x) =5이므로 f(x) =g(x) 이려면 x =0이다.5 x 7 이면 g(x) =x 이므로 f(x) =g(x) 이려면 x = 25 4 이다. x>7 이면 g(x) =7이므로 f(x) =g(x) 이려면 x =10이다. 답 0, 25 4,10 56. a, b, c 는모두양수이고, a<5 이다. 이수들이 a 2 a 2b 2c =0,a +2b 2c +3=0 을만족할때, a, b, c 사이의대소관계를밝혀라. (1995 교육청경시 ) 풀이 a 2 a 2b 2c =0 (1) a +2b 2c +3=0 (2) (1) (2) : a 2 2a 4b 3=0 ) a 2 2a =4b +3> 3 ) a 2 2a 3 > 0 ) (a 3)(a +1)> 0 ) a>3: 이때 4b +3=a 2 2a <15(a <5) ) b<3: (1) + (2) : a 2 4c +3=0. a 2 4a +3 = (a 1)(a 3) > 0=a 2 4c +3 = 0 ) c>a: 종합하면 c>a>3 >b 이므로, c>a>b 의대소관계가있다.
2.1 대수중급문제 73 57. ½ ¾ 4 1 1 3 a = a 를만족하는유리수 a의개수를구하여라.( 단,0<a<1, [x] 는 x를넘지않는최대 a 정수 ) 풀이 a = q p 라고두고위의식에대입한다.( 이때p>q) 4 3 f p q bp q cg = q p ) 4 3q fp qb p q cg = q p 이때 (p qb p c) q 는 p를 q로나눈나머지이다. p>q이므로 p = kq + r(r <q) 라고하고대입하면 4r 3q = q p ) 4rp =3q2 ) 4r(kq + r) =3q 2 ) 4r 2 +4kqr 3q 2 =0) r = 2kq p 4k 2 q 2 +12q 2 = 4 q 2 ( k p k 2 +3): r 0이므로 r = q 2 ( k + p k 2 +3) 인데 r이자연수이려면 (k 2 +3) 이제곱수여야하고 (k +2) 2 > k 2 +3>k 2 이므로 k 2 +3=(k +1) 2 이다. 따라서 k =1이고이때r = q 2, p = kq + r = q + q 2 = 3q 2 이다. 즉 a로가능한값은 2=3 하나뿐이다. 답 :1개. 58. x i 들은 1 <x i < 1 이고 jx 1 j + jx 2 j + + jx nj =19+jx 1 + + x nj 을만족하는실수들이다. n 의가능한최소값은얼마인가? (AIME 1988-4) 풀이 %EEE lhs < n, so we need n 20. Taking x_i = alternately ±19/20 works for n = 20. 답 20 59. f 1 (x) =1 jxj 이고, n =1; 2; 3;::: 에대해 f n+1 (x) =1 jf n(x)j 로정의된다. 임의의자연수 m 에대해, f k (m) =0 이되는최소의자연수 k 를구하여라. ( 플란더즈예선 1998/1999 2 차변형 ) 풀이 jf k (x)j 의절대값이 1보다클때 f k+1 (x) =1 jf k (x)j의절대값은 jf k (x)j보다 1 작아진다. 임의의자연수 m이있을때f 1 (m) =1 jmj =1 m이고 f n (m) 에서 n =1; 2; 3;::: 로바뀌어갈수록 f n (m) 의절대값은 1씩작아진다. jf 1 (m)j = m 1이므로 jf m (m)j = m m =0이최초로 0이고, 그후1과 0이반복된다. 답 : k = m: 60. 다음식을계산하여라. ( 플란더즈예선 1999/2000 1차 ) µ 1+ 1 µ 1+ 1 µ 1+ 1 µ 1+ 1 2 3 1999 2000 풀이 3 2 4 3 2001 답 : 2 : 2000 1999 2001 2000 = 2001 2 : 61. 1 + 1 4 + 1 9 + 1 16 + 1 ¼2 + = 25 6 임을알고있다고할때,1+1 9 + 1 25 + 1 49 + 1 + 의값을구하여 81 라. ( 플란더즈예선 2003 2차 ) 풀이 1+ 1 9 + 1 25 + 1 49 + 1 81 + =1+( 1 3 )2 +( 1 5 )2 +( 1 7 )2 +( 1 9 )2 = f1+( 1 2 )2 +( 1 3 )2 +( 1 4 )2 +( 1 5 )2 + g f( 1 2 )2 +( 1 4 )2 +( 1 6 )2 + g = f1+( 1 2 )2 +( 1 3 )2 +( 1 4 )2 +( 1 5 )2 + g 1 4 f( 1 1 )2 +( 1 2 )2 +( 1 3 )2 + g = ¼2 6 1 4 ¼2 6 = ¼2 8 : 답 ¼ 2 =8 62. 정의역이자연수인함수 f(x) 가있다. f(1) = 1 이고모든자연수 m, n 에대해서 f(m + n) =f(m) + f(n) 2f(mn) 을만족시킬때 f(2005) 와 f(2006) 의값을차례로구하여라. (IT 꿈나무올림피아드 2006 1 차 ) 풀이 n =1 을대입하면 f(m +1)=f(m) +1 2f(m) =1 f(m). 따라서, f(1) = 1, f(2) = 0, f(3) = 1, f(4) = 0, :::; f(2005) = 1, f(2006) = 0.
74 대수 63. 정수의집합에서유리수의집합으로의두함수 f, g 가임의의정수 n 에대해다음조건을만족한다 : f(0) = 4, f(1) = 12, g(n) =f(n +1), f(n) =2g(n +1) 2g(n 1). (1) f(2), g(2) 의값을구하여라. (2) f(2n), g(2n) 을각각n의식으로나타내어라. (3) f(1995) + g(1995) 의값을구하여라. (1995 서울시 ) 풀이 (1) f(n) =2g(n +1) 2g(n 1) = 2f(n +2) 2f(n) ) 3f(n) =2f(n +2)) f(n +2)= 3 2 f(n) f(2) = 3 f(0) = 6: 2 g(2) = f(3) = 3 f(1) = 18: 2 (2) f(2n) =( 3 2 )n f(0) = ( 3 2 )n 4= 3n 2 n 2 g(2n) =f(2n +1)=( 3 2 )n f(1) = ( 3 2 )n 12 = 3n+1 2 n 2 : (3) f(1995) + g(1995) = g(1994) + f(1996) = ( 3 2 )997 12 + ( 3 2 )998 4= 3999 2 996 64. 다음등식을관찰하여라. 1 3 =1 2 3 =3+5 3 3 =7+9+11 4 3 =13+15+17+19. 위와같은방식으로 50 3 = m +(m +2)+(m +4)+ + n 이된다고할때,(m; n) 을구하여라. ( 플란더즈예선 2003 2 차 ) 풀이규칙을살펴보면 1부터홀수를늘어놓고앞에서부터각각한개, 두개, 세개씩묶어더하면세제곱수가되는규칙을알수있다. 그러므로 50 3 은 (1+2+3+ 49 + 1( 번째홀수부터 (1+2+3+ 49) + 50번째홀수까지의합이다. 1+2+3+ 49+1번째홀수 : (1+2+3+ 49 +1) 2 1 =(49 50=2+1) 2 1 =49 50+1 = 2451: 1+2+3+ 49 + 50번째홀수 :(51 50=2) 2 1 = 2550 1 = 2549: 답 (2451; 2549) 65. 임의의실수 a, b, c, d, e 에대해다음부등식을증명하여라. ( 몰도바 1998 최종 -y9-1) 증명다음식을전개하면준식과같다. a 2 + b 2 + c 2 + d 2 + e 2 a(b + c + d + e) ( 1 2 a b)2 +( 1 2 a c)2 +( 1 2 a d)2 +( 1 2 a e)2 0 따라서, 준식은성립하고, 등호는 1 2 a = b = c = d = e 일때성립한다. 주산술 - 기하평균부등식에서 1 4 a2 + b 2 2q 1 4 a2 b 2 = jabj ab 임등을이용해도된다. 산술 - 기하평균부등식은음이아닐경우에만적용되므로이와같이절대값으로풀어놓은후비교해야한다는점을주의할것. 별증 a 에대한식으로보고 (b; c; d; e 는상수로보고 ) 내림차순으로정리하자. a 2 (b + c + d + e)a +(b 2 + c 2 + d 2 + e 2 ) 0 이 a 에대한이차식이모든실수 a 에대해항상 0 이상이어야하므로그래프가 x- 축과접하거나그보다위쪽에놓여야한다. 즉, 이부등식이항상성립할필요충분조건은판별식 D 0 이다. D =(b + c + d + e) 2 4(b 2 + c 2 + d 2 + e 2 ) 인데, 코시 - 슈바르츠부등식에서 (1+1+1+1)(b 2 + c 2 + d 2 + e 2 ) (b + c + d + e) 2 이므로 D 0 은틀림없이성립하고, 따라서, 문제도성립한다.
2.1 대수중급문제 75 66. m 이임의의자연수이고 x = m 2 ; y =(m +1) 2 ; z =2(x + y +1) 이면다음여섯개의수는모두제곱수임을보여라. ( 셈본중등초급도전문제 3.1.1) (1) xy + x + y (2) yz + y + z (3) zx + z + x (4) xy + z (5) yz + x (6) zx + y 증명 (1) 은다음과같이확인된다. xy + x + y = fm(m +1)g 2 + m 2 +(m +1) 2 =(m 2 + m) 2 + m 2 +(m +1) 2 = m 4 +2m 3 + m 2 + m 2 + m 2 +2m +1 = m 4 +2m 3 +3m 2 +2m +1=(m 2 + m +1) 2 (2){(6) 도다m에대한식으로풀어보면다음과같다. (2) yz + y + z =4m 4 +12m 3 +21m 2 +18m +9=(2m 2 +3m +3) 2 (3) zx + z + x =4m 4 +4m 3 +9m 2 +4m +4=(2m 2 + m +2) 2 (4) xy + z = m 4 +2m 3 +5m 2 +4m +4=(m 2 + m +2) 2 (5) yz + x =4m 4 +12m 3 +17m 2 +12m +4=(2m 2 +3m +2) 2 (6) zx + y =4m 4 +4m 3 +5m 2 +2m +1=(2m 2 + m +1) 2 따라서, 모두제곱수이다. 주 (1) 을 ( p x p y ) 2 +( p xy 1) 2 1=( p xy +1) 2 등으로좀더요령있게정리할수도. 67. 1,2,3,:::; 1000 중에서적당한실수 x에대해 [2x]+[4x]+[6x]+[8x] 꼴로나타나는것은모두몇개인가? (AIME 1985-10) 풀이 %EEE Put f(x) = [2x] + [4x] + [6x] + [8x]. We find f(1/8) = 1, f(1/6) = 2, f(1/4) = 4, f(1/3) = 5, f(3/8) = 6, f(1/2) = 10, and f(x + n/2) = f(x) + 10n. So we realise 6 out of 10, or 600 out of 1000. 답 600 68. 수직선에서좌표가 10 인점 P (10) 을중심으로양쪽에대칭으로놓여있는 24 개의서로다른정수좌표를갖는점 P 1 (x 1 ), P 2 (x 2 ), P 3 (x 3 ), :::; P 24 (x 24 ) 에대하여 (jx 1 1j + jx 1 +1j)+(jx 2 1j + jx 2 +1j)+ +(jx 24 1j + jx 24 +1j) 의최소값을구하여라. ( 한국 2006 1 차 -J15) 풀이한점의좌표를k라하면P (10) 에대칭인점의좌표는 20 k가된다. 이렇게쌍으로묶어생각하면 k<10 으로둘수있다. 정수 n 6= 0에대해jn 1j + jn +1j =2jnj 이므로 (i) 1 k<10 이면 : 20 k 1 이므로 A =(jk 1j + jk +1j) +(j20 k 1j + j20 k +1j) = 2k +2(20 k) = 40. (ii) k =0이면 :20 k =20이므로 A =2+2 20 = 42. (iii) k 1 이면 :20 k 1 이므로 A = 2k +2(20 k) =40 4k. k 1 이므로 k가작아질수록 A의값은커지게된다. 즉 A 44 가된다. 즉, k가클수록a값이작고, 가장작은 24 =12 2 개를합하면 9 40+42+44+48=494 답 69. 가우스함수와사칙연산만으로, 0이면 1을, 아니면 0을돌려주는함수를구현해보아라. (ML프로포절 136-2) 1 풀이 1+a 2 (a 2 = a a 로구현한다.) 답은이외에도여러가지가있을수있다.
76 대수 70. 수열 1; 2; 4; 5; 10; 11; 22; 23; 46; 47;::: 는다음과같이만들어진것이다 :1 에 1 을더해 2 를얻고,2 를 2 배하여 4 를얻고 ;4 에 1 을더해 5 을얻고,5 를 2 배하여 10 을얻고 ; ::: ; 이렇게 1 을더하는것과 2 배하는것을계속하여수열을얻는다. 이수열의 100 번째항은 3 2 k 1 꼴이다. k 를구하여라. (1989 뉴욕주수학리그 ) 풀이 a 2 =2,a 2k+2 =2a 2k +1 이므로 a 2k+2 +1=2(a 2k +1)= =2 k (a 2 +1)=3 2 k 따라서, a 2k+2 =3 2 k 1 이다.2k +2=100 일때이므로 k =49 답 71. 임의의실수들의수열 A =(a 1 ;a 2 ;a 3 ;:::) 에대해, A 를수열 (a 2 a 1 ;a 3 a 2 ;a 4 a 3 ;:::) 로정의한다. ( A) 의모든항이 1 이고 a 19 = a 92 =0 일때, a 1 은얼마인가? (AIME 1992) 풀이 ( A) =(1; 1; 1; 1;:::) 이므로 A =(b; b +1;b+2;b+3;:::) 가된다. 그럼또 즉, A =(a; a + b; a + b +(b +1);a+ b +(b +1)+(b +2);:::) a n = a + b +(b +1)+ +(b +(n 2)) = a +(n 1)b + 이다. a 19 = a 92 =0 임에서 a +18b +17 9=0 a +91b +45 91 = 0 이고, 여기서 b 를소거하기위해두식에각각 91, 18 을곱하여빼면 73a +91 9 (17 90) = 0 (n 2)(n 1) 2 즉, a =91 9=819 답 72. 방정식 x 6 x 5 + x 4 x 3 + x 2 x + 3 =0 4 이실수해를갖지않음을보여라. ( 스웨덴 1986-1) 증명 1 준식 =(x 1 2 )2 x 4 + 3 4 (x 2 3 )2 x 2 + 2 3 (x 3 4 )2 + 3 8 3 8 > 0. 증명2 준식 = x(x 1)(x 4 + x 2 +1)+ 3 3. x 1 4 또는 x 0 이면 x(x 1) 0 이므로준식 4. 0 <x<1 에서는 jx(x 1)j = j(x 1 2 )2 1 4 j 1 4, x4 + x 2 +1< 3 이므로준식 > 1 4 3+ 3 4 =0. 73. a b c 와 a + b + c 1 을만족하는양수 a, b, c 가있다. a 2 +3b 2 +5c 2 1 임을보여라. (Towns 1989 봄 JO1) 증명 a 2 +3b 2 +5c 2 a 2 +(2ab + b 2 )+(c 2 +2ac +2bc) =(a + b + c) 2 1. 74. 다음방정식이무한히많은해를갖게되도록하는 a 의값을모두구하여라. jx aj + jx +3a 8j =4 ( 오클랜드 2001-4) 풀이 b = 3a +8이라하고 jx aj + jx bj =4의방정식으로생각하자. 또, 수직선에서 x, a, b의위치의점을각각x, A, B라하자. 그럼방정식은 XA + XB =4로다시쓸수있다. ² AB > 4 이면 XA + XB AB 이므로해가없다. ² AB < 4 이면선분 AB 내에서는 XA + XB = AB 이므로해가없고, 선분 AB의밖에서는이선분에서멀어질수록 XA + XB 가단조증가하므로해가양쪽에각각하나뿐이다. 즉, 해를유한개밖에갖지않는다. ² AB =4일때선분AB 내의모든점이 XA + XB = AB =4를만족하므로, 이것이무한히많은해를가질필요충분조건이된다. 그러므로, AB = ja bj = ja ( 3a +8)j =4를풀면j4a 8j =4,ja 2j =1, 따라서 답 a =1 또는 3
2.1 대수중급문제 77 주 y = jx aj + jx bj 의그래프를이용하여풀수도있고 (y = c 와무한히많은교점을갖는경우는 a와 b 사이의평평한선분이 y = c 가되는경우임 ), x a 와 x b 의부호에따라경우를나누어풀수도있다. 75. n n 표의각각의칸에실수가하나씩채워져있다. 임의의십자교차영역 ( 한행과한열의합집합 ) 의수를모두합하면항상 A 이상이다. 모든칸에적힌수의총합의가능한최소값은얼마인가? ( 독일 BW 1972 1 차 -1) 풀이모든칸에대해그칸을교차점으로포함하는십자교차영역을모두합해보자. 그럼 n 2 개의십자교차영역을합하므로그값은 n 2 A 이상이고, 또한각칸은그칸을지나는십자교차영역이 n+n 1개씩있으므로 (2n 1)S가된다 (S는모든칸에적힌수의총합). 즉 (2n 1)S n 2 A 이므로 S n2 A 2n 1. 실제로모든칸의수가 A 2n 1 이면문제의조건을충족하면서등호가성립하는경우가된다. 답 n2 A 2n 1 76. 다음을증명하여라. (Towns 1991 가을 JO3) 1 1 2+ 1 3+ 1 4+ + 1 1991 1 + 1 1+ 1 1+ 1 3+ 1 4+ + 1 1991 =1 1 증명 n = 1 3+ 1 4+ + 1 2007 이라하자. 그럼 준식 = 1 2+n + 1 1+ 1 1+n = 1 2+n + 1 2+n 1+n = 1 2+n + 1+n 2+n =1 로문제의식이성립함을알수있다. 77. n, m 은 m>n 인자연수이다. q n + p m + p q n 과 m + p n + p m 중어느것이더큰가? (Towns 1992봄 JA3 변형 ) 풀이둘다양수이므로둘을제곱한 A 2 = n + p m + p n 과 B 2 = m + p n + p m 을비교하면된다. B 2 A 2 =(m n)+( p n + p m p m + p n ) =(m n)+ (n + p m ) (m + p n ) p p p p n + m + m + n =(m n) m n p p m n X + (X := p n + p m + p m + p n ) X µ > (m n) 1 1 (* p m p n>0 이므로 ) X > 0 (X>1임이자명하고 m>n이므로 ) 로 B 2 >A 2 이다. 즉, 후자가항상더크다. 78. 양의실수 2 개가주어져있다. 두수의합이두수의곱보다작다면그합은 4 보다큼을증명하여라. (Towns 1990 가을 JO1) 증명 (a + b) 2 4ab > 4(a + b) 이므로 a + b>4.
78 대수 79. 1 < p 3 < 2 임을이용하여,6< 3 p3 < 7 임을증명하여라. (IMTS R10-2) 증명 0 < p 3 1 < 1 이고제곱하면 0 < 4 2 p 3 < 1, 즉 3 2 < p 3 < 2. 다시 0 < 2 p 3 < 1 2 이고제곱하면 0 < 7 4 p 3 < 1 4, 즉 27 16 < p 3 < 7 4. 이제 327=16 < 3 p3 < 3 7=4 이므로 6 16 < 3 27 과 3 7 < 7 4 을확인하면충분함. 전자는 2 16 < 3 11 과동치인데 2 16 =(2 3 ) 5 2 < (3 2 ) 5 3=3 11 로확인되고, 후자는 3 7 =(3 3 ) 2 3 < (4 7) 2 3 < 7 4 으로확인됨. 80. 방정식 x [x] =[ 1 x 2] 2 의모든실수해를구하여라. ( 몰도바 1996 최종-y8-6) 풀이 x는두가우스식의합이므로정수. 그럼좌변은 0. 그럼 0 1 x 2 < 1 2 이고 4 x<6. 정수인것은x =4; 5뿐이고각각대입해보면실제로성립함. 답 4, 5 81. 1 보다큰임의의자연수 n 에대해다음을증명하여라. ( 인도지역예선 2002-6) 1 2 < 1 n 2 +1 + 2 n 2 +2 + + n n 2 + n < 1 2 + 1 2n 증명분모를 n 2 이나 n 2 + n 으로통일해보면끝. 82. 1월에 K와 V 는학교에서 20번평가를받았고 20회씩점수를받았다. 각점수는 2에서 5까지의정수이다. K가 5점을받은횟수는 V 가 4점을받은횟수와같고, K가 4점을받은횟수는 V 가 3점을받은횟수와같으며, K가 3점을받은횟수는 V 가 2점을받은횟수와같고, K가 2점을받은횟수는 V 가 5점을받은횟수와같다. 두사람의평균점이같다면, K가 2점을받은횟수는몇번인가? (Towns 1988봄 JO1) 풀이 K가 5점,4점,3점,2점을받은횟수를각각 a, b, c, d라하자. 그럼 V 가 5점,4점,3점,2점을받은횟수는각각 d, a, b, c. 두사람의총점이같으므로 5a +4b +3c +2d =4a +3b +2c +5d, 여기서 a + b + c =3d, 즉 4d = a + b + c + d =20이고 d =5 답 83. 양의실수 a, b 에대해다음부등식을증명하여라. ( 러시아 1993 4 차 -y9-1) a 2 + ab + b 2 3(a + b 1) 증명 (a 1) 2 +(a 1)(b 1) + (b 1) 2 0 84. 포물선 y = x 2 에접하는모든포물선 y = x 2 + bx + c 의꼭지점의자취를구하여라. ( 러시아 1991 4 차 -y9-1) 풀이같다고놓고 D =0 이용하면 c = b2 8. 꼭지점은 ( b 2 ; b2 8 ). 답 y = 1 2 x2. 85. 방정식 ax 5 + bx 4 + c =0이서로다른세근을갖는다. 방정식 cx 5 + bx + a =0도서로다른세근을가짐을보여라. ( 러시아 1994 4차-y9-5) 증명 c 6= 0 이고, 그럼양변을 x 5 으로나눠보면끝. 86. 다음식을인수분해하여라. ( 방글라데시 2004 2 차 -1) (a + b) 2 (b + c) 2 (c + a) 2 + abcf2(a + b)(b + c)(c + a)+abcg 풀이 f(a + b)(b + c)(c + a)+abcg 2 =(ab + bc + ca) 2 (a + b + c) 2 87. a 0 = >0 이고 a n+1 = a n 1+a n 으로주어진수열 (a n) 의일반항을구하여라. ( 이탈리아 1986-2) 풀이역수를생각하면쉽... 1 = 1 +1 a n+1 a n 의등차수열이므로 1 = 1 +(n 1). a n 88. a, b, c, x, y 는 a 3 + ax + y =0,b 3 + bx + y =0,c 3 + cx + y =0 을만족하는다섯개의실수이다. a, b, c 가모두서로다르면그합이 0 임을보여라. ( 주니어발칸 1999-1)
2.1 대수중급문제 79 증명 f(t) =t 3 + xt + y =0 의세근이 a, b, c 이므로근과계수와의관계로끝. 89. 어떤정수 m; n 2 에대해서도, m p n 과 np m 중적어도하나는 3p 3 보다크지않음을증명하여라. ( 몰도바 1996 최종 -y10-5) 증명 k 2 일때 3 k k 3 임을이용. 귀류법을쓰면 m 3 n 3 > 3 n 3 m m 3 n 3 으로모순. 90. 다음등식을증명하여라. ( 헝가리 1941-1) (1 + x)(1 + x 2 )(1 + x 4 )(1 + x 8 ) (1 + x 2k 1 )=1+x + x 2 + x 3 + + x 2k 1 증명지수를 2 진법으로전개한다고생각. 91. f가다음함수방정식을만족한다고하자. µ 2x +29 2f(x)+3f =100x +80 x 2 f(3) 의값을구하여라. (IMTS R19-4) 풀이 g(x) = 2x+29 x 2 라하면g(g(x)) = x. g(3) = 35, g(35) = 3. x =3; 35 대입하고연립방정식풀면 f(3) = 1996. 92. 모든실수 x에대해다음을만족하는함수 f : R! R 를모두구하여라. x 2 f(x)+f(1 x) =2x x 4 ( 오스트리아 1985-6) 풀이 연립방정식으로풀면될테고... 93. 삼차방정식 x 3 3px 2 +3q 2 x r 3 =0 의세근이 p, q, r 일때 p = q = r 임을증명하여라. ( 인도지역예선 1999-4) 증명근과계수와의관계에서 (1) p + q + r =3p, (2)pq + qr + rp =3q 2,(3)pqr = r 3. 만일 r =0 이면 (1) 에서 q =2p 이고 (2) 에서 2p 2 =12p 2 이므로 p =0이되어성립. p, q 중에 0인것이있으면 (3) 에서 r =0이되어마찬가지. 이제 p; q; r 6= 0일때를보자.(3) 에서 pq = r 2 이고이것과 (1) 을 (2) 에대입하면 3pr =3q 2, 즉 pr = q 2. 이것을다시 (2) 에대입하면q(p + q + r) =3q 2, 즉 p + q + r =3q 이고 (1) 에의해p = q. 그럼 r도같게된다. 94. 실수 a, b, c, d 가부등식 (a + b + c + d) 2 4(a 2 + b 2 + c 2 + d 2 ) 을만족할때, 방정식 (a x)(b x)+ (c x)(d x) =0 에실수해가있음을증명하여라. ( 몰도바 1999 최종 -y8/9-2) 증명 D =(a + b + c + d) 2 8(ab + cd) (a + b + c + d) 2 4(a 2 + b 2 + c 2 + d 2 ). 95. 용량이각각 a리터와 b리터인두통이각각다른주스로채워져있다. 단, 두주스는리터당단가가서로다르고, a와 b는자연수이다. 각각의통으로부터동시에 999리터의주스를빼내서각각상대편의통에옮겨부었더니두통안의최종혼합물의리터당단가가같게되었다. 이런상황이가능한 a와 b 중에서 a + b를최소로하는것을찾아라. ( 몰도바 1998 최종-y7/8-4) (a 999)x +999y (b 999)y +999x 풀이 = 를정리하면 (a 999)(b 999) = 999 2. a + b = a b (a 999) + (b 999) + 2 999 2 p (a 999)(b 999) + 2 999 = 4 999. 등호는 a = b =2 999 일때. 96. 양수 x, y에대해다음부등식을증명하여라. ( 러시아 1995 4차-y9-1) x x 4 + y 2 + y y 4 + x 2 1 xy 증명양변에 xy를곱한후분자를기하-산술부등식으로전개하면끝. 곱하지않고그냥분모만산술-기하로풀어도마찬가지네.
80 대수 97. 다음식을증명하여라. ( 러시아 1994 최종 -y9-5) a 1 a 2(a 1 + a + a 2 2) a 3(a 2 + a + + a n 3) a 1(a n + a = a 2 1) a 1(a 1 + a + a 3 2) a 2(a 2 + a + + a 1 3) a n(a n + a 1) 풀이 1 왼쪽식에서오른쪽식을뺀후좀정리해보면금방... 풀이 2 부분분수로각변을전개해도금방. 98. 모든자연수 n 에대해다음을보여라. ( 아일랜드 1996-7) 2 1=2 4 1=4 8 1=8 (2 n ) 1=2n < 4 증명 S = 1 2 + 2 4 + 3 8 + + n 2 n < 2 를증명하면됨. 등비수열과등차수열이복합된꼴.2S S = 1+ 1 2 + 1 4 + + 1 2 n 1 n 2 n, S =2 1 2 n n 2 n < 2. 99. 다음연립방정식의모든실수해 x 1 ;x 2 ;:::;x 1999 을구하여라. ( 몰도바 1999 최종 -y7-3) 1+x 2 1 =2x 2 ; 1+x 2 2 =2x 3 ; :::; 1+x 2 1999 =2x 1 풀이다합하여좌변으로다넘기면 P (x i 1) 2 =0 이므로 x 1 = = x 1999 =1. 100. 다음연립방정식을만족하는실수순서쌍 (x; y; z) 를모두구하여라. ( 남미 1989-1) x + y z = 1 x 2 y 2 + z 2 =1 x 3 + y 3 + z 3 = 1 풀이세식을각각 (1), (2), (3) 이라할때 (1) 2 (2) 하면 (y + x)(y z) =0 으로거의끝. 답 ( 1; 1; 1), (1; 1; 1) 101. 자연수 a, b, c 에대해다음부등식을증명하여라. ( 몰도바 1999 최종 -y7-7) a 2 a 2 +2bc + b 2 b 2 +2ca + c 2 c 2 +2ab 1 증명 AM-GM 으로분모를 a 2 + b 2 + c 2 으로통일하면끝. 102. (x + p x 2 +1)(y + p y 2 +1)=1 이면 x + y =0 임을보여라. ( 러시아 1994 최종 -y9-1) 증명 (x + p x 2 +1)(y + p y 2 +1)=(x p x 2 +1)(y p y 2 +1) 을전개해서소거. 103. a 는 a 3 =6(a +1) 인양의실수이다. 방정식 x 2 + ax + a 2 6=0 는실수해를갖지않음을증명하여라. ( 주니어발칸 2007-1) 증명 a>2 p 2 만보이면됨. 104. 0이아닌임의의실수 x에대해x 8 x 5 1 x + 1 0 임을증명하여라. ( 아일랜드 1998-1) x4 증명그냥인수분해 (x 9 1)( 1 x 1 x 4 )= 1 x 4 (x 3 1) 2 (x 6 + x 3 +1) 0
2.1 대수중급문제 81 105. 서로다른실수 a 1 ;a 2 ;:::;a n 이주어졌을때, 다음함수의최소값을구하여라. ( 폴란드 1969 3 차 -2) y = jx a 1 j + jx a 2 j + + jx a n j; x 2 R 풀이 WLOG a 1 <a 2 < <a n 이라할때, 삼각부등식으로 y ja 1 a n j + ja 2 a n 1 j + + ja b n 2 c a d n 2 ej. 106. 다음연립방정식의모든실수해를구하여라. (Towns 1985 봄 JO3) (x + y) 3 = z (y + z) 3 = x (z + x) 3 = y 풀이 1 양변에 1=3 승을취하면 x + y = z 1=3 y + z = x 1=3 z + x = y 1=3 즉, x + y + z = z + z 1=3 = y + y 1=3 = x + x 1=3 함수 f(x) =x + x 1=3 은일대일대응이다. 따라서 x = y = z 이다. 대입해보면 8x 3 = x 에서 x =0 또는 8x 2 =1. 따라서,(x; y; z) =(0; 0; 0), (1= p 8; 1= p 8; 1= p 8) 풀이 2 첫번째식에서두번째식을빼보면 z x =(x z)f(x + y) 2 +(x + y)(y + z)+(y + z) 2 g 이되어서 x = z 가아니라면 1 =f(x + y) 2 +(x + y)(y + z)+(y + z) 2 g 가된다. 그런데우변은 x, y, z 가실수일때절대로 0 보다작아질수없다. 따라서 x = z. 결국 x = y = z 가되고, 이제앞의풀이처럼대입하여해를구하면된다. 풀이 3 개요 (1) 하나가 0 이면나머지도 0. (2) x>0 >y 이면 x + z<0 그럼 x + z<0 그럼 z<0 또 x<0 모순. (3) x; y; z > 0 이면 x y z 가정. 그럼 y x, 즉 x = y = z. 107. 임의의실수 a 에대해서다음부등식을증명하여라. ( 몰도바 2000 최종 -y8-7) (a 3 + a 2 +3) 2 > 4a 3 (a 1) 2 증명 1 전개해서몇개씩완전제곱의합으로나타내기. 증명2 a<0 일땐L 0 >R이라서당연.0 a 1 일땐L 9 > 4 >R로역시당연. a>1 일땐우변의 8a 4 을좌변으로이항해놓고, 좌변의항들중 a 6 +4a 4 4a 5 과 6a 3 > 4a 3 만을생각하면당연.
82 대수 108. 다음방정식의실수해 x 는둘뿐임을증명하여라. ( 아일랜드 2004-4) (x 1)(x 2)(x 3)(x 4)(x 5)(x 6) = 720 증명 720 = 6! 이므로 0과 7은해가되고나머지는 x에대한증가감소를생각하면더이상없음이금 방... 109. 두이차다항식 x 2 + ax + b 와 x 2 + bx + a 가모두삼차다항식 x 3 + px 2 + qx +72 의인수라고한다. 단, a, b, p, q 는상수이고, a 6= b 이다. 이삼차다항식의세근을구하여라. ( 뉴멕시코 1992 최종 -7) 풀이두이차식의공통근 가있어야하고, 두식을변변빼면 =1. 두식의나머지근은각각 b와 a. 근과계수와의관계로대충풀면세근은 1, 8, 9. 1 110. 함수 f(x) = 를모든실수x에대해g( x) =g(x), h( x) = h(x) 를만족하는두함수의 1 x + x2 합 f(x) =g(x) +h(x) 로나타내어라. ( 뉴멕시코 1995 1차-7) 풀이 f( x) =g(x) h(x) 이므로 g(x) = 1 2 x 1+x 2 +x 4. 1+x2 (f(x)+f( x)) = 1+x 2 +x 4, h(x) = 1 (f(x) f( x)) = 2 111. (1) 임의의실수 x 6= 1에대해f( x )=x x 1 를만족하는함수 f를구하여라. (2) 임의의실수 x 6= 1에대해g( x )=g(x) x 1 를만족하는함수 g가무한히많음을보여라.( 단, 하나가다른것의상수배가되는것은다시세지않는다.) ( 뉴멕시코 1998 최종-7) 증명 (1) y = x y x 1 로치환하면 f(y) =. (2) f(f(x)) = x y 1 임에서, f와항등함수 I를대칭적으로사용한함수는모두 g가됨. 예를들어 (f(x)i(x)) n 이나, f(x) n + I(x) n 등. 112. 0 x 1, 0 y 1 인실수x, y에대해항상 x 1+y + y 1 이성립함을증명하여라. 1+x ( 레닌그라드 1988-y7-1) 증명 x 1+y + y 1+x x x + y + y =1. x + y =0일때도당연. x + y 113. 임의로주어진실수상수 a<b<c 에대해, 다음등식이해를딱두개가짐을증명하여라. ( 레닌그라드 1988-y9-3) 1 x a + 1 x b + 1 x c =0 증명구간 (a; b), (b; c) 에각각한개씩해가있음. 중간값정리의개념을풀어서서술해줄것. 114. p 2+ p 3 과 p 2+ 3 p 3 을모두근으로갖는정계수다항식을하나찾아라. ( 스웨덴 1964-3) 풀이각근을갖는다항식을구한후곱하면되겠고, 그냥열심히 (x p 2) 2 =3등으로근호를차근차근벗겨가면됨. 답 (x 4 10x 2 +1)(x 6 6x 4 6x 3 +12x 2 36x +1) 115. 1이아닌실수x에대해 (1 + x + x 2 ) 2 < 3(1 + x 2 + x 4 ) 이성립함을증명하여라. ( 스웨덴 1971-1) 증명전개해서이항 / 소거한후 2로나누면준식은 x 4 x 3 x +1> 0 과동치이고, 인수분해하면 (x 1)(x 3 1) > 0 과동치. 이것은참. 별증 1+x 2 + x 4 =(1+x + x 2 )(1 x + x 2 ) 으로인수분해되고 1+x + x 2 > 0 이므로이것으로준식의양변을나눠주면 1+x + x 2 < 3(1 x + x 2 ) 과동치. 이것은정리하면 0 < (1 x) 2 과동치.
2.1 대수중급문제 83 116. 임의의자연수 n 과양의실수 r 에대해다음부등식이성립함을증명하여라. 1 1 r n(r1=n 1) r 1 ( 스웨덴 1974-2) 증명 r 1=n = a 로치환하자. a>1 이면 an 1 a 1 = an 1 + a n 2 + + a +1 >n임에서, a<1 일때도비슷하게하여오른쪽부등식이성립. 왼쪽부등식도비슷하게, a > 1 이면 1 a n an 1 a 1 = a 1 + a 2 + + a n <n임에서, a<1 일때도비슷하게하여성립. 별증 r; 1;:::;1 에산술-기하평균부등식을쓰면 r 1=n r+(n 1), n 정리하면 n(r 1=n 1) r 1 로오른쪽부등식이성립. 왼쪽부등식은 r (n+1)=n ;:::;r (n+1)=n ; 1( 총 n +1개항 ) 에산술-기하평균을사용하면됨. 117. 양수 a, b, c에대해다음부등식이성립함을보여라. ( 러시아 1990 4차-y10-1) p p p p ab(a + b)+ bc(b + c)+ ca(c + a) > (a + b)(b + c)(c + a) 증명한번제곱하여항들을적당히소거하고나면우변이 2abc만남고, 양변 2로나누면좌변의각항이우변의 abc보다큼. 혹은제곱하여소거한후좌변에남은세항중한항만으로도 2 p ab 2 c(a + b)(b + c) > 2 p ab 2 c(a +0)(0+c) =2abc 이므로끝. 118. a;b;:::;n이 3보다큰어떤소수로도나누어지지않는서로다른자연수들일때, 다음부등식을증명하여라. ( 헝가리 1922-3) 1 a + 1 b + + 1 n < 3 증명 1 a + 1 b + + 1 n < (1 + 1 2 + 1 2 2 + )(1 + 1 3 + 1 3 2 + )=2 3 2 =3 119. 연립방정식 x = a y 2, y = a x 2 이서로다른실수해 x, y 가존재하는실수 a 를모두구하여라. ( 스웨덴 1976-2) 풀이두식을변변빼면 x y = x 2 y 2, x y 로나누면 x + y =1. 즉 x, y 는둘다이차식 t 2 t +(1 a) =0 을만족. 판별식이양이어야하므로 1 > 4(1 a), 즉 a> 3 4. 120. cd =1 인양수 a, b, c, d 가주어져있다. 폐구간 [ab; (a + c)(b + d)] 에완전제곱수가적어도하나포함됨을증명하여라. ( 레닌그라드 1987-38) 증명완전제곱수가포함되지않는다면, n 2 <ab<(a + c)(b + d) < (n +1) 2 을만족하는음아닌정수 n이존재. 그런데,(n +1) 2 = n 2 +2n +1<ab+2 p ab +1=ab +2 p abcd + cd ab + ad + bc + cd = (a + c)(b + c) 로모순. 121. 다음연립방정식을풀어라. ( 스웨덴 1979-1) x 1 +2x 2 +3x 3 + +(n 1)x n 1 + nx n = n 2x 1 +3x 2 +4x 3 + + nx n 1 + x n = n 1 3x 1 +4x 2 +5x 3 + + x n 1 +2x n = n 2. (n 1)x 1 + nx 2 + x 3 + +(n 3)x n 1 +(n 2)x n =2 nx 1 + x 2 +2x 3 + +(n 2)x n 1 +(n 1)x n =1 풀이모든식을다더하면 x 1 +x 2 + +x n =1. 이웃한두식끼리빼면 x 1 +x 2 + +x n nx k = 1 (k =2;:::;n). 그럼 x k = 2 n (k =2;:::;n) 이고 x 1 = 2 n 1.
84 대수 122. x, y 는 x p x y 1 4 x + p x 를만족하는양의실수들이다. y p y x 1 4 y + p y 임을증명하여라. ( 스웨덴 1991-2) 증명 ( p x 1 2 )2 y ( p x + 1 2 )2 이므로 p x 1 2 py px + 1 2. 즉 p y 1 2 px py + 1 2. 123. x 4 2ax 2 + x + a 2 a =0 의근이모두실수가되도록하는실수 a 의값을모두구하여라. ( 인도지역예선 2000-7) 풀이 (x 2 + x a)(x 2 x (a 1)) 로인수분해됨. 답 a 3=4 124. u 와 v 는이차방정식 ax 2 + bx + c =0 의두근이다 ( 단, ac 6= 0). px 2 + qx +1=0 이 u=v 와 v=u 를두근으로갖는이차방정식이라면, q 는얼마인가? a, b, c 에대한식으로나타내어라. ( 플란더즈예선 1998/1999 2 차 ) 풀이 2 b2 ac 125. a, b, c가양의실수들일때 abc (a + b c)(b + c a)(c + a b) 임을증명하여라. ( 스웨덴 1982-2) 증명 (x; y; z) =(b + c a; c + a b; a + b c) 로치환하면준식은 (x + y)(y + z)(z + x) 8xyz 와동치.AM-GM. 126. R 은실수전체의집합, S 는 0 과 1 을제외한실수전체의집합이다. 임의의 x 2 S 에대해 f(x)+ 1 µ 1 2x f =1 1 x 을만족하는함수 f : S! R 를모두구하여라. ( 호주 2003-2) 풀이 x 대신에 x, 1 1 x, x 1 x 을각각대입하면, µ 1 f 1 x µ x 1 f x f(x)+ µ 1 1 2x f 1 x µ x 1 + 1 x f 2 + x =1 (1) =1 (2) x f(x) =1 (3) 2(x 1) (1) 2x (2) 를하면 (3) 1 x 2 +(4) 를하면 2xf(x) 1 x f 2 µ x 1 x =2x 1 (4) 7x 4 f(x) = 3 2 x 1 2 ) f(x) = 6x 2 7x 이것을다시문제의식에대입해보면성립하므로이것이해가된다. 127. 다음연립방정식을풀어라 : ab + c + d =3; bc+ d + a =5; cd+ a + b =2; da+ b + c =6 단, a, b, c, d는실수이다. ( 영국 2003/2004 1차-1) 풀이 답 2, 0, 0, 3
2.1 대수중급문제 85 128. 임의의 c>1 에대해, 두수 p c +1 p c 와 p c p c 1 중어느쪽이큰지결정하여라. ( 캐나다 1969-2) 풀이 부등호의자리를비워두고다음과같이정리해보자. p p p p c +1 c c c 1 () p c +1+ p c 1 2 p c () (c +1)+2 p c 2 1+(c 1) 4c () 2 p c 2 1 2c () c 2 1 c 2 따라서, 자리에들어갈부등호는 < 이고, p c p c 1 이항상크다. 별해무리식의유리화를이용해보자. p p 1 1 c +1 c = p p < p p = p c p c 1 c +1+ c c + c 1 이므로 p c p c 1 이더크다. 주 y = p x 의그래프가위로볼록함을이용해도된다. 129. 조건 2 x 4, x y, 3 y 5 를만족시키는실수 x, y 에대하여, 2+xy x + y 의최대값은? ( 한국 2005 1 차 -J4) 풀이 x = a 로일정할때 2+ay a + y = a(y + a) (a2 2) = a a2 2 y + a y + a 이므로이것은 y가최대일때 ( 즉, y =5일때 ) 최대값을갖는다. 따라서, y =5일때중에만최대값이있고, 그럼 5x +2 23 =5 x +5 x +5 도역시x가최대일때최대값을갖는다. 따라서, 최대값은 (x; y) =(5; 4) 일때 답 22 9. 130. 자연수 n 에대하여 p n 의정수부분을 a, 소수부분을 b 라할때 a 3 9ab + b 3 =0 이만족된다고한다. a n 의값을구하여라. (1991 서울시 ) 풀이 a 3 + b 3 =9ab 에서 a 3 이면좌변이크고 a 1 이면우변이커서해없음. a =2일때는 (b 4)(b 2 +4b 2) = 0 으로인수분해되고 n =6이답이됨. 131. a 1 >a 0, a 1 > 0, a r+2 =3a r+1 2a r (r =0; 1;:::;98) 을만족시키는정수 a 0 ;a 1 ;:::; a 100 이주어져있다. a 100 > 2 99 임을증명하여라. ( 소련 1962-3) 증명점화식을풀어 a r =(2 r 1)a 1 (2 r 2)a 0... 132. a, b, c 가 a 2 + b 2 + c 2 =1 을만족하는실수일때, 다음부등식을증명하여라. ( 헝가리 1910-1) 1 ab + bc + ca 1 2
86 대수 증명 0 (a + b + c) 2 =(a 2 + b 2 + c 2 )+2(ab + bc + ca) =1+2(ab + bc + ca) 로부터왼쪽부등식이얻어지고, 0 (a b) 2 +(b c) 2 +(c a) 2 =2(a 2 + b 2 + c 2 ) 2(ab + bc + ca) =2 2(ab + bc + ca) 로부터오른쪽부등식이얻어진다. 133. f(x)+xf(1 x) =1+x 를만족시키는함수 f : R! R 을구하여라. ( 통신강좌 1991-2-18) 풀이 f(x)+xf(1 x) =1+x (1) 에서 x 대신 1 x 를대입하면 f(1 x)+(1 x)f(x) =2 x (2) 를얻는다.(1) (2) x 를하면 f(x) x(1 x)f(x) =1+x x(2 x). 따라서 (x 2 x+1)f(x) =x 2 x+1. x 2 x +1> 0 이므로,(x 2 x +1) 로양변을나누면, f(x) =1,x 2 R. 134. 정의역이 0 이상 11 이하인실수들의집합이고함수값이항상 0 이상인함수 f 가다음두조건을만족시킬때, f(2) 2 + f(10) 2 의값을구하여라. (1) f(0) = 0, f(6) = 1; (2) x 0, 0 y; x + y 11 이면 f(x + y) = f(x)+f(y). ( 한국 2006 1차-J19) 1 f(x)f(y) 풀이 f(6) 에대한식을구하여 6 차식을적당히인수분해하면뭔가나옴. f(2) 2 +f(10) 2 =(2 p 3) 2 + (2 + p 3) 2 =14 답 135. a + b + c =6,a 2 + b 2 + c 2 = 18, abc =4일때, 다음식의값을구하여라. (1) (a + b) 2 +(b + c) 2 +(c + a) 2 (2) a 2 b 2 + b 2 c 2 + c 2 a 2 풀이 답 (1) 54 (2) 33 136. 이차방정식 x 2 (3m +1)x +8m =0 의두근의비가 2:3 일때, 정수 m 의값을구하여라. 풀이두근을2a, 3a라고하면근과계수의관계에의해서 2a +3a =3m +1, 6a 2 =8m 이다. 이를연립하면 8(5a 1) = 3(6a 2 ) 이고이를정리하면 (9a 2)(a 2) = 0 이다. 따라서 a는 2 혹은 2 9 이고 m이정수가되려면 a는 2가되어야한다. 그러면 m은 3이다. 137. a, b, c 를 a 2 + b 2 + c 2 6=0 인실수라하자. x, y, z 가 ax + by + cz =0,bx + cy + az =0,cx + ay + bz =0 을동시에만족하면 x = y = z 또는 x + y + z =0 이성립함을증명하여라. (1991 서울시 ) 증명세식을모두합하면 (a + b + c)(x + y + z) =0. x + y + z 6= 0이라면 a + b + c =0. a = b c 를세식에대입하면 b(x y) =c(z x), b(z x) =c(y z), b(y z) =c(x y). x, y, z 중어느둘이같은것이있으면모두같음. x 6= y 6= z 6= x 라하면이세식을모두곱하여b 3 = c 3, 즉 b = c 를얻음. 대칭적이므로 a = b = c 임을알수있고, 그럼 a = b = c =0이되어모순. 138. 1 2! + 2 3! + 3 4! + + 9 의값은얼마인가? 10! 풀이 telescoping 으로 1 1 10!
2.1 대수중급문제 87 139. 함수 f(x) 가다음두조건을만족할때, f(1999) 를구하여라. (i) f(3) = 5 (ii) f(x +2)= f(x) 1 f(x)+1 (1999 교육청경시 ) 풀이 5; 2 3 ; 1 5 ; 3 2 ; 5;::: 로 f(2n +1) 은주기수열. 답 1 5 140. 함수 f(x) =x 3 x 를생각하자. f(x + a) =f(x) 를만족하는 x 가존재한다면 jaj 2 임을증명하여라. ( 아일랜드 1988-7) 증명 (x + a) 3 (x + a) =x 3 x 를정리하면 (x + a) 3 x 3 =(x + a) x, a(3x 2 +3ax + a 2 )=a. a =0일땐틀림없이결론이성립하므로 a 6= 0이라하면 3x 2 +3ax + a 2 1=0. 해를가지므로, 판별식 D =9a 2 12(a 2 1) 0, 즉 3a 2 12. 141. 다음연립방정식이실수해를갖지않음을증명하여라. ( 레닌그라드 1989-y8-1) x + y + z =0 1 x + 1 y + 1 z =0 증명 1 x = 1 y 1 z = y z = x yz yz 이므로 x2 = yz 로 x, y, z는모두같은부호임을알수있다. 그럼합이 0이될수없다. 142. abc =1 이고 a + b + c> 1 a + 1 b + 1 c 인세양의실수a b c 가있다. a>1 >b임을보여라. ( 스웨덴 1995-4) 증명 a 1 이면 b; c 1 이고 3 a + b + c> 1 a + 1 b + 1 3 c 으로모순이므로 a>1. a + b + 1 ab = a + b + c > 1 a + 1 b + 1 c = 1 a + 1 b + ab 이므로 ab a b < 1 ab 1 a 1 b, 정리하면 (a 1)(b 1) < (1 1 a )(1 1 (a 1)(b 1) )=, b ab 즉 (a 1)(b 1)(1 1 ) < 0. b 1 ab 이면이것은성립하지않음. 143. 0이아닌세실수a, b, c가 abc =1과 a + b + c = 1 a + 1 b + 1 c 증명하여라. 을만족하면, a, b, c 중에 1 인것이있음을 ( 레닌그라드 1988-y8-1) 증명 a + b + c = ab + bc + ca 이므로 (a 1)(b 1)(c 1) = abc (ab + bc + ca)+(a + b + c) 1=0. 144. 실수에서정의되는실함수 f가모든실수x에대해다음의 (i), (ii) 를만족한다. (i) f(10 + x) =f(10 x) (ii) f(20 + x) = f(20 x) f는주기함수이며 f( x) = f(x) 임을증명하여라. ( 이탈리아 1997-2) 증명 f(x +40)=f(x) 등을뭐적당히하면얻어낼수있음. 셈PS 에도대칭성쪽에비슷한문제있었던것같은데. 145. 임의의양수 a, b 에대하여다음두조건이동치임을증명하여라 : (i) p a +1> p b. (ii) 임의의 x>1에대하여ax + x >b. ( 폴란드 1994/1995 1차-5) x 1 증명 (ii) 를이차식 f(x) 로전개한후축이1보다크면 D<0, 축이 1 이하이면 f(1) 0 임을이용함 ( 근데항상 f(1) = 1). 두경우를각각 (i) 과비교.
88 대수 146. x[x[x[x]]] = 88 을만족하는실수 x 를모두구하여라. 단,[x] 는 x 를넘지않는가장큰정수를나타낸다. ( 체코슬로바키아 1998-1) 풀이 f(x) = x[x[x[x]]] 는 ( 단조가아닌 ) 증가함수. f(3) = f( 3) = 81 이므로그근처에서찾아보자. x< 3 이면 [x] 4 이고,[x[x]] 12, [x[x[x]]] 37, x[x[x[x]]] > 111 이므로해가없다. x = 88 28 = 22 88 7 만해가됨.[x[x[x]]] = x < 88 3 =291 88 3 임에서 x = 27 ; 88 28 ; 88 29 만후보가되는데, 각각검산해보면 x = 88 28 = 22 7 만실제가능함. 147. (1) p x + p y + p xy = p x + p y + xy +2y p x 임을증명하여라. (2) p p 3+ 10 + 2 p p 3= 5+ p q 22 + 8 p p 22 + 2 15 3 p 22 임을보여라. ( 스페인 1990-1) 증명 (1) p y + xy +2y p x =(1+ p x) p q y. (2) 8 p p 22 + 2 15 3 p 22 = p p 3+ 5 p 22. 그다음은 p 3 소거하고양변제곱해서비교하면끝. 148. a 6= c 이고 p(x) =x 4 + ax 3 + bx 2 + cx +1, q(x) =x 4 + cx 3 + bx 2 + ax +1 이라하자. p(x) =0 과 q(x) =0 이두개의공통근을가질 a, b, c 의조건을구하고, 그공통근을구하여라. ( 스페인 2000-1) 풀이 p(x) q(x) =(a c)(x 3 x) =0. a 6= c 이므로 x 3 = x, x =0; 1; 1. p(0) 6= 0이므로 0은해가아니고, 따라서 1이두공통근일수밖에. 대입하면 a + b + c +2= a + b c +2=0, 즉 b = 2 이고 a + c =0. 149. 연립방정식 x 2 y 2 =0,(x k) 2 + y 2 =1 이딱두개의실근을가질 k 의조건을구하여라. 딱세개의실근을가질조건도구하여라. ( 브라질 1981-1) 풀이두번째식이 2x 2 2kx + k 2 1=0이되고, 이것은판별식을조사하면 k 2 > 2 일때는해가없고, k 2 =2일때는중근, k 2 < 2 일때는서로다른두실근. k 2 =2일때, 즉 k = p 2 일때를각각조사하면 y가 2개씩존재하므로두실근의경우가됨. k 2 < 2 일때는각각의 x에대해y = x 로 y가 2개씩존재하므로 x =0일때가아니면근은항상 4개가됨. x =0일때를조사하면세실근. 주그래프를그려보아서판단해도쉽다. 첫식이y = x 두직선이고두번째식은x-축위에중심을둔원이므로딱두개의실근을가질때는두직선에모두접할때. 150. x 3 + ax + b =0 이서로다른세실근을가지면 a<0 임을증명하여라. ( 브라질 1992-1) 증명세실근을 p; q; r 이라하면 p+q +r =0 이므로 a = pq +qr+rp = pq +r(p+q) =pq (p+q) 2 = (p 2 + pq + q 2 ) 0. a =0 일때는 p = q =0 일때이므로서로다른세실근이아님. 2.2 대수고급문제 1. abcd =1 인네양의실수 a, b, c, d 에대해,(1+a)(1 + b)(1 + c)(1 + d) 16 임을증명하여라. ( 인도지역예선 1993-6) 증명산술 - 기하. 2. n 은 n 2 인자연수이다. 다음을증명하여라. ( 캐나다 1998-3) µ 1 1+ 1 n +1 3 + + 1 > 1 µ 1 2n 1 n 2 + 1 4 + + 1 2n
2.2 대수고급문제 89 증명양변에 n +1을곱한후, 다음과같이양변에같은값을더하거나빼면서동치변형을해보자. ( 준식 ) () 1+ 1 3 + + 1 µ 2n 1 > 1+ 1 µ 1 n 2 + 1 4 + + 1 2n () 1+ 1 2 + 1 3 + + 1 µ 2n > 2+ 1 µ 1 n 2 + 1 4 + + 1 2n µ () 1+ 1 2 + + 1 µ 1+ 1 2n 2 + + 1 > 1 µ 1 n n 2 + 1 4 + + 1 2n 1 () n +1 + 1 n +2 + + 1 2n > 1 2n + 1 4n + + 1 2n 2 여기서 n + k<nk+ nk =2nk 이므로 1 n+k > 1 2nk 가되어이부등식은성립한다. 3. a, b, c 를음이아닌실수라하자. 이때 a + b + c abc 이면 a 2 + b 2 + c 2 abc 임을증명하여라. ( 아일랜드 1997-4) 증명 a 2 + b 2 + c 2 <abc라고하면 a 2 <abc이므로 a<bc, 비슷하게얻은세식을모두합하면 (abc )a + b + c<bc+ ca + ab( a 2 + b 2 + c 2 ) 으로모순. 4. 만일 n개의자연수 x 1 ;x 2 ;:::;x n 이있어다음n개의방정식 2x 1 x 2 =1 x k 1 +2x k x k+1 =1 (k =2; 3;:::;n 1) x n 1 +2x n =1 을만족한다면, n 은짝수임을증명하여라. ( 스웨덴 1983-3) 증명 점화식을열심히풀면 을얻을수있다. 여기서 x 1 = n 2 x n = nx 1 n(n 1) 2 = n x 1 이므로 n 은짝수. 5. 다음식을간단히하여라. [m ]n 단, m 과 n 은양의정수이며, 는 n 보다큰임의의무리수이다. 풀이 는무리수이므로 m = k + e(0 <e<1)(k는정수 ) 로표현이가능하다. 따라서 [m ]n = nk이다. m = m = k + e 이고 [m ]n = nk 이다. 이때n< 이므로 따라서 이므로 이고 이다. [m ]n = nk < nk + ne [m ]n < nk +1(e<1; n < 이므로 ne < a) <mn< [m ]n mn 1 < [m ]n [m ]n +1 <mn = mn 1
90 대수 6. 함수 f :(0; 1)! R 가성질 f(x) =f µ 1 을만족한다. 모든 x>0 에대해서 x u Ã! x + 1 x = f(x) 2 를만족하는함수 u :[1; 1)! R 가존재함을증명하여라. 풀이 t = 1 µ x + 1, x>0라하면 2 x ) x 2 2tx +1=0 ) x = t p t 2 1 ) u(t) =f(t p t 2 1) for x 0. cf) f(t + p µ t 2 1 1) = f p t+ t 2 1 = f(t p t 2 1) ( 통신강좌 1989-C14) 7. a, b, c; p, q, r이실수이고, 임의의실수 x에대해부등식 ax 2 +2bx + c 0 와 px 2 +2qx + r 0 이성립할때, 임의의실수 x에대해다음부등식이성립함을증명하여라. apx 2 + bqx + cr 0 ( 헝가리 1918-3) 증명 ax 2 +2bx + c 0 이므로판별식 D = b 2 ac 0 이다. 마찬가지로 q 2 pr 0 이다. 즉, 0 b 2 ac; 0 q 2 pr 이다. 그러므로 (bq) 2 acpr 이다. apx 2 +bqx+cr 의판별식 D 0 =(bq) 2 4apcr 을생각하자. apcr 0 이므로 (bq) 2 4apcr (bq) 2 apcr 이고 (bq) 2 acpr 에서 D 0 0 이다. 판별식이 0 보다작거나같으므로모든 x 에대해 apx 2 +bqx+cr 0 이다. 8. 방정식 4x 2 40[x]+51=0 의모든실근을구하여라. 단,[x] 는 x 보다작거나같은가장큰정수를나타낸다. ( 캐나다 1999-1) 풀이 x 1 < [x] x 이므로 4x 2 40x +51 4x 2 40[x]+51< 4x 2 40x + 91. 4x 2 40x +51 0 < 4x 2 40x +91 (2x 3)(2x 17) 0 < (2x 7)(2x 13) 3 2 x< 7 2 또는 13 2 <x 17 2 따라서, 가능한 [x] 는 1, 2, 3, 6, 7, 8뿐이다. ² [x] =1일때 :4x 2 +11=0으로곤란. ² [x] =2일때 :4x 2 29 = 0, 여기서 [x] =2를만족하는것만구하면 x = ² [x] =3일때 :4x 2 69 = 0, 여기서 [x] =3을만족하는것은없음. ² [x] =6일때 :4x 2 189 = 0, 여기서 [x] =6을만족하는것만구하면 x = ² [x] =7일때 :4x 2 p 229 = 0, 만족하는해는 x = 229. 2 ² [x] =8일때 :4x 2 p 269 = 0, 만족하는해는 x = 269. 2 p p p 따라서, 구하는모든실근은 x = 29 2, 189, 229, 2 2 p 29 2. p 189 2. p 269 2 답
2.2 대수고급문제 91 9. 모든실수 a, b, c, d 에대하여다음의부등식을증명하여라. ( 한국 2006-J6) a + b + c + d (1 + a 2 )(1 + b 2 )(1 + c 2 )(1 + d 2 ) < 1 증명 ( 서울당산서중 2학년박민재 ) A.M-G.M 부등식에의해서어떤양수 x에대해서 r x =2 x 2 1 4 x2 + 1 4 ( ) 가성립한다. 이로부터 준식 = a + b + c + d (1 + a 2 )(1 + b 2 )(1 + c 2 )(1 + d 2 ) jaj + jbj + jcj + jdj (1 + a 2 )(1 + b 2 )(1 + c 2 )(1 + d 2 ) a 2 + b 2 + c 2 + d 2 +1 (1 + a 2 )(1 + b 2 )(1 + c 2 )(1 + d 2 ) a2 + b 2 + c 2 + d 2 +1 a 2 + b 2 + c 2 + d 2 +1 =1 가된다. 마지막부등식은분모를전개해보면쉽게알수있고, 마지막부등식의등호는 a, b, c, d 중에 3 개이상이 0 일때성립한다. 그러나, 그경우 ( ) 에의한두번째부등식이성립하지않으므로결국원래부등식의등호는성립하지않는다. 10. a, b, c 는 a + b + c =1 을만족하는양의실수들이다. 다음을증명하여라. (1 + a)(1 + b)(1 + c) 8(1 a)(1 b)(1 c) ( 러시아 1991 4 차 -y11-3) 관찰제한식 a + b + c =1을무효화시키기위해동차식으로변형해주고싶은것이인지상정. 그래서,1+a =(a + b + c)+a, 1 a =(a + b + c) a 등으로대체하면준식은 (a + b + c + a)(b + c + a + b)(c + a + b + c) 8(b + c)(c + a)(a + b) 가된다. 이제 a, b, c 를똑같이 k 배를해줘도이부등식은똑같은식이되므로더이상 a + b + c =1 의제한식은의미를갖지못하고없는것이나마찬가지이다. 이식은또잘관찰하면 b + c = X, c + a = Y, a + b = Z 로치환하고싶은유혹을느낄것이다. 유혹에는넘어가주는것이또한인지상정. 그럼 (Y + Z)(Z + X)(X + Y ) 8XY Z 의식이된다. 이것은 Y + Z 2 p YZ 등의 A.M G.M 부등식으로바로풀수있음이이제눈에띌것이다. 그렇게관찰된최종적인구조를보니굳이치환을하고말고할것이없었다는생각이든다. 그래서, 증명만다시정리한다면다음과같이하면되겠다. 증명 a + b + c =1 이므로 1+a =(1 b)+(1 c) 1+b =(1 c)+(1 a) 1+c =(1 a)+(1 b) 모든항이다양수이므로, 여기에 A.M G.M 부등식을적용하여변변곱하면 (1 + a)(1 + b)(1 + c) =[(1 b)+(1 c)][(1 c)+(1 a)][(1 a)+(1 b)] 2 p (1 b)(1 c) 2 p (1 c)(1 a) 2 p (1 a)(1 b) =8(1 a)(1 b)(1 c) 따라서, 준식이증명되었다. 등호는 1 a =1 b =1 c 일때, 즉 a = b = c = 1 3 일때성립.
92 대수 11. 다음식을증명하여라. [x]+[y]+[x + y] [2x]+[2y] 증명 x =[x]+a, y =[y]+b일때0 a b< 이라두자. 그럼 2b a + b 이다. [2x]+[2y] = [2[x]+2a]+[2[y]+2b] =2[x]+2[a]+2[y]+2[b] 2[x]+2[y]+[2b] 2[x]+2[y]+[a + b] = [x]+[y]+[[x]+[y]+a + b] =[x]+[y]+[x + y] 이다. 12. az 3 + bz 2 + cz + d =0 의세근은모두실수부가음수라고한다. ab > 0 이고 bc > ad > 0 임을보여라. ( 아일랜드 1992-8) 주세근을모두음의실근이거나, 아니면세근중하나가음근 ( p이라하자 ) 이고나머지둘은실수부가음수인켤레허근 ( q ir라하자 ). 후자의경우세근의합은 p 2q 로음수, 세근의곱은 p(q 2 +r 2 ) 으로음수, 세근을둘씩곱해합한값은 p( 2q)+(q 2 +r 2 ) 으로양수. 전자의경우도마찬가지로, 세근의합, 세근의곱은항상음수, 세근을둘씩곱해합한값은항상양수이다. 즉, b a ; c a ; d a > 0 이고, a, b, c, d는모두같은부호. 이제 bc > ad 만보이면됨. b a c a > d a 와동치. 세근을,, 라하면 ( + + )( + + ) < 와동치. 넘겨서 ( + )( + )( + ) < 0 의인수분해식과동치가됨. 세근모두음근일때는당연히성립하고, 켤례허근이있을때는 ((p + q) 2 + r 2 )( 2q) < 0 로역시성립. 13. 실수의수열 u 1 ;u 2 ;::: 는 1 u 1 =1; u n = u 1 + + u n 1 (n>1) 의점화식을만족한다. u 1 + u 2 + + u N > 1989 가되는양의정수 N 이존재함을증명하여라. ( 호주 1989-3) 증명개요귀류법 u 1 + + u n 1989 이면 u n+1 1 1989. ) N 19892. 별증 ( 대전전민중 3 학년정명진 ) 1 u i > 0 이므로 u 1 + + u n 1 (n 1), 즉 u n 1 n 1. u 1 + + u N 1+ + 1 N 1!1. 14. a 1, a 2, :::; a n 을음이아닌실수들이라하자. 모든가능한짝의곱 a i a j (i<j) 들을모두합한값을 M 으로정의하자. 즉, M = a 1 (a 2 + a 3 + + a n )+a 2 (a 3 + a 4 + + a n )+ + a n 1 a n 이다. a 1, a 2, :::; a n 중적어도하나는그제곱이 2M n(n 1) 을넘지않음을보여라. ( 캐나다 1972-2) 증명 a i 들중가장작은것을 a 라하자. 그럼 M = X a i a j X ³ n a 2 = a 2 2 i<j i<j 따라서, a 2 2M n(n 1) 이다. 15. 관계식 x 2 + y 2 =2 를만족시키는실수 x, y 에대하여, 15x2 2 +4xy 의최대값을구하여라. ( 한국 2006 1차-J11)
2.2 대수고급문제 93 풀이 (KAIST 05 학번이은정 ) y = p 2 x 2 을 k = 15 2 x2 +4xy 에대입하면 2k 15x 2 = 8x p 2 x 2 4k 2 60kx 2 + 225x 4 =64x 2 (2 x 2 ) 289x 4 2(30k +64)x 2 +4k 2 =0 이식을 x 2 에대한이차방정식으로볼때, 실수해를가져야하므로판별식이 0 이상이다. 즉, D 0 =(30k +64) 2 289 4k 2 =(30k +64) 2 (34k) 2 =(30k +64+34k)(30k +64 34k) =64(k +1) ( 4)(k 16) 0 따라서, 1 k 16 이다. x 2 = 544 (< 2) 289 일때실제로 k =16이성립하므로, 최대값은 답 16 별해1 ( 김해장도한 ) G.M A.M 부등식에의해 15 2 x2 +4xy 15 2 x2 + 1 2 (x2 +(4y) 2 )=8(x 2 + y 2 )=16 이다. 등호가성립하는경우는 x 2 =4y 2 = 2 5 답 이고 xy > 0 일때로존재한다. 따라서, 최대값은 16 별해2 ( 광주동성고 3학년방재혁 ) x 2 + y 2 =2이므로, x = p 2cosµ, y = p 2sinµ 로치환하자. 준식 = 15 2cos2 µ +4 p 2cosµ p 2sinµ 2 =15cos 2 µ +8sinµ cos µ 15(1 + cos 2µ) = +4sin2µ 2 = 15 2 + 15 2 cos 2µ + 8 sin 2µ 2 s = 15 2 + 8 2 +15 2 sin(2µ + ) (tan = 8 4 15 ) = 15 2 + 17 2 sin(2µ + ) 16 즉 2µ + =90 ± 일때, 최대값 16을가진다. 16. 모든자연수 n에대해다음부등식을증명하여라. ( 플란더즈 1986-2) µ n +1 n n! 2 힌트 ((n +1)=2+ )((n +1)=2 ) ((n +1)=2) 2 만확인하면충분 17. 실수 a, b, x, y 가 ax + by =3,ax 2 + by 2 =7,ax 3 + by 3 =16,ax 4 + by 4 =42 를만족한다. ax 5 + by 5 의값을구하여라. (AIME 1990-15) 풀이 %EEE This needs a trick: (ax^{n+1 + by^{n+1)(x+y) - (ax^n + by^n)xy = ax^{n+2 + by^{n+2. Hence 7(x+y) - 3xy = 16, 16(x+y) - 7xy = 42. Solving x+y = -14, xy = -38. Applying again: ax^5 + by^5 = 42(x+y) - 16xy = 20. 답 20
94 대수 18. 어떤자연수를 3x 2 + y 2 꼴로나타낼수있으면, 이수는u 2 + uv + v 2 꼴로도나타낼수있음을보여라. 또한, 그역도성립함을보여라. 단, x, y, u, v는음아닌정수이다. (IMTS R4-3) 풀이 u = x + y, v = x y 라고하면, 3x 2 + y 2 = u 2 + uv + v 2 이된다. 즉, x; y가정수이면 3x 2 + y 2 = u 2 + uv + v 2 인 u; v가존재한다. 역방향에서 u, v가짝수일때는위와같이하면되고, u, v의홀짝이다를때는 WLOG u = 짝수 =2w 라하면u 2 + uv + v 2 =4w 2 +2wv + v 2 =3w 2 +(w + v) 2. 19. b<c<d 일때다음부등식이임의의 a 에대해성립함을증명하여라. ( 아벨콘테스트 1993 결선 2) (a + b + c + d) 2 > 8(ac + bd) 풀이 (a + b + c + d) 2 8(ac + bd) 는 a 에관한이차식이다. 이것을 a 에관해묶으면 a 2 +2(b 3c + d)a +(b + c + d) 2 8bd 이된다. 판별식을살펴보면, D=4 = (b 3c + d) 2 (b + c + d) 2 +8bd = (b 3c + d + b + c + d)(b 3c + d b c d)+8bd = (2b 2c +2d)( 4c)+8bd = 8(c 2 bc + bd cd) =8(c b)(c d) < 0 이므로식은항상양수가된다. 그러므로모든 a에대해 (a + b + c + d) 2 > 8(ac + bd) 이다. 20. 모든실수 x 에대해 f(x) =f(398 x) =f(2158 x) =f(3214 x) 가성립한다. f(0);f(1);f(2);:::;f(999) 에나타나는서로다른실수는최대몇개인가? (AIME 2000 1 차 -12) 풀이 %EEE From the first two, f(x) = f(1760+x). From the second two, f(x) = f(1056+x). gcd(1056, 1760) = 352, so f(x) = f(x+352). Then f(x) = f(398-x) gives f(x) = f(46-x). Conversely, it is easy to check that f(x) = f(46-x) and f(x) = f(352+x) imply the relations given. Evidently, f(x) = f(352+x) means the values repeat after any block of 352. Take that block centered on x = 23. We can choose separately the values at x = 23-176, 23-175,..., 23-1, 23. Then the reflection determines the values at 23+1, 23+2,..., 23+176. The period then determines all other values. So we get at most 177 different values. If we take them all different, they certainly appear in x = 0, 1,..., 999, which contains two complete periods. 답 177 21. 다음을만족하는실수 x 가존재하는가장큰자연수 n 을구하여라 : 2 1 <x 1 + x 2 < 2 2 2 2 <x 2 + x 3 < 2 3. 2 n <x n + x n+1 < 2 n+1 ( 아일랜드 1990-4) 풀이 x 2 + x 4=0 은만족하는 1 과 2 사이의근을 라하면 1 <x< <1:6. x 4 + x 5 < 4 + 5 =4 3 =4 (4 ) =16 4(4 ) =20 16 < 2 4 이므로 n 4 이면성립하지않는다. 한편, x =1:5 를대입하면 n =3 일때잘성립한다.
2.2 대수고급문제 95 22. 모든자연수 k 는 k = a 1 1! + a 2 2! + + a m m! 꼴로승수전개할수있고그방법은유일하다. 단,0< a m <m+1, 0 a i <i+1. 16! 32! + 48! 64! + + 1968! 1984! + 2000! = a 1 1! + a 2 2! + + a nn! 이승수전개한것일때, a 1 a 2 + a 3 a 4 + +( 1) j+1 a j 의값을구하여라. (AIME 2000 2 차 -14) 풀이 %EEE We have (n+1)! = n n! + n! = n n! + (n-1)(n-1)! + (n-1)! etc. Hence 2000! - 1984! = 1999 1999! + 1998 1998! +... + 1984 1984!, and 1999-1998 + 1997-1996 +... + 1985-1984 = 8. Thus the 62 pairs (2000! - 1984!) + (1968! - 1952!) +... + (48! - 32!) give 8 62 = 496. Finally + 1 16! gives -1. Total 495. 답 495 23. 두실수 x, y 가 2x 2 15x +26 y x 2 8x +20 을만족시킬때,4(x 2 + y) 의최대값과최소값의합을구하여라. ( 한국 2006 1 차 -J7) 풀이 ( 한국과학영재학교 1학년김민기 ) 우선 2x 2 15x +26 x 2 8x +20이어야한다. x 2 7x +6 0, (x 6)(x 1) 0, 즉 1 x 6 이다. 이제준식을변형하면 3x 2 15x +26 x 2 + y 2x 2 8x +20 4(3x 2 +5x +26) 4(x 2 + y) 8(x 2 4x +10) 이고, 이식은원래의식과동치이다. 여기서좌변과우변은각각 L =12x 2 +20x +104 =12(x 5 2 )2 +29 29 ( 최소 ) R =8(x 2) 2 +48 176 (x =6일때최대 ) 이된다. 실제로등호가성립하는경우를구하면 (x; y) =( 5 2 ; 1), (6; 8) 일때4(x2 + y) = 29, 176이되므로이것이틀림없는최소값과최대값이다. 따라서, 구하는합은 29 + 176 = 205 답 주 2x 2 15x +26 x 2 8x +20 을만족하는영역의그래프를그려풀수도있다. 24. bxc는내림기호이고, x보다작거나같은가장큰정수를뜻한다. 비슷하게, dxe은올림기호이고, x보다크거나같은가장작은정수를뜻한다. x는 1 x 100 인실수일때, bxcdxe = x 2 이성립하는 x 는모두몇개인가? (IT 꿈나무올림피아드 2006 1 차 ) 풀이우선, x가정수일때는 bxc = dxe = x 이므로항상성립한다. x가정수가아닐때를보자. 좌변이정수이므로우변 x 2 도정수이다. x = p m 이라하고, n< p m<n+1이라하자 (m, n은정수 ). 그럼 bxcdxe = n(n +1) 이므로 m = n(n +1) 이다. 즉, x = p 1 2, p 2 3, :::; p 99 100 등 99개의값에서성립한다. x가정수일때는 x =1; 2;:::;100 등 100개의값이있으므로, 모두 100 + 99 = 199( 개 ). 25. 다항식 f 와그계수조건이다음과같이주어져있다. f(x) =a 0 + a 1 x + a 2 x 2 + + a n x n ; 0 a i a 0 (i =1; 2;:::;n) b 0 ;b 1 ; ;b 2n 을다항식 (f(x)) 2 의계수, 즉 (f(x)) 2 =(a 0 + a 1 x + a 2 x 2 + + a n x n ) 2 = b 0 + b 1 x + b 2 x 2 + + b n+1 x n+1 + + b 2n x 2n 라고하자. 이때, 다음을증명하여라. ( 캐나다 1974-3) b n+1 1 2 (f(1))2
96 대수 증명 b n+1 은 x n+1 의계수이므로 b n+1 = a 1 a n + a 2 a n 1 + + a na 1 주어진조건에서 0 a i a 0 이므로 b n+1 a 0 (a 1 + a 2 + + a n) 이것과비교하면 f(1) 2 =(a 0 + a 1 + + a n ) 2 = a 2 0 +2a 0 (a 1 + + a n )+(a 1 + + a n ) 2 2a 0 (a 1 + + a n) 따라서, f(1) 2 2b n+1 임을알수있다. 26. u 는 0 <u<1 이고, u 1 =1+u; u 2 = 1 u 1 + u; :::; u n+1 = 1 u n + u; n 1 로정의된다. 모든 n =1; 2; 3;::: 에대해 u n > 1 임을보여라. ( 캐나다 1977-6) 증명우선 u 1 > 1, u 2 > 1 임을확인할수있다 ( 산술 - 기하평균부등식에서 u 2 = 1 1+u +(1+u) 1 2 1=1 이고등호는성립하지않음 ). n 1 과 n 일때성립한다고가정하면 u n+1 = 1 u n + u = 1 u n + u n 1 u n 1 2 1 u n 1 > 1 로 n +1 일때도성립하므로, 수학적귀납법으로증명되었다. 별증수학적귀납법으로다음을증명하자. 1 <u n 1+u n =1 일때는자명하고, n = k 일때 1 <u k 1+u 이성립한다고가정하면, 1 < 1+u + u2 1+u = 1 1+u + u 1 u k + u(= u k+1 ) < 1+u 로 n = k +1 일때도성립한다. 주 u k >u k+1 이면 1 u k < 1 u k+1 이므로 u k+1 <u k+2 가된다. 또그럼, 다시비슷하게 u k+2 >u k+3 이된다. 이렇게 n 이증가함에따라 u n 은증가와감소를반복하는수열이다. 그리고, x = 1 x + u 가되는한근 (> 1) 로수렴할것을예상할수있다. 실제로, 임을어렵지않게증명할수있다. ju n+1 j < ju 1 j n! 0 27. 이차방정식 (bc 1)x 2 +(a b + c abc)x + ab 1=0 이정수근을갖도록하는한자리의양의정수 a, b, c 에대하여 100a +10b + c 의최대값을구하여라. ( 한국 2005 1 차 -S17)
2.2 대수고급문제 97 풀이 ( 제주과학고 2 학년강진호, 수정됨 ) 정수계수이차방정식이므로정수근을갖는다면판별식이완전제곱수라야한다. D =(a b + c abc) 2 4(bc 1)(ab 1) = a 2 b 2 c 2 2abc(a + b + c)+(a 2 + b 2 + c 2 )+2(ab + bc + ca) 4 =(abc) 2 2(abc)(a + b + c)+(a + b + c) 2 4 =(a + b + c abc) 2 4=m 2 즉,(a + b + c abc) 2 m 2 =4 이다. 완전제곱수들 0, 1, 4, 9, 16, ::: 의분포를관찰하면두완전제곱수의차가 4 가되는경우는 4 0 일때뿐이다. 따라서, m =0 이고 a + b + c abc = 2 ( ) 이다. 그럼 a(bc 1) = b + c 2, 즉 bc 1 j b + c 2 이다. b + c 2=0 일때는 bc 1=0 이기도하므로, bc 1 b + c 2 이항상성립한다. 즉, 이다.( ) 은대칭식이므로, 마찬가지로 (b 1)(c 1) 4 (a 1)(b 1) 4; (a 1)(c 1) 4 이기도하다. a>5 이면 b = c =1 이어야하므로문제의식에서최고차항이 0 이되어이차방정식이아니므로곤란하다. a =5 일때는 b 1;c 1 1 이고, 이경우 (a; b; c) =(5; 2; 1) 또는 (5; 1; 2) 일때 ( ) 이만족된다.(a; b; c) =(5; 2; 1) 일때문제의방정식은 x 2 6x +9=0 으로 x =3 의정수근을갖는다. 따라서, 100a +10b + c 의최대값은 521 이다. 28. 다음을증명하여라. ( 인도지역예선 1992-6) 1 < 1 1001 + 1 1002 + 1 1003 + + 1 3001 < 1 1 3 풀이 1 2001 < 1 µ 1 2 2000 + 1 < < 1 µ 1 2002 2 1001 + 1 < 4=3 3001 2001. 주 y = 1 x 의막대그래프에서넓이를적당히비교하는것을생각한것과동치인식임. µ c 29. a, b, c는실수이고 abc > 0 이다.3 ab + a bc + b µ 1 +4 ca a + 1 b + 1 > 0 임을증명하여라. c (1998 교육청경시 ) 풀이 abc > 0 이므로왼쪽식에 abc 를곱한후 0 과비교하면된다. abc 를곱하면 3(a 2 + b 2 + c 2 )+4(ab + bc + ca) =2(a + b + c) 2 +(a 2 + b 2 + c 2 ) > 0 이므로준식이성립한다. 30. 집합 S 에서연산 를다음의법칙이만족하도록정의한다. x (x y) =y; (x; y 2 S) (y x) x = y; (x; y 2 S) 이때, 연산 는교환법칙은성립함을보여라. 그리고, 결합법칙이성립하지않는경우가있음을보여라. (Putnam 1972)
98 대수 증명 두번째법칙의 x 대신 y x 를대입하면 y =(y (y x)) (y x) =x (y x) 이된다. 이로부터 x y = x (x (y x)) = y x 로교환법칙이성립함을확인할수있다. 한편, a b c a b a c b a c b c c b a 로 를정의하면 (a b) c = a c = c, a (b c) =a b = a 로서로다르므로결합법칙이성립하지않는예가된다. 각원소 a, b, c 는왼쪽에서곱할때어떤두원소를서로바꾸는작용을하므로왼쪽에서두번곱하면항등원으로작용한다. 오른쪽에서곱할때도마찬가지이므로문제의조건을만족한다는것을쉽게알수있다. 31. 다음을만족시키는함수 f(x) 에대하여, f( 9) 의값을구하여라. f(3) = 1; (x 2 x +1)f(x 2 )=f(x) ( 한국 2004 1 차 -J16) 풀이 x =3 을대입하면 f(9) = 1 7 f(3) = 1 7. x =9 를대입하면 f(81) = 1 73 f(9) = 1 73 7. x = 9 를대입하면 f( 9) = 91f(81) = 13 73. 32. a, b, c, d, e 는 1 a<b<c<d<e 인정수이다. 1 [a; b] + 1 [b; c] + 1 [c; d] + 1 [d; e] 15 16 임을보여라. 단,[m; n] 은 m 과 n 의최소공배수를나타낸다. ( 캐나다 1979-3) 증명준식의좌변을 S라하자. x<y일때 [x; y] 2x 이고등호는 y =2x 일때만성립함을기억해두자. (i) c =3일때 ; a =1,b =2,d 4 이고,[a; b] =2,[b; c] =6,[c; d] 6, [d; e] 8. d 6= 6이면 [c; d] 9 이므로 S 1 2 + 1 6 + 1 9 + 1 8 < 2 3 + 1 4 = 11 12 < 15 16 으로성립. d =6이면 [c; d] =6,[d; e] 12 이므로 S 1 2 + 1 6 + 1 6 + 1 12 = 2 3 + 1 4 = 11 12 < 15 16 으로역시성립. (ii) c =4일때 ;[a; b] 2, [b; c] 4, [c; d] 8, [d; e] 10. d 6= 8이면 [c; d] 12 이므로 S 1 2 + 1 4 + 1 12 + 1 10 = 3 5 + 1 3 = 14 15 < 15 16 으로성립. d =8이면 [d; e] 16 이므로 S 1 2 + 1 4 + 1 8 + 1 16 = 15 16 으로역시성립. (iii) c 5 일때 ;[a; b] 2, [b; c] 6, [c; d] 10, [d; e] 12 이므로이때도역시성립. 따라서, 준식은항상성립한다. 33. 모든 x 2 R 에대해다음을만족하는함수 f : R! R 을모두구하여라. f(x) 3 = x 12 (x2 +7x f(x)+16 f(x) 2 ) ( 플란더즈 1987-3) 힌트모두이항하여인수분해할수있음. 연속성을감안하면 f(x) = x=2 또는 f(x) = x=3 또는원점에서이두가지가서로교체되는함수들. 34. 0 이아닌모든실수에서정의되고다음두조건을만족하는함수 f 는유일하게존재함을증명하여라. (i) 0 이아닌임의의실수 x 에대해, f(x) =xf( 1 x ). (ii) x 6= y 인임의의 0 이아닌실수쌍 (x; y) 에대해, f(x)+f(y) =1+f(x + y). ( 호주 1991-4)
2.2 대수고급문제 99 증명 g(x) =f(x) 1 로치환하면두조건은 g(x)+1=x(g( 1 )+1) x 과 g(x)+g(y) =g(x + y) 로바뀐다. 두번째식의y에x를대입하면 g(2x) =2g(x) 이므로, 첫번째식과연관시키면 g(2x) =2xg( 1 2x )+2x 1=xg( 1 x )+2x 1 2g(x) =2xg( 1 x )+2x 2 의두식이같아야한다. 즉, xg( 1 x )=1,g( 1 x )= 1 x 이다. x 대신 1 x 를대입하면 g(x) =x. 따라서, f(x) =x +1뿐이다. 35. 다음방정식을풀어라. ( 캐나다 1992-4) x 2 + x 2 (x +1) 2 =3 풀이양변에 (x +1) 2 을곱해주고전개하여정리하면다음을얻는다. 이것은다음과같이인수분해된다. x 4 +2x 3 x 2 6x 3=0 (x 2 x 1)(x 2 +3x +3)=0 x 2 +3x+3 은판별식이음이므로항상양의값을갖고, 따라서, x 2 x 1 =0 의근을구하면 x = 1 p 5 2 답 36. 실수 a, b, c가임의의실수 x, y, z에대하여다음부등식을만족시킬때,3a +2b c 의최대값은얼마인가? ( 한국 2004 1차-J3) x 2 +4xy +4y 2 + axz + byz + cz 2 = 0 풀이 x 에대한식으로보고 x 에대한내림차순으로정리하자. x 2 +(4y + az)x +(4y 2 + byz + cz 2 ) 0 이이차식이임의의 x 에대해항상성립해야하므로판별식 D =(4y + az) 2 4(4y 2 + byz + xz 2 ) 0 이어야한다. 이번에는 y 에대한내림차순으로정리하면 (8az 4bz)y +(a 2 4c)z 2 0 z =0일때는당연히성립하고, z 6= 0일때도임의의y에대해성립해야하므로 8az 4bz =0이고 a 2 4c 0 이어야한다. 즉, b =2a; c a2 4 그럼문제의식은 X =3a +2b c 7a a2 ³ a 4 = 2 7 2 +49 49 로최대값은 a =14,b =28,c =49일때49가된다. 관찰준식을 (x +2y) 2 z(ax + by + cz) 로정리할수있고, 여기에 z =1,x = 3, y = 2 를대입하면 49 3a +2a c 의부등식을얻는다. 여기서등호조건만찾아말해줘도답은확실히구할수있다.
100 대수 37. a, b, c 는양의실수이고 a b c 이다. 다음부등식이성립함을보여라. ( 통신강좌 1997-14-6) a 2 b 2 c + c2 b 2 a + a2 c 2 b 3a 4b + c 증명 a + b 2c이므로양변에 a b 을곱하면 a2 b 2 2(a b) 이다. c c a + c>b이므로양변에 a c 를곱하면 a2 c 2 a c이다. b b b + c 2a이므로양변에 b c a 를곱하면 b 2 c 2 2(b c) 이고 c2 b 2 2(c b) 이다. a a 위세결과를더하면 a 2 b 2 c + c2 b 2 a + a2 c 2 b 3a 4b + c 이다. 38. x 는 1 x 3 범위의실수이다. x 2 의소수부가 x 의소수부와같다고한다. 이런 x 는모두몇개인가? ( 플란더즈 1995-2) 풀이 1 x 2 9이므로 x 2 의정수부크기에따라방정식을 9개얻을수있다. 따라서답은 9개 답 39. 실수 x, y, z, w 가 n =2; 4; 6; 8 에대해다음식을만족한다 : x 2 n 2 1 2 + y2 n 2 3 2 + z2 n 2 5 2 + w2 n 2 7 2 =1 x 2 + y 2 + z 2 + w 2 의값을구하여라. (AIME 1984-15) 풀이 %EEE Solving is messy, x^2 = 11025/1024 etc. So trick needed. Write equations as x^2/(t - 1) + y^2/(t - 9) + z^2/(t - 25) + w^2/(t - 49) = 1. Multiply out to get quartic in t, namely t^4 - (84+x^2+y^2+z^2+w^2)t^3 +... = 0, which has 4 known roots 4, 16, 36, 64, so must be t^4-120t^3 +... = 0. 답 36 40. a, b, c, p는실수이고a + 1 b = b + 1 c = c + 1 = p a 일때, abc + p =0임을증명하여라. 단, a, b, c가모두같지는않다. (ML프로포절 161-1) 증명 (KAIST 과학영재센터연구원고봉균 ) (1) a + 1 b = b + 1 c 에서 (a b) =(b c)=bc, 그럼순환적이므로 (a b) = (b c) bc = = (a b) (abc) 2 : 따라서 (abc) 2 =1. jabcj =1. (2) 문제에주어진식을조금변형하여 abc = pbc c = pca a = pab b 를얻을수있고,(1) 에서처럼둘씩빼면 (a b) = pa(b c) = = p 3 abc(a b): 따라서 p 3 abc = 1. (1), (2) 의결과에의해 jp 3 j =1, 즉 jpj =1이고,(2) 의결과에서 p와 abc가부호가달라야함을알수있다. 따라서, abc + p =0. 41. 실수 a 1 ;a 2 ;:::;a 100 들이다음을만족한다. a 1 a 2 a 100 0; a 1 + a 2 100; a 3 + a 4 + + a 100 100 a 2 1 +a2 2 + +a2 100 이가질수있는최대값을구하고, 이최대값일때의모든가능한수열 a 1 ;a 2 ;:::;a 100 을찾아라. ( 캐나다 2000-5)
2.2 대수고급문제 101 풀이아래그림과같이 200 100 크기의사각형에서 a 2 1 + + a2 100 을정사각형들의넓이의합으로생각해보자. 100 A B C a 1 a 2 a 3 a 4 a 5 a 6 직사각형 A와 B는높이가같다.0<a 2 < 50 일때는A보다 B가더넓으므로 A<B+ C 따라서, a 2 1 + + a 2 100 <a 2 1 + + a 2 100 +(B + C A) =50 200 = 10000 이된다. a 2 =0일때와a 2 =50일때는각각 (a 1 ;:::;a 100 )=(100; 0;:::;0) 과 (50; 50; 50; 50; 0;:::;0) 으로모두 a 2 1 + + a2 100 = 10000 이고이때가최대이다. 42. 정수 a, b, c가 a +2b +3c =35; ab+ bc + ca =61 을만족시킬때, a의최대값을구하여라. ( 한국 2003 1 차 -J16) 풀이 ( 전남과학고 1 학년심민수 ) a =35 2b 3c 를오른쪽식 a(b + c)+bc =61 에대입하면 35(b + c) 2b 2 3c 2 5bc + bc =61 이고, 이것을 b 에관한내림차순으로정리하면 이된다. 정수해 b 가존재해야하므로근의공식 2b 2 +(4c 35)b +(3c 2 35c +61)=0 b = (35 4c) p D 4 ( ) 꼴에서 p D =4b (35 4c)X 가정수, 즉 D 는완전제곱수라야한다. D =(4c 35) 2 8(3c 2 35c + 61) = 737 8c 2 에서 D 는홀수임을알수있으므로 D =(2n +1) 2 이라하면 (n 0), 4n(n + 1) = 736 8c 2, 즉 n(n +1) 2 =92 c 2 여기서 jcj 9 임을알수있고, 이범위에서 92 c 2 이 n(n+1) 2 =1+2+ + n 꼴의수가되는것을찾으면된다. 92 c 2 =92; 91; 88; 83; 76; 67; 56; 43; 28; 9 n(n +1) =0; 1; 3; 6; 10; 15; 21; 28; 36; 45; 55; 66; 78; 91; ::: 2 등이 p 가능하므로, 겹치는것은 28과 91뿐이다. 즉, jcj = 1 or 8이고, 그때각각n = 13과 7이어서 D = 27, 15이다.( ) 에대입하여 b가정수가되는경우들을구하면 (b; c) =(1; 1); (3; 1); ( 3; 8); (13; 8) 이다. 각각의경우 a =35 2b 3c = 30, 32, 17, 33 이므로 a 의최대값은 33 이다.
102 대수 43. 음이아닌실수 a, b, c, d, e, f, g 가 a + b + c + d + e + f + g =1 을만족한다. M =maxfa + b + c; b + c + d; c + d + e; d + e + f;e + f + gg 라할때 M 의최소값을구하여라. ( 통신강좌 1998-17-10) 풀이 ( 권익재 ) M = 1 3 일때, a = 1 3, d = 1 3, g = 1 이면성립한다. 3 이제 M< 1 이면모순임을보이자. 3 a + b + c; e + f + g< 1 3 이므로 a + b + c + e + f + g<2 3 a + b + c + d + e + f + g =1에의해서 d> 1 3 c + d + e> 1 3 이므로이는 M<1 에모순이된다. 3 ) M 의최소값은 1 3 이다. 별해 ( 손서연 ) M =maxfa; a + b; a + b + c; b + c + d; c + d + e; d + e + f;e + f + g; f + g; gg 로도쓸수있다. 여기에는각문자가 3 번씩포함되어있으므로이집합의평균은 3=9 =1=3 이다. 따라서, 그중최대값인 M 은 M 1=3 이다. 등호는 a = d = g =1=3 이고나머지가 0 일때성립한다. 44. 함수열 f n 은 f 1 (x) =x 이고 f n (x) = p f n 1 (x) 1 4 로정의된다 (n 2 N, n 2). (a) 두함수가정의되는모든실수 x에대해f n (x) f n 1 (x) 임을증명하여라. (b) 각각의 n에대해, f n (x) =x 를만족하는 x를구하여라. ( 플란더즈 1994-4) 풀이 f n 1 (x) f n (x) =( p f n 1 (x) 1 2 )2 0이므로항상 f n (x) f n 1 (x) 이다. 한편위의부등식에서등호는 f n 1 (x) = 1 4 일때성립하는데, f 1( 1 4 ) = 1 4 이고 f n 1( 1 4 ) = 1 4 이면 f n ( 1 4 )= 1 4 이므로귀납적으로임의의자연수 n에대해f n( 1 4 )= 1 4 이며 f n(x) =x의유일한해가된다는알수있다. 즉모든n에대해f n(x) =x의해는x = 1 4 이다. 45. a 1 ;a 2 ;:::;a 121 은 1000 을넘지않는자연수들의수열이다. 값 n 은이수열에서다른수들보다더여러번나타나고, m 은이수열의모든항의평균이다.[m n] 의가능한최대값은무엇인가? (AIME 1989-11) 풀이 n에해당하는모든항들을 1씩줄이면m은 1보다는덜줄어들고n은 1 줄어들고하니 m n이증가함. 따라서, n =1일때만보면되고, 이때m의최대값을생각하면됨. %EEE Optimum is evidently n+1 terms = 1, n terms = 1000, 999, 998,... (as far as necessary). n = 1 gives 2 of 1, 1 each of 881,..., 1000, mean 924.9, diff 923. n = 2 gives 2 each of 942,..., 1000, mean 946.9, diff 945. n = 3 gives 3 each of 962,..., 1000, mean 948.6, diff 947. n = 4 gives 4 each of 972,..., 1000, mean 945.3, diff 944. In the general case the mean is (1001 - (120-n)/n + 1000) (120 - n)/2 + n+1)/121, so we wish to maximise (2002-120/n)(120 - n)/2 + n and hence to maximise -1000n - 7200/n, or equivalently to minimise 36/n + 5n. Easily seen this is at n = 3. 답 947 46. F 는 2 개이상의원소를갖는유한집합이다. 모든 x; y; z 2 F 에대해다음을만족하는연산 가존재하는지증명혹은반증하여라. (1) x z = y z =) x = y ( 오른쪽소거법칙이성립 ) (2) x (y z) 6= (x y) z ( 결합법칙이성립하는경우가없음 ) (Putnam 1984)
2.2 대수고급문제 103 풀이존재한다. F 의원소를원형으로배열해놓고각원소 x에대해그오른쪽원소를x 0 으로말하기로하자. 그럼 x y = x 0 로하면 (1), (2) 를모두잘만족함을쉽게알수있다. 47. a 1 =1,a 2 =1, a n = a n 2a n 1 a n 2 + a n 1 (n 3) 으로주어진수열 fa n g 에서, a 12 를구하여라. (1990 뉴욕주수학리그 ) 풀이 양변에역수를취하면 1 a n = a n 2 + a n 1 a n 2 a n 1 = 1 a n 1 + 1 a n 2 즉, b n = 1 a n 으로치환하면 b 1 = b 2 =1,b n = b n 1 + b n 2 의피보나치수열이된다.1,1,2,3,5,8, 13, 21, 34, 55, 89, 144 에서 b 12 =144이므로 a 12 = 1 = 1 답 b 12 144 48. 모든정수 x 에대해 f(f(x)) = x +1 을만족하는함수 f : Z! Z 는존재하지않음을보여라. ( 아벨콘테스트 1994 결선 3b) 증명먼저 f가일대일대응임을보이겠다. 만일 f(x) =f(y) 라하면f(f(x)) = f(f(y)) 이므로 x+1 = y +1이되어x = y이다. 따라서 f는일대일함수이다. 또, 임의의 x에대해f(f(x 1)) = x이므로 f(y) =x인 y = f(x 1) 이존재한다. 그러므로 f는일대일대응이다. f가일대일대응이므로 f(x) =0인정수x가유일하게존재한다. 귀납적으로 f(x + n) =n임을보이자. 먼저 n =0인경우, 가정에의해성립한다. 만일 n = k일때 f(x + k) =k가성립한다하자. 그러면 f(f(x + k)) = f(k) =x + k +1 이고양변에다시 f 를취하면 f(x + k +1)=f(f(k)) = k +1 되어 f(x + k +1)=k +1 이성립한다. 따라서모든정수 n 0 에대해 f(x + n) =n 이다. 만일 n = k 일때 f(x k) = k 가성립한다하자. 그러면 f(f( k 1)) = k = f(x k) 이다. f 가일대일대응이므로 f( k 1) = x k 이고 x k = f(f(x k 1)) 에서 f( k 1) = f(f(x k 1)) 이다. 그러므로 f(x k 1) = k 1 이다. 즉, 모든정수 n 0 에대해 f(x + n) =n 이성립한다. n = x +1 이라하자. 그러면 f(2x +1)=x +1 이다. 그런데 f(x) =0 에서 f(0) = f(f(x)) = x +1 이고, f 가일대일대응이므로 2x +1=0 여야한다. 이는 x 가정수라는데모순이다. 즉, 조건을만족하는 f 가존재하지않는다. 49. 서로다른두양의홀수 a 와 b 가,(a b) 2 8 p ab 를만족시킨다고하자. n = ab 일때, 방정식 x 2 2([ p n ]+1)x + n =0 의해가모두양의정수임을보여라. 단, 임의의실수 r에대하여 [r] 은 r보다크지않은가장큰정수를나타낸다. ( 한국 2003-J2)
104 대수 증명 ( 부산건국고 1 년신승현 ) a =2a 1 +1,b =2b 1 +1(a 1 ;b 1 2 N) 1 ⅰ) (a + b) 2 > 4ab (* a 6= b) 1 을대입하여정리하면,(a 1 + b 1 +1) 2 >ab a ⅱ) (a b) 2 8 p ab < 4(a + b) (* 산술 - 기하평균, a 6= b) (a b) 2 < 4(a + b) 에 1 을대입하여정리하면, a, b 에서 양변에적당히더하고빼면, a 2 1 + b 2 1 2a 1 b 1 < 2a 1 +2b 1 +2 a 2 1 + b2 1 +2a 1b 1 1 < 4a 1 b 1 +2a 1 +2b 1 +1 (a 1 + b 1 ) 2 1 < (2a 1 +1)(2b 1 +1)=ab (a 1 + b 1 ) 2 1 <ab () (a 1 + b 1 ) 2 ab b (a 1 + b 1 ) 2 ab < (a 1 + b 1 +1) 2 a 1 + b 1 pab < a 1 + b 1 +1 ) [ p ab ]=[ p n ]=a 1 + b 1 이것을주어진방정식에넣고인수분해를해서답을구해보면, x =2a 1 +1; 2b 1 +1=a; b 따라서, 두해는 a, b 가되어해가모두양의정수이다. 50. 다음조건을만족하는함수 f : R! R 과 g : R! R 를모두구하여라. (1) 임의의실수 s, t에대해,2f(s) g(s) =f(t) t. (2) 임의의실수 s에대해, f(s)g(s) s +1. ( 플란더즈 1999-3) 풀이 (1) 의 t에 s를대입하면 f(s)+s = g(s). 이것으로 g를소거할수있음.(1')f(s) s = f(t) t. 즉 f(s) s = f(0) 0 으로상수. f(s) = s+a. (2') f(s)(f(s)+s) s+1 에대입하면 (s+a)(2s+a) s+1, 즉 2s 2 +(3a 1)s+(a 2 1) 0. 판별식으로풀면 D =(3a 1) 2 8(a 2 1) = a 2 6a+9 = (a 3) 2 0. 따라서, a =3. 51. 1 보다큰모든자연수 n 에대해다음을증명하여라. ( 헝가리 1938-2) 1 n + 1 n +1 + 1 n +2 + + 1 n 2 1 + 1 n 2 > 1 증명 분모가클수록수는작아지므로 준식 = 1 µ 1 n + n +1 + 1 n +2 + + 1 n 2 > 1 n + µ 1 n 2 + 1 n 2 + + 1 n 2 {z n 2 n 개 = 1 n + n2 n n 2 = 1 n + n 1 =1 n 로성립함을알수있다. 위부등식에서 n 2 일때 fn +1;:::;n 2 1g 6=? 이므로등호를분명히배제할수있다.
2.2 대수고급문제 105 별증 1 두번째항부터 n 개씩항을묶어다음과같이분석할수있다. 준식 = 1 µ 1 n + n +1 + + 1 2n ³ n 3n 1 n + ³ n 2n + = 1 n + µ 1 2 + 1 3 + + 1 n µ 1 + 2n +1 + + 1 3n ³ n + + n 2 µ + + 1 (n 1)n +1 + + 1 n 2 n 4 이면괄호안에 1 2 + 1 3 + 1 4 이있으므로 1 보다항상크고, n =3 일때도 1 3 +(1 2 + 1 3 ) 이있으므로역시 1 보다크다. n =2 일때는준식에서따로확인하여 1 2 + 1 3 + 1 4 > 1 로역시성립한다. 별증 2 k 1 일때다음의성질을이용하자. 1 2 k 1 +1 + 1 2 k 1 +2 + + 1 2 k 1 2 k + 1 2 k + + 1 1 2 k =2k 1 2 k = 1 2 등호는항이딱하나뿐일때인 k =1일때만성립한다. 즉,[2 k 1 +1; 2 k ] 꼴의구간이구간 [n; n 2 ] 안에두번포함되면 1 2 + 1 =1 2 보다크게됨을알수있다. 즉, [2 k 1 +1; 2 k ]; [2 k +1; 2 k+1 ] ½ [n; n 2 ] 인 k 가언제존재하는지살펴보자. n 2 k 1 +1 을만족하는가장작은자연수 k 를잡자 (n 2 이므로 k 1). 그럼 k 의최소성에서 2 k 2 +1<n, 즉 2 k 2 +2 n 이다.2 를이항하고양변에 8 을곱하면 2 k+1 8n 16 이다. 따라서,8n 16 n 2 이면원하는 k 가틀림없이존재하는데, 이것은 n 2 8n +16=(n 4) 2 0 으로항상잘성립한다. 그러므로, 문제의부등식은항상잘성립하는부등식이다. 별증 3 즉, 코시 - 슈바르츠부등식에서 µ 1 (n +(n +1)+(n +2)+ + n 2 ) n + 1 n +1 + 1 n +2 + + 1 n 2 (1 + 1 + +1) 2 이것이 1 보다크면충분하다. (n + n 2 )(n 2 n +1) ( 준식 ) (n 2 n +1) 2 2 ( 준식 ) 2(n2 n +1) n 2 + n 2(n 2 n +1) n 2 > 1 + n 2n 2 2n +2>n 2 + n n 2 3n +2> 0 (n 1)(n 2) > 0 으로동치변형되므로, n>2 이면이부등식은항상잘성립한다. n =2 일때도직접확인하거나코시부등식의등호가성립하지않음등을관찰하면역시성립한다. 52. 모든음아닌정수 x, y (x y) 에대해다음을만족하는함수 f : N 0! R 을모두구하여라. (N 0 은모든음아닌정수들의집합이다.) ( 오스트리아 - 폴란드 1979-4) f(x + y)+f(x y) =f(3x)
106 대수 풀이 x = y =0 을대입하면 f(0) + f(0) = f(0) 이므로 f(0) = 0. y =0 을대입하면 2f(x) =f(3x) 이고, y 에 x 를대입하면 f(2x)+f(0) = f(3x), 여기서 f(0) = 0 이므로 f(3x) =f(2x) =2f(x) 가된다. 이제 x; y 에 2x; x 를대입하면 f(3x)+f(x) =f(6x) 2f(x)+f(x) =2f(2x) =4f(x) f(x) =0 따라서, 문제의함수방정식을만족하는 f(x) 는영함수뿐임을알수있다. 53. 두문자 x, y 에대한이차식이두일차식의곱으로인수분해되려면, 한문자에대한식으로보았을때의판별식의판별식이 0 이되어야함을증명하여라. 증명 두일차식의곱을 (ax + by + c)(a 0 x + b 0 y + c 0 ) = aa 0 x 2 +(ab 0 + a 0 b)xy + bb 0 y 2 +(ac 0 + a 0 c)x +(bc 0 + b 0 c)y + cc 0 이라하면, 이것을한문자x에대한식으로보았을때의판별식은 D =((ab 0 + a 0 b)y +(ac 0 + a 0 c)) 2 4aa 0 (bb 0 y 2 +(bc 0 + b 0 c)y + cc 0 ) =(ab 0 a 0 b) 2 y 2 +2(ab 0 a 0 b)(ac 0 a 0 c)y +(ac 0 a 0 c) 2 =((ab 0 a 0 b)y +(ac 0 a 0 c)) 2 으로 y에대한완전제곱식이된다. 54. n 은자연수이고 k 는자연수 a 1 ;a 2 ;:::;a n 들의합보다큰자연수이다. a 1! a 2! a n! <k! 임을증명하여라. 단, n! 은 1 부터 n 까지의자연수를모두곱한것이다. ( 헝가리 1937-1) 증명좌변 a 1! a 2! a n! 에는 a 1 + a 2 + + a n 개의수가곱해져있는데, 대개는 (a i! 의안에서는 )1씩증가시키면서곱하다가 (a i!=1 2 a i ) 가끔 (a i! 의 a i 를곱한후다음의a i+1! 의 1을곱하는것으로넘어갈때 )1로부터다시곱셈을시작하게된다. 우변은 1부터 a 1 + a 2 + + a n 개 ( 좌변에곱해진수의개수 ) 보다더많은수들을계속1씩증가시키면서곱한것이다. 따라서, 각각의곱해진항을차례로비교하면우변이더크거나같고, 우변에는남는항도있으므로우변이더큼을알수있다. 별증서로다른 n종류의물건이각각 a 1 ;a 2 ;:::;a n 개씩있을때, 이들을일렬로나열하는경우의수는 (a 1 + a 2 + + a n )! 으로세어진다 ( 고등학교과정 ). 따라서, 이것은경우의수이므로자연수이고, a 1! a 2! a n! 즉 1 이상이다. 이로부터, k>a 1 + + a n 이므로 k! a 1! a 2! a n! > (a 1 + a 2 + + a n )! 1 a 1! a 2! a n! 임을알수있다. 55. a, b, c 가한삼각형의세변일때, 다음을증명하여라 : ( 인도지역예선 1999-5) 증명 a c + a b + b a + b c + c b + c a 3 a = y + z, b = z + x, c = x + y 의 Ravi 치환을하면, 좌변은 y+z 2y + z+x 2z + x+y 2x = 1 2 ( z x + x z + q y x )+ 3 2 3 2 3 z x x z x y + 3 2 = 3 2 + 3. 2 마지막부등식은 AM-GM.
2.2 대수고급문제 107 56. n개의수를모았을때 (n > 2) 각수가전체합보다작으면이모음을 `북적대는 ' 모음이라부른다. n 1 fa;b;c;:::g는합이s인 n개의수들의북적대는모음이라하자. 다음을증명하여라. (a) 이수들은모두양수이다. (b) 항상 a + b>c가성립한다.(a, b, c는이모음의임의의세수 ) (c) 항상 a + b S n 1 가성립한다. (Towns 1992봄 JA1) 증명임의의항 a 에대해, S 에서 a 를제외한나머지 n 1 개의항을더한것을생각해보자. S S a = b + c + < (n 1) {z n 1 = S n 1 개 따라서, S 를소거하고 a 를넘기면 a>0 임을알수있다. a 는임의의항이었으므로, 모든항은양수. 즉 (a) 가증명되었다. 이번에는임의의두항 a, b 에대해, S 에서 a, b 를제외한나머지 n 2 개의항을더한것을생각하자. S S (a + b) =c + d + < (n 2) {z n 1 = S S n 1 n 2개 정리하면 a + b> S n 1 이되므로 (c) 도증명되었다 ( 특히, 등호는성립하지않는다는것도알수있다 ). (c) 가증명되었으면 a + b S >c n 1 이므로 (b) 는당연히성립한다. 57. 다음을만족하는함수 f : R +! R 를모두찾아라. ( 오스트리아 - 폴란드 1978-1) 임의의 x; y > 0 에대해 f(x + y) =f(x 2 + y 2 ) 58. m 1 ;m 2 ;:::;m k 는합이 S 인양의실수들이다. 다음을증명하여라. ( 베트남 1980-2) kx i=1 µ m i + 1 2 µ k k m i S + S 2 k 59. 다음연립방정식의모든실수해를구하여라. (Towns 1988 봄 JA3) (x 3 + x 4 + x 5 ) 5 =3x 1 (x 4 + x 5 + x 1 ) 5 =3x 2 (x 5 + x 1 + x 2 ) 5 =3x 3 (x 1 + x 2 + x 3 ) 5 =3x 4 (x 2 + x 3 + x 4 ) 5 =3x 5 풀이 윤환식이므로 WLOG: x 1 이최대라하자. 그럼 x 3 + x 4 + x 5 x 4 + x 5 + x 1 이므로그 5제 곱끼리도부등호방향은불변이고따라서 3x 1 3x 2. 즉, x 2 도최대. 마찬가지로 x 3, x 4, x 5 들도 차례로모두최대이므로다섯수가모두같은값. (3x) 5 = 3x 를풀면3x = 1; 0; 1, 즉 답 x 1 = x 2 = x 3 = x 4 = x 5 = 1 3 또는 0 또는 1 3 60. A, B, C는다음을만족하는음아닌실수들이다. A 4 + B 4 + C 4 2(A 2 B 2 + B 2 C 2 + C 2 A 2 ) (a) A, B, C 중어느것도나머지두수의합보다크지않음을증명하여라. (b) A 2 + B 2 + C 2 2(AB + BC + CA) 임을증명하여라. (c) 원래의부등식이 (b) 로부터얻어질수도있는가? (Towns 1987가을 J5)
108 대수 증명이것은헤론의공식을전개했을때나오는꽤알려진인수분해식임 (A 2 에대한내림차순으로정리하거나하면풀만함 ). (A + B + C)( A + B + C)(A B + C)(A + B C) 0 A + B + C =0 이면모두 0 이므로 (a) 가성립. B + C = A 이면 A + B =2B + C C 등이바로확인되므로역시 (a) 가성립. 이제 0 인항이없을때만보자. A B + C; A + B C<0 이라하면둘을합했을때 2A <0 로모순. 즉, 음수인항은많아야하나뿐이고, 곱이양이므로그럼음수인항은없음. 결국 (a) 는항상성립. 혹은,(A 2 (B + C) 2 )(A 2 (B C) 2 ) 0 까지만인수분해한후 (B C) 2 A 2 (B + C) 2, 즉 jb Cj A B + C 등으로도 (a) 가확인됨. (b) 는 A 2 A(B + C) 등의식을모두합하면바로확인됨.(c) 는 (A; B; C) =(1; 2; 4) 등의반례가있음. 61. a, b, c 는삼각형의세변의길이이고, p + q + r =0 이다. a 2 pq + b 2 qr + c 2 rp 0 임을증명하여라. (Towns 1988 가을 SA2) 증명 p, q, r의부호를한꺼번에바꿔도상관없으므로,wlogp q r 이라할때q 0 일때만보면된다. r = (p + q) 를대입하면준식은 a 2 pq b 2 q(p + q)+c 2 (p + q)p =(b 2 + c 2 )pq +(bq) 2 +(cp) 2 과동치. a<b+ c 이므로 (b + c) 2 pq (b 2 + c 2 )pq +(bq) 2 +(cp) 2 만보이면충분하고, 이것은소거하고정리하면 0 (bq cp) 2 이므로성립. 62. f n (k) =k +1(k =1; 2; 3;:::;n 1), f n (n) =1 이성립한다. 각자연수 n 2 에대해서, a n, b n, c n, d n 이 n 에의해서만결정되는함수 f n(x) =a n + b nx + c njx d nj 을구하여라. (IMTS R10-3) 63. a, b, c, d는 a b c d 와 a + b + c + d 1 을만족하는양수들이다. a 2 +3b 2 +5c 2 +7d 2 1 임을증명하여라. (Towns 1989봄 SO1) 증명 a 2 +3b 2 +5c 2 +7d 2 a 2 +(2ab+b 2 )+(2ac+2bc+c 2 )+(2ad+2bd+2cd+d 2 )=(a+b+c+d) 2 1. 64. f(x) =x 4 +17x 3 +80x 2 +203x +125 라하자. f(3 p 3) = g(3 p 3) 과 f(5 p 5) = g(5 p 5) 를만족하는다항식 g(x) 중에서최소차수인것을구하여라. (IMTS R11-5) 풀이 f(x) g(x) 는 3 p 3 과 5 p 5 를모두근으로가지므로 f(x) g(x) =((x 3) 2 3)((x 5) 2 5)Q(x) 로 4차식이상임. Q(x) 가상수1일때g(x) 는 3차식이고, Q(x) 가다른상수이면g(x) 는 4차식이고, Q(x) 가 1차이상이면 g(x) 는 5차이상임. 따라서,3차식인것이최소차수. 답 f(x) ((x 3) 2 3)((x 5) 2 5) 65. 방정식 x 2 1 2 [x2 2] = 2 + 3[ 1 2 x2 3] 의모든실수해를구하여라. ( 몰도바 1996 최종 -y9-6) 66. 수열 (x n), (y n), (z n) 이다음과같이주어져있다. x n+1 = y n + 1 x n ; y n+1 = z n + 1 y n ; z n+1 = x n + 1 z n (n 0) 단, x 0, y 0, z 0 는주어진양수이다. 이세수열에는얼마든지큰수가있음을증명하여라. ( 오스트리아 - 폴란드 1981-6) 증명 M n =max(x n ;y n ;z n ) 이라하자. 그럼 M n 은증가수열이고 M n+1 M n + 1. M n 귀류법으로 이수열이위로유계, 즉항상M n <N 이라하자. 그럼 M n+1 >M n + 1 N, 따라서 M n >M 0 + n N 이 되고, 특히 M N 2 >M 0 + N>N. 67. n 은 2 이상의정수이다. 3p n 3 +2n 2 + n 의십진전개에서소수점이하첫번째자리의수를구하여라. ( 아일랜드 2004-8)
2.2 대수고급문제 109 풀이 (n +0:6) 3 <n 3 +2n 2 + n<(n +0:7) 3. 답 6 68. x 1 x 2 x 3 x n 인양의실수들의수열 fx n g n 1 이모든n에대해다음을만족한다. x 1 1 + x 4 2 + x 9 3 + + x n 2 n 1 모든 k에대해다음이성립함을보여라. ( 인도지역예선 2000-3) x 1 1 + x 2 2 + x 3 3 + + x k k 3 69. 모든 x 에대해서 (2x 1) 20 (ax + b) 20 =(x 2 + px + q) 10 을만족하는실수 p; q; a; b 를모두찾아라. ( 소련 1963-12) 70. 자연수 c 가주어졌을때, 수열 ff n g 은다음과같이정의된다. f 1 =1; f 2 = c; f n+1 =2f n f n 1 +2 (n 2) 각첨자 k 에대하여, 다음을만족하는첨자 r 이항상존재함을보여라. f k f k+1 = f r (IMO-SL 1984, 캐나다출제 ) 증명 f n =(n 1)c +(n 2) 2. 그다음은그냥전개확인. r = f k + k. 71. ab + cd =2pq 인수 a, b, c, d, p, q 가있다. ac p 2 > 0 이면 bd q 2 임을보여라. ( 러시아 1990 4 차 -y9-2) 증명 bd > q 2 가정하면 (ab + cd) 2 =4p 2 q 2 < 4abcd 로모순. 72. a 2 x 2 + ax +1 7a 2 =0 의두해가모두정수가되도록하는양의실수 a 의값을모두구하여라. ( 러시아 1990 4 차 -y9-5) 풀이두근의합이정수이므로 a = 1 b 꼴. 대입하고 b2 곱해주면 x 2 + bx + b 2 7=0. 판별식이제곱수임을이용.28 3b 2 = 제곱수 0. b =1; 2; 3뿐. 73. 3p 2+ 3 p 4 가어떤정계수삼차방정식의근이면, 이삼차방정식의실근은이것뿐임을증명하여라. (IMTS R12-4) 증명 x = a + a 2, a 3 =2라두면x 3 = a 3 (1 + a) 3 =6(a 2 + a +1)=6(x +1). y = x 3 그래프와 y =6(x +1) 그래프를그려서한점에서만만남을확인...; 74. x, y, z는서로다른양의실수들이다. 다음의세수 x y y x ; y z z y ; z x x z 들이한삼각형의세변의길이가될수있는지그렇지않은지알아내어라. ( 인도지역예선 1997-5) 풀이 ( 김대현 ) 일반성을잃지않고 x y z 라고하자. 그러면위의세수는 x y y x ; y z z y ; x z z x 삼각형의세변이될수있는지알아보기위해서는변의비가필요하므로 xyz 를곱해주면 (x 2 y 2 )z; (y 2 z 2 )x; (x 2 z 2 )y (x 2 y 2 )z +(y 2 z 2 )x (x 2 z 2 )y =(y x)(z x)(z y) < 0 로인수분해정리되어삼각부등식을만족하지않으므로삼각형을이루지못한다.
110 대수 75. x>y 0 일때다음부등식을증명하여라. ( 유고슬라비아 1979 고 2-2) x + 4 (x y)(y +1) 2 3 증명분모를 x y; y+1 2 ; y+1 2 로분해하면산술-기하로 ( 좌변 ) x +( 3 x+1 )3 이되고... 왼쪽의 x를다시 x+1 + x+1 + x+1 1 3 3 3 로분해하면다시산술-기하로 ( 좌변 ) 4 4p 1 1 이됨. 76. 다음연립방정식을풀어라. ( 러시아 1992 4 차 -y9-1) x 3 5 y2 x = 6 y ; y3 5 x2 y = 6 x 증명 xy 곱하고, 두식을더해보고또빼보고. 77. a, b, c, d 는다음을만족하는서로다른실수들이다. a + b + c + d =3; a 2 + b 2 + c 2 + d 2 =45 이때, 다음식의값을구하여라. (IMTS R18-4) a 5 (a b)(a c)(a d) + b 5 (b a)(b c)(b d) + c 5 (c a)(c b)(c d) + d 5 (d a)(d b)(d c) 풀이 a 5 (a b)(a c)(a d) + b 5 (b a)(b c)(b d) + c 5 (c a)(c b)(c d) + d 5 (d a)(d b)(d c) 통분하고, 분자의인수들찾고, 대칭식임을보이고, 그럼결과는약분되고 2 차식만남아서수치대입법. 78. a; b; c 0 이고 a + b + c 3 이다. 다음부등식을증명하여라. ( 러시아 1989 4 차 -y10-4) a 1+a 2 + b 1+b 2 + c 1+c 2 3 2 1 1+a + 1 1+b + 1 1+c 증명왼쪽은분모산술 - 기하. 오른쪽은산술 - 조화 ( 혹은코시 ). 79. Milo는 Mindbender고등학교의학생이다. 각과목의시험을치른후, 그는항상누적평균을계산하고, 그평균값은반올림된가장가까운정수로만나타낸다.( 즉, 85.49점은내림을하여 85로, 85.50점은올림을하여 86으로나타낸다.) 오늘그는 2개의시험을쳤다. 먼저프랑스어시험에서 75점을받았고, 이것은평균을 1점떨어뜨렸다. 두번째역사시험에서는 83점을받았고, 이것은평균을 2점더떨어뜨렸다. 그의현재평균점수는얼마인가? (IMTS R13-1) 풀이어제까지과목수를 n, 총점을 S, 반올림된평균을 m +3 이라하자. m +2:5 S S +75 S + 158 <m+3:5; m+1:5 <m+2:5; m 0:5 n n +1 n +2 <m+0:5 n, S, m 이정수임에유의하여 S 를소거해주는방향으로이부등식을풀면됨.( 문제를음미해보면평균은 m +3:5, m +1:5, m +0:5 에가까웠을테니까, 아마도첫식의오른쪽, 둘째식의왼쪽, 세째식의오른쪽부등식이서로연결된의미가클듯 )
2.2 대수고급문제 111 80. x 2 + y 2 x 2 y 2 + x2 y 2 x 2 = k 일때다음식을k에관한식으로나타내어라. + y2 E(x; y) = x8 + y 8 x 8 y 8 x8 y 8 x 8 + y 8 ( 주니어발칸 1997-2) 풀이통분하면 k = 2(x4 + y 4 ) x 4 y 4. 마찬가지로 k 2 + 2 k = 2(x8 + y 8 ) x 8 y 8 := r. 즉, E = r 2 2 r = k 2 +4 4k 4k k 2 +4 = (k +2)2 (k 2) 2 4k(k 2. +4) 81. x 1 + + x n 1 2 인음아닌실수 x 1;x 2 ;:::;x n 에대해다음을증명하여라. (1 x 1 )(1 x 2 ) (1 x n ) 1 2 ( 폴란드 1966 3 차 -3) 증명 (1 x)(1 y) (1 (x + y)) 이므로귀납법. 뉴질랜드 2000-2 의확장 82. x, y 는다음을만족하는양의실수들이다. x 3 + y 3 +(x + y) 3 +30xy =2000 x + y =10 임을보여라. ( 주니어발칸 2000-1) 증명 x+y = p, xy = q 라하면준식은 2p 3 3pq +30q 2000 = 0, (p 10)(2p 2 +20p+200 3q) =0. 산술 - 기하평균부등식에서 p 2 4q 이므로오른쪽항은양수이고, 고로 p 10 = 0. 83. 모든양의실수 a, b, c 에대하여다음부등식이성립함을증명하여라. 1 b(a + b) + 1 c(b + c) + 1 a(c + a) 27 2(a + b + c) 2 ( 주니어발칸 2002-4) 증명 a; b; c와 b + c; c + a; a + b는크기순서가반대이므로재배열부등식의최소일때에의해 LHS 1 a(b+c) + 1 b(c+a) + 1 c(a+b). 그담은산술-조화 ( 혹은코시 ) 를쓰면LHS 9 2(ab+bc+ca) 가되고이것을 RHS와비교하는것은껌. 혹은그냥처음부터쭈욱산술-기하부등식만반복해서사용해도됨. 84. x, y 가둘다 0 은아닌실수일때다음부등식을증명하여라. ( 주니어발칸 2004-1) x + y x 2 xy + y 2 2 p 2 p x 2 + y 2 증명 (x + y) 2 2(x 2 + y 2 ) 임에서 x + y p2 p x 2 + y 2 이고, 또 x 2 xy + y 2 1 2 (x2 + y 2 ) 이므 p p 2 x 로, LHS 2 +y 2 1 = RHS. 2 (x2 +y 2 ) 85. a 1, a 2, b 1, b 2, c 1, c 2 는 a 1 a 2 > 0, a 1 c 1 b 2 1, a 2c 2 b 2 2 을만족하는실수들이다.(a 1 + a 2 )(c 1 + c 2 ) (b 1 + b 2 ) 2 임을증명하여라. ( 헝가리 1939-1) 증명 a 1 ;a 2 ;c 1 ;c 2 는모두부호가같다. 모두음수일때는모두부호만바꿔줘도식은똑같으므로, 모두양수일때만생각해도된다. 그럼코시부등식으로직접끝. 혹은 (a 1 + a 2 )x 2 2(b 1 + b 2 )x +(c 1 + c 2 )= (a 1 x 2 + b 1 x + c 1 )+(a 2 x 2 + b 2 x + c 2 ) 가 0이될수없음을말해도끝. 86. 방정식 4x 4 ax 3 + bx 2 cx +5=0 의네근 r 1 ;r 2 ;r 3 ;r 4 가모두양의실수이고다음을만족함이알려져있다. 이네근을구하여라. ( 남미 1985-3) r 1 2 + r 2 4 + r 3 5 + r 4 8 =1
112 대수 풀이 G:M에의해 ( 김대현 ) 근과계수와의관계에서 r 1 r 2 r 3 r 4 = 5 4 이다. 네근이모두양의실수이므로A:M 1= r 1 2 + r 2 4 + r 3 5 + r r r 4 8 4 r1 r 4 2 r 3 r 4 1 2 4 5 8 =44 4 4 =1 따라서등호성립조건에서 r 1 2 = r 2 4 = r 3 5 = r 4 8 = 1 4 (r 1 ;r 2 ;r 3 ;r 4 )=( 1 2 ; 1; 5 4 ; 2) 87. a 2 + b 2 = c 2 과 x 2 + y 2 = z 2 을만족하는자연수 a; b; c 와 x; y; z 를생각하자. (a + x) 2 +(b + y) 2 (c + z) 2 임을증명하고등호조건을구하여라. (IMTS R15-4) 풀이 양변전개후소거하면 ax + by cz 만증명하면됨. 양변제곱하면정확히코시부등식 or 완전 제곱꼴. 88. x, y는양의정수이고 y>3 이다. x 2 + y 4 =2[(x 6) 2 +(y +1) 2 ] 일때, x 2 + y 4 =1994임을증명하여라. ( 아일랜드 1994-1) 89. 양이아닌실수 a 1 ;a 2 ;:::;a 1996 에대해다음부등식을증명하여라. ( 몰도바 1996 최종-y10-6) 2 a 1 +2 a 2 + +2 a 1996 1995 + 2 a 1+a 2 + +a 1996 증명 2 a +2 b 1+2 a+b. 귀납적으로. 등호조건은 ab =0일때이므로 1995개의 a i 가 0일때. 90. 다음을만족하는함수 f 를모두구하여라. (i) 실수값을갖는다. (ii) x 6= 2 3 인모든실수에대해f(x) 가정의된다. (iii) 2 3 를제외한모든실수 x에대하여다음을만족한다. 498x f(x) = 1 µ 2x 2 f 3x 2 ( 호주 1992-6) 풀이 x à 2x 대입... 연립방정식 3x 2 91. 모든실수에대하여정의되고, 다음두성질을만족하는각각의함수 f에대하여f( p 5753 ) 의값을구하여라. ( 호주 1993-2) f(xy) =xf(y)+f(x)y f(x + y) =f(x 1993 )+f(y 1993 ) 풀이 f(0) = 0, f(1) = 0, f(x +1)=f(x 1993 )=f(x). 고로 f(n) =0. f( p n p n )=::: 92. 임의의자연수 n 에대하여 f(n) =b2 p n c b p n 1+ p n +1c 라할때, f(n) =1 을만족시키는 n 을모두구하여라.( 단, bxc 는 x 를넘지않는최대의정수이다.) ( 호주 1993-4)
2.2 대수고급문제 113 풀이 (m 1 ) p n 1+ p n +1<m 2 p n( <m+1) 이라놓고부등식풀이. 제곱해서풀다보면아마n 2 일때2 p n 2 1 > 2n 1 이성립함을유용하게써먹... 93. 서로다른세양수가주어져있다. 이세수에 a b + b c > a c + c a 가성립하도록적당한순서로 a, b, c 의이름을붙일수있음을보여라. ( 러시아 1991 4 차 -y9-3) 증명 a>c>b 로하면됨... 94. P (x) 를 n 3 차의정수계수다항식이라고하자. P (x 1 )=P (x 2 )= = P (x m )=1 를만족하는서로다른정수 x 1 ;:::;x m (m 3) 이있을때, P 는정수근을가질수없음을증명하여라. ( 체코슬로바키아 1986-2) 증명 x i 들중 3개만택하여보면충분할. P (x) 1=(x a)(x b)(x c)q(x) 라두고n이정수근이라하면 1 =(n a)(n b)(n c)q(n), 즉 n a; n b; n c j 1, n a; n b; n c 2f 1; 1g. 이것은 a, b, c가서로다름에모순. 95. x 6= 1,y 6= 1,x 6= y 인실수 x; y; z 는다음식을만족한다. yz x 2 1 x = zx y2 1 y 이분수의값이 x + y + z 와같음을보여라. ( 남미 1985-4) 증명변은 x+(yz-x)/(1-x) 이므로 (yz-x)/(1-x)=y+z 임을보이면충분하다. 양변에 1-x를곱하면, x+y+z=xy+yz+zx 임을보이면된다. 그런데주어진조건에서한변에모두넘기고 x-y로나누면아마바로이식이나옴 96. 다음등식을증명하여라. ( 몰도바 1996 최종 -y11-1) 1 666 + 1 667 + + 1 1996 =1+ 2 2 3 4 + 2 5 6 7 + + 2 1994 1995 1996 증명 2 (n 1)n(n+1) = 1 (n 1)n 1 n(n+1) = 1 n 1 2 n + 1 n+1 이므로 2 (3n 1)3n(3n+1) = 1 3n 1 + 1 3n + 1 3n+1 1. n 이것으로전개하면끝. 97. a 1 = a n =0 이고 k =2; 3;:::;n 1 에대해 a k 1 +a k+1 2a k 를만족하는실수 a 1 ;a 2 ;:::;a n (n 3) 이있다. 이수들이모두 0 이하임을증명하여라. ( 폴란드 1967 3 차 -1) 증명양의항이있다고가정하고, 최대항 ( 중에서도가장오른쪽항 ) 을잡으면그좌우항이역시최대항이되어야해서모순. 98. 다음을만족하는함수 f : N! N 를모두구하여라. (a) 모든 n; m 2 N 에대해, f(n + m) =f(n)f(m). (b) 방정식 f(f(x)) = f(x) 2 이적어도하나의자연수해를갖는다. ( 몰도바 1997 최종 -y11-5) 풀이 f(1) = a 라할때 f(n) =a n. (b) 는 a an = a 2n 이되고, a>1 이면지수가같아야하므로 a n =2n. 그런데 a 3 이면 a n 3 n > 2n 으로해없음. 답 f 1, f(n) =2 n.
114 대수 99. 다음성질을만족하는함수 f : R +! R 를모두구하여라 (R + 는양의실수의집합 ). (i) 임의의 x; y 2 R + 에대해f(xy) =f(x)f( 3 y )+f(y)f( 3 x ) (ii) f(1) = 1 ( 2 호주 1995-8) 풀이 f(3) = 1 2. f(3x) =f(x) =f( 3 x )=f( 1 x ). f(xy) =2f(x)f(y) =f( x y ). f(x2 ) 1. 100. 임의의삼각형의세변의길이 a, b, c 에대해다음부등식이성립함을증명하여라 : ( 인도지역예선 1995-5) a 2 + b 2 + c 2 > p 3maxfja 2 b 2 j; jb 2 c 2 j; jc 2 a 2 jg 증명 WLOG a b c 이면우변은 p 3(a 2 c 2 ). cos 제 2 법칙으로 b 2 소거하고 (2 p 3)a 2 +(2+ p 3)c2 2ac AM-GM 이용하면. 101. p(x) =x n + a n 1 x n 1 + + a 1 x + a 0 은정수계수의다항식이고,(p(x)) 2 은음이아닌계수들만을갖는다항식이라고한다. p(x) 의계수도모두음이아니어야하는가? 당신의주장을증명하여라. (IMTS R43-3) 증명반례 : p(x) =x 4 +2x 3 2x 2 +4x +4,p(x) =x 4 +100x 3 x 2 +100x +100 등 102. 모든실수 x 에대해 a(x) > 0 이고 b(x) 2 4a(x)c(x) < 0 인함수 a, b, c 에의해정의된함수 f(x) = a(x)x 2 + b(x)x + c(x) 는모든실수 x 에대해 f(x) > 0 임을증명하여라. 증명이식을일반화해서두변수 x, y에관한식f(x; y) =a(y)x 2 + b(y)x + c(y) 로생각. 각각의고정된 y에대해항상성립하므로 OK. 103. 모든자연수 n에대해다음을증명하여라. ( 아일랜드 1995-6) µ (n +1)(n +2) n n n (n!) 2 6 증명왼쪽부등식은 1:2 로짝을지어n (k +1)(n k) 임을보여서끝. 오른쪽은 n이충분히클때대충 n! nn 로만들어서비교. 8 104. 2 이상의각정수 n 에대해다음을만족하는두양의실수 a 와 b 중에서어느것이더큰지밝혀라. ( 미국 1993-1) a n = a +1; b 2n = b +3a 풀이 a; b > 1 임은당연. f(a) =a n a 1, g(b) =b 2n b 3a 라두면f, g 모두 1보다큰실수범위에서단조증가. g(1) = 3a <0 이고 g(a) =a 2n a 3a =(a +1) 2 4a =(a 1) 2 > 0 이므로 g(b) =0의근b는구간 (1;a) 에존재. 105. 다음의방정식이모든실수a 1 ;a 2 ;:::;a n+1 들에대해서항상실수해를갖도록하는자연수 n을모두구하여라. ( 오스트리아 1985-4) q a n+1 x 2 2x a 2 1 + a2 2 + + a2 n+1 + a 1 + a 2 + + a n =0 풀이판별식두번사용. x에대해한번, 다시 a n+1 에대해한번. a 1 = a 2 = = a n =1을대입해보면 n =1; 2; 3; 4 만가능. 코시로실제됨을확인.
2.2 대수고급문제 115 106. x; y; z > 1 일때, 다음을증명하여라. ( 주니어발칸 2003-4) 1+x 2 1+y + z 2 + 1+y2 1+z + x 2 + 1+z2 1+x + y 2 2 증명 x 1+x2 2 이용하여분모를풀어주고 1+x 2 = a,... 로치환하면 c a+2b + a b+2c + b c+2a 1 꼴로변형됨. 이것은 (a +2b)c +(b +2c)a +(c +2a)b 를양변에곱해주면코시부등식으로됨. 107. a 1 = 1 2 이고, n>1 에대해서 a n = a n 1 2na n 1 +1 로정의된수열이있다. a 1 + a 2 + + a 1998 을계산하여라. ( 몰도바 1998 최종 -y11-5) 풀이 1 = 1 +2n = = n(n +1). a a n a n = 1 n 1 n 1 n+1 로 telescoping 108. f : N! N 는증가함수이고, f(2) = a>2, 또모든자연수 m, n 에대해 f(mn) =f(m)f(n) 을만족한다. a 의가능한최소값을구하여라. ( 북유럽 1987-3) 풀이 f(n) =n 2 일때모든조건만족하므로 a =4는가능. a =3은불가능함을보이자. f(3) = b 라하면f(3 2 ) >f(2 3 )=27임에서 b>5. f(3 5 ) <f(2 8 ) = 6561 임에서 b<6. 109. w, a, b, c 는서로다른실수들이고, 다음연립방정식을만족하는실수 x, y, z 가존재한다. x + y + z =1 xa 2 + yb 2 + zc 2 = w 2 xa 3 + yb 3 + zc 3 = w 3 xa 4 + yb 4 + zc 4 = w 4 w 를 a, b, c 에대한식으로나타내어라. ( 아일랜드 1994-9) 풀이 ' 서로다른 ' 에주목하여먼저 a, b, c 중에 0이없음을확인하고, 그담엔아래세식에서아무생각없이 x, y, z를연립하여푼다음, 이것을첫식에대입하여 w에대한4차식을얻고, 원래이연립방정식이 w = a; b; c 를해로가진다는것과이4차식이일차항이없음에착안하여 abc + bcw + caw + abw =0 에서마지막해 w = abc=(bc + ca + ab) 를구함. 110. a, b, c 는실수이고 a; c 6= 0 이다. ax 2 + bx + c =0 의한실근을 r 이라하고 ax 2 + bx + c =0 의한실근을 s 라할때, r 과 s 사이에 a 2 x2 + bx + c =0 의한실근이있음을증명하여라. ( 몰도바 2000 최종 -y9-1) 증명 f(x) = a 2 x2 + bx + c 이라하면 f(r)f(s) =( a 2 r2 )( 3a 2 s2 )= 3 4 (ars)2 < 0. 중간값정리 111. 주어진세실수 a, b, c 가 a + b + c>0, bc + ca + ab > 0, abc > 0 을만족한다. a; b; c > 0 임을보여라. (Towns 1985 봄 SO3) 증명근과계수. p; q; r > 0 일때, x 가음수이면 x 3 px 2 + qx r<0 으로근이될수없다. 112. A 1 = f1g, A 2 = f2; 3; 4g, A 3 = f5; 6; 7; 8; 9g,... 등의집합들을생각하자. b n 을 A n 의최소원소와최대원소의산술평균이라고하자. 다음식의값이소수임을보여라. 2000 b 2 1 + 2000 2000 + + b 3 1 b 2000 1 ( 몰도바 2000 최종 -y7-3) 증명 b n = 1 2 ((n 1)2 +1+n 2 )=n 2 1 n +1. b n 1 = 1 n(n 1) = 1 n 1 1 telescoping. n 답 1999
116 대수 113. x 1 = 1 2, x k+1 = x 2 k + x k 로주어진수열이있다. 1 x 1 +1 + 1 x 2 +1 + + 1 x 100 +1 의정수부를구하여라. (Towns 1985 가을 J6) 풀이부분분수에서 1 x k +1 = 1 x k 1 x k+1. telescoping 114. n 개의실수 a 1 ;a 2 ;:::;a n 이있는데, 총합은 0 이고, 처음 k 개의항은양이아니며, 그이후의항들은음이아니라고한다. a 1 +2a 2 +3a 3 + + na n 0 임을증명하고, 등호는 a i 들이모두 0 일때만성립함을보여라. ( 스웨덴 1966-2) 증명 a 1 +2a 2 + + ka k +(k +1)a k+1 + + na n ka 1 + ka 2 + + ka k + ka k+1 + + ka n =0 이고등호는모든항이 0 일때만. 115. 임의의자연수 n 과음아닌실수 a, b, c 에대해다음을증명하여라. ( 스웨덴 1975-3) a n + b n + c n ab n 1 + bc n 1 + ca n 1 증명 1 AM-GM 관계에서 a n + b n + + b n nab n 1. 증명 2 재배열부등식. 116. (x; y) 6= (0; 0) 일때, 함수 f(x; y) = x2 xy + y 2 2x 2 + y 2 의최소값을구하여라. ( 대학생경시 2002-4 0 ) 풀이 f(x; y) =(x 2 xy + y 2 )=(2x 2 + y 2 )=k 로두고 k 의최소값을구하자. 분모를넘기면 x 2 xy + y 2 =2kx 2 + ky 2, 이것을정리하면다음과같다. (1 k)y 2 xy +(1 2k)x 2 =0 k 가구하는최소값일때이 y 에관한 2 차식은해를가지므로, 판별식 D = x 2 4(k 1)(2k 1)x 2 0 이다. x =0 이면 k =1 이되고, x 6= 0 이면 1 4(k 1)(2k 1) 0, 즉 8k 2 12k +3 0. 근의공식으로풀면 3 p 3 4 k 3+p 3 ; 따라서최소값은 3 p 3 4 4 117. 다음연립부등식을풀어라. ( 스웨덴 1992-3) 2x 1 5x 2 +3x 3 0 2x 2 5x 3 +3x 4 0. 2x 23 5x 24 +3x 25 0 2x 24 5x 25 +3x 1 0 2x 25 5x 1 +3x 2 0 풀이다더하면 0 0 으로모두등호성립. 모두똑같지않다면, 윤환식이므로, 최대인항과그렇지않은항이이웃한경우가존재. 그런데, x k 가최대라면 5x k 2x k 1 +3x k+1 로 x k 1 = x k+1 = x k 여야함. 따라서, 모든항이똑같을때뿐.
2.2 대수고급문제 117 118. 0 이아닌실수들에대해정의되는연산 가 0 아닌임의의실수 a, b, c 에대해다음을만족한다 :(1) a a =1,(2)a (b c) =(a b)c (a b 와 c 의곱셈 ). 방정식 x 36 = 216 을풀어라. ( 스웨덴 1993-4) 풀이 216 36 = (x 36)36 = x (36 36) = x 1=x (x x) =(x x)x = x. 일반적으로 a b = a=b 임을증명할수있음. 119. 주어진상수 a, b 에대해함수 f(x) = 1 ax + b 를생각하자. f(x 1)=x 2, f(x 2 )=x 3, f(x 3 )=x 1 인서로다른세실수 x 1, x 2, x 3 이존재할때, a 와 b 는어떤수인가? ( 스웨덴 1993-6) a + b 2 + abx 풀이 x = f(f(f(x))) = b 3 +2ab +(a 2 + ab 2 )x 를풀면 (a2 + ab 2 )x 2 +(b 3 + ab)x (a + b 2 )=0. 이이차방정식이서로다른세실근을가져야하므로, 이식은항등식, 즉계수가모두 0이어야한다. 고로, a + b 2 =0. a = b 2 일때는f(f(f(x))) x 로항상성립. 120. 모든실수 x 에대해 p(x 1) + p(x +1)=2p(x)+2 가되는실계수다항식 p(x) 를모두구하여라. ( 스웨덴 2003-4) 풀이 p가 1차이하, 즉 p(x) =ax + b 라하면2ax +2b =2ax +2b +2 로성립하지않는다. p가 2차이상이라면 p(x) =ax n + bx n 1 + cx n 2 + 라두자. a((x 1) n +(x+1) n )+b((x 1) n 1 +(x+1) n 1 )+c((x 1) n 2 +(x+1) n 2 )+ =2ax n +2bx n 1 +2cx n 2 + 여기서좌변의 n 2차항의계수는 n(n 1)a +2c 6= 2c 로, n>2 일때는우변의n 2차항의계수와같을수없다. 즉, n =2이고,2a +2c =2c +2로 a =1이다. p(x) =x 2 + bx + c 일때는식이항상잘성립한다. 121. x + y =2 인음아닌실수 x, y 에대해다음을증명하여라. ( 아일랜드 2000-6) x 2 y 2 (x 2 + y 2 ) 2 증명동차식의방법으로하면준식은다음과차례로동치 :32x 2 y 2 (x 2 + y 2 ) (x + y) 6, x 6 +6x 5 y 17x 4 y 2 +20x 3 y 3 17x 2 y 4 +6xy 5 + y 6 0, (x y) 2 ((x 2 y 2 ) 2 +8xy(x 2 + y 2 )) 0. 그리고이것은성립. 별증 xy ( x+y 2 )2 =1이므로 x 2 y 2 (x 2 + y 2 ) xy(x 2 + y 2 ) 1 8 (x + y)4 =2. 마지막부등식은 (x y) 4 0 과 (x + y) 4 8xy(x 2 + y 2 ) 이동치임에서. 122. 다음연립방정식이모두가 0 은아닌음아닌실수해 x 1 ;x 2 ;x 3 ;x 4 ;x 5 를갖는다 : x 1 +x 3 =2tx 2, x 2 +x 4 = 2tx 3, x 3 + x 5 =2tx 4. t 의최소값은얼마인가? ( 스웨덴 1974-5) 풀이만일 x 3 =0이면 x 2 + x 4 =0이고그럼 x i 들이모두 0임을금방알수있음. 즉, x 3 6=0. 연립하여 4t 2 2= x 1+x 5 x 3 의식을얻을수있고, 이것은 0 이상이므로 t 2 1 2. t도 0 이상이므로 t 1p. 2 실제 x 1 = x 5 =0,x 2 = x 4 =1,x 3 = p 2 이면등호성립. 123. a; b 0 에대해다음을증명하고등호조건은 a = b 임을보여라. ( 아일랜드 2004-5) p µq 2 a(a + b) 3 + b p a 2 + b 2 3(a 2 + b 2 ) 증명 (a+b) 2 2(a 2 +b 2 ) 임을이용하면 LHS p2 p a 2 + b 2 ( p 2a(a + b)+b) p2(a 2 + b 2 )( 3a+b + 2 b), 즉,(a + b) p2 p a 2 + b 2 와동치이므로끝. 124. f(1) = 1 이고, 모든실수 x, y 에대해다음을만족하는함수 f : R! R 를모두구하여라. ( 아일랜드 2006-5) f(xy + f(x)) = xf(y)+f(x) 풀이 x =1을대입하면f(y +1)=f(y)+1. (x; y) =(n; n), (n +1;n 1) 을대입한두식을비교하면됨.
118 대수 별해 (x; y) =(1; 0) 을대입하면 f(0) = 0. y =0을대입하면 f(f(x)) = f(x) 임을이용하여다음과같이할수도있다. x = 1 y 를대입하면 1+f( 1 y )=1+f(f( 1 y )) = f(1 + f( 1 y )) = 1 y f(y)+f( 1 ). y 따라서,1= 1 f(y), y 즉 f(y) =y. 125. 양의실수 a, b, c 에대해다음을증명하고, 각부등식의등호조건을구하여라. a + b + c 3 s a 2 + b 2 + c 2 3 ab c + bc a + ca b 3 ( 아일랜드 2007-7) 풀이좌변은체비셰프나코시나전개. 우변은 ab c = Z 등으로치환후전개하면 X2 + Y 2 + Z 2 XY + YZ+ ZX 가됨. 126. a, b, c 는정수이고 a>0 이다. 방정식 ax 2 + bx + c =0 이구간 (0; 1) 에서서로다른두해를갖는다고하자. a 5 임을보이고, a =5 일때그와같은방정식의예를하나들어라. ( 스웨덴 1979-5) 증명 f(x) =ax 2 dx + c 라할때f(0) > 0, f(1) > 0, D>0, 0 < 축 < 1 등에서잘정리하면다음두부등식을얻음 : d 2 > 4ac 이고 0 < 2c <d<a+ c. 정수이므로 a와 c는 2 이상의차이가있어야함. 만일 a 4 이면 c 2. c =2이면 a =4이고 d =5인데 d 2 > 4ac 성립하지않음. 즉 c =1. d 2 > 4ac 12 이고 d<a+ c 5 이므로 d =4. 그럼 a =4인데역시 d 2 > 4ac 성립하지않으므로모순. a =5일때실례는5x 2 5x +1=0. 127. 임의의실수 x, y 에대하여다음을만족하는함수 f : R! R 를모두구하여라. f(x + y) f(x y) =f(x)f(y) ( 폴란드 1992/1993 1 차 -4) 풀이 x = y =0대입하면 f(0) = 0. x =0만대입하면 f(y) =f( y). y = x; x 대입하여비교하면 f(2x) =f(x) 2 = f(2x). 즉 f 0. 128. 0 이아닌실수 a b c 에대해다음부등식을증명하고, 등호가성립할조건을구하여라. ( 북유럽 1988-2) a 3 c 3 3 µ a b abc + b c c a 증명 3 배하고다왼쪽으로넘기면 (a b) 3 +(b c) 3 0 으로정리됨. 별증 a3 b 3 3 + b3 c 3 3 =(a b) a2 +ab+b 2 3 +(b c) b2 +bc+c 2 3 (a b)ab +(b c)bc. 129. 모든실수 x 에대해다음조건을만족하는함수 f : R! R 을모두구하여라. ( 폴란드 1993 3 차 -5) f( x) = f(x); f(x +1)=f(x)+1; µ 1 f = f(x) x x 2 (x 6= 0) 풀이마지막식에서 f(x) =x 2 f( 1 x ) 임을참조하여, x! 1 x! 1 x +1! x x+1! 1! (x +1)! x+1 x +1! x 로순환시키며식을연속적으로변화시켜보면그냥 f(x) =x 가정리됨. 130. a, b, x, y 는 a + b + x + y<2 인양의실수들이다. a + b 2 = x + y 2 이고 a 2 + b = x 2 + y 이면, a = x 이고 b = y 임을보여라. ( 스웨덴 1995-3) 증명 a x =(y b)(y + b) 이고 (a x)(a + x) =y b. 즉 a x =(a x)(a + x)(y + b). 만일 a 6= x 라면 (a + x)(y + b) =1. 그럼 2 > (a + x)+(y + b) 2 p (a + x)(y + b) =2로모순. 즉, a = x 이고 b = y.
2.2 대수고급문제 119 131. 임의의양의실수 A, B, C 에대해다음부등식을증명하여라. max(a 2 B;B 2 C; C 2 A) max(a 2 A; B 2 B; C 2 C) 단,max(x; y; z) 는 x, y, z 중가장큰수를나타낸다. ( 레닌그라드 1991-y9-1) 증명좌변을 X, 우변을 Y 라하고, 귀류법으로 X<Y 라하자.WLOG,Y = A 2 A 라할수있다. A 2 B X<Y = A 2 A 이므로 A<B. 그럼또 A 2 C<B 2 C X<Y = A 2 A 이므로 A<C. 그럼 X C 2 A>A 2 A = Y 가되어모순. 132. a, b, c 가삼각형의세변의길이일때, 다음부등식이성립함을보여라. 1 a + 1 b + 1 c 1 a + b c + 1 b + c a + 1 c + a b ( 폴란드 1993/1994 1 차 -3) 증명 Ravi 치환하면금방. 1 y+z + 1 2 p yz + 1 2x +. 133. 양의실수 a, b, c, d 에대해다음부등식을증명하여라. ( 레닌그라드 1988-y9-E3) 증명보조정리 : 1 a + 1 b + 4 c + 16 d 64 a + b + c + d 1 a + 1 b 4 a+b 를반복하여적용. 134. 0 과 1 사이의세실수 X, Y, Z 에대해다음을증명하여라. ( 레닌그라드 1989-y9-E5) 2(X 3 + Y 3 + Z 3 ) (X 2 Y + Y 2 Z + Z 2 X) 3 증명 X 3 X 2 Y + Y 3 1 임을 X Y, X<Y 일때로나누어각각확인된다. 이런식셋을합하면 준식. 135. x + y + z = 1 2 인 x; y; z 0 에대해다음을증명하여라. ( 레닌그라드 1988-y8-6a) 1 x 1+x 1 y 1+y 1 z 1+z 1 3 증명보조정리 : a; b 0 일때 1 a 1+a 1 b 1+b = 1 a b+ab 1+a+b+ab 1 (a+b) 1+(a+b) 임을이용하면됨. 136. 주어진실수 a에대하여, 수열 fa n g을다음과같이정의하자 : a 1 = a, 8 µ < 1 a n 1 (a n 6= 0일때 ) a n+1 = 2 a n : 0 (a n =0일때 ) fa n g 에양수가아닌항이무한히많이나타남을증명하여라. ( 폴란드 1978 3 차 -5) 증명양이아닌항을하나만찾을수있어도충분. 귀류법으로모두양이라고가정. 그럼각항의앞항을생각하면모두 1보다커야함. 그럼또모두1+ p 2보다커야함. 귀납적으로, 모든항이 b보다크다면각항의앞항을고려했을때모든항은b + p b 2 +1> 2b 보다커야함. 그럼모든항이무한히커야한다는것이므로모순. 137. x, y, z는 1보다작은양의실수들이다. 다음셋중적어도하나는 1 를넘지않음을보여라. 4 ( 헝가리 1961-2) (1 x)y; (1 y)z; (1 z)x 증명귀류법. 1 < 2 p (1 x)y <(1 x) +y (GM-AM) 들을다더하면 3 < 3 의모순. 항이 6개짜리인 AM-GM으로봐도되고.
120 대수 138. 모든 x 에대해다음을만족하는함수 f : R! R 을모두구하여라. xf(x) =[x]f(fxg)+fxgf([x]) 단,[x] 와 fxg 는각각 x 의정수부와소수부를나타낸다. ( 몰도바 1999 최종 -y10-5) 풀이 x에정수를대입하면 f(n) = f(0). x에 0 < r < 1 을대입하면f(r) = f(0). 마지막으로 n + r 을대입하면 f(n + r) =f(0). 상수함수. 139. 모든실수 x 에대해다음을만족하는정수계수의다항식 P 를모두구하여라. P (P (P (P (P (x))))) = x 28 P (P (x)) ( 오스트리아 - 폴란드 2005-2) 풀이우선상수함수일때는없음. P (x) =ax n + bx n 1 + 로두면좌우변의최고차항의차수를비교할때 n 5 =28+n 2, 즉 n =2. P (x) =ax 2 + bx + c 로둘때최고차항의계수를비교하면 a 31 = a 2, 즉 a =1. 다시제2최고차항 (31차항) 을비교하면 16b =2b 에서 b =0. 다시제3최고차항 (30차항) 을비교하면 16c =2c 에서 c =0. P (x) =x 2 뿐. 140. 자연수 n 2 에대하여다음연립방정식을풀어라. ( 폴란드 1994/1995 1 차 -2) x 1 jx 1 j = x 2 jx 2 j +(x 1 1)jx 1 1j; x 2 jx 2 j = x 3 jx 3 j +(x 2 1)jx 2 1j;. x n jx n j = x 1 jx 1 j +(x n 1)jx n 1j 풀이 x i 가어떤값이어도 (x i < 0, 0 x i < 1, 1 x i 등으로경우를나눠따져보면됨 ) 항상 x i+1 > 0. 또한, x i 들은모두한꺼번에 1 이상이거나모두한꺼번에 1 미만임. 각각의경우대입하여식을모두변변합하면제곱들의합만남고그럼모두 1일때뿐. 141. 다음을만족하는실수해 (x; y; z) 를모두구하여라. ( 남아공 2001-2) x(1 y 2 )(1 z 2 )+y(1 z 2 )(1 x 2 )+z(1 x 2 )(1 y 2 )=4xyz =4(x + y + z) 풀이 xyz = x + y + z 임에서첫번째식은전개해보면그냥당연한식이되어버린다. 그래서 xyz = x + y + z 만풀면되고, 그럼임의의 x, y 에대해 (x; y; z) =(x; y; x+y xy 1 ) 이답. 142. 수열 fx n g 이다음과같이주어진다. x 1 = 1 2n 3 ; xn = 2 2n x n 1 (n 2) 임의의 n 2 N 에대하여 x 1 + x 2 + + x n < 1 이성립함을보여라. ( 폴란드 1994/1995 1차-12) 증명귀납법으로 x n 1 n(n+1) 임을증명할수있고, 그것으로 telescoping. 143. a; b; c; d 는양의무리수들이고 a + b =1 이다. c + d =1 일필요충분조건은모든자연수 n 에대하여 [na]+[nb] =[nc]+[nd] 가성립하는것임을증명하여라. ( 폴란드 1995 2 차 -3) 증명정방향의명제는 [na]+[nb] =n 1=[nc]+[nd] 로쉬움. 역명제는 c + d 0 =1로두면 ( 정명제에의해 ) 모든자연수 n에대해 [nd] =[nd 0 ]. 그럼 d = d 0. 144. 다음연립방정식의양의실수해는 x = y = z =1 뿐임을증명하여라. x + y 2 + z 3 =3 y + z 2 + x 3 =3 z + x 2 + y 3 =3 ( 스웨덴 1986-4) 증명 x; y 1 이면 x + y 2 x 2 + y 3 이므로 z 3 z, 즉 z 1. 그럼 x = y = z =1뿐. x; y 1 일때도비슷하게 x + y 2 x 2 + y 3 에서 z 1 로 x = y = z =1뿐.
2.2 대수고급문제 121 145. 두실계수삼차방정식 x 3 + mx n =0 과 nx 3 2m 2 x 2 5mnx 2m 3 n 2 =0 이딱하나의공통근을가질때, 첫번째식이중근을가짐을보이고, 이두방정식의근을 n 에대한식으로모두구하여라. ( 스페인 1995-5) 풀이삼차방정식이라했으므로우선 n 6= 0. m =0이면두식이 x 3 = n 으로일치하여공통근이딱하나가아니므로 m 6= 0. 공통근을 a라하고, a 3 = n ma 를둘째식에대입하여정리하면 a 2 + 3n m a + m = 0. 이걸다시첫식에대입하여정리하면 a2 = m 2m2 3 이고 a = q q 9n. 둘을연립하여풀면 4m 3 = 27n 2, 즉 m = 3 3 n 2 4 이고공통근은 a = 3 n 2, m = 3a2. 그럼첫식은 (x a)(x 2 +ax+ n a )=(x a)2 (x+2a) =0, 둘째식은 n으로나눠주고나면 x 3 +9ax 2 +15a 2 x 25a 3 = (x a)(x +5a) 2 =0. 146. a, b 는주어진실수이다. 방정식 ax + b[x] =ay + b[y] 가 x 6= y 인실수해를갖지않을 a, b 의조건을구하여라. ( 브라질 1985-5) 풀이 b a =0이면해가무한히많으므로 a 6= 0. WLOGx<y. ([x] [y]) = y x 6= 0 a 에서 [y] =[x]+n 이라할때 n 1 임을알수있다. n 1 <y x<n+1 임에서 1 1 n < y x n < 1+ 1 n, 즉 0 < y x n < 2 이므로 0 < b a < 2 임을알수있고, 실제이범위에서 n =1로하면 b a = x y 는항상해를갖는다. 답 a 6= 0이고, b a 0 또는 b a 2. 147. f(x) = ax + b 로주어진함수f가 f(0) 6= 0,f(f(0)) 6= 0을만족한다. F (x) =f( f(x) ) 이라할 cx + d 때 (f를 n번합성 ), F (0) = 0 이면이함수가정의되는모든 x에대해f (x) =x 임을증명하여라. ( 브라질 1990-5) 증명 f를 k번합성한것을 f k (x) = a kx+b k c k x+d 라하면, f k+1 (x) =f(f k (x)) = f k (f(x)) 임에서두점 k 화식이나오고, 그것을비교하면 bc k = cb k 이고 (a d)b k = b(a k d k ). f n (0) = 0 이라면 b n =0인데, 그럼 bc n =0이고, b 6= 0이므로 c n =0. 또 b(a n d n )=0에서 a n = d n. 그럼 f n (x) x. 148. 임의의실수 a, b, c, d 에대해다음을증명하여라. ( 레닌그라드 1987-y8-3) (1 + ab) 2 +(1+cd) 2 +(ac) 2 +(bd) 2 1 증명 LHS =1+(1+ab + cd) 2 +(ac bd) 2 1 149. f(0) = 0 이고, 모든음아닌실수 x 에대해 f(2x +1)=3f(x)+5 를만족하는, 실수값을갖는함수 f 를하나만찾아라. ( 브라질 1993-5) 풀이구간 I n =[2 n 1; 2 n+1 1) 들은음아닌실수들의집합을분할하고, x 7! 2x +1 에의해 I n! I n+1 로대응된다. 따라서, I 0 =[0; 1) 에서만아무렇게나정의하면나머지구간에서는귀납적으로역시잘정의된다. 150. x + y + z + t =0 과 x 2 + y 2 + z 2 + t 2 =1 을만족하는임의의실수 x, y, z, t 에대해다음부등식을증명하여라. ( 오스트리아 - 폴란드 1996-4) 1 xy + yz + zt + tx 0 증명준식은 (x + z)(y + t) = (x + z) 2 이므로오른쪽부등식은바로나옴. 왼쪽부등식은 j(x + z)(y + t)j 1 2 ((x + z)2 +(y + t) 2 ) (x 2 + z 2 )+(y 2 + t 2 )=1 로증명됨.
122 대수
제 3 장 기하 3.1 기하중급문제 1. 한원위에세점 A, B, C 가있다. 호 BC, CA, AB 의중점을각각 M, N, K 라하자. 그럼세현 AM, BN, CK 가한점에서만남을보여라. 증명만나는한점은삼각형 ABC 의내심이다. 2. ABCD 의점 D 에서 AB, BC 에내린수선의발을각각 P, Q 라하고, B 에서 AD, CD 에내린수선의발을각각 R, S 라하자. \PSR = \SPQ 이면 PR = QS 임을보여라. ( 아일랜드 2003-7) 증명 ( 대원외국어고 3 학년강성환 ) P A R D S B Q C \BPD = \BRD = \BQD = \BSD = 90 ± 이므로지름 BD에대한원주각이되고, 다각형 BQSDRP는원에내접한다. 그러므로원주각 \PSR = \SPQ 이면 PR = QS 이다. 3. 다음세가지가주어졌을때사각형 ABCD를구하는방법을설명하여라. (1) 네변의길이 (2) 변 AB와 CD가평행하다. (3) 변 BC와 DA가만나지않는다. ( 캐나다 1972-4) 풀이일반성을잃지않고 AB CD 라하자. 선분 AB 위에 BE = CD 가되는점E를잡는다. A를중심으로반지름 AD인원을그리고, E를중심으로반지름 BC인원을그린다. 두원의교점하나를
124 기하 D 라한다. 조건을만족하는사각형이존재한다면, D에서 BC와평행한직선을그려 AB와만나는점 E가위와같은점이므로, 두원의교점D가반드시있다. 다만, AB = CD 여서 A와 E가일치할때는두원도일치하므로교점이무한히많고, 그중아무것이나하나택하면된다. 이제평행사변형 BCDE가되도록 C를잡으면된다. 4. 삼각형 ABC 가주어져있다. BE = EF = FC 가되도록변 AB, BC 위에각각점 E, F 를작도하여라. 풀이 \EFB =2\ECB 임에서착안한다. \ABC의이등분선을그려 BC의수직이등분선과만나는점을 D라하자. CD의연장선이 AB와만나는점을 E라한다. 그리고, E를중심으로하고 B를지나는원이 BC와만나는점을 F 라하면된다. 5. 삼각형 ABC 에서세내각중에 \A 가가장크면세변중에가장긴것은 BC 임을증명하여라. 증명삼각형의임의의두내각 \B, \C와그에대응하는두변CA, AB에대해, 둘중큰각에대응하는변의길이가항상더큼을증명하자. 남은한변BC에서수직이등분선 L을올려, 남은한꼭지점 A가 L에서왼쪽에있는지오른쪽에있는지에따라살피면각의크기와변의길이사이의상관관계를관찰할수있다. 좀더엄밀히한다면 A에서 BC에수선을내려피타고라스의정리로논하면된다. 주고등부로간다면 cosine 제 2 법칙으로논할수도있다. 6. ABCD 는한변의길이가 1 인정사각형이다. 변 AB, BC, CD, DA 위에각각점 A 0, B 0, C 0, D 0 을잡는데, AA 0 = BB 0 = CC 0 = DD 0 =1=n 이되도록한다. 두사각형 AA 0 CC 0 과 BB 0 DD 0 이겹치는영역은정사각형이되는데넓이가 1=1985 이라고한다. n 을구하여라. (AIME 1985-4) 풀이 %EEE AP = (1-^2), so by similar triangles, x/1 = (1-x^2)/(1-1/n). Hence x = 1/ (1 + (1-1/n)^2), so the side of the small square is x/n = 1/ (n^2 + (n-1)^2). It is easy to check that n^2 + (n-1)^2 = 1985 for n = 32. [ 그림 ] 답 32 7. K, L, M, N 을, 어떤마름모의네변위에 ( 바깥쪽으로 ) 세운정사각형들의중심이라고하자. 다각형 KLMN 이정사각형임을증명하여라. ( 헝가리 1906-2) 힌트 (1) 이웃한두정사각형의중심과그사이에위치한마름모의한꼭지점을연결한삼각형이넷있는데, 이들이서로합동이고 90 ± 회전한위치에있음을보이면된다. (2) 똑같은마름모와정사각형타일들을무한히복제하여평면을가득깔아나가는것에서대칭성을발견한다. 8. AB < AC 인삼각형 ABC 가외접원 S 를갖는다. A 에서 BC 에내린수선이 S 와 P 에서다시만난다. X 가선분 AC 위의점일때, BX 가 S 와 Q 에서다시만난다. BX = CX 이면 PQ 가 S 의지름이고그역도성립함을보여라. ( 영국 2002/2003 1 차 -2)
3.1 기하중급문제 125 증명 (1) () 의증명 ) BX = CX 라가정하자. XBC 가이등변삼각형이고 XAQ 도그와닮았으므로 AQ 는 BC 와평행하고, 따라서 AP 와수직이다. 따라서, PQ 는원 S 의지름이다. (2) (( 의증명 ) PQ 가원 S 의지름이라가정하자. \PAQ =90 ± 이므로 AQ 와 BC 가평행. 따라서, XBC 는이등변삼각형이고 XB = XC 이다. (1), (2) 에의해두조건이동치임이증명되었다. 9. 두원 C 1, C 2 가점 S 에서만난다. S 에서의 C 1 의접선이 C 2 와 A( 6= S) 에서만나고, S 에서의 C 2 의접선이 C 1 과 B( 6= S) 에서만난다. A, B, S 를지나는원을 C 3 이라하자. S 에서의 C 3 의접선이 C 1, C 2 와각각 P ( 6= S), Q( 6= S) 에서만난다. PS = QS 임을증명하여라. ( 플란더즈 2003-2) 증명 선분 AS, SB, PQ 가각각원 C 1, C 2, C 3 의접선이되므로, 원과접선의성질에의해 \QAS = \SAB = \PSB \AQS = \ASB = \SPB \QSA = \SBA = \PBS 가성립한다. 이것에의해 4QSA 와 4SBA 와 4PBS 는닮음임을알수있다 (AA 닮음 ). 닮음인삼각형의각변의비는같으므로 4QSA 와 4SBA 에서 AB : AS = SB : SQ { 1 4SBA 와 4PBS 에서 AB : AS = SB : SP { 2 1, 2 에서 AB : AS = SB : SQ = SB : SP 가성립하고 SB : SQ = SB : SP 에서 SB 는같으므로 SP = SQ 이다. 10. 삼각형 ABC 에서 BC 의중점을 D 라하자. 직선 AD 위에 \CEA 가직각이되는점 E 를잡았더니 \ACE = \B 가되었다. \A 가직각이아니라면 AB = AC 가됨을보여라. ( 호주 2003-6)
126 기하 증명 AB 6= AC 인 4ABC 가존재한다고가정하자. A C B E D C! CE 위에 CE = C 0 E 인점 C 0 (6= B) 을잡자. 4ACC 0 는이등변삼각형이므로 \ACE = \AC 0 E (1) 4CED»4CC 0 B (SAS) 이므로 \BC 0 C =90 ± (2) 문제의조건 \ACE = \ABC 와 (1) 에의해, 따라서, AC 0 BC 는원에내접.(2) 에의해 \AC 0 E = \AC 0 C = \ABC \BAC = \BC 0 C =90 ± 이것은조건에모순. 따라서, AB = AC. 11. 삼각형 ABC 내부의점 O를지나는세직선DE, FK, MN을각각AB, AC, BC에평행하게그렸다. 단, F 와 M은 AB 위에, E와 K는 BC 위에, D와 N은 AC 위에있는점이다. AF AB + BE BC + CN CA 의값을구하여라. ( 플란더즈 2003-J2) 풀이 AF AB = 4AF C 4ABC = 4AOC 4ABC BE BC = 4ABE 4ABC = 4ABO 4ABC CN CA = 4CBN 4ABC = 4BOC 4ABC 세식을더하면 AF AB + BE BC + CN 4AOC + 4ABO + 4BOC = = 4ABC CA 4ABC 4ABC =1
3.1 기하중급문제 127 12. 다음그림과같이세변의길이가각각5, 6, 7인삼각형 ABC의외부에세개의정사각형을그리면세개의빗금친삼각형을얻는다. 그림에서세선분 AP, BQ, CR은각각빗금친삼각형의중선이다. 이세중선의길이의합은얼마인가? ( 한국 2004 1차-J10) 풀이 Q 가지나는선분의양끝점을 D, G 라하자. 4BQG 를 Q 에대해대칭시킨것을 4EQD 라하자. 그럼도형 DEQB 는삼각형이되고, DB = AB, DE(= BG) =BC, \EDB = \EDQ + \BDQ = \BGQ + \BDQ =180 ± \DBG = \B 이므로 4BDE 4ABC (SAS 합동 ) 가된다. 따라서, BQ = 1 2 BE = 1 2 AC 마찬가지로, AP = 1 2 BC, CR = 1 AB 이고, 세중선의합은 2 AP + BQ + CR = 1 2 (BC + CA + AB) = 1 2 (7+6+5)=9 이다. 별해 4DBG를, DB가 AB에일치하도록, B를중심으로 90 ± 회전하자. 그럼 D 0, B, C는일직선을이루고, BQ 0 은중점연결선으로 AC의길이의 1 2 임을알수있다. 13. QB 는 PA 를지름으로하는원의 PA 와평행한현이다. 두직선 PB 와 QA 가점 R 에서만난다. 이원의중심을 O 라하고, PORS 가평행사변형이되도록점 S 를잡자. 그럼 SP = SQ 가됨을보여라. ( 아일랜드 2003-2) 증명 ( 부산과학고 1 학년이태희 ) B R Q S A O P QB k PA 이므로 RO 에대해좌우대칭인그림이되고 (\QAP = \QBP = \BPA), PORS 는직사각형이다. \RSP = \RQP = \R 이므로 QRP S 는한원에내접하는사각형이고, \SPQ = \SRQ = \PAQ = \AP R = \SRP = \SQP 로 4SPQ 는 SP = SQ 인이등변삼각형이다.
128 기하 14. AB =12 이다. A 를중심으로하고반지름 8 인원과 B 를중심으로하고반지름 6 인원을그렸을때만나는교점중하나를 P 라하자. P 를지나는직선이두원과각각 Q, R 에서다시만나는데, QP = PR 이된다고한다. QP 2 을구하여라. (AIME 1983-14) 풀이 %EEE Put QP = 2a, take perp dist A, B from QP be b, c. Take line parallel to QP through B meeting perp bisector of QP at D. Considering triangle ADB gives 4a^2 + (b-c)^2 = 144. Also b^2 = 64 - a^2, c^2 = 36 - a^2. Hence 4a^2 = 130. 답 130 15. P 를평행사변형 ABCD 내부의임의의점이라하고, 세점 A, B, C 를지나는원의반지름을 R 이라하자. P 로부터가장가까운꼭지점에이르는거리는 R 보다크지않음을보여라. ( 헝가리 1907-2) 증명삼각형 ABC와 CDA는합동이므로일반성을잃지않고 P 가삼각형 ABC 안에있다고할수있다. 삼각형 ABC의외심에서세꼭지점에선분을긋고세변에도수선 ( 수직이등분선 ) 을내린다. 그럼삼각형 ABC는여섯개의직각삼각형으로영역이분할되고, 이들직각삼각형은빗변의길이가모두 R이다. 따라서 P 가어느직각삼각형에있든지그쪽꼭지점에이르는거리가 R 이하가된다. 16. AB = BC 인예각이등변삼각형 ABC의외접원의반지름의길이가 3이다. 점 A와외접원의중심 O를잇는직선과변 BC와의교점 P 에대하여 AP = 21 4 일때, 선분 AB의길이의제곱은얼마인가? ( 한국 2004 1차-S6) 풀이 AP 의연장선이원과만나는점을 G 라하자. B G P C O A 그럼 BP PC = AP PG = 21 4 3 4 = 63 16 한편, 각의이등분선정리에의해 AB : BP = AO : OP =3: 21 4 3=4:3 따라서, BP PC = 3 4 AB 1 4 AB = 3 16 AB2 이되고, 그럼 AB 2 =21. 17. 삼각형의두꼭지점중하나씩을지나는두직선이이삼각형을세개의작은삼각형과하나의사각형으로나눈다. 이네작은도형의넓이가모두같을수있는가? ( 영국 1989 주니어 ) 18. 원주위에세점이 A, B, C 순으로있을때호 BC 위에 AD = DC 가되게 D 를정하고, D 에서현 BC 에수선을내려그발을 E 라고하면 AB + BE = CE 임을증명하여라. 주 (1) BD에대해A를대칭이동 (2) E에대한B의대칭점을 B 0 으로할때4ABD 4CB 0 D
3.1 기하중급문제 129 19. 두개의 5 1 크기의직사각형을아래그림과같이 2 쌍의꼭지점이일치하도록겹쳐놓았다. (a) 겹쳐진부분의넓이를구하여라. (b) 두직사각형의교점중꼭지점이아닌두점A와 B 사이의거리를구하여라. ( 플란더즈 2004-J1) 풀이 (a) KA = x, AN = y 라하자. 그럼 x + y =5 대칭성에의해 KA = AR = PB = BQ = x, AN = NB = BL = LA = y 이다. 직각삼각형 ARN 에서 y 2 = x 2 +1, 즉 (y + x)(y x) =1 이고 y x =0:2 이다. 연립하여풀면 y =2:6, x =2:4. 겹쳐진마름모영역 ANBL 의넓이는 AN 을밑변으로볼때높이가 1 이므로 S = AN 1=y =2:6. (b) 마름모의넓이는두대각선의곱에 1 2 을하여구할수도있으므로, 2:6 =S = 1 2 LN AB = p 26 2 AB 따라서, AB = 10 1 p 26 2 p = 26 26 5. 20. 세점 A, B, C 가주어져있을때, A 를중심으로하고두이웃한변 ( 또는그연장선 ) 이각각 B 와 C 를지나는정사각형을작도하여라. 21. 한원의둘레위에 A, B, C, D 네점이차례로있고, 호 AB, BC, CD, DA 의중점으로역시원주위에있는네점 P, Q, R, S 를잡자. 이때 PR 과 QS 가서로수직임을보여라. ( 캐나다 1975-5) 풀이 PR 과 QS 의교점을 M 이라하자. 현 XY 에대한원주각을 \(XY ) 로쓰기로하자. \PMS = \PQM + \QP M = \(PS)+\(QR) = 1 (\BCD + \BAD) =90± 2 따라서, PR? QS 이다.
130 기하 22. 둘레의길이와넓이가같은삼각형이있다. 이삼각형의내접원의반지름을구하여라. ( 플란더즈예선 1990/1991) 풀이 S = 1 (a + b + c)r 2 에서 S = a + b + c 이므로 r =2 답 23. Q 를원위의임의의점이라하고, P 1 P 2 P 3 P 8 을이원에내접하는정 8 각형이라하자. Q 에서네지름 P 1 P 5, P 2 P 6, P 3 P 7, P 4 P 8 에이르는거리들의제곱의합은 Q 의위치에관계없음을증명하여라. ( 헝가리 1911-2 변형 ) 증명원위의점Q에서 P 1 P 5, P 3 P 7 에내린두수선은서로직교하며, 원의중심 O까지포함하여대각선의길이가원의반지름 r이되는직사각형을이룬다. 따라서, 피타고라스의정리에의해두수선의길이의제곱의합은 r 2 과같다. P 2 P 6, P 4 P 8 에대해서도마찬가지이며, 따라서네거리들의제곱의합은 2r 2 으로일정하다. 24. 한변의길이가 1 인정육면체에두정사면체 A 와 B 가내접하고있는데, 정사면체 A 는이정육면체의밑면에있는한대각선의양끝점과윗면에있는한대각선의양끝점을네꼭지점으로하고, 정육면체의나머지네꼭지점은정사면체 B 의꼭지점이된다. A [ B 의부피를구하여라. ( 플란더즈 1994-3) 풀이교집합은정팔면체인데합집합은... 방사팔면체로군하!1 12 1 1 3 8 = 1 2 25. 원에정사각형이내접하고있고, 이원과정사각형사이의한영역에작은정사각형이내접하고있다. 작은정사각형은큰정사각형과변으로접한다. 작은정사각형의넓이가 1 일때, 큰정사각형의넓이를구하여라. (AIME 2001 2 차 -6) 풀이 %EEE Let the large square have side 2, the small square side x. Then the radius is 2. The lines containing vertical sides of the small square pass a distance x from the center. So by Pythagoras, 2 = (1 + 2x)^2 + x^2, so (5x-1)(x+1) = 0, so x = 1/5. So ratio = 1/x^2 = 25. 답 25 26. 지름이 AB인원에서 \CAB =22 ± 인현AC를긋자. 주어진원에점 B에서내접하고, 현 AC에도접하는작은원을그린다. 작은원과직선 AC의접점P에서선분 AB에그은수선의발을 M이라할때, \CMB의크기는얼마인가? ( 한국 2004 1차-S10) 풀이 BQ 를작은원의지름이라하자. 접선과현이이루는각은원주각과같으므로 x = \AP Q = \QBP = \QP M 따라서, x = 1 2 \AP M = 1 2 (90± 22 ± )=34 ± 이다. \PMB + \PCB =180 ± 로 PMBC 가원에내접하는사각형이므로, \CMB = \CPB =90 ± x =56 ± 27. 두점에서만나는두원의교점중하나를 P 라하자. 두원의공통현을포함하지않는 P 를지나는직선중에서, 두원에의해잘린현의길이가같은직선 l 을작도하여라. 풀이 P 에대해두원중하나를점대칭이동하여얻은교점과 P 를연결하면됨.
3.1 기하중급문제 131 28. ABCD 는볼록사각형이다. 변 AB, BC, CD, DA 의중점을각각 A 0, B 0, C 0, D 0 이라하고, A 0 C 0 과 B 0 D 0 의교점을 P 라하자. 네영역 PD 0 AA 0, PA 0 BB 0, PB 0 CC 0, PC 0 DD 0 의넓이를각각 a, b, c, d 라할때, a + c = b + d 임을증명하여라. ( 아벨콘테스트 1993 결선 1a) 증명 j4pad 0 j = j4pdd 0 j 등을모두합하면 a + c 와 b + d 에모든넓이가동등하게기여하므로둘은같다. 29. 이등변삼각형 ABC 의꼭지각 \A 는 20 ± 이고, 이등변삼각형 DAC 의꼭지각 \D 는 100 ± 이다. 단, D 는직선 AC 에대해 B 의반대쪽영역에있다. AB = BC + CD 임을증명하여라. ( 플란더즈 1996-1) 증명 ABCD의바깥으로 CDE가정삼각형이되는점 E를잡자. 그럼 BCE는일직선을이루고, DEA는이등변삼각형이다. \DAB = \DEB =60 ± 이므로대칭성에의해 BAE도이등변삼각형이된다. 따라서, AB = BC + CE = BC + CD. 30. AB =13,BC =15,CA =17 인삼각형 ABC 가있다. D, E, F 는각각변 AB, BC, CA 위의점이고, AD=AB = p, BE=BC = q, CF=CA = r 이라하면 p + q + r =2=3 이고 p 2 + q 2 + r 2 =2=5 이다. j4defj=j4abcj 를구하여라. (AIME 2001 1 차 -9) 풀이 %EEE area ADC = α area ABC. area ADF = (1-γ)area ADC = α(1-γ) area ABC. Similarly for the other two triangles, so area DEF/area ABC = 1 - (α+β+γ) + (αβ+βγ+γα) = 1/3 + ½( (α+β+γ)^2 - (α^2+β^2+γ^2) ) = 1/3 + 1/45 = 16/45. 답 16=45 주 AB =13,BC = 15, CA =17 이라는것은불필요한조건인듯. 31. A 와 B 를원 k 위의점이라하자. 또다른원 l 의한호 k 0 이 A 와 B 를연결하고 k 를똑같은넓이의두부분으로나눈다고한다. 호 k 0 이원 k 의지름보다더길다는것을증명하여라. ( 헝가리 1914-1) 힌트 AB에평행한원 k의지름, 혹은 A를지나는원k의지름을그려그것과 k 0 의호AB를최단거리부등식으로비교한다. 풀이 k D B k' O A 원호 k 0 가원k의면적을이등분하므로 k 0 는임의의지름의한쪽에위치할수없다. 그러므로 k의모든지름은원호 k 0 와만나고k의중심P는원L의내부에존재한다. 그러므로 k의반지름 AO는원L안에있고 k 0 와지름AC의교차점 D는반지름OC상에있다. k 0 는 AD + BD보다크므로 DB > DC임을보이면된다. 중심이 D이고 C를지나는원이원 k안에있으므로 DB > DC이다. 32. 한직선위에있지않은세점 P, Q, R 이주어져있다. P, Q, R 을세변의중점으로하는삼각형을찾아라.
132 기하 풀이삼각형 PQR을그리면이것이원래삼각형의중점삼각형. 따라서, P 를지나고 QR에평행한직선, Q를지나고 RP 에평행한직선, R을지나고 PQ에평행한직선을그려, 세직선이둘씩만나는점을찾으면됨. 33. 서로수직인두평면에정사각형 ABCD 와 ABEF 가각각놓여있다. 직선 AE 와 BF 의교점을 O 라하자. AB =4 일때다음을계산하여라 : ( 가 ) 점 B에서평면 DOC와평면DAF 의교선에이르는거리 ( 나 ) 두직선AC와 BF 사이의거리 단, 공간상의두직선사이의거리는양끝점이각각두직선위에있고두직선과동시에수직인선분의길이를뜻한다. ( 루마니아지역예선 2005y8-2) 풀이 ( 가 ) 왼쪽그림과같이, BE, AF 의중점을각각G, H라하자. 그럼평면 DOC는평면CDHG와같고, 이평면과평면 DAF 와의교선은 DH이다. B에서 DH에내린수선의발을 P 라하고, 삼각형 BDH의넓이를이용해 BP의길이를구해보자. BDH는세변의길이가 2 p 5, 2 p 5, 4 p 2=BD 인이등변삼각형이므로, BD를밑변으로하는삼각형으로볼때넓이는 S = 1 2 BD h =2p 2 p 20 8=4 p 6 이된다. 또, S = 1 2 DH BP 이기도하므로 4p 6= p 5 BP, 즉 BP = 4p 30 답 5 ( 나 ) 오른쪽그림과같이, ACBI 가평행사변형이되도록하자. 그럼두직선 AC 와 BF 사이의거리는점 A 에서평면 FIB 에이르는거리와같다. IB 의중점을 M 이라할때, 대칭성에의해평면 FIB 는평면 FMA 와수직이므로, A 에서평면 FIB 에내린수선의발 Q 는직선 FM 위에있다. 삼각형 AF M 의넓이로부터, FM AQ = AF AM, 즉 p 16 + 8 AQ =4 2 p 2, AQ = 4p 3 3 답 34. ABC 는삼각형이고 X, Y, Z 는각각 BC, CA, AB 위의점으로선분 AX, BY, CZ 는 ABC 내부의한점 D 에서만난다. 사각형 DY AZ, DZBX, DXCY 중두개가원에내접하면나머지하나도또한원에내접함을보여라. ( 통신강좌 1989-C11 변형 ) 증명내대각얘기로풀면초간단. 35. 원 K 의지름 AB 의연장선위에점 C 를잡고, C 를지나원 K 와점 N 에서접하는접선을그리자. \ACN 의이등분선이직선 AN, BN 과만나는점을 P, Q 라할때, PN = QN 임을증명하여라. ( 아벨콘테스트 1994 결선 1b) 증명 \ACP = \PCN(= x) 이고 \BNC = \BAN(= y) ( 접현각 ) 이다. 이로부터삼각형의외각을계산하면 \NPQ = \PAC + \PCA = x + y \NQP = \QNC + \QCN = x + y 로 \NPQ = \NQP 이므로 4NPQ 는이등변삼각형이다. 36. 한변의길이가 6 p 2 인정사각형 ABCD 와, 이정사각형이놓인평면에평행하고길이가 12 p 2 인선분 EF 가있다. BCF 와 ADE 가둘다정삼각형이라고할때, 다면체 ABCDEF 의부피를구하여라. (AIME 1983-11)
3.1 기하중급문제 133 풀이 ABEF 는등변사다리꼴이되고, 이런다면체를 2개준비하여정사각형 ABCD와 B 0 C 0 D 0 A 0 이맞붙도록하면한변의길이가 12 p 2인정사면체가된다. 답 288 37. 두점 A 와 B 에서시작하여다음의순서로그림처럼원과교점들을작도하였다. ² A 를중심으로하고 B 를지나는원 ² B 를중심으로하고 A 를지나는원 ² C 를중심으로하고 A 를지나는원 ² D 를중심으로하고 B 를지나는원 ² E 를중심으로하고 A 를지나는원 ² F 를중심으로하고 A 를지나는원 ² G 를중심으로하고 A 를지나는원 M 이 AB 의중점임을보여라. ( 플란더즈 2005-J2) 증명 AB = r 이라하자. 정삼각형들에의해 \ABC + \CBD + \DBE =3 60 ± 으로평각이되므로 B는 AE의중점이다. EF = EA =2r =2AF 이므로 4EAF는세변의길이의비가 2:2:1인이등변삼각형이다. F 와 G는 AE에대칭이므로 M은 AE 위에있고, AF = FM 이므로 4FAM도이등변삼각형이다. 특히 \A가공통이므로 4FAM»4EAF 이고, 따라서 AM = 1 r 2 이다. 그러므로, M은 AB의중점이다. 38. 삼각형 ABC 에서 B 와 C 로부터그은두중선이서로직교한다. BC, CA, AB 의길이를각각 a, b, c 라할때, b 2 + c 2 을 a 에관한식으로나타내어라. ( 플란더즈예선 1992/1993) 풀이파푸스의중선정리와무게중심의성질, GBC 에서피타고라스정리를응용하면 5a 2. 39. 한변의길이가 a 인정사각형의네꼭지점을중심으로반지름이 a 인네원을그렸을때사각형중앙에그려지는영역의넓이를구하여라. 풀이네꼭지점을 ABCD라하고, 각변AB, BC, CD, DA에가장가까운교점을각각 E, F, G, H라하자. 주어진정사각형의한변의길이를 a, 구하는도형 EFGH의넓이를S, 돛꼴 DAEH의넓이를 X라하자. 삼각형 DEC가정삼각형이되므로, 각 ADE가 30 ± 임을먼저확인해두자. X = j부채꼴daej + j정삼각형decj j부채꼴cehdj = ¼ p 3 12 a2 + 4 a2 ¼ 6 a2 p 3 =( 4 ¼ 12 )a2 따라서, S = a 2 4X =(1 p 3+ ¼ 3 )a2.
134 기하 40. 정삼각형 ABC가주어져있다.! BC의연장선위에점 D,! BA의연장선위에점 E를잡아AE = BD 가되게한다. CE = DE 임을증명하여라. 증명! BD 연장선위에 BD = CF 가되도록점 F 를잡으면AE = CF 이므로 EBF는정삼각형. 그리고, CD의중점은 BF의중점과일치함. 41. a, b, c 는어떤직각삼각형의세중선의길이이고 a b c 이다. a2 + b 2 풀이세변의길이를 2x 2y <2z 라하면 z 2 = x 2 + y 2 이고, a 2 = x 2 +(2y) 2 ; b 2 =(2x) 2 + y 2 ; c = z c 2 은얼마인가? ( 플란더즈예선 1996/1997 1 차 ) 따라서, a 2 + b 2 c 2 = (x2 +4y 2 )+(4x 2 + y 2 ) x 2 + y 2 =5 답 5 42. 직사각형 ABCD 내부에두점 P, Q 가있는데, PQ 는 AB 에평행하다. X, Y 는변 AB 위의점으로 A, X, Y, B 의순서로있고, Z, W 는변 CD 위의점으로 C, Z, W, D 의순서로있다. 네영역 AXP W D, XPQY, BY QZC, WPQZ 의넓이는모두같다. BC =19,PQ =87,XY = YB+ BC + CZ = WZ = WD + DA + AX 일때 AB 는얼마인가? (AIME 1987-6) 풀이 %EEE Let distance of PQ from XY be h and XY = x. Then area XPQY = area WPQX, so (PQ + XY)h/2 = (PQ + WZ)(19-h)/2. Hence 2h = 19. Also AX + BY + DW + CZ = 2x - 38, so 2 AB = 4x - 38, or AB = 2x - 19. Hence area ABCD = (2x - 19)19. But 4 x area XPQY = (87 + x)19. Hence x = 106, so AB = 193. 답 193 43. P 는삼각형 ABC 내부의점이고, 직선 PA, PB, PC 는각각변 BC, CA, AB 와점 D, E, F 에서만난다. PD = PE = PF =3 이고 PA+ PB + PC =43 일때, PA PB PC 를구하여라. (AIME 1988-12) 풀이 %EEE Area PAB/area ABC = 3/(c+3). Hence 1 = 3/(a+3) + 3/(b+3) + 3/(c+3). Multiplying out we get abc = 9(a + b + c) + 54. 답 441 44. 어떤볼록사각형 ABCD 가지름 AB 인원에내접한다. 대각선 AC 와 BD 의교점을 S 라하고, S 에서 AB 에내린수선의발을 T 라하자. ST 가각 CTD 를이등분함을증명하여라. ( 뉴질랜드 2003-7) 증명 ( 과천고 2 학년홍성준 ) \ADB = \ACB = \STA = \STB =90 ± 이므로 ADST, BCST 는원에내접하는사각형들이다.
3.1 기하중급문제 135 따라서, 원주각에서 \STD = \SAD = \CAD = \CBD = \CBS = \CTS 이므로 ST 는 \CTD 의이등분선이다. 45. 중심이 O 인원에볼록사각형 ABCD 가내접하고있고, 대각선은서로직교한다. 꺾은선 AOC 에의해이사각형이같은넓이의두영역으로나뉘어짐을증명하여라. (Towns 1981 J3) 증명 1 ( 제주대기고 1 학년김대철, 대전어은중 1 학년임준혁 ) O 에서현 BD 에내린수선의발을 M 이라하자. OM 과 AC 가모두 BD 에수직이므로 OM k AC 이다. 따라서, 등적변형으로 j4oacj = j4macj 이고, 그럼 j OABCj = j MABCj 가된다. 또한, M은 BD의중점이므로, j4ambj = j4amdj, j4cmbj = j4cmdj 이다. 즉, j ABCMj = j ADCMj = 1 2 j ABCDj. 따라서, j OABCj = 1 j ABCDj = j OADCj 2 가된 다. 증명2 \AOB + \COD =2\ADB +2\CAD =2 90 ± 이므로 4OAB와 4OCD는 1 2 r2 sin µ 의넓이공식에의해넓이가같다.( 혹은 4OCD를 O를중심으로적당히회전시켜 4OAB와변으로접하도록하면넓이가같다는것이직접관찰된다.) 4OBC와 4ODA도마찬가지이고, 따라서두사각형은넓이가같다. 46. 정삼각형 ABC가한원과6개의점 D, E, F, G, H, J에서만나고있다. 이점들은삼각형의둘레를따라 A; H; J; B; D; E; C; F; G의순서로있다. CF =1,FG =13,GA =2,HJ =7일때선분DE의길이를구하여라. 답 2 p 22 47. 평면위에원 C 와점 P 가주어져있다. P 를지나고 C 와만나는직선들은 C 의현을하나씩결정한다. 이런현들의중점은한원위에있음을보여라. ( 캐나다 1991-3) 증명원의중심을 O 라하고현의중점을 M 이라하자. 그러면 OM 과 PM 은항상직교한다. 따라서, M 은 OP 를지름으로하는원위에있다.
136 기하 48. 예각삼각형 ABC 의세수선이외접원과각각점 D, E, F 에서만난다. 이세수선은다음중삼각형 DEF 의어떤선에해당하는가?(1) 수선 (2) 중선 (3) 각의이등분선 (4) 변의수직이등분선 (5) 앞의어느것도아니다. ( 플란더즈예선 1997/1998 1 차 ) 답 (3) 49. 세점 A( 8; 5), B( 15; 19), C(1; 7) 로이루어진삼각형에서각 A 의이등분선은 ax +2y + c =0 이다. a 와 c 를구하여라. (AIME 1990-7) 풀이 %EEE AB has gradient (5+19)/(15-8) = 24/7, AC has gradient (19-7)/(15+1) = 3/4. [ 그림 ] By Pythagoras, AB = 25, AC = 15. Extend AC to X with AX = 25, then X must be (7,-15), so the midpoint of BX is (-4,-17), and the equation of the line joining it to A is (x+8)/(y-5) = -4/22, or 11x + 2y + 78 = 0. hence a = 11, c = 78. 답 11, 78 별해각의이등분선정리를이용하면된다. 50. 한모서리의길이가 1 인정이십면체의한꼭지점 A 에서그대칭인꼭지점 B 까지표면을따라움직이는최단거리는얼마인가? ( 플란더즈예선 2000/2001 2 차 ) 풀이 전개도를생각하면... 정삼각형 4개를이어붙인도형에서양끝점의최단거리니까... p 답 7 51. ABCD 는 AB =13 p 3, AD =12 p 3 인직사각형이고 O 는이사각형의중심이다. 4OBC 를잘라내고남은도형에서, 선분 OA 와 OD 를접어서한면이트여있는사면체를만들었다. 이사면체의부피를구하여라. (AIME 1990-14) 풀이 %EEE I cannot see any elegant solution to this. It seems to be a horrible slog. Take the base ADC to have sides m, n, n where m = 432, n = 507. Then the three sides from the base to the apex O are all equal to ½ (m^2 + n^2). So if the foot of the altitude from O is X, then OXA, OXC, OXD are congruent, so X is the circumcenter of ADC. [ 그림 ] By similar triangles CE/AC = CD/CY, so CE = n^2/ (n^2 - m^2/4). Hence OX^2 = (m^2 + n^2)/4 - n^4/(4n^2 - m^2) = ½m^2(3n^2-m^2)/(4n^2-m^2). Area ADC = ½m (n^2 - m^2/4), so vol = (1/3) OX area ADC = m^2/24 (3n^2-m^2) = 594. 답 594 주 AB =2a, AD =2b, 사면체의높이를 h 라하면 Ã µ! h 2 =3b 2 b4 b 2 a 2 = b2 3 a 이식의결과가잘나오도록 a 와 b 를좋은값으로주는것이어떨까. 52. 직사각형이반원에내접한다는것은직사각형의밑변이반원의밑변위에있고직사각형의윗변이반원의원호에내접함을뜻한다. 반지름이 3 인반원에높이가 p 3 인직사각형이내접한다. 이반원에내접하는또다른직사각형이있는데, 먼저주어진직사각형과넓이가같다고한다. 이두번째직사각형의높이를구하여라. (IT 꿈나무올림피아드 2006 1 차 )
3.1 기하중급문제 137 풀이그림과같이직사각형의밑변의길이를 2x, 높이를 y 라하면, x 2 + y 2 =3 2. 처음의직사각형은 y = p 3 일때이므로 x = p 6. 이제두번째직사각형의 x, y 에대해살펴보자. 처음의직사각형과넓이가같다고했으므로 xy = p 3 p 6. 즉, x 2 y 2 =18. x 2, y 2 에대한연립방정식이다. 합과곱을알고있으므로이차방정식으로풀리고, 한근 ( 처음의직사각형 ) 을알고있으므로나머지한근은둘을서로바꾼것이다. 즉, x 2 =3,y 2 =6. 답 p 6 53. 원에내접하는볼록사각형 ABCD 가있다. 대변 AB 와 CD 의연장선이만나는교점을 P 라하고, AD 와 BC 의연장선이만나는교점을 Q 라하자. 그리고, 4PBC 의외접원과 4QAB 의외접원이만나는 B 이외의교점을 R 이라하자. P, Q, R 이한직선위에있음을보여라. 증명원주각에의해 \PRB = \PCB = \C, \QRB = \QAB = \A 이다. 그리고, ABCD 가원에내접하므로대각의합 \A+\C =180 ± 이다. 따라서, \PRB+\QRB =180 ± 이고, 이것은 P, R, Q 가한직선을이룸을뜻한다. 54. 삼각형 ABC에서 BC =4,AC =2,AB =3이고삼각형 A 0 B 0 C 0 에서 A 0 B 0 =1이다. \BAC + \B 0 A 0 C 0 =180 ± 이고 \ABC = \A 0 B 0 C 0 일때, 삼각형 ABC의넓이를S, 삼각형 A 0 B 0 C 0 의넓이를 S 0 S 이라고하자. S 0 의값을구하여라. ( 한국 2006 1차-J13) 풀이 1 (KAIST 응용수학과 05 학번이준경 ) 4A 0 B 0 C 0 을 2 배확대한삼각형 4A"B"C" 을 4ABC 에 AC 와 A"B" 이일치하도록맞붙이자. 그럼 \A + \A" =180 ± 이므로 BA(A")C" 은일직선을이룬다. 또, \B = \B" 이므로 4C"BC»4C"B"A" 이다. A"C" =x, B"C" =y 라하면이닮은삼각형의비례관계에서 x : y = y : x +3=2:4
138 기하 가되고, 그럼 y =2x, x +3 = 2y 이므로이것을풀면 x =1 임을알수있다. 따라서, j4abcj : j4a"b"c"j =3:x =3:1 이고, j4a"b"c"j =4S 0 이므로 S=S 0 =12 답 풀이 2 (KAIST 과학영재센터연구원고봉균 ) \B = \B 0 이므로 B 와 B 0 이일치시키고 C 0 2 AB, A 0 2 BC 가되도록놓자. 그리고, BC 위에 D 를잡아 AD k C 0 A 0 이되도록하자. 그럼 \A =180 ± \A 0 = \C 0 A 0 C = \ADC 가되므로 4CAB»4CDA. 여기서 CD : CA = CA : CB, 즉 x :2=2:4이므로 x =1. 따라서, BD =3이고 j4abdj = 3 4 S. 4ABD와 4C0 B 0 A 0 의닮음비가 BD : B 0 A 0 =3:1이므로 j4a 0 B 0 C 0 j = 1 9 j4abdj = 1 S 12 이다. 답 12 55. 삼각형 ABC 에서 \ACB 의이등분선이 AB 와만나는점을 D 라하자. 삼각형 ABC 의외심이삼각형 BCD 의내심과일치한다고한다. 이때, AC 2 = AD AB 임을증명하여라. ( 루마니아 2005 지역예선 y7-4) 증명 4ABC 의외심을 O 라하자. OBC 는 OB, OC 가외접원의반지름인이등변삼각형이므로 \OBC = \OCB. O 는 4BCD 의내심이기도하므로 \OBD = \OBC = \OCB = \OCD, 따라서 \DBC = \DCB 가된다. CD 는 \C 의이등분선이므로 \ACD = \DCB. 이로부터 \ACD = \DBC 이다. \A 는공통이므로 4ABC»4ACD 의닮음이되고, 따라서 AB : AC = AC : AD 이다. 56. 네면의넓이가모두같은사면체 ABCD 가있다. 이사면체의내부에있는임의의점 P 에서이사면체의네면에이르는거리의합이일정함을보여라. 증명사면체 PABC, PBCD, PCDA, PDAB 의부피의합을생각하면간단. 57. 수학적인거미줄기술이있는거미가다음그림과같이모든선분의길이가자연수인거미줄을만들었다.
3.1 기하중급문제 139 x 를구하여라. ( 플란더즈예선 2003 2 차 ) 풀이 x +5> 17 이고 x<5+9 이므로 x =13 답 58. 평면위에볼록사각형 ABCD 가주어져있을때, 네꼭지점에이르는거리의합 PA+ PB + PC + PD 가최소가되는점 P 의위치를찾아라. ( 통신강좌 1993-7-6) 풀이 최단거리부등식에의해 PA+ PC AC; P B + PD BD 이고, 각각등호는 P 가 AC 위에있을때, BD 위에있을때성립한다. 따라서, P 가두대각선 AC 와 BD 의교점일때, PA+ PB + PC + PD 는 AC + BD 와같게되고이때가최소이다. 59. AB 와 CD 가평행한사다리꼴 ABCD 에서, AB = 92, BC = 50, CD =19,DA =70 이라고한다. P 는변 AB 위의점이고, P 를중심으로하는한원이변 AD 와 BC 에접한다. AP 의길이를구하여라. (AIME 1992-9) 풀이 %EEE Extend AD, BC to meet at X. Then B lies on the bisector of AXB. But AP/PB = AX/BX. By similar triangles AX/BX = DX/CX = AD/BC = 7/5. Hence AP = (7/12)92 = 161/3. 답 161=3 60. AB = 1995, BC = 1994, CA =1993 인삼각형 ABC 가있다. CX 를한수선이라고하자. 두삼각형 ACX 와 BCX 의내접원이 CX 와만나는두접점사이의거리를구하여라. (AIME 1993-15) 풀이 %EEE We have AC = AU + CU = AT + CR = (AX - r) + (CX - r), where r is the inradius of ACX. Put AX = x, CX = z, AC = b. Then 2r = x + z - b. Similarly, if r' is the inradius of BCX and BX = y, then 2r' = y + z - a, so 2RS = 2r - 2r' = x - y - b + a = x - y - 1. Now x^2 = b^2 - z^2, y^2 = a^2 - z^2, so x^2 - y^2 = b^2 - a^2, and x - y = (b^2 - a^2)/c = 3987/1995. Hence RS = 996/1995. [ 그림 ] 답 332=665 61. 중심이 O 인원의접선 AD 와할선 ABC 가있다 (B, C, D 는원 O 위의점 ). D 에서 AO 에내린수선의발을 E 라고하면 \AEB = \ACO 임을증명하여라. (1993 도대회 ) 증명 직각삼각형의닮음과원의방멱에의해 AE AO = AD 2 = AB AC 따라서, 4ABE»4AOC 이다 (SAS 닮음 ).
140 기하 62. 볼록사각형 ABCD의변AB, BC, CD, DA 위에각각점 P, Q, R, S를잡는데, AP PB = BQ QC = CR RD = DS = k 로비가일정하도록한다. PQRS의넓이가 ABCD의넓이의 52 % 일때, k의값을구하여 SA 라. (IMTS R3-3) 풀이 j4ap Sj = k k+1 1 j4abdj k+1 임등을이용하면 0:48j ABCDj = j4ap Sj + j4bqpj + j4crqj + j4dsrj = 2k (k +1) 2 j ABCDj 이된다. 여기서 k에대한이차방정식을풀면 6k 2 13k +6=0,(3k 2)(2k 3) = 0, 즉 k = 2 3 or 3 2 답 63. 삼각형 ABC의변AB의중점을M이라하자. \ABC의이등분선이 AC와만나는점을 D라하자. MD? BD 이면 AB =3BC 임을증명하여라. ( 루마니아 2005 지역예선 y7-2) 증명 MD \ BC = N, 즉두직선 MD 와 BC 의교점을 N 이라하고, AC 의중점을 K 라하자. 4BNM 은 \B 의이등분선이밑변과수직으로만나므로이등변삼각형이다. 따라서, BM = BN 이고 MD = DN. 삼각형의합동과중점연결정리에의해 BC =2MK =2CN, 그럼 BC = 2 3 BN = 2 3 BM = 1 3 AB 가된다. 64. 삼각형 ABC 의각 B, C 의이등분선을각각 BP, CQ 라하자. 그리고, 꼭지점 A 에서직선 BP, CQ 에내린수선의발을각각 H, K 라하자. KH 와 BC 가평행함을증명하여라. ( 호주 1989-2) 풀이 AK, AH 의연장이 BC 와만나는점을 L, M 이라하면 KH 는 4ALM 의중점연결선. 65. 중심이 O 인원에두현 AC 와 BD 가원내부의점 X 에서만나는데, AC = BD =78,OA = 42, OX =18 이라고한다. 부채꼴과비슷하게생긴영역 CXD 의넓이를구하여라. (AIME 1995-14) 풀이 %EEE OY^2 = OA^2 - AY^2, so OY = 9 9. XY^2 = OX^2 - OY^2, so XY = 9 and OXY = 60^o. Hence DX = AX = AY + XY = 48, BX = CX = 30. Also AXB = 180^o - 2 60^o = 60^o. So area triangle AXB = ½AX BX sin 60^o = 360 3. ACB = XBC and their sum is AXB = 60^o, so ACB = 30^o. Hence AOB = 60^o. So area sector OAB = 422π/6 and area between AB and minor arc AB = 294π - 422( 3)/4. Hence required area = 294π - 81 3. [ 그림 ] 답 294¼ 81 p 3
3.1 기하중급문제 141 66. 정삼각형 ABC 의변 BC, CA, AB 의중점을각각 D, E, F 라하자. P, Q, R 은각각선분 EF, FD, DE 위의점으로, AP R, BQP, CRQ 가각각일직선을이룬다고한다. 4PQR 의넓이가 1 일때, 4ABC 의넓이를구하여라. (AIME 1998-12) 풀이 %EEE Compare the figures APRE and CRQD. If PE > RD, then RE > DQ, so QF > RD. Similarly that implies QF > PE, and that implies PE < RD. Contradiction. Similarly if PE < RD. So PE = RD. Similarly PE = QF. Put PF = QD = RE = x, AF = 1. Then BQD, PQF are similar, so x/1 = (1-x)/x or x^2 + x - 1 = 0, so x = ( 5-1)/2 (we discard the negative root). By cosine rule PQ^2 = x^2 + (1-x)^2 - x(1-x) = 3x^2-3x + 1 = 3(x^2+x-1)+4-6x = 7-3 5. Area ABC/area PQR = 22/x2 = 7 + 3 5. 답 7+3 p 5 67. M 은한변의길이가 4 인정삼각형 3 개를붙여만든등변사다리꼴이다. M 0 은 M 을뒤집어놓은사다리꼴이다. M 과 M 0 의밑변을같은직선위에두고두사다리꼴을이직선위에서미끄러뜨릴때, 두사다리꼴이겹치는영역의넓이는최대얼마인가? ( 플란더즈예선 1998/1999 2 차 ) 답 10 p 3 68. AB =8,BC =6,BD = 10, \A = \D, \ABD = \C 인사각형 ABCD 가있다. CD 의길이를구하여라. (AIME 2001 2 차 -13) 풀이 %EEE Extend AB, CD to meet at E. Then triangles ECB, EBD are similar. Hence BD/ED = CB/EB, so 10/ED = 6/(ED-8). Hence ED = 20. Also EC/CB = EB/BD, so EC = 7.2. Hence CD = 12.8 = 64/5. 답 64=5 69. 한변의길이가 1 인정육면체 ABCD-A 1 B 1 C 1 D 1 가있다. 모서리 BC 의중점을 E 라하고, 면 CDD 1 C 1 의중심을 M 이라하자. 평면 AEM 이이정육면체와만나는단면의넓이를구하여라. ( 플란더즈 1998-2) 풀이이평면이CC 0, DD 0 과만나는점을P, Q라하자. 단위정육면체 CDGH-C 1 D 1 G 1 H 1 을맞붙여그리면, M은 AH 1 의중점. D 1 G 1 의중점을 F 라하면EF의중점도 M이고, 따라서 AEH 1 F p 는평행사변형이고구하는단면의넓이는이것의절반. 대충구하면넓이는 14 답 4 70. 사각형 ABCD 에서 AB =5,BC = p 50, CD =4,AD =3 이고 AC =5,BD = p 58 이다. X, Y 가각각대각선 BD 와 AC 의중점일때 XY 2 을구하여라. (2000 대전. 충남영재수학교실 2 차평가 ) 풀이 파푸스의중선정리에의해 2(AX 2 + BX 2 )=AB 2 + AD 2 2(AY 2 + XY 2 )=AX 2 + CX 2 2(CX 2 + BX 2 )=CD 2 + BC 2 첫번째식과세번째식을더한후두번째식을이용해 2(AX 2 + CX 2 ) 을소거하면 AC 2 + BD 2 +4XY 2 = AB 2 + BC 2 + CD 2 + DA 2 25 + 58 + 4XY 2 =25+50+16+9 따라서,4XY 2 =17, 즉 XY 2 = 17 4 답
142 기하 71. AB =6,AC =8 인 4ABC 에서 \BAC 의이등분선이 BC 와만나는점을 D 라했더니 AD = AB 가되었다. C 에서직선 AD 에내린수선의발을 M 이라할때, AM 의길이를구하여라. ( 통신강좌 1990-1-1) 풀이각의이등분선정리에서 BD : DC = BA : AC =3:4. 따라서, BD =3x 라하면DC =4x. BD의중점을 N이라하면, AN 2 + ND 2 = AD 2, AN 2 + NC 2 = AC 2 임에서 µ 11 2 µ 3 2 28 = AC 2 AD 2 = NC 2 ND 2 = 2 x 2 x =28x 2 따라서, x =1. 4AND와 4CMD가 AA-닮음이므로 CD : DM = AD : DN =6: 3 =4:1. CD =4 2 이므로 DM =1. 따라서, AM =7. 별해 AB, AC의중점을각각 E, F 라하고, EF와 AD의교점을N이라고하면, EF k BC 이고, 가정에서 AB = AD 이므로, AN = AE = 1 AB. 2 직각삼각형 AMC에서 F 는빗변의중점이므로 AF = CF = MF = 1 2 AC 이다. 따라서삼각형 FAM은이등변삼각형이고, \FMA = \FAM = \MAB = 1 2 \A. 그러므로 AB k FM. 한편 EF k BC 이므로, \EFM = \B = \ADB = \MNF 이고삼각형 MNF는이등변삼각형. 고로 MN = MF = 1 2 AC. 따라서 AM = AN + MN = 1 (AB + AC). 2 72. 직각삼각형 ABC 의넓이를 S 라하고외접원과내접원의반지름을각각 R, r 이라하자. r(2r + r) =S 임을증명하여라. 증명직각삼각형의둘레의길이는 2r +4R. 73. 한변의길이가 1 인두개의정사각형이같은중심을갖고공통의팔각형영역을갖는다. 이팔각형의어느한변의길이가 43=99 일때, 이팔각형의넓이를구하여라. (AIME 1999-4) 풀이 %EEE Let the sides of the triangle be x, y, (x^2+y^2). It is clear from the diagram that the perimeter of the triangle is the side of the square 1. Solving, y = (1-2x)/(2-2x) and octagon side = 1-x-y = (2x^2-2x+1)/(2-2x) = 43/99. Hence 198x^2-112x + 13 = 0. Product of roots is 13/198. So area octagon = area square - 4 x area triangle = 1-2xy = 1-26/198 = 86/99. [ 그림 ] 답 86=99 74. P 와 Q 는 x 축위를움직이는점이고 PQ =8 로일정하다. A(0; 6), B(16; 10) 에대해사각형 ABQP 의둘레의길이가최소가될때, PQ 의중점 M 의 x- 좌표를구하여라. ( 플란더즈예선 1998/1999 1 차 ) 힌트최단거리부등식,7 75. AB k DC, BC? AB, AC? BD 인사다리꼴 ABCD 가있다. AB = p 11, AD = p 1001 일때, BC 의길이를구하여라. (AIME 2000 2 차 -8)
3.1 기하중급문제 143 풀이 %EEE Put BC = x. Then AC = (x^2+11). ABC and BCD are similar. So BD/BC = AC/AB, so BD = x( (x^2+11))/ 11. Also cos ABD = cos ACB = x/ (x^2+11). Applying cosine rule to ABD, we get 1001 = AD^2 = AB^2 + BD^2-2AB BD cos ABD = 11 + x^2(x^2+11)/11-2x^2. Hence (x^2+99)(x^2-110) = 0, so x = 110. p 답 110 76. 4PQR의세변의연장선! PQ,! QR,! RP 의연장선위에 PA PQ = QB QR = RC = a 가되도록각각점 A, B, RP C를잡자. 4ABC의넓이가 4PQR의 61배가되도록하려면 a를얼마로해야하는가? (1991 도대회 ) 77. AB =5,BC =6,CA =7 인 4ABC 내부에한점 P 가있다. P 를지나는이삼각형의현을각변에평행하게하나씩그렸더니, 이세현의길이가모두같았다. 변 AB 와만나는두현이변 AB 의길이를분할하는비를구하여라. (1995 도대회 ) 78. 세변의길이가 90,120,150 인직각삼각형이있다. 이직각삼각형의내접원을그리고, 빗변이아닌두변과평행하며이내접원에접하는두직선을그리면, 원래직각삼각형의내부에작은직각삼각형영역두개를얻는다. 이두작은직각삼각형의내심을잇는선분의길이를구하여라. (AIME 2001 2 차 -7) 풀이 %EEE Let B be the vertex with angle 90^o. Suppose it is a distance x from the points of contact of the incircle. Then chasing around the triangle using the fact that the tangents from a point have equal length, we get x = (90 + 120-150)/2 = 30. Evidently this is also the radius of the incircle. The other two triangles are similar. The top one has sides 30, 40, 50 and inradius 10, and the right-hand one has sides 45, 60, 75 and inradius 15. We can regard the dotted line as the hypoteneuse of a right-angled triangle with vertical side 60+10-15 = 55, horizontal side 60+15-10 = 65. Hence length (55^2 + 65^2) = 5 290. 답 5 p 290 79. 원 O 를중심으로하는원 C 가있다. AB 는이원의지름이고, BM 은 B 에서의이원의접선이다. 원주위의또다른점 E 에서의접선이 BM 와 F 에서만난다. AE 의연장선이 BM 과만나는점을 D 라할때, BF = FD 임을증명하여라. (1995 교육청경시 ) 80. 직각삼각형 ABC 의내접원의반지름을 r 이라하자. r 은빗변의 1=4 보다짧고, 또다른두변의 1=2 보다짧음을보여라. ( 헝가리 1925-3) 증명직사각형 ABCD를생각하면내접원이완전히포함되므로지름이한변의길이보다짧아서 2r <a;c. 직각점에서빗변에내린높이를 h, 중선의길이를 d라하면, 직각점을지나빗변에평행한선에대해직각삼각형이완전히한쪽에있으므로내접원도완전히한쪽에있어서두평행선사이의거리 h보다지름이짧으므로 2r <h d = 1 2 b. 81. 다음그림은한변의길이가 a 인정사각형의각꼭지점과그꼭지점을포함하지않는변의중점을연결한선분들을그려얻은도형이다. 빗금친영역의넓이를구하여라. ( 플란더즈예선 1990/1991)
144 기하 풀이정사각형 ABCD라하고, AB부터각변의중점을K, L, M, N. 또, 빗금친팔각형의꼭지점들을 N에가까운것부터 PQRSTUVW라하자. 그럼구하는넓이는 S = a 2 4(j4PAKj + j4qkbj) = a 2 4( 1 8 a2 + 1 12 a2 )= 1 6 a2 답 82. A, B, C 는 AB = AC 를만족하는한원위의세점이다. 선분 BC 위의임의의점을 E 라하고, 직선 AE 가원과다시만나는점을 D 라하자. AD AE 는 E 의선택과관계없이항상일정함을증명하여라. ( 아벨콘테스트 1997 결선 2b) 증명이등변삼각형이므로 \ABC = \ACB 이고, 원주각에의해 \ABC = \ADC 이다. 즉, \ACB = \ADC 가되고 \CAD 는공통이므로 4ADC»4ACE 따라서, AD : AC = AC : AE 이고풀면 AD AE = AC 2 으로일정한값이된다. 별증1 A에서 BC에내린수선이BC와원과만나는점을각각 M, N이라하자. 이등변삼각형이므로 AN에대해좌우대칭의그림이되고따라서 AN은지름이다. 그럼지름에대한원주각으로 \ADN = \R = \AME 이고, \DAN 은공통이므로 4ADN»4AME 따라서, AD : AN = AM : AE 이고풀면 AD AE = AM AN 이다. M, N 은 4ABC 에의해결정된점이므로이값은일정한값이다.
3.1 기하중급문제 145 별증 2 A 에서 BC 에내린수선의발을 M 이라하고, 그림과같이 AB = AC = a, AM = h, BM = CM = b, ME = x 라하자. 그럼다음과같이식을정리할수있다. AD AE =(AE + ED)AE = AE 2 + AE ED = AE 2 + BE EC (* 방멱 ) =(h 2 + x 2 )+(b + x)(b x) = h 2 + b 2 = a 2 따라서, 일정한값이다. 83. 두원 k 1, k 2 가한점 P 에서접한다. P 를지나는한직선이원 k 1, k 2 와각각 A 1, A 2 에서다시만나고, P 를지나는또다른직선이원 k 1, k 2 와각각 B 1, B 2 에서다시만난다. 삼각형 PA 1 B 1 과 PA 2 B 2 가닮았음을증명하여라. ( 헝가리 1933-3) 증명먼저두원이외접하는경우를생각하자. 두원의공통내접선을 XY 라하자 (X는 \A 1 PB 2 쪽에, Y 는 \B 1 PA 2 쪽에있는점이다 ). 접선과현이이루는각과맞꼭지각에의해 \A 1 B 1 P = \A 1 PX = \A 2 PY = \A 2 B 2 P 이된다. 마찬가지로 \B 1 A 1 P = \B 2 A 2 P 가된다 ( 대신에맞꼭지각 \A 1 PB 1 = \A 2 PB 2 를얘기해도된다 ). 따라서, 4PA 1 B 1 과 4PA 2 B 2 는 AA 닮음이다. 두원이내접하는경우에도 \A 1 B 1 P = \A 1 PX = \A 2 B 2 P 등으로비슷하게된다 ( 맞꼭지각을생각할필요가없으므로더쉽다 ). 별증반직선 PA 1, PB 1, 원 k 1 을모아집합X라하고, 반직선 PA 2, PB 2, 원 k 2 를모아집합Y라하자. 두원이외접하든내접하든, 집합 X와집합Y는 P 를중심으로하는확대-축소닮음관계에있다. 따라서, 4PA 1 B 1 과 4PA 2 B 2 도 P 를중심으로같은닮음관계에있다.
146 기하 84. 한변의길이가 1 인정오각형 ABCDE 가주어져있다. M 을이도형의내부혹은선분위의점이라하자. 점 M 으로부터가장가까운꼭지점까지의거리를 r 1, 그다음으로가까운꼭지점까지의거리를 r 2, ::: 이런식으로계속하여, M 으로부터다섯꼭지점까지의거리를 r 1 r 2 r 3 r 4 r 5 이되도록정하자. (a) r 3 이최소값이되는점M의자취를구하여라. (b) r 3 이최대값이되는점M의자취를구하여라. ( 소련 1962-8) 풀이 Let X be the midpoint of AB and O the center of ABCDE. Suppose M lies inside AXO. Then ME = r 3. So we maximise r 3 by taking M at X, andweminimiser 3 by taking M as the intersection of AO and EB. AXO is one of 10 congruent areas, so the required loci are (a) the 5 midpoints of the diagonals, and (b) the 5 midpoints of the sides. 85. 세점 ( 6; 0), (0; 12), (16; 0) 을꼭지점으로하는삼각형의둘레와내부에있는격자점들중에서, 두점 (0; 0), (5; 0) 과이루는삼각형이둔각삼각형이되는격자점의개수를구하여라. ( 플란더즈예선 1996/1997 1 차변형 ) 풀이어떤꼭지점이둔각인가에따라나누어 (0; 0); (5; 0) 과둔각삼각형을이루는격자점을구한다. ² (0; 0) 이둔각인경우삼각형내부나둘레에있는점중에서 x 좌표가 0보다작은점들이다. x = 1 :y =0 10 (11개) x = 2 :y =0 8(9개 ) x = 6 :y =0(1개 ) 격자점개수 : 11+9+ 1=36개. ² (5; 0) 이둔각인경우삼각형내부나둘레에있는점중에서 x 좌표가 5보다큰점들이다. (0; 12) 와 (0; 16) 을잇는기울기 3 4 인직선아래에있는점으로 x좌표가 4 단위로변할때규칙성이있다. x =5:y =0 8(9개 ) Ã 답에는포함되지않는다. x =6:y =0 7(8개 ) x =7:y =0 6(7개 ) x =8:y =0 6(7개 ) x =9:y =0 5(6개 ) 격자점개수는 (8+7+7)+(6+5+4+4)+(3+2+1+1)=48이다. ² 나머지한점이둔각인경우는그점이 (0; 0) 과 (5; 0) 을잇는선을지름으로하는원내부에있을경우이다. 원내부와둘레에있는점중에서 y 0 인점은삼각형안에포함되므로,(x 5 2 )2 +y 2 ( 5 2 )2 인점의개수를구한다. y =0:x =0 5 y =1:x =1 4 y =2:x =1 4 ) 6+4+4=14 개. 총 36 + 48 + 14 = 98. 답 :98 개. 86. 양의정수 a, b 에대하여, y =(x a) 2 의그래프와 y = b 2 x 2 의그래프의두교점과두그래프의 y- 축과의교점들을꼭지점으로하는사각형의넓이가 12 p 7 일때, a + b 의값을구하여라.( 단, a<b 이다.) ( 한국 2005 1 차 -J13) 풀이두그래프의 y-절편은 a 2 과 b 2 이고, 두교점은그래프를그려보면 x-좌표의부호가서로다르게나타난다. 즉,(x a) 2 = b 2 x 2 의두근을, 라하면구하는사각형의넓이는삼각형 2개의넓이의합으로보아서 1 2 (b2 a 2 )(j j + j j) = 1 2 (b2 a 2 )j j =12 p 7 이된다.2x 2 2ax + a 2 b 2 =0에서 + = a, = 1 2 (a2 b 2 ) 이므로 위의넓이의식을제곱한것에이것을대입하면 ( ) 2 =( + ) 2 4 =2b 2 a 2 (b 2 a 2 ) 2 (2b 2 a 2 )=2 6 3 2 7
3.1 기하중급문제 147 이된다. 만일 a 와 b 가홀짝이서로다르면 b 2 a 2 은홀수이고 2b 2 a 2 2 0or0 1(mod4) 로좌변이 4 의배수가될수없어모순. 따라서, a 와 b 는홀짝이같고, 그럼 b a 2, b + a 4 이므로 b 2 a 2 8. (b 2 a 2 ) 2 이 2 6 3 2 7 의약수임에서 b 2 a 2 = 8, 12, 24 만이가능하다. 이때, 각각 2b 2 a 2 = 63, 28, 7 이고, b 2 = 55, 16, 17 이되어, 가능한것은 b 2 =16 뿐이다. 즉, b =4,a =2. a + b =6 답 87. a 1995 c 크기의직육면체꼴의나무토막이있다 (a 1995 c). 이나무토막을어느한면과평행한면으로두조각으로자르는데, 그중한조각이원래의나무토막과닮은꼴이되도록하려고한다. 주어진 (a; c) 에대해, 이렇게자르는방법은몇가지가가능한가? (AIME 1995-11) 풀이 %EEE We must cut the longest edges, so the similar piece has dimensions a x 1995 x k for some 1 k < c. The shortest edge of this piece cannot be a, so it must be k. Thus a x 1995 x c and k x a x 1995 are similar. Hence c = 1995^2/a, k = a^2/1995. Now 1995 = 3 5 7 19, so 1995^2 has 3^4 factors, of which (3^4-1)/2 = 40 are < 1995. 답 40 88. J 씨는직사각형꼴의농장을갖고있는데, 그림과같이 9 개의정사각형구역으로나누었다. 가장작은구역의넓이는 1 이다. 두번째로작은영역 A 의넓이를구하여라. ( 플란더즈예선 2000/2001 2 차 ) A 풀이 A의한변의길이를 a라하고, 그오른쪽사각형의한변의길이를 b라하자. 그럼그오른쪽은 b+1, 그아래는 b+2, 그왼쪽은b+3짜리가된다. 이다섯사각형구역의폭을비교하면 a+2b+1 = 2b+5, 따라서 a =4 답 89. 중심 O인원위에다섯점A, B, C, D, E가차례로있다. AOC, BOD, COE, DOA, EOB는각각B, C, D, E, A를내부에포함하는어떤정다각형의일부들 ( 연속한세꼭지점들 ) 이라고한다. 이 5개의정다각형의변의개수를각각 a, b, c, d, e라할때, 1 a + 1 b + 1 c + 1 d + 1 의가능한값을모두구하여라. e (IT꿈나무올림피아드 2006 1차 ) 풀이정 n 각형의한외각의크기는 360± n 이므로, 한내각의크기는 180 ± 360± n. 따라서, 720 ± = \AOC + \COE + \EOB + \BOD + \DOA = µ180 ± 360± + µ180 ± 360± + µ180 ± 360± a b c µ 1 2= 2 1 µ 1 + a 2 1 µ 1 + b 2 1 µ 1 + c 2 1 + d + µ 1 2 1 e µ180 ± 360± d + µ180 ± 360± e 따라서, 1 a + 1 b + 1 c + 1 d + 1 e =5 1 2 2= 1 2. 답 1 2
148 기하 90. 중점삼각형이란원래삼각형의각변의중점을세꼭지점으로하는삼각형을말한다. 부피가 1 인정사면체가있다. 이정사면체의각삼각형면에중점삼각형을그리고, 그중점삼각형을밑면으로갖는작은정사면체를각각붙여새로운다면체를얻는다. 이런과정을두번더거치자 ( 그래서모두세번거친다 ). 이결과로얻어진다면체의부피는얼마인가? (AIME 2001 2 차 -12) 풀이 %EEE The side length of each small tetrahedron on the face of the original tetrahedron is 1/2 the side length of the original, so its volume is 1/8. There are 4 of them so total volume 1/2. The new figure has 24 faces. The second time, each small tetrahedron has volume 1/64, so their total volume is 24/64 = 3/8. The new figure has 3 24 = 72 faces which are subsets of faces of the previous figure and another 72 outside, total 144. The tiny tetrahedra have vol 1/512, so total 144/512 = 9/32. Hence total vol = 1 + 1/2 + 3/8 + 9/32 = (32 + 16 + 12 + 9)/32 = 69/32. 답 69=32 91. 임의의삼각형에서, 대변보다긴수선은많아야하나뿐임을증명하여라. ( 헝가리 1942-1) 증명한꼭지점 A에서대변에내린수선은그변에이르는최단거리이므로변 AB, AC의길이이하임. 즉, h A >a라하면, b; c h A >a h B ;h C 이므로그런경우는많아야하나뿐. 92. 원 1의중심이원 2 위에있고, 두원은두점A, B에서만난다. B에서그린원 2의접선이원 1과점C에서다시만난다. AB = BC 임을증명하여라. (Towns 1991가을 JO1) 증명 원 1 의중심을 O 라하자. 접현각의성질로부터 \OBC = \OAB. 따라서, 4OAB 와 4OBC 는같은크기의밑각을갖는 OA = OB = OC 인이등변삼각형이므로합동이다. 이로부터 AB = BC. 93. 한각의내부에중심이각각 A, B인두원이있다. 두원과각의두변은서로접한다. AB를지름으로하는원도각의두변에접함을증명하여라. (Towns 1991가을 SO1) 증명각을 XOY, AB의중점을 M이라하고, A, M, B에서 OY 에내린수선의발을각각 C, N, D라하자. 그리고, 원 A와 B, M의반지름을각각a, b, m이라하자. AB가원M의지름이므로 m = AB 2 = a+b 2 이다. 그런데, 사다리꼴 ACDB에서 MN은중점연결선 이므로윗변 MN = AC+BD 2 = a+b 2 이다. 이것은 m과같으므로, 원 M은 OY 에접한다. 대칭적으로, OX에도접한다.
3.1 기하중급문제 149 94. 삼각형 ABC 의외심을 O, 수심을 H, 그리고변 BC 에대한 O 의대칭점을 D 라하자. AHDO 가평행사변형임을보여라. 증명 O에대한B의대칭점을E라하면중점연결정리에의해 OD = CE. 그리고각을좀고려해주면 AHCE가평행사변형... 95. 볼록사각형 ABCD에서변 BC와 DA의중점을각각 M, N이라하자. 대각선 AC가 MN을이등분한다고할때, 두삼각형 ABC와 ACD의넓이가같음을증명하여라. (Towns 1988봄 JO2) 증명 MN \ AC = P 라하자. j4abcj =2j4AMCj =2(j4AMP j + j4cmpj) =2(j4ANP j + j4cnpj) =2j4ANCj = j4acdj. 96. 정사각형 ABCD의내부에점M이있다. \MAC = \MCD = x 라할때, \ABM 의크기는얼마인가? (Towns 1988가을 JA2) 풀이 MAC의외접원은 CD와 C에서접함. 즉 B를중심으로함. 따라서, \ABM = 2\ACM = 2(45 ± x). 97. AM은 4ABC의한중선이다. 4ABM의내접원의반지름이 4ACM의내접원의반지름의 2배가될수있는가? (Towns 1989봄 JO2) 98. 사다리꼴 ABCD에한원이내접한다. 빗변을지름으로하는두원이서로접함을증명하여라. (Towns 1989봄 SO2) 99. 오각형 ABCDE는내접원을가지며대각선 AD와 CE가내심O에서만난다. BO와 DE가서로수직임을증명하여라. (Towns 1991봄 JA2) 100. 어떤사다리꼴이있고, 한직선이이사다리꼴의각변, 각대각선과 6개의점에서만나는데, 교점으로끊긴이직선의 5개의선분의길이가모두같다 ( 윗변, 밑변과는그연장선에서만난다 ). 이사다리꼴의윗변과밑변의길이의비를구하여라. (Towns 1988봄 SA1) 101. 4ABC의수심을N이라하자. 4ABN, 4ACN, 4BCN의외접원들은모두같은크기임을증명하여라. (Towns 1988가을 SO2) 102. 반지름이각각 1, r, r 인세원이서로외접하고있다. 이세원에외접하는 ( 즉, 세원을내부에포함하고, 각원이두변에접하며, 각변도두원에접하는 ) 삼각형이존재하는 r 의값의범위를구하여라. (Towns 1991 봄 JO2) 풀이 r의크기를연속적으로움직여가며관찰하자. r이최소일때는 r인원과1인원의공통외접선이서로평행에이를때의직전. 즉, r> 1. r 2 이최대일때는두공통외접선이완전히일치하여한직선을이루게될때의직전. 그때의상황은피타고라스정리로 (r +1) 2 =(r 1) 2 + r 2 일때이고구하면 r =4. 즉, r<4. 답 1 2 <r<4 103. 세높이가 4, 7, 10 인삼각형은존재하지않음을증명하여라. (IMTS R8-1) 증명세변의길이는 x=4, x=7, x=10 이고 1=4 > 1=7+1=10 이므로삼각부등식실패. 104. 4ABC 에서 \C 의이등분선이외접원과변 AB 와만나는점을각각 D, K 라하자. CA CB = CK CD 임을보여라. 증명 \CDB = \A 이므로 4DBC»4AKC. 따라서, CD : CB = CA : CK. 105. 그림과같이, 단위정사각형을 7 개의닮은직각삼각형으로분할하는실수 x (0 <x<1) 가존재한다.
150 기하 이와같은 x 는최고차항의계수가 1 인 5 차다항식을만족한다. 이다항식을구하여라. (IMTS R8-2) 풀이직각삼각형들이모두닮았고세변의길이의비는 1:x : p x 2 +1. 이비에의해오른쪽변에접한직각삼각형부터차근차근변의길이들을구해보면윗변의길이는 x + x x 2 +1 + x (x 2 +1) 2 =1이됨. (x 2 +1) 2 곱해서정리하면 x((x 2 +1) 2 +(x 2 +1)+1)=(x 2 +1) 2, x 5 +3x 3 +3x = x 4 +2x 2 +1. 답 x 5 x 4 +3x 3 2x 2 +3x 1 106. 두원이두점 M, N 에서만난다. M, N 과다른점 A 를첫번째원위에잡는다. 직선 AM 과 AN 이두번째원과각각점 B 와 C 에서다시만난다. 점 A 에서첫번째원의접선이 BC 와평행함을증명하여라. ( 이탈리아 1986-1) 증명접선위에점 K 를 M 쪽으로하나잡으면 \MAK = \MNA = \MBC. 107. 삼각형의세변을각각 p :1 p, q :1 q, r :1 r 로나누는세점을잡자. 이세점을이은삼각형의넓이를구하여라. 단, 원래삼각형의넓이를 1 로한다. 풀이 pqr +(1 p)(1 q)(1 r). 이것은대칭식이어서쓸모가있을수. 베트남 1980-3 의보조정리로쓰이거나. 셈PS의그림활용부등식과연관. 108. 사각형의세내각의이등분선이한점에서만난다면, 네내각의이등분선이모두그점에서만남을증명하여라. (IMTS R11-4) 증명1 ( 김지훈 ) 각 A, B, C의이등분선이한점 P 에서만난다고할때 P 와 D를이었을때그PD가각D의이등분선임을보이면된다. P 에서 AB, BC, CD, AD에수선의발을내리고순서대로 Q, R, S, T 라하자. 그러면 4AQP 4AT P (RHA), 마찬가지로 4PQB 4PRB 도되고또한4PRC 4PSC 도성립. 따라서 PT = PS 이고따라서 4TDP 4SPD 도성립 (RHS). 따라서 \PDT = \PDS. 증명2 ( 이광섭 ) 각 A, B, C의이등분선이한점에서만났다고하자. DA와 CB의연장선의교점을 X라고하고, 문제에서의교점을 I라고하자. I는삼각형 ABX의외각의이등분선의교점이므로방심이되고, \AXI = \IXB. 따라서 XI는 \AXB의이등분선이고, 삼각형 DXC에서 \BCI = \DCI 이므로 I는삼각형 DXC의내심. 따라서 DI는 \ADC의이등분선. DA k CB 일때는따로간단히확인해보면역시성립함. 증명3 ( 김지욱 ) 사각형 ABCD에서 \A의이등분선은변 AB, 변 AD에서의거리가같은점들의집합이다. 따라서, 한점에서만나는세내각의이등분선들의교점을 O라하면O에서부터모든변들까지의거리는같다. 그러므로나머지한각에서의이등분선은그각을끼고있는변들에서의거리가같은점들의집합이다. 따라서이선은 O를지난다. 109. 삼각형 ABC 가주어져있다. C 를지나고 \B 의이등분선과평행한직선이 \A 의이등분선과점 D 에서만난다. 또점 C 를지나고 \A 의이등분선과평행한직선과 \B 의이등분선의교점을 E 라고하자. 이때, DE k AB 라면 CA = CB 임을증명하여라. ( 소련 1963-9) 힌트그냥각계산.or 원에내접하는사각형 110. S 는삼각형 ABC 내부의점으로, 세삼각형 ABS, BCS, CAS 의넓이가모두같다고한다. S 는삼각형 ABC 의무게중심임을증명하여라. ( 헝가리 1936-2) 증명꼭지점 : 변에이르는거리의비가 2:1 이므로점이유일하게결정됨. 그럼무게중심뿐. 111. 평행사변형 ABCD에서, E는 AD의중점이고F는 B에서 CE로내린수선의발이다. ABF 가이등변삼각형임을보여라. ( 몰도바 1997 최종-y7-6) 증명 BC 의중점을 G 라할때, AG 는 4BCF 의중점연결선이되어 BF 를수직이등분함.
3.1 기하중급문제 151 112. ABC 는 AB = AC 이고 \BAC =30 ± 인삼각형이다. BC 에대한 A 의대칭점을 A 0 ; CA 에대한 B 의대칭점을 B 0 ; AB 에대한 C 의대칭점을 C 0 이라하자. A 0 B 0 C 0 이정삼각형임을증명하여라. ( 인도지역예선 1998-4) 증명각계산하고직각이등변삼각형몇개찾고하면각변의길이가모두 AB p 2. 113. BE 와 CF 는삼각형 ABC 의두높이이고 O 는그교점, 즉수심이다. O 를지나는임의의직선 KL 을잡자. 단, K 는 AB 위에, L 은 AC 위에있는점이다. 또, KM? BE 이고 LN? CF 가되도록 BE 와 CF 위에각각점 M, N 을잡자. FM k EN 임을증명하여라. ( 인도지역예선 2001-1) 증명 \FMO = \KFO = \NLO = \NEO 114. 예각삼각형 ABC 에서, 점 A, B, C 에서대변에내린수선의발을각각 D, E, F 라하자. H 를수심이라할때, 다음을증명하여라. ( 호주 1993-6) AH AD + BH BE + CH CF =2 증명넓이로따지면간단. H 대신삼각형내부의임의의점으로해도다성립. 셈본수준. 115. 단위길이의두선분 AB 와 CD 가점 O 에서만나고 \AOC =60 ± 이다. AC + BD 1 임을보여라. ( 러시아 1993 최종 -y9-2) 증명 BD 를평행이동해서 B 0 이 A 와만나도록하면정삼각형한변에대한최단거리부등식. 116. 도시 A, B, C, D가있는데,A에서의거리가 D보다 C가가깝고,B에서도 D보다 C가더가깝다.A에서 B에이르는직선도로위에어떤점에서도 D보다 C가가까움을보여라. ( 러시아 1994 4차-y9-2) 증명 CD의수직이등분선 l에대해, A, B는모두C와같은쪽의영역에있음. 선분 AB도그영역에있음 ( 반평면은볼록함 ). 117. 삼각형 ABC 에서변 BC 위에 B;K; L;C 의순서가되도록두점 K, L 을잡는데, BC KL = BK CL 이고 AL 이 \KAC 의이등분선이되도록하였다. AL? AB 임을증명하여라. ( 인도지역예선 1995-1) 증명 XK k AC 가되도록 AB 위에점 X를잡자. XK=AC = BK=BC = KL=CL = AK=AC 이므로 XK = AK. 이등변삼각형이므로 \XKA의이등분선KH가 AB와직교하고 KH는 AL과평행하니까. 118. D 는삼각형 ABC 의선분 BC 위의점이고, DC =2BD 를만족한다. \ABC =45 ±, \ADC =60 ± 일때, 삼각형 ABC 의세각의크기를모두구하여라. ( 유고슬라비아 1980 고 1-3) 풀이 A에서 BC에내린수선의발을 H라하고, BD =1,DH = x 라할때, x = p 1 p = 3+1. 3 1 2 싸바싸바계속계산해서구하면 답 60 ±,45 ±,75 ± 별해 C 에서 AD 에내린수선의발을 O 라하면 OA = OB = OC 를확인할수있음. 119. 삼각형 ABC 에서 P 는무게중심이고 M 과 N 은각각 AC, AB 의중점이다. 만약사각형 ANP M 에내접하는원이존재한다면 4ABC 는이등변삼각형임을증명하여라. ( 남미 1987-2) 증명 AM + PN = AN + PM 이랑파푸스의정리두개를함께연립하여풀면... 120. 사각형 ABCD 에서 AB 는 CD 와평행하고대각선 AC 와 BD 는서로수직이다. 다음을보여라 : (1) AD BC AB CD (2) AD + BC AB + CD ( 인도지역예선 1997-4) 증명대충제곱해서피타고라스로정리하고인수분해.(2) 는제곱하면 (1) 과동치
152 기하 121. 평면위의서로만나지않는두원 k 1, k 2 의중심을각각 O 1, O 2 라하자. 한공통외접선이 k 1 과 A 에서, k 2 와 B 에서접한다. 선분 O 1 O 2 는 k 1, k 2 와각각 C, D 에서만난다. (a) 네점A, B, C, D가한원위에있음을보여라. (b) 직선 AC와 BD가서로수직임을보여라. ( 몰도바 1996 최종 -y11-3) 증명 \AO 1 C =2, \BO 2 D =2 라하면 + =90 ±. 따라서, \CAB + \DBA = + =90 ± 로 (b) 가확인되고, \O 1 CA = = \DBA 로 (a) 도확인됨. 122. 삼각형 ABC 의변 BC 위에 BM = MN = NC 를만족하는두점 M, N 을잡자. AC 에평행한한직선이직선 AB, AM, AN 과만나는점을각각 D, E, F 라하자. EF =3DE 임을보여라. ( 호주 1994-1) 증명확대-축소변환에의해, AC에평행한직선이 B를지난다고가정할수있다. 그럼삼각형의닮음에서 BD : CA = MB : MC =1:2,CA : BF = NC : NB =1:2이므로 BD : BF =1:4. 별증평행사변형의한꼭지점에서두대변에그은두중선이대각선을 3등분함을이용해도같은그림에서간단. 123. 삼각형 ABC의중선BM( 혹은그연장선 ) 위에 AN = BC 가되도록점 N을잡자. 직선 AN이 BC와만나는교점을 K라할때, BK = KN 임을증명하여라. ( 몰도바 1999 최종-y7-4) 증명 ABCD 가평행사변형이되는점 D 를잡으면 AN = AD 이고 4AND»4KNB. 124. P 를이등변삼각형 ABC의밑변BC 위의임의의점이라하자. P 에서 AB, AC 위로내린수선의발을각각 D, K라고하고, 선분 AD와 AK 위에 2MD = MB 이고 2NK = NC 인점M과 N을각각잡자. MC = NB 임을증명하여라. ( 몰도바 1999 최종-y7-8) 증명 DM = DB, KN = KC 이므로 4PBM»4PCN»4ABC 이고 4BPN 4MPC 125. \A 가직각인직각삼각형 ABC 에서, 한원이중선 AD 와 A 에서접하고있다. 그원은직선 BC 와두점 M, N 에서만난다. 이삼각형의한옆변이 \MAN 을이등분함을증명하여라. ( 몰도바 1999 최종 -y8/9-8) 증명 \DBA = \DAB 와 \NMA = \NAD 를변변빼면끝. 126. 두원S 1 과 S 2 가점F에서서로외접한다. 두원의한공통외접선이 S 1 과 A에서접하고 S 2 와 B에서접한다. AB에평행하고 S 2 와 C에서접하는직선이 S 1 과두점D, E에서만난다. A, F, C가한직선위에있음을보여라. ( 러시아 1994 최종-y9-2) 증명 AO 2 k CO 1 이므로 \AO 2 F = \CO 1 F, \AF O 2 = \CFO 1. 127. ABC 는 \C =90 ± 이고 CA 6= CB 인삼각형이다. CH 는수선, CL 은각의이등분선이라하자. 직선 CL 위의 C 가아닌임의의점 X 에대하여 \XAC 6= \XBC 임을보여라. 또한직선 CH 위의 C 가아닌임의의점 Y 에대하여 \YAC 6= \YBC 임을보여라. ( 주니어발칸 2001-2) 증명 (1) \XAC = \XBC 이면 4ACX 4BCX (ASA). (2) WLOG BC > AC 라하면 BY > AY. 그럼 \CBA > \CAB 이고 \YBA>\YAB 라서 \YBC >\YAC. 128. 정사각형 ABCD 내부에점 M 이있다. 삼각형 ABM, BCM, CDM, DAM 의무게중심들이정사각형의네꼭지점이됨을증명하여라. (Towns 1983 가을 J1) 증명 M 을중심으로 3=2 배확대하면중점연결사각형. 변환기하 129. 정삼각형내부의한점 P 로부터세꼭지점에이르는거리가각각 3, 4, 5 라고한다. 이정삼각형의넓이를구하여라. ( 아일랜드 1998-2, 한국 1993-5 변형 )
3.1 기하중급문제 153 풀이 P 에의해나뉘어진세조각을각각한꼭지점을기준으로 60 ± 회전시키면원래삼각형의넓이의 2배가되는육각형이얻어지는데, 그육각형의넓이가 S(3; 3; 3) + S(4; 4; 4) + S(5; 5; 5) + 3S(3; 4; 5) 가됨 (S(a; b; c) 는세변의길이가a, b, c인삼각형의넓이 ). 답 50 p 8 3+9 130. CA = CB 인삼각형 ABC 의외접원의호 AB 위 ( 직선 AB 에대하여 C 의반대쪽 ) 에점 P 가있다. C 에서 PB 에내린수선의발을 D 라할때, PA+ PB =2 PD 임을보여라. ( 주니어발칸 2002-1) 증명 4CAP 를 C를중심으로회전하여 CA 0 과 CB가일치하도록하면 CPP 0 은이등변삼각형이고 2 PD = PP 0 = PB + AP 0 = PB + AP. 131. 두원S 1, S 2 가두점A, B에서만나고 S 2 의중심O가S 1 위에있다. S 1 의현OC가 S 2 와점D에서만난다 (D는 O와 C 사이의점이다 ). D가삼각형ABC의내심임을보여라. ( 러시아 1990 4차-y10-6) 증명 \DAB = 1 2 \DOB 1 2 \COB = 1 2 \CAB 등. 132. 삼각형 ABC에서 \B의이등분선이 AC와 D에서만나고 \C의이등분선이 AB와 E에서만난다. 이두이등분선은점 O에서만나고 OD = OE 가성립한다. \A =60 ± 이거나 ABC가이등변삼각형임을증명하여라. (Towns 1984가을 JO1) 증명 D, E를각각이등분선에대해대칭시켜 BC 위에얻은점을 D 0, E 0 이라할때 ID 0 E 0 이이등변삼각형임에서 ABC도. 만일 D 0 = E 0 이면각계산좀해보면 \A =60 ±. 133. ABCD 는 AB k CD 와 AB + CD = AD 인사각형이다. AD 위에 AP = AB 와 PD = CD 인점 P 를잡자. (1) \BPC =90 ± 임을증명하여라. (2) BC의중점을Q라하고, BAQ의외접원과직선 AD의교점을R이라하자. 네점B, P, R, C가한원위에있음을증명하여라. ( 중미 2004-5) 증명 (1) P X k AB 라하면 P B, P C 는 \AP X, \DP X 의이등분선. (2) \QP R = \QRP 로 QP R 은이등변삼각형인데, Q 는직각삼각형 BPC 의외심이므로. 134. 4ABC에서 \B = \C =40 ± 이다. \B의이등분선이 AC와만나는점을 D라하자. BD + DA = BC 임을증명하여라. (Towns 1984가을 SO3) 증명 A를 BD에대칭시킨점을 A 0, 이것을다시 D에서 BC에내린수선을축으로대칭시킨점을 A 00 로하면. 135. 어떤삼각형의한중선, 한각의이등분선, 한수선이내부의한점 O에서만난다. 이중각의이등분선의꼭지점으로부터 O까지연결한선분의길이가수선의꼭지점으로부터 O까지연결한선분의길이와같다고한다. 이삼각형은정삼각형임을증명하여라. (Towns 1985봄 JO1) 증명중선이수선이됨을확인하면됨. 136. 사각형 ABCD 가 AB = BC = l, \ABC = 100 ±, \CDA =130 ± 를만족한다. BD 의길이는얼마인가? (Towns 1985 봄 SO1) 풀이 100 ± =2=180 ± 130 ± 이므로, B를중심으로하고A, C를지나는원위에D도있음. 답 l 137. O를중심으로하는원의한지름 AB와그에수직인현 CD가있다. 현 AE가반지름 OC를이등분할때, 직선 DE가현BC를이등분함을보여라. ( 러시아 1995 최종-y9-2) 증명 \CAB = \CDB, \AOC = \DBC 로 4AOC»4DBC (AA닮음). \CAE = \CDE 이므로 4AOC 및그중선을함께 C를중심으로회전확대하여 4DBC에겹쳐주면.
154 기하 138. 예각삼각형 ABC 의변 BC, CA, AB 위에각각점 D, E, F 가있고 CD CE = CA CB ; AE AF = AB AC ; BF BD = BC BA 를만족한다. AD, BE, CF 가삼각형 ABC 의수선들임을증명하여라. ( 인도지역예선 2002-1) 증명문제의조건에의해 ABDE, BCEF, CAFD cyclic. 원주각에의해 \BEF = \BCF(= DCF )=\DAF (= DAB) =\DEB 이므로직선 BE 는삼각형 DEF 의한내각의이등분선. AD, CF 도마찬가지. 문제의조건에서또 \AEF = \B = \CED 이므로 BE 는 \AEC 의이등분선, 즉수선. AD, CF 등도마찬가지. 139. 정사각형 ABCD 의변 AB, CD 위에각각점 K, L 을잡고, 선분 KL 위에점 M 을잡자. 4AKM 과 4MLC 의외접원이다시만나는점은대각선 AC 위에있음을보여라. (Towns 1986 봄 J4) 증명 \KAN = \KMN = \NCL. 정사각형일필요는없고평행선 AB k CD 이기만하면충분. 140. 볼록오각형 ABCDE 에서, 삼각형 ABE, BCE, CDE, DAE 의무게중심을각각 P, Q, R, S 라하자. PQRS 가평행사변형임을보이고그넓이는 ABCD 의넓이의 2=9 임을보여라. ( 중미 2000-3) 증명 E 를중심으로 PQRS 를 3 2 배확대. 141. 삼각형 ABC 에서변 BC 의중점을 D 라하고, 변 AC 위에 BE =2AD 를만족하는점 E 가있다. BE 와 AD 의교점을 F 라할때 \FAE =60 ± 라한다. \FEA 의크기를구하여라. ( 중미 2002-4) 풀이 BEE 0 B 0 을 E 0 2 AC, D 2 B 0 E 0 인평행사변형이라하자. 4DBB 0 4DCE 0 (ASA) 이므로 AD = DE 0. \FEA = \DE 0 A =60 ±. 142. 두원이두점 P 와 Q 에서만난다. P 를지나는한직선이두원과각각 A 와 A 0 에서다시만난다. 그와평행한 Q 를지나는직선이두원과각각 B 와 B 0 에서다시만난다. 두삼각형 PBB 0 과 QAA 0 의둘레의길이가같음을보여라. ( 중미 2003-4) 증명 4PBB 0 4QAA 0 143. A 와 B 는원 T 위의서로다른점이다. BC 는 B 에서의 T 의접선이고 AB = AC 이다. \ABC 의이등분선이 AC 와점 D 에서만난다고하자. D 가 T 의내부에있다면, \ABC > 72 ± 임을보여라. ( 아일랜드 2004-7) 증명 AC가원T와다시만나는점을 E라하고, \ABC =2a 로두자. 4ABC의내각의합과접현각에서 180 ± 4a = \BAC = \EBC < \DBC = a. 즉,5a>180 ±,2a>72 ±. 144. 예각삼각형에서세수선의발을서로연결하여작은삼각형을얻었다. 이삼각형이두변의원래삼각형의변과평행하다고한다. 그럼세번째변도역시평행임을증명하여라. (Towns 1987봄 JO2) 증명체바의정리에의해평행변에직교하는수선의발은중점. 즉, 수심과무게중심이동일하게됨. 145. 두점 A, B 에서만나는두원이있다. A 에서의두원의접선이서로직교한다. 한쪽원위에있고다른쪽원의내부에놓이는임의의점 M 에대해, 직선 AM, BM 이나중의원과다시만나는점을각각 X, Y 라하자. XY 는지름임을증명하여라. ( 레닌그라드 1987-26) 증명 M 이놓인원의중심을 O, 다른원의중심을 P 라하자. \XBY = \BMA \BXA = 1 2 (360± \BOA) 1 2 BPA = 180± 1 2 (\BOA + \BPA) =90±. 146. 중심이각각 O, O 0 인같은크기의원, 0 의두교점중하나를 A 라하자. B 는 위의한점이고, AB 가 0 과다시만나는점을 C, 그리고 OBDO 0 이평행사변형이되도록 0 위에점 D 를잡자. CD 의길이는 B 의위치에상관없이일정함을보여라. ( 중미 2006-2)
3.2 기하고급문제 155 증명 OAEO 0 이평행사변형이되도록하면 \BCD = \AED = \ABD 이므로 BD = CD. 147. 예각삼각형 ABC 의외심을 O 라하자. \A 의이등분선이 BC 와 D 에서만나고, D 에서 AO 에내린수선이 AC 와 P 에서만난다. AB = AP 임을보여라. ( 이탈리아 1995-4) 증명 \ABC = 1 2 \AOC =90± 1 A = \AP D 2 이므로 AD를축으로선대칭합동. 148. 지름 AB 가원을두반원으로나눈다. 점 P 1 ;P 2 ;:::;P n 이한반원위에이순서대로배열되어있다. 다른반원위에서점 C 를잡는데, 삼각형 CP 1 P 2 ;CP 2 P 3 ;:::;CP n 1 P n 들의넓이의합이최대가되게하려면 C 는어떤점이어야하는가? ( 폴란드 1970 3 차 -1) 풀이 P 1 P 2 P n 의넓이는일정하고, 삼각형 P n P 1 C의넓이가최대가되도록하면됨. 즉, C에서의접선이 P 1 P n 에평행할때. 혹은 P 1 P n 의수직이등분선이다른반원과만나는점을 C. 149. 점 O 를중심으로하는한원의내부에점 A 가주어져있고, 이점을지나는, 지름이아닌현 PQ 가있다. 점 P, Q 에서의원의접선을각각 p, q 라하자. 점 A 를지나고 OA 에수직인직선 l 이 p, q 와각각점 K, L 에서만난다. AK = AL 임을보여라. ( 폴란드 1993/1994 1 차 -4) 증명 OAKP, OAQL cyclic. 4OAK 4OAL. 150. AB 를긴밑변으로하는사다리꼴 ABCD 에서대각선 AC 와 BD 는서로수직이다. 삼각형 ABC 의외심을 O 라하고, 직선 OB 와 CD 의교점을 E 라하자. BC 2 = CD CE 임을증명하여라. ( 이탈리아 1998-4) 증명 \CEB = \ABE =90 ± 1 2 \AOB =90± \ACB = \CBD 이므로 4CBD»4CEB 3.2 기하고급문제 1. 컴퍼스만을사용하여 ( 직선자는사용하지말것 ) 수직으로만나는 2 개의원을그려라. 한평면위의두원이수직으로만난다는것은두원의교점에서그은각각의접선이서로직교한다는것을뜻한다. (IMTS R13-5) 풀이 X를중심으로하고 Y 를지나는원을원 X(Y ) 로쓰기로하자. 우선원 O(A) 를그리자. 원 A(O) 를그려원O와만나는한점을B라하고, 원 B(O) 를그려원O와만나는 A가아닌점을C라하고, 원 C(O) 를그려원O와만나는B가아닌점을D라하자. 그럼 OAB, OBC, OCD들은정삼각형들이고, AD는원O의지름이다. 그럼지름의원주각으로 AC? CD 이므로, 원 A(C) 와원D(C) 를그리면이두원의접선CD와 CA는서로직교한다. 주두원을똑같은크기가되도록할수도있다. 위의풀이에서처럼 O, A, B, C, D를잡고, 원 A(C) 와원 D(B) 의한교점을 E라하자. OA = r 이라할때, AE = DE = AC = DB = p 3 r 이므로 OE = p 2 r 이다. 따라서, E를중심으로하고반지름 r인원을그리면원o와직교한다 ( 두중심과두교점으로이루어진사각형이한변의길이가 r인마름모인데대각선의길이가 p 2 r이므로정사각형이다 ).
156 기하 2. 원 O 위에이원의중심 O 를지나지않는고정된현 AB 가주어져있다. XY 가움직이는지름일때, 직선 AX 와직선 BY 의교점 P 의자취를구하여라. ( 캐나다 1973-6) 풀이 P 를원의안쪽에생기는점, P 0 을원의바깥에생기는점이라하자. 현 AB 의원주각을 라하면 \AP B =90 ± + \AP 0 B =90 ± 로일정하고두각의합이 180 ± 이므로 A, P, B, P 0 은한원위에있다. 즉, 위와같은원주각에의한원이점 P 의자취가된다. 3. 직선 MN 은원 O 와 A 에서접하고, 현 BC 는원 O 의지름 AD 와점 Q 에서만난다. 또, P 는 PB? BC 를만족하는직선 MN 위의점이다. (1) \CAN =27 ±, \ACQ =33 ± 일때, \PBO의크기를구하여라. (2) PQ AC = AQ 2 + BQ CQ 임을보여라. (1995 서울시 ) 풀이 (1) \BOC = \BOA + \AOC =2\ACB +2\CAN =2(27 ± +33 ± )=120 ±. (2) 방멱에의해 AQ 2 + BQ CQ = AQ 2 + AQ QD = AQ AD 이므로, 준식은 PQ AC = AQ AD, 즉 PQ : QA = DA : AC 와동치이다. 따라서 4PQA»4DAC 임을, 이것은직각삼각형이므로 \AP Q = \CDA 임을보이면충분하다. PBQA가 PQ를지름으로하는원에내접하므로, \AP Q = \ABQ = \ADC 가잘성립한다. 4. 삼각형의꼭지점과, 내접원과대변의접점을이은직선은한점에서만남을보여라. ( 통신강좌 1993-6-16) 풀이 A; B; C의대변에대한접점을각각 D; E; F라하자. 그러면 EA = AF; F B = BD; DC = CE이다. 따라서 AF FB BD DC CE EA = AF BD BD CE CE AF =1 이고체바의정리에의해 AD; BE; CF 는한점에서만난다. 5. 주어진삼각형 ABC 의변 AC 와 BC 에대해이삼각형의바깥쪽으로평행사변형 AA 0 C 0 C 와 BB 00 C 00 C 를각각그린다. 선분 A 0 C 0 과 B 00 C 00 의연장선의교점을 P 라하고, 선분 CP 와평행하면서길이도같도록변 AB 의바깥쪽에선분 AP 0 과 BP 00 을잡는다. 이때 j ABP 00 P 0 j = j AA 0 C 0 Cj + j BB 00 C 00 Cj 가성립함을보여라. 단, jsj 는도형 S 의넓이를뜻한다. ( 셈본중등초급도전문제 4.1.2)
3.2 기하고급문제 157 증명 CP 가 AB 와만나는점을 X 라하자. 등적변형으로 j4acc 0 j = j4acp j = j4ap 0 Cj = j4ap 0 Xj 마찬가지로 j4bcc 00 j = j4bb 00 Xj. 따라서, j ABP 00 P 0 j =2j4AP 0 Xj +2j4BB 00 Xj =2j4ACC 0 j +2j4BCC 00 j = j AA 0 C 0 Cj + j BB 00 C 00 Cj 가성립한다. 6. 삼각형 ABC 에서, A 와 B 를지나는한원이변 AC, BC 와각각 D, E 에서만난다. 직선 AB 와 DE 는 F 에서만나고직선 BD 와 CF 는 M 에서만난다. MF = MC 이면, 또그때만 MB MD = MC 2 임을증명하여라. ( 미국 2003-4) 단계 1 MF = MC 이면 MB MD = MC 2 임을보이자. DM 의연장선위에 MD = MD 0 인점 D 0 을잡고 D 0 F, D 0 C, AE 를연결하자. 사각형 DCD 0 F 는평행사변형이다. D 0 F==AC, EF==D 0 C 삼각형 ADE 와삼각형 FD 0 C 는 B 를닮음의중심으로닮음. 삼각형 BMC 와 CMD 는닮음. ) AE==F C; \DBE = \EAD = \ACF MD : MC = MC : MD; MB MD = MC 2 단계2 MF = MC 일때만MB MD = MC 2. 즉, MB MD = MC 2 이면 MF = MC 임을보이자. MB : MC = MC : MD 이므로삼각형 MBC와 MDC가닮음. AE를연결하면 \MBC = \AEC = \MCA 이므로 AE==MC; \EFM = \FEA = \ABM 따라서삼각형 FBM과 FMD도닮음. ) FM 2 = MD MB 즉, FM 2 = CM 2, MF = MC. 별증 ( 박경태 )()) 체바의정리에의해서 FA=AB = EC=BE. 잘정리하면 FA=EC = AB=BE. 가비의리에의해서 FA=EC = AB=BE = FB=BC. 따라서 4BAE»4BFC. 즉, AE k FC. 그러면 \EBD = \EAD( 원에내접 )= \ACF ( 엇각 ) 이므로 4MDC»4MCB. 성립함. (Ã) 체바의역정리쓰면됨. 7. 원 O 에내접하고 \A <\B 인예각삼각형 ABC 를생각하자. 원외부의어떤점 P 가 \A = \PBA =180 ± \PCB 를만족시킨다고하자. 직선 PB가원O와만나는 B가아닌점을D라하고, 점 A에서원 O에접하는접선이직선 CD와점Q에서만난다고하자. 이때, CQ : AB = AQ 2 : AD 2 임을보여라. ( 한국 2003-J3)
158 기하 증명 \A = \PBA 이므로 \A = \DBA = a. 점 A 는접점이므로, QA 2 = DQ CQ 1 점 A; B; C; D 는한원위에있으므로 \CAB = \CDB = a. a = \ABD 이므로 AB k CD. 또한 A 는접점이므로 4ABD 에서 \ABD = a = \DAQ 이다. 3 2, 3 에의해서 4ABD»4DAQ (AA 닮음 ) 1, 4 에의해서 ) \DAB = \ADQ 2 ) AD : AB = DQ : AD ) AD 2 = AB DQ 4 AQ 2 : AD 2 = DQ CQ : AB DQ = CQ : AB 가성립한다. 8. 직사각형 ABCD 에서 AC 의길이는 e 라하고점 A, B, C, D 를중심으로하고반지름이각각 a, b, c, d 인네원이있다. 이때 a + c = b + d<e 를만족한다. 원 A, C 의두공통외접선과원 B, D 의두공통외접선을변으로하는사각형에내접하는원을그릴수있음을증명하여라. ( 소련 1961-2) 풀이직사각형의중심 O 에서각공통외접선까지의거리가일정 : r =(a + c)=2 =(b + d)=2. 9. A, B, C, D 가다음을만족하는공간위의네점이라할때, \ABC = \BCD = \CDA = \DAB = ¼ 2 A, B, C, D 는한평면위에있음을증명하여라. ( 캐나다 1976-6) 증명 A 에서평면 BCD 에내린수선의발을 H 라하자.
3.2 기하고급문제 159 AB; AH? BC 이므로삼수선의정리에의해 HB? BC, 마찬가지로 HD? CD 이다. 따라서, HBCD는직사각형. BAD와 BHD가모두직각삼각형이고 AHB와 AHD도직각삼각형이므로, 피타고라스의정리에의해 BD 2 = AB 2 + AD 2 =(AH 2 + HB 2 )+(AH 2 + HD 2 ) =2AH 2 + BD 2 즉, AH =0 으로 A = H. 따라서, 네점은한평면위에있다. 별증 피타고라스의정리에서 AC 2 = AB 2 + BC 2 = CD 2 + DA 2 BD 2 = BC 2 + CD 2 = DA 2 + AB 2 따라서, AC 2 = 1 2 (AB2 + BC 2 + CD 2 + DA 2 )=BD 2 이로부터비슷한방식으로하여 AC = BD; AB = CD; AD = BC 임을알수있다. 한편, B는 A를직선BC 위로사영시킨점이므로두평면 ADC와 BCD가이루는각을 µ 라하면 j4bcdj = j4adcj cos µ 이다. 4ADC 4BCD 임에서 j4adcj = j4bcdj 이므로 µ =0이어야한다. 10. 직선 ` 이 4ABC 의무게중심 G 를지난다. 각점 A, B, C 에서 ` 에그은수선의발을 X, Z, Y 라하자. CY = AX + BZ 임을보여라. 단, ` 은 AC, BC 와만난다. ( 통신강좌 1993-7-11) 풀이 D 를 AB 의중점이라하고, D 에서 l 에그은수선의발을 Q 라하자. GC : GD =2:1 이므로 CY =2DQ 또, 사다리꼴 ABZX 에서 D 는 AB 의중점이므로,2DQ = AX + BZ ) CY = AX + BZ 11. 단위원에외접하는사다리꼴에서평행인두변의길이를 x, y 라할때, 부등식 x + y 4 와 xy 4 가성립하는지각각검토하고, 성립할경우등호조건을찾아라. (ML 프로포절 147-3) 풀이아래그림처럼 x = AD = a + b, y = BC = c + d 라두자. 직각삼각형 ABX 에서피타고라스정리에 (a c) 2 +2 2 =(a + c) 2 이성립하므로, ac =1 이된다. 마찬가지로 bd =1. 따라서, 산술 - 기하평균관계에의해 x + y = a + b + 1 a + 1 b = a + 1 a + b + 1 b 2+2=4 µ 1 xy =(a + b) a + 1 =2+ a b b + b a 2+2=4 로두부등식이모두성립함을알수있다. x + y =4일등호조건은 a = b =1일때이므로정사각형일때, xy =4일등호조건은 a = b 일때이므로등변사다리꼴일때이다. [ 주 ] \AOB = 1 \EOF = \R 2 임으로부터 AG : GO = GO : GB, 즉 ac =1임을알아낼수도있다.
160 기하 별해 x + y 4 는다음과같이증명할수도있다. (1) x + y = a + b + c + d = AB + CD 이고, AB와 CD는두평행선 AD와 BC 사이를잇는선분이므로각각두평행성의거리 2보다크거나같다. 따라서, x + y 2+2=4. 등호는 AB = CD =2 일때, 즉 AB와 CD가높이가될때이므로정사각형일때이다. (2) 사다리꼴 ABCD의중단 ( 높이의 1 ) 2 을가르는직선을그려, 내접원과만나는점을 E, F, 사다리꼴의옆변과만나는점을 G, H라하자. 그럼 EF는원의지름이고, 사다리꼴이원에외접하므로 G, H는원의외부에있어서 GH EF 이다. 따라서, x + y =2GH 2EF =4. 등호는 GH = EF 일때, 즉옆변이중단에서접할때이므로정사각형일때이다. 12. 삼각형 ABC 에서변 BC 의중점을 M 이라하고, P 와 R 을각각변 AB 와 AC 위의점이라하자. AM 과 PR 의교점 Q 가선분 PR 의중점이라면, PR 과 BC 가평행임을증명하여라. ( 호주 1991-7) 풀이 PR 0 k BC 인점 R 0 을변 AC 위에잡고 Q 0 은 PR 0 의중점이라하면, 중점연결정리에의해 QQ 0 k RR 0, 즉 AM k AC 가되어모순. 13. 한직선 l 과이직선밖의한점 P 가주어져있다. 이때, 자없이컴퍼스만을이용하여, PQ 가 l 과평행하게되는점 Q 를하나작도하여라. 풀이 마름모법 14. 고정된원 C 와그중심 O 를지나는직선 L 이있다. 직선 L 위를움직이는점 P 를중심으로하고점 O 를지나는원 K 를그린다. 원 C 와원 K 의공통접선이원 K 와만나는점을 T 라고할때, 점 T 의자취를구하여라. ( 소련 1962-2) 풀이 Let the common tangent meet C at S. LetX be the intersection of C and OP lying between O and P. PT = PO, hence \POT = \PTO,so\OPT = 180 ± 2\POT. But PT and OS are parallel, because both are perpendicular to the common tangent. Hence \POS = 2\POT, so \SOT = \XOT. Hence TX is tangent to C, inotherwordst lies on the ( xed) tangent to C at X. Conversely, it is easy to see that any such point can be obtained (just take P such that PO = PT). Thus the required locus is the pair of tangents to C which are perpendicular to L. 15. O 를중심으로하는어떤원의내부에 O 와는다른점 A 가주어져있다. 각 OP A 가최대가되는원 O 의원주위의점 P 를모두구하고작도하여라. ( 캐나다 1977-2) 풀이 O, A 를지나고원 O 와접하는작은원 Q 를생각하자. 접점을 P 라하면, 원 O 의원주위에있고직선 OA 에대해같은쪽영역에있는점 X (6= P ) 는모두원 Q 밖에있으므로 \OXA < \OPA 로원주각보다작게된다. 따라서, 이 P 가구하는점이다. 이런 P 는 OA 에대칭인것까지 2 개있다.
3.2 기하고급문제 161 주 OP 가원 Q 의지름이므로, 작도는 A 에서 OA 에수선을올려원 O 와만나는점을찾으면된다. 16. A, B, C 를한평면위의세점이라하자. 원 k 2 와 k 3 이 A 에서공통접선을갖고, k 3 과 k 1 이 B 에서, k 1 과 k 2 가 C 에서공통접선을갖는세원 k 1, k 2, k 3 을작도하여라. ( 헝가리 1924-3) 풀이 A; B; C의외심O를작도하면 joaj = jobj = jocj이다. 그러면 OB; OC를접선으로갖고 B;C를지나는원을그릴수있고, 마찬가지로 C; A, A; B에대해서도그리면그원들이각각 k 1 ;k 2 ;k 3 가된다. 17. M 은 AB = BC 인이등변삼각형 ABC 의외접원의호 AC 위의한점이다. P 는이호의중심이고, N 은현 BM 의중심이다. P 에서 MC 에내린수선의발을 K 라할때, 삼각형 ANK 가이등변삼각형임을보여라. ( 통신강좌 1991-3-12) 풀이 \OAB = \PAC = \OBA = \PCA 에서 4OAB v 4PAC (1) 이고, \ONB = \PKC, \OBN = \PCK (PM 에대한원주각 ) 에서 4ONB v 4PKC (2) 이다. B B N O O N A M K P C A P K M C (1), (2) 에서 KC NB = PC OB = AC 이고, \NBA = \KCA (AM에대한원주각) 이므로 AB 4NAB v 4KAC (3) 이다. 다시 (3) 에서 \NAB = \KAC 이므로 \BAC = \NAK 가되고, AC AB = AK AN 이므로, 4BAC v 4NAK (4) 가된다. 4ABC 는이등변삼각형이므로,(4) 에서 4ANK 도 AN = NK 인이등변삼각형이된다. 18. \B = \C 이고, \A >60 ± 인이등변삼각형 ABC 의외접원을 O 라하자. 꼭지점 A 에서원 O 에접하는접선을 ` 이라하고, ` 위의점 P 가 \BPA = \BAC 를만족시킨다고하자. 선분 PC 가원 O 와만나는점을 E 라하고, 직선 BE 가접선 ` 과만나는점을 F 라할때, AF = FP 임을보여라. 단, E 6= C. ( 한국 2002 1 차 -S3)
162 기하 증명 1 직선 ` 과원이점 A 에서접한다. 할선의정리에의해서, AF 2 = FE FB (1) 문제에서주어진것처럼 \BAC = \BPA 이고, \BAC 와 \BEC 는같은현을공유하고있으므로크기가같다. 또한, \BEC 와 \FEP 는맞꼭지각이므로같다. 결국, \PEF = \BPF 이고, 그러므로 4EPF 와 4PFB 는닮았다 (\PFE 는공통 ). 즉, PF : FE = FB : PF 이고, PF 2 = FE FB (2) (1), (2) 에서 AF = PF. 증명2 \BPA = \BAC, \ABC = \BAP 이므로 4ABC»4PAB 의닮은이등변삼각형이다. PA = PB =1,AB = AC = a 라하면BC = a 2 =2AD. 피타고라스정리와 PA 2 = PE PC 로부터 PC = p 2a 2 +1; PE = 1 p 2a 2 +1 또 4EFP»4EBC 에서 PF : PF + BC = PE : PC =1:2a 2 +1. 따라서 PF + a 2 = PF (2a 2 +1); 즉 PF = 1 2 = 1 2 PA 그러므로 AF = FP = 1 PA 이다. 2 19. 삼각형 ABC 의수심을 H, 변 AC 의중점을 M 이라하자. M 을지나 \AHC 의이등분선에평행한직선을 l 이라하자. l 은이삼각형의둘레를이등분함을증명하여라. ( 중미 2005-5) 증명높이 AD, CE, 그리고변 AB, BC가 l과만나는점을각각 I, J, S, T 라하고, A를지나l에평행한직선이 CE와만나는점을 G라하자. J는 CG의중점이고 AI = JG. 따라서, 4AIS 4CJT. 20. A, B, C 는 x; y- 좌표평면의세점이고, 선분 AB, BC, CA 의중점을각각 X, Y, Z 라하자. 또, \CPZ = \YXZ 를만족하는직선 BC 위의한점을 P 라하자. AP 와 BC 가직교함을증명하여라. ( 호주 1991-3) 풀이 Z 는 ZA = ZP = ZC 로 4AP C 의외심. 21. AB 와 BC 의길이가주어져있을때 A 에서그은중선과 B 에서그은중선이직교하도록삼각형 ABC 를작도하여라. ( 소련 1962-6) 풀이 Let M be the midpoint of AB and X the midpoint of MB. Construct the circle center B, radius BC=2 and the circle diameter AX. If they do not intersect (so BC < AB=2 or BC > AB) then the construction is not possible. If they intersect at N, thentakec so that N is the midpoint of BC. LetCM meet AN at O. ThenAO=AN = AM=AX =2=3, so the triangles AOM and ANX are similar. Hence \AOM = \ANX =90 ±.
3.2 기하고급문제 163 22. 원 k 가삼각형 ABC 의변 BC, CA, AB 들과점 A 1, A 2 ; B 1, B 2 ; C 1, C 2 에서만난다. 각각 A 1, B 1, C 1 을지나는 BC, CA, AB 의수선들이한점 M 에서만난다. 각각 A 2, B 2, C 2 를지나는 BC, CA, AB 의수선들도한점에서만남을증명하여라. ( 헝가리 1914-3) 힌트구하는점은원 k 의중심에대한 M 의대칭점이다. 증명원 k의중심을 O라하자. O에서세변 BC, CA, AB에내린수선의발은각각 A 1 A 2, B 1 B 2, C 1 C 2 의중점이된다. 따라서, MN 의중점이 O 가되도록하는점 N 을잡으면사다리꼴의빗변을같은비율만큼늘린것이되어 NA 2, NB 2, NC 2 는각각 MA 1, MB 1, MC 1 에평행이된다. 즉, A 2, B 2, C 2 를지나는 BC, CA, AB 의수선들은모두 N 을지난다. 23. AB, CD 는한원의서로수직인두현으로각각길이가 2, 1 이고 AC + BD = AD + BC 일때그원의반지름의길이를구하여라. ( 통신강좌 1994-9-4) 풀이 조건에서 AC + BD = AD + BC (5) 이고피타고라스정리에의해 AC 2 + BD 2 = AD 2 + BC 2 (6) 이다.(1) 을제곱하여정리하고 (2) 를사용하면 AC BD = AD BC (7) 이다. 톨레미정리에의해 AC BD + AD BC = AB CD =2 (8) 가된다.(3) 과 (4) 로부터 AC BD = AD BC =1 이되고 (1) 에의해 AC +1=AC = AD +1=AD 가되어 AC = AD 또는 AC =1=AD = BC 가된다. X 를 AB 와 CD 의교점이라하면방멱에의해 AX BX = CX DX 이다. AC = BC 이면 CD 는 AB 를이등분하고 AX = BX =1=CX DX 이다. 그러나, 이것은 CX;DX 둘다 1 보다작기때문에불가능하다. 그러므로 AC = AD; CX = DX =1=2 이다. AB 는 CD 의수직이등분선이므로지름이된다. 따라서반지름의길이는 1 이다. 풀이 ( 과기원기계과 96 학번이원철 ) 피타고라스정리에의해 AC 2 + BD 2 = AD 2 + BC 2 조건식과위식을이용하여 AC BD = AD BC
164 기하 톨레미정리에의해 AC BD + AD BC = AB CD =2 즉, AC BD = AD BC =1 조건식과위식을합치면 AC + 1 AC = AD + 1 AD AC = AD or AC = 1 AD = BC X를 AB와 CD의교점이라하면방멱에의해 AX BX = CX DX이다. AC = BC이면 CD는 AB를이등분하고 AX = BX =1=CX DX이다. 그러나이것은 CX, DX < 1이므로불가능하다. 그러므로, AC = AD, CX = DX = 1 2 이다. AB는 CD의수직이등분선이므로지름이다. 즉, 반지름의길이는 1이다. 24. 평면위에세원이있다. 각원의중심을O 1, O 2, O 3, 반지름을 r 1, r 2, r 3 이라하자. 임의의두원이두점에서만나공통현을갖는다고하자. (1) 세공통현은한점P에서만남을증명하여라. (2) 각각의원 O i 에서 (i =1; 2; 3), P 를지나고PO i 와수직인현을생각하자. 이런세현의길이가모두같음을보여라. (ML프로포절 141-1) 증명 (1) 은두현AB, CD의교점이 E라하면AE EB = CE ED 가성립한다는것을이용하면된다.( 셈본중등고급에이문제있다. 몬즈의문제 ) (2) 는원의중심에서어떤현 AB에수선의발 M을내리면M이 AB의중점이된다는것과 (1) 에서이용했던것을다시이용하면된다. 주이문제는지구과학에서진원의깊이를잴때의원리로사용되는방법이다. 25. 삼각형 ABC의내심을 I, 직선 AI가삼각형 ABC의외접원과만나는점을 D, 선분 BI의중점을 P 라하자. CI =2AI 이면 AB = PD 임을보여라. 단, D 6= A. ( 한국 2002-J7) 증명각 A, B, C의크기를각각 2o, 2u, 2x라하자. 그리고직선 CI가원과만나는점을 F, ID의중점을 G라하자. 이제 FAB와 GP D가합동인이등변삼각형임을보이겠다. \DBI = \DBC + \CBI = \DAC + \CBI = o + u \DIB = \IAB + \IBA = o + u 로 DBI는이등변삼각형이다. \BDI = \BDA = \BCA =2x 이고 \GP D = \PDB = \GDP 이므로, 4GP D는밑각이 x인이등변삼각형이다. 또한 \FAB = \FBA = 1 \ACB = x 로 4FAB도 2 밑각이 x인이등변삼각형이다. DCI도 DBI와마찬가지로 DI = DC 인이등변삼각형이고 4ICD와 4IAF의닮음비가 CI : AI =2:1이므로, DG = 1 2 DI = 1 DC = FA 2 FAB와 GP D의두이등변삼각형이합동임이확인되었으므로, PD = AB.
3.2 기하고급문제 165 주선분 BC의중점을M이라할때삼각형 PMD와 AIB가합동임을보여해결할수도있다. 또, 선분 IA의중점을 E라할때삼각형AIC와 EPD가 1:2의변을갖는닮은삼각형임을보여해결할수도있다. 26. 삼각형의외심, 무게중심, 수심은일직선위에있음을보여라. 증명 D; E 는각각 A; B 에서 BC, AC 에내린수선의발이고, M; M 0 은각각 BC; AC 의중점이다. H 를 4ABC 의수심, O 를외심이라하면 H 는 AD 와 BE 의교점이고, OM? BC, OM 0? AC 가된다. AM 과 HO 의교점을 X 라하자. AD k OM 이므로 4AHX 와 4OMX 는닮음이고, 따라서 같은방법으로밑의그림에서, HX : XO = AH : OM 1 BE k OM 0, 즉 4BHY»4M 0 OY 이고, HY : YO= BH : OM 0 한편, AB k MM 0 ( 중점연결정리 ), AD k OM이므로 \HAB = \OMM 0 이고, AB k MM 0, BE k OM 0 이므로 \HBA = \OM 0 M 이다. 따라서 4HAB»4OMM 0 이고, AH : OM = BH : OM 0 = AB : MM 0 (= 2 : 1) 이로부터 1, 2 에의해 HX : XO = HY : YO=2:1, 즉 X = Y 이다. 이점은두중선 AM 과 BM 0 의교점이므로 4ABC 의무게중심이다. 따라서, 무게중심은수심과외심을이은선분을 2:1 로내분하며, 세점은한직선위에있다. 27. \C =2\B 인삼각형 ABC 가있다. 이삼각형의내부에 BD = CD 이고 AC = AD 인점 D 를잡자. \A =3\BAD 임을보여라. ( 호주 2003-3) 증명 \ACE = \ECB = \EBC =, \DCB = \DBC = 라하자. A 2 2α E C α α β D β α B
166 기하 4ACE»4ABC (AA) )4ADE»4ABD (SAS). \ACD = \ADC =2. ) AC 2 = AE AB = AD 2 ) \ADE = ) \CAD =180 ± 2\ACD =180 ± (4 2 ) (1) \CAB =180 ± \ACB \ABC =180 ± 3 (2) \CDE 에대해, 90 ± + =(2 )+( ) =3 2 ) =30 ± (3) 그런데, (1) 3=(2) 2 () 540 ± (12 6 ) = 360 ± 6 () 180 ± =6( ) () 30 ± = 이것은 (3) 에의해성립. ) 2\CAB =3\CAD ) \CAD = 2 3 \CAB =) \DAB = 1 3 \CAB ) \CAB =3\DAB 28. 세변과한수선의길이가네개의연속한양의정수이고이수선이원래의삼각형을각변의길이가정수인두직각삼각형으로나누는삼각형을하나구하여라. 이런삼각형은당신이찾은것이유일함을보여라. ( 캐나다 1993-1) 풀이그림과같이각변과수선의길이를 a = x + y, b, c, h 로두자. h<b;c 이므로, 일반성을잃지않고 b<c 라하면, 크기순서는다음과같이네가지경우가나온다 : ahbc, habc, hbac, hbca. 피타고라스의정리를응용하면 c 2 b 2 =(h 2 + y 2 ) (h 2 + x 2 )=y 2 x 2 =(y + x)(y x) 가성립하고, 위의네가지경우각각에대해여기에 y + x = a, b, c 등을모두 h 에관한식으로대입하면 (h 1)(y x) =2h +3 (i) (h +1)(y x) =2h +5 (ii) (h +2)(y x) =4h +8 (iii) (h +3)(y x) =2h +3 (iv) 의식을각각얻는다.(i) 의경우는 h > 6 이면 2(h 1) < 2h +3 < 3(h 1) 이므로해가없고, h =1; 2; 3; 4; 5; 6 일때도직접구해보면아무것도해가되지않는다. (ii) 의경우는 h>2 이면 2(h+1) < 2h +5< 3(h +1) 이므로해가없고, h =1; 2 일때도직접구해보면해가되지않는다.(iv) 의경우는
3.2 기하고급문제 167 언제나 h +3< 2h +3< 2(h +3) 이므로해가없다. 이제 (iii) 의경우만남았다. 이때 y x =4, 즉 y = x +4 이고, 이것을이용해모든값을 x 로나타내면다음의왼쪽그림과같다. 왼쪽직각삼각형에서 x 2 =4x +5, 즉 (x +1)(x 5) = 0 이므로, x =5일때오른쪽그림과같은삼각형이유일하다. 29. 직선 ` 위에네점 A, B, C, D 가주어져있다. 이때정사각형하나를작도하여그두대변이 ( 또는그연장선이 ) 각각 A 와 B 를지나도록하고, 또나머지두변이 ( 또는그연장선이 ) 각각 C 와 D 를지나도록하여라. ( 헝가리 1898-3) 힌트 CD 를 90 ± 회전하여한꼭지점이 B 에붙도록한다. 30. 정삼각형 ABC의내접원의한접선이변AB, AC와각각점D, E에서만난다. AD DB + AE =1임을증 EC 명하여라. ( 폴란드 1995/1996 1차-4) 증명 j4adej = 1 2 (AD + EA DE)r = 1 2 AD DE sin 60± 와준식이동치. 31. 4ABC 는 AB = AC 인이등변삼각형이다. \B 의이등분선이변 AC 와 D 에서만나고, BC = BD + AD 가된다고한다. \A 를구하여라. ( 캐나다 1996-4) 풀이 BC 위에 BD = BE 가되는점 E 를잡자. 그럼문제의조건에의해 EC = AD 가된다. 각의이등분선의성질에의해 AB : BC = AD : DC = EC : CD 이고 \C 가공통이므로 4ABC 와 4ECD 가닮았다. 따라서, 4ECD 도이등변삼각형. \B =2 라두면 \BDE = \BED = \ECD + \ECD = \B + \B =4. 즉, 4BCD 의내각의합에서 +6 +2 =9 =180 ± 이므로 =20 ± 이고, 4ABC 의내각의합에서 \A =180 ± 4 =100 ± 답 32. AB = AC 인이등변삼각형 ABC 의변 AB, AC 와각각 P, Q 에서접하고 4ABC 의외접원에내접하는원이있다. 선분 PQ 의중점은 4ABC 의내심임을증명하여라. ( 셈본중등중급도전문제 4.2.3) 증명 T 를두원의접점이라하고,M 을선분 PQ 의중점이라하자. 접선과현의성질에의하여 \BPT = \PQT, 이등변삼각형의대칭성에의하여 \BPT = \PQT = \QP T 가된다. \PBT = \PMT = ¼ 2 이므로 4PBT 와 4PMT 는 RHA 합동이다.
168 기하 A P M Q B T C )\PBM = \PMB = \MBC 따라서 M 은 4ABC 의내심이다. 33. 정삼각형 ABC 에서변 BC 와평행한직선이변 AB, AC 와 D, E 에서만난다. 선분 CD 의중점을 F 라하고, 정삼각형 ADE 의중심을 G 라하자. 삼각형 BFG 의세내각의크기를구하여라. ( 호주 2004-4) 풀이하자. (KAIST 과학영재센터연구원고봉균 ) 점 E, A, G 를 F 에대해점대칭시킨점을각각 I, J, H 라 그럼 (A; D; E; G) 와 (C; I; J; H) 는그순서대로점 B 에대해 60 ± 회전이동한점이된다. 따라서, BG = BH, \GBH =60 ± 가되고, 그럼 BHG 는정삼각형이다. F 는변 GH 의중점이므로, 4BFG 는세내각의크기가 30 ±,60 ±,90 ± 인직각삼각형이다. 34. AB = AC 인이등변삼각형 ABC 에서변 BC 의중점을 D 라한다. \ABC 의삼등분선이 AD 와만나는점을 A 로부터차례대로 M, N 이라하고, CN 의연장과 AB 의교점을 E 라하면 EM==BN 임을증명하여라. (1991 한국계절학교선발시험 ) 풀이 \BEC =180 ± 4 \ABD = \BMC 3 이므로 EBCM은원에내접한다. 따라서 \EMB = \ECB = \MBN( 원주각 ) 이고따라서 EM k BN이다. 35. \A =90 ± 인직각삼각형 ABC 가있다. \B 와 \C 의이등분선이 AC, AB 와만나는점을각각 D, E 라하고, BD 와 CE 의교점을 O 라고하면, 4BOC 의넓이는사각형 BEDC 의넓이의절반이됨을보여라. 증명 CO에대한 D의선대칭점을 D 0, BO에대한 E의선대칭점을 E 0 이라하자. 그럼두삼각형 ODE와 OD 0 E 0 의넓이가같다. 36. 중심이각각 O 1, O 2 인원S 1, S 2 가두점A, B에서만난다. O 1, O 2, A를지나는원이 S 1, S 2, 직선 AB와각각 D, E, C에서다시만난다. CD = CB = CE 임을보여라. ( 러시아 1995 4차-y9-6) 증명 \CBD = 라하면 \AO 1 D =2, \ACD =180 ± 2. \CDB =180 ± \CBD \ACD =.
3.2 기하고급문제 169 별증 1 O 2 A = O 2 C 이므로 \O 2 O 1 C = \O 2 O 1 A = \O 2 O 1 B, 즉 O 1 BC 는일직선. 마찬가지로 O 2 BD 도일직선. 그럼, \EBC = \ABO 1 = \EAO 1 = \ECO 1. 별증2 \DO 1 C = \DAC = \DAB = 1 2 \DO 1B 이므로 O 1 C는 BD를수직이등분. 마찬가지로 O 2 C도 BE를수직이등분. C는외심. 37. 점 P 는이등변삼각형의밑변위의점이다. 옆변과평행하고, 점 P 를지나는두직선이삼각형과만나는 P 가아닌점을각각 Q, R 이라하자. 점 P 를직선 QR 에대칭이동시킨점은삼각형의외접원위에있음을증명하여라. ( 헝가리 1949-2) 증명 P 의대칭점을 P 0 이라하면 AQRP 0 cyclic. 180 ± \P 0 AQ = \P 0 RQ = 1 2 \P 0 RP (Q쪽)= \P 0 BP. (R이 P 0 BP의외심이라서 ) 38. 삼각형 ABC 의내부에 \PBC = \P CA < \PAB 를만족하는점 P 가있다. 직선 BP 가삼각형 ABC 의외접원과만나는점을 E 라하고, 직선 CE 가삼각형 AP E 의외접원과만나는점을 F 라하자. 사각형 AP EF 와삼각형 ABP 의넓이의비는점 P 의선택에관계없이일정함을증명하여라. ( 폴란드 1996 3 차 -2) 증명 \PCA = \PBC = \EAC 이므로 AE k PC. 등적변형으로 j4ap Ej = j4acej, 즉 j AP EF j = j4acf j. 4ACF»4ABC 이고닮음비는 AB : AC 로일정. 39. 삼각형 ABC 에서 I 는내심이고 D, E, F 는각각내접원이변 BC, CA, AB 와만나는접점이다. 직선 AD 는 P 에서내접원과다시만난다. M 이 EF 의중점이라면, 네점 P, I, M, D 가한원위에있음을증명하여라. ( 남미 1990-2) 증명 AP AD = AF 2 = AI AM 40. \A =2\C 인 4ABC 가있다. 변 BC 의길이 x 를다른두변 AC, AB 의길이 b, c 에관한식으로나타내어라. (1994 서울시 ) 풀이 \A의이등분선과 BC의교점을D라하자. 그러면 BD CD = c b 이다. 따라서, BD = cx bx ; CD = b + c b + c 가된다. 한편 \BAD = \BCA 이고 \ABC 는공통이므로 4ABD 와 4CBA 는닮음이다. 닮음비에의해 b : x = bx b+c : c 이고여기서 x = p bc + c 2 이다. 41. 평행사변형 ABCD 의내부에 \AOB + \COD =180 ± 가되도록점 O 를잡는다. 그럼 \OBC = \ODC 임을보여라. ( 캐나다 1997-4) 증명 4AOB 4DPC 가되도록 O 를평행이동한점을 P 라하자. \COD + \DPC = 180 ± 이므로 OCPD 는원에내접하는사각형이고, 따라서, 원주각과평행사변형의대각에의해 \ODC = \OPC = \OBC. 42. 임의의삼각형에서둘레의길이를 L, 세중선의길이의합을 M 이라할때 M L > 3 4 임을증명하여라. ( 한국 1993-9)
170 기하 증명 AG + GB > AB 등을모두변변더하면됨. 43. 임의의예각삼각형이주어져있다. 몇개의선분을그려이삼각형을세영역으로분할하는데, 각각의분할된영역은선대칭도형 ( 어떤직선에대해대칭이동시켰을때자기자신이되는도형 ) 이되도록하려고한다. 이런분할방법이항상 3가지이상있음을보여라. (IMTS R1-4) 풀이 (1) 외심과각꼭지점을이으면 3 개의이등변삼각형이생긴다. (2) 내심에서각변에수선을내리면 3 개의선대칭사각형이생긴다. (3) AB, AC 의중점을 M, N 이라하고 A 에서 BC 에내린수선의발을 H 라하자. 그럼선대칭사각형 AMHN 과두개의이등변삼각형이생긴다. 44. 직사각형 ABCD 가있을때, 어떤직선 ` 이각변 AB, CD, 그리고 AD, BC ( 또는그연장선 ) 들과만나는점을각각 M, N, 그리고 P, Q 라하자. 네점 M, N, P, Q 와변 AB 의길이 p 가주어져있을때, 원래의직사각형을작도하여라. 이문제는어떤조건에서해를구할수있고, 또얼마나많은해가있는가? ( 헝가리 1897-3) 힌트 AB 즉 p 를평행이동하여한쪽끝이 P 혹은 Q 에오도록한다. 45. 한직선위에있지않은세점 M, N, P 가있다. M, N 은어떤삼각형의두변의중점이고 P 는그삼각형의수심이다. 이조건을만족하는삼각형을작도하여라. ( 남미 1991-6) 풀이 M에대한P의대칭점을 Q라하자. QN을지름으로하는원과 P 에서 MN에그은수선의교점을 A라하면됨. 46. N 은 \BAC 의이등분선위의점이다. P 와 O 는 AB 와 AN 위의점으로 \ANP =90 ± = \AP O 이다. Q 는 NP 위의임의의점이고 Q 를지나는임의의직선이 AB, AC 와각각 E, F 에서만난다. 이때 \OQE =90 ± 와 QE = QF 는동치임을보여라. (IMO 1994-2) 증명먼저 \OQE =90 ± 라고가정하자. PN의연장선이AC와 R에서만난다고하자. 그러면 OEPQ와 ORFQ는각각동일원주상의사각형이므로 4OEQ 4OFQ가되어QE = QF 를얻을수있다. 이제 \OQE 6= 90 ± 라고가정하자. 점 O 를지나고 EF 에수직인직선이 NP 와 Q 0 6= Q 에서만난다고하자. Q 0 을지나고 EF 에평행한직선은 AB; AC 와각각 E 0 ;F 0 에서만난다고하자. 그러면위에서한것처럼 Q 0 E 0 = Q 0 F 0 을얻을수있다. AQ 0 이 EF 와 M 6= Q 에서만난다고하자. 그러면 ME = MF 가되어 QE 6= QF 이다. 즉 QE = QF 이면 \OQE =90 ± 47. 사다리꼴 ABCD 의두대각선의교점을 M 이라하자. AD k BC, AB? AD 이고이사다리꼴의내접원이존재한다. 이내접원의반지름을 R 이라했을때 4DCM 의넓이를구하여라. ( 셈본중등고급도전문제 4.1.3)
3.2 기하고급문제 171 풀이내점원과 AB, BC, CD, DA 의교점을각각 M, N, P, Q 라하자. A R Q a D R M R M a P b B R N b C 이때, AM = MB = BN = AQ = R 이고, DQ = DP = a, CN = CP = b 라놓자. D 에서 BC 에내린수선의발을 D 0 이라하면피타고라스정리에의해 (a + b) 2 =(b a) 2 +(2R) 2 ) R 2 = ab S 4DCM = S 4BCD DM BD = 1 AD 2R(R + b) 2 AD + BC = R(R + b) R + a 2R + a + b = R R2 + R(a + b)ab 2R + a + b = R R(2R + a + b) 2R + a + b = R 2 48. 세점 X, Y, Z 가주어져있다. X 를외심으로하고, BC 의중점이 Y 이고, BZ 가한수선이되는삼각형 ABC 를작도하는방법을찾아라. ( 아일랜드 1991-1) 풀이직선 BC 를그리고원 Y (Z) 를그려교점을 B, C. 49. 삼각형 ABC에대하여 \BAC의이등분선이삼각형 ABC의외접원과만나는점을 M, CM과 AB의교점을 P 라하자. P 를지나고 AM과수직인직선이 AC와만나는점을 X, P 를지나고AC와수직인직선이 AM와만나는점을 Y, P 를지나고 BC와수직인직선이 MB와만나는점을 Z라하자. 세점X, Y, Z가한직선위에있음을보여라.( 단, \B는 \C보다크다.) ( 한국 2005-J5) 증명 (KAIST 과학영재센터연구원고봉균 ) P 에서 AC, BC 에내린수선의발을각각 Q, S 라하고, XY 와 AB 의교점을 T 라하자.
172 기하 Z, Y, X 가일직선을이룸을보이면되겠다. PQCS 가 PC 를지름으로하는원에내접하므로, 내대각과원주각으로 \ZPY \SPQ = \SCA \BCA = \BMA \ZMA 따라서, ZPY M 은원에내접하는사각형이다. 또이등변삼각형 AP X 는축 AM 에대해대칭이므로 \XTP = \PQX = \R 이고, 따라서 AT Y Q 도원에내접하는사각형이다. 그럼다시원주각과내대각에의해 \ZY P = \ZMP = \BAC \TAQ = \XY Q 가되고, 그럼맞꼭지각에서 X, Y, Z 가일직선을이룸을확인할수있다. 50. ABC 는 \BAC =40 ± 이고 \ABC =60 ± 인삼각형이다. D 와 E 는각각변 AC 와 AB 위의점으로서, \CBD =40 ± 과 \BCE =70 ± 을만족한다. F 는두직선 BD 와 CE 의교점이다. 두직선 AF 와 BC 가직교함을증명하여라. ( 캐나다 1998-4) 증명직선 BF 에대한 C 의대칭점을 G 라하고, GF X 가정삼각형이되는 B 에서먼점을 X 라하자. 그리고, F 에서 BC 에내린수선의발을 H 라하자. 4BGF 4BCF 이고이들은세내각이 40 ±, 70 ±, 70 ± 인이등변삼각형이다. 그럼 BX는밑변이맞붙은두이등변삼각형 4BGF와 4XGF의꼭지각을이등분하므로, \XBC =60 ± 이다 (1). 한편, \XFH =60 ± +70 ± +50 ± =180 ± 로 XFH는한직선을이루므로 \HFC는 4FCX의외각이고, FX = FG = FC 로이등변삼각형이므로 \FXC = 1 2 \HFC =10± 이다. 그러므로 \BXC =40 ± 가된다 (2). (1), (2) 에의해4XBC와 4ABC는세내각이같고 BC가일치하므로합동, 즉 X = A 이다. 따라서, AF? BC 가확인된다. 51. 볼록오각형 ABCDE 에서 BC, CD, DE 의길이가같고각대각선이이오각형의다른어떤변에평행하다면 ( 예를들어 AC 는 DE 와평행하다 ) 주어진볼록오각형 ABCDE 는정오각형임을보여라. ( 통신강좌 1995-10-21, 미국 1972-5 유사 ) 증명 ( 대전과학고윤재문 ) 문제의조건의의해 BC = CD = DE 이므로, 4BCD, 4CDE는이등변삼각형이다. \CBE = \CDB \DCE = \DEC BE =CD이므로 \CBD = \CDB = \DBE \DCE = \DEC = \CEB 그림처럼 ED =E 0 C가되도록 BE상에 E 0 을잡으면 E 0 CDE는평행사변형 E 0 C = ED = BC
3.2 기하고급문제 173 2\CBD = \CBE 0 = \CE 0 B = \DEB =2\DEC ) \CBD = \DEC 각대각선이평행이므로엇각을이용하여같은각을모두표시하면같다. 그런데, ABCD에서 \ABD = \ACD이므로이사각형은원에내접하고그러면원주각으로 \ADB = \ACB가된다. 다시엇각을적용하면 \ADB = \BCA = \CAD = \ADE 그러면 4ABC, 4ADE 도이등변삼각형이되어 5 개변, 각이모두같아정 5 각형이된다. Α Α Β Ε Β Ε ' Ε Β Ε Χ Χ Χ 52. P 와 Q 는삼각형 ABC 의 \BAC 내부에있고 ( 삼각형의내부에있을필요는없다 ) PQ 의연장선이 BC 를수직이등분하며, \ABP + \ACQ = ¼ 이다. \BAP = \CAQ 임을보여라. ( 통신강좌 1995-10-42) 증명 ( 상문고이경용 ) 4ABP 의외접원과직선 PQ의교점을 R이라하자. 점 A, B, P, R은같은원주상에있으므로 \ABP = \ARP, \ABP + \ACQ = ¼이므로 \ARP + \ACQ = ¼. 그러므로, ARQC도원에내접하는사각형이다. \BAP = \BRP \CAQ = \CRQ 4RBC 는이등변삼각형이므로 \BRP = \CRQ ) \BAP = \CAQ R A B C P Q 53. 원 O의외부에있는점 P 를지나는한직선이원 O와두점A, B에서만나고, P 를지나는또다른직선이원 O와두점C, D에서만난다. 단, A는 P 와 B 사이에, C는 P 와 D 사이에있다. 선분 AD와 BC의교점을 L이라하고, 반직선 PA 위에 BL PE = DL PD 가되도록점 E를잡자. 직선 PL과직선DE의교점을 M이라할때, M은선분DE의중점임을보여라. ( 한국 2005-S5)
174 기하 증명 ( 고양백석고 2 학년정윤범 ) L; M 에서 PB, PD 에내린수선의발을각각 H, I; J, K 라하자. 4LAB 와 4LCD 가닮았으므로그닮음비를 a : b 라하자. 그럼 LH : LI = a : b. 또, P 를중심으로한닮음에서 MJ : MK = a : b 이기도하다. 그런데, 문제의조건에서 PD : PE = BL : DL = a : b = MJ : MK 이므로, PD MK = PE MJ, 즉 j4pemj = j4pdmj 으로넓이가같다. 그럼이두삼각형을각각 EM, MD 를밑변으로하고같은높이를갖는삼각형으로보면 EM = MD 임을알수있다. 따라서, M 은 DE 의중점이다. 54. 한원이 ABCD 에내접한다. AB 와 CD 는평행하고 BC = AD 이다. 대각선 AC, BD 는 E 에서만난다. 삼각형 ABE, BCE, CDE, DAE 의내접원의반지름은각각 r 1 ;r 2 ;r 3 ;r 4 이다.1=r 1 +1=r 3 =1=r 2 +1=r 4 임을증명하여라. ( 소련 1964-15) 증명삼각형 ABE, BCE, CDE, DAE를각각X 1, X 2, X 3, X 4 로나타내고, 삼각형 X i 의넓이와둘레의길이를각각 S i, d i 로나타내자. 일반적으로 S = dr 이성립하고, 또 AB : CD = p : q 라할때셈본을참조하면 S 1 : S 2 : S 3 : S 4 = p 2 : pq : q 2 : pq 이다. 또한삼각형의닮음에서 d 1 : d 3 = r 1 : r 3 = p : q 이다. 원에외접하는사각형이므로 AB + CD = BC + DA 임에서 S 1 S 2 d 1 + d 3 = d 2 + d 4 S 1 + S 3 = S 2 + S 4 r 1 r 3 r 2 r 4 1 r 1 + S 3 S 2 1 = 1 + 1 r 3 r 2 r 4 p q 1 + q r 1 p 1 = 1 + 1 r 3 r 2 r 4 1 + 1 = 1 + 1 r 3 r 1 r 2 r 4 가확인된다. 55. AB =36,AC =72,\B =90 ± 인종이삼각형 ABC 가있다. 이삼각형의내부에점 P 를잡고, A, B, C 가각각 P 에오도록종이를한번씩접었다가펴서세개의접힌자국의선분이생기도록한다. 이세선분이서로가로지르지않도록 P 를잡을수있는영역의넓이를구하여라. (AIME 1994-15) 풀이 %EEE [ 그림 ] The folds to bring A and B to P are the perpendicular bisectors of AP and BP, which meet at O. We want O to lie below AB. Note that O is the circumcenter of ABP, so APB = ½ AOB. We want AOB > 180^o, so APB > 90^o, so P must lie inside the semicircle with AB as diameter. [ 그림 ] Similarly P must lie inside the semicircles on AC and BC.
3.2 기하고급문제 175 The semicircle on AC does not impose any constraint since B lies on it and so the entire triangle lies inside it. Thus we want the area of the intersection of the semicircles on AB and BC. The upper arc BC has radius 18 and subtends an angle 120^o, so the area between it and the chord BD is (1/3)π18^2-18^2cos 30^o sin 30^o = 108π - 81 3. Similarly, the other arc has radius 18 3 and angle 60^o, so area (1/6)π3 18^2-243 3. So total area 270π - 324 3. 답 270¼ 324 p 3 56. 삼각형 ABC 의내접원이변 AB, AC 와각각점 D, E 에서접한다. \ACB 와 \ABC 의이등분선이직선 DE 와만나는점을각각 X 와 Y 라하고, 변 BC 의중점을 Z 라하자. 삼각형 XY Z 가정삼각형일때, 또그때만 \A 가 60 ± 임을증명하여라. ( 발칸 2005-1) 증명내심을 I라하자. \DXI = \DEC + \ECX =90 ± + A 2 + C 2 이므로, \DXI + \DBI =180±. 따라서, BIXD는원에내접하는사각형이다. 그럼, ID? AB 이므로, BX? CI 가된다. 즉 CXB가직각삼각형이므로, ZX = ZB = ZC 이다. 같은방법으로 ZY = ZB 도얻을수있고, 따라서 XY Z는항상이등변삼각형이다. ZX = ZC 이므로 \ZXC = \ZCX = \ACX 이고, 이로부터 ZX k AC 이다. 마찬가지로 ZY k AB 도확인할수있다. 이상으로부터 XY Z가정삼각형일필요충분조건은 \XZY =60 ±, 즉 \A =60 ± 임을알수있다. 57. 는반지름 r 인원이다. A 와 B 는 AB < p 3 r 인 위의서로다른두점이다. B 를중심으로하고반지름 AB 인원이 와 C 에서다시만난다고하자. 그리고 P 를 ABP 가정삼각형이되는 내부의점이라고하자. 마지막으로, CP 가 와 Q 에서다시만난다고하자. PQ = r 임을증명하여라. ( 캐나다 2002-4) 풀이원 의중심을 O 라하고 \QCB = 라고하자. 원 B 와원 에서원주각과중심각의관계를고려하면 \QOA =2\QCA =2\PCA = \PBA 로꼭지각이 60 ± 인이등변삼각형이므로 OQA 는정삼각형이다. 그럼 A 를중심으로 Q 와 P 를 60 ± 회전한점이각각 O 와 B 가되므로 4QAP = 4OAB. 따라서 QP = BO = r. 58. 서로만나는두원의교점의하나를 O 라하자. O 를지나는직선이두원과만나는점을각각 P, Q 라하자. 선분 PQ 의중점의자취를구하여라. ( 통신강좌 1995-11-4) 풀이 ( 서울과학고이지운 ) 두원의다른교점 O 0 이라하자. 우선그림과같이 P, Q 가각각원의바깥
176 기하 쪽호와만난경우를생각하자. 선분 PQ 이외의다른임의의선분을 P 0 Q 0 이라하자. 선분 OQ 에대한원주각으로써 \OPO 0 = \OP 0 O 0 ; \OQO 0 = \OQ 0 O 0 이다. 따라서 이된다. PQ; 4PO 0 Q 4P 0 O 0 Q 0 (AA 닮음 ) P 0 Q 0 의중점을각각 M, M 0 라하면 O 0 P 0 : O 0 P = P 0 M 0 : PM; \OP 0 O 0 = \OP O 0 이므로 이다. 따라서 4MPO 0»4M 0 P 0 O 0 (SAS 닮음 ) \OM 0 O 0 = \OMO 0 = 일정 이다. 그러므로 M은 OO 0 을지나는원주위의점이다. P, Q가다른쪽호와만날때의경우도비슷하게증명할수있다. 59. r 1 <r 2 <r 은각각원 1, 2, 의반지름이다. 1 과 2 가각각점 A, B 에서 에내접하고있으며, 둘은서로다른두점에서만난다. 만약 AB 가 1 과 2 의한교점을지날필요충분조건은 r 1 + r 2 = r 임을증명하여라. ( 이탈리아 1999-3) 증명세원의중심을O 1, O 2, O라하자. O 1, O 2 는각각OA, OB 위에있다. AB가지나는두원의교점을 C라하면OAB, O 1 AC, O 2 CB는모두닮은이등변삼각형. 역도싸바싸바하면성립. 60. 주어진한원에접하고이원밖에주어진두점을지나는원을작도하여라. ( 셈본중등고급도전문제 4.2.4) 풀이원 O와원밖의두점A, B가주어져있다고하자. A, B를지나고원O와두점에서만나는원 O 0 을그린다. A E A C F O B O B D
3.2 기하고급문제 177 O 0 과 O와의교점을 C, D라하고, Ã! CD와 Ã! AB의교점을E라하자.(CD와 AB가평행하면다른원을잡는다.) E에서원 O에접선을그어접점을 F 라하자. 4ABF 의외접원이우리가구하려는원이다.(* 세원이각각접하거나두점에서만날때, 각각의교점을잇는세직선은한점에서만남.) 61. 원에내접하는사각형 ABCD에대하여변 AB 위에중심 O를갖는다른원이사각형ABCD의다른세변 AD, DC, CB와각각E, F, G에서접한다. FC = p 2, AO =3AE 일때AE의길이를 a라고하자. 4a 2 의값을구하여라. ( 한국 2006 1차-J17) 풀이 4OFC 를 O 를중심으로회전하여 4OEH 가되도록하자. \OHE = \OCF = µ 라하면 \A =180 ± \C =180 ± 2µ. 따라서, 4AOH 는 \AHO = \AOH = µ 인이등변삼각형이고, AH = AO 이다. 그럼 p 2=FC = EH = AH AE = AO AE =2a. 따라서,4a 2 =2 답 주 `평면기하의아이디어 '( 박승동편저, 현재는절판됨 ) 라는책의 41쪽예10에이와거의똑같은문제가실려있다. 위의풀이는거기서옮겨온것이다. 출제가잘못된듯하다. 별해 ( 광주동성고 3 학년방재혁 ) 피타고라스의정리에서 OE = OF = OG = p (3a) 2 a 2 =2 p 2 a. \A = µ 라하자. 사각형 ABCD가원에내접하므로, \C = ¼ µ. 따라서 \FCO = 1 (¼ µ). 2 그러므로 \FOC = µ 2 4AOE 와 4OCF 에서 tan µ = 2p 2a a =2 p 2 tan µ 2 = p 2 2 p 2 a = 1 2a
178 기하 이것을탄젠트배각공식 tan µ = 2tan µ 2 1 tan 2 µ 에대입하면 2 2 p 1 a 2= 1 1 = 4a 4a 2 1 4a 2 4 p 2a 2 p 2=2a p 2 a 2 4 a 1 4 =0 Ã p!ã p! 2 2 a a + =0 2 4 이된다. a>0 이므로 a = p 2 2. 따라서,4a2 =2 답 62. ABC 는예각삼각형이다. B 에서내린수선이 AC 를지름으로하는원과점 P, Q 에서만나고, C 에서내린수선이 AB 를지름으로하는원과점 R, S 에서만난다. 네점 P, Q, R, S 는한원위에있음을증명하여라. ( 체코슬로바키아 1992-6) 증명 BD, CE를수선, H를수심이라하면 D, E는각각원AC, AB 위에있고 BCDE는 cyclic이므로방멱에의해 PH HQ = CH HE = BH HD = RH HS. 63. 점 A, B, C는원O 위의서로다른점이다. A, B를지나는 O의두접선이 P 에서만난다고하자. C를지나는 O의접선은 AB와 Q에서만난다. 다음을증명하여라. PQ 2 = PB 2 + QC 2 ( 폴란드 2002/2003 1차-3) 증명 ( 여주홍지현 ) X 2 P X 1 Q A B O C QC는원O의접선이므로, QC 2 = QB QA (1) 직선 PQ 위에 PB = PX 1, PA = PX 2 인점X 1, X 2 를찍으면, PA = PB 이므로 PA = PB = PX 1 = PX 2 로네점A, B, X 1, X 2 는 P 를중심으로하는한원위에있다. 그러면할선의비례에관한성질에서 QA QB = QX 1 QX 2 =(PQ PB)(PQ+ PB) (2) (1), (2) 에의해, QC 2 = PQ 2 PB 2 으로문제가증명되었다. 64. \ABC + \ABD =180 ± 이고 AC DE = BC CE 인사각형 ABCD 가있다. 단, E 는두대각선의교점이다.2(AB 2 + BC 2 )=AC 2 + AD 2 임을증명하여라. (ML 프로포절 158-3, 이탈리아 2000-2 변형 ) 증명 B 에대한 C 의대칭점 F 를잡자.
3.2 기하고급문제 179 그럼 4AF C 에서 AB 가중선이므로파푸스의중선정리에의해 AC 2 + AF 2 =2(AB 2 + BC 2 ) 따라서, 문제의식과비교하면 AF = AD ( ) 임을증명하면됨을알수있다. E 에서 AB 에평행한직선을그려 BC 와만나는점을 H 라하자. 그럼 AC : CE = BC : CH 이므로문제의조건과비교하여 임을알수있다. 또, DE = CH (1) \EHB = \ABF =180 ± \ABC = \ABD = \HEB 이므로 4BHE 가이등변삼각형이되어 BE = BH (2) 이다.(1) 과 (2) 를변변더하면 BC = BD 이다. 이로부터 BF = BD, \ABF = \ABD, AB는공통이므로, 4ABD 4ABF (SAS합동) 이고, 이로부터 ( ) 이확인된다. 65. A, B, C, D 는한원위에시계반대방향의순서로놓인네점이고 AB < AD 와 BC > CD 를만족한다. 각 BAD 의이등분선이이원과만나는점을 X 라하고, 각 BCD 의이등분선이이원과만나는점을 Y 라하자. 이원주위의 6 개의점으로이루어진 6 각형을생각하자. 이 6 각형의 6 개의변중에서 4 개의길이가같으면, BD 가이원의지름이됨을증명하여라. ( 캐나다 2004-3) 증명 원주각의크기가같으면현의길이도같으므로, AX 가 \A 의이등분선임에서 BX = XD, CY 가 \C 의이등분선임에서 BY = YD 가된다. 즉, AB; AY < Y D; CX; CD < BX 이고,6 각형의변중 4 개의길이가같으려면 AB = AY = CX = CD 일수밖에없다. 그럼 \A =2\(XD)=4\(CD)=4\(AB) =2\(BY )=\C 가되고 (\(PQ) 는현 PQ 의원주각을의미함 ), \A = \C =90 ± 일수밖에없으므로 BD 는지름이된다. 66. 평행사변형 ABCD 가있다. P 는 \PAB = \PCB 를만족하는이평행사변형외부의점이다 ( 단, 두각의방향은서로반대이다 ). \AP B = \DPC 임을증명하여라. ( 헝가리 1976-1)
180 기하 증명평행사변형 CDPQ 를만들면, 4ADP 4BCQ, \AP B = \PBQ = \PCQ = \DPC, 그리고각비교로 BCQP cyclic. 67. ABC 는 AC = BC 인이등변삼각형이다. M 은 AC 의중점이고, z 는 AB 와수직이고 C 를지나는직선이다. 세점 B, C, M 을지나는원이직선 z 와두점 C, Q 에서만난다. 삼각형 ABC 의외접원의반지름을 m = CQ 에대한식으로나타내어라. ( 주니어발칸 2004-2) 풀이 ABC의무게중심과외심을각각 G, O, AB의중점을 N이라하자. AMGQ, AMON cyclic 이므로 CG CQ = CM CA = CO CN. 따라서, CO = CG CN m = 2 3 m. 68. 원 O의원주위에반시계방향으로여섯개의점 A, B, C, D, E, F 가있다. 선분 BD와 CF는직교한다. 또한, 세선분CF, BE, AD는한점에서만난다. 점 B에서 AC에내린수선의발을 M, 점 D에서 CE에내린수선의발을 N이라할때, AE와 MN이평행함을보여라. ( 한국 2006-J3) 증명 ( 고양백석고 3 학년조상영 ) CF \ BE \ AD = L, BD \ CF = K 로정의하자. \BKC = \BMC =90 ± 이므로 B;M; K;C 는한원위에있다. 따라서원주각에의해 \LAC = \DAC = \DBC = \KBC = \KMC 즉, LA k KM 동일한원리로 LE k KN 이로부터 C를중심으로하여 CALE» CMKN 이고, 따라서 4AEC»4MNC, 또 AE k MN 임을알수있다. 69. AB 를지름으로하는원의 B 에서그은접선위에두점 C 와 D 가 B 에대해서로반대쪽에있다. AC, AD 가원과각각 E, F 에서다시만난다. 또, CF, DE 가원과각각 G, H 에서다시만난다. AG = AH 임을보여라. ( 중미 2003-2) 증명 AK 를접선이라하자 (K 는 D 와같은방향 ). \CDF = \FAK = \FEA 이므로 FDCE 는 cyclic. 그럼 \FCD = \FED = \FGH 로 GH k CD. 70. 삼각형 ABC 의점 C 를지나고 AB 에평행한직선을 ` 이라하자. \A 의이등분선이 BC 와점 D 에서만나고 ` 과점 E 에서만난다. \B 의이등분선이 AC 와점 F 에서만나고 ` 과점 G 에서만난다. DE = FG 일때 CA = CB 임을보여라. ( 아일랜드 1991-8) 증명 \A =2, \B =2 라하고, > 라가정하자. 그럼 BC > CA 이고 BG > AE. BG AB + CG AB + CE = < = AE FG CG CE DE 가되어모순.
3.2 기하고급문제 181 71. 육각형 ABCDEF 는대칭중심을가진다. AB 와 EF 는점 A 0 에서만나고, BC 와 AF 는점 B 0 에서, AB 와 CD 는점 C 0 에서만난다. 다음을증명하여라. AB BC CD = AA 0 BB 0 CC 0 ( 폴란드 1992/1993 1 차 -3) 증명 세변의길이가a, b, c인삼각형에서 a b b c c =1 a 임을이용. 이것을닮은삼각형에적용하여변 형. 72. 삼각형 ABC 와점 P 가같은평면위에있으며점 P 는 ABC 의외심이아니다. P 에서삼각형 ABC 의각변의수직이등분선에내린수선의발을세꼭지점으로하는삼각형이 ABC 와닮았음을증명하여라. ( 이탈리아 1993-4) 증명세수선의발이모두 OP를지름으로하는원위에있음. 세각이둘씩같은AA닮음. 경우를좀간략히따지는요령은필요할듯. 73. 사각형 ABCD 를어느두변도평행하지않은볼록사각형이라하자. 대변의연장선에의해만들어지는두각의이등분선이사각형 ABCD 의네변과각각 P, Q, R, S 에서만난다고하자. 사각형 ABCD 가외접원을가지는것과사각형 PQRS 가마름모라는것이동치임을보여라. ( 통신강좌 1997-15-6) 증명 ( 과기원 97 학번이수인 ) 그림과같이 AD 와 BC 의연장선의교점을 M, AB 와 CD 의연장선의교점을 N 이라하자. 각 \AMB 와 \BNC 의이등분선이각변과만나는점 P, Q, R, S 가그림과같다고하고 MP 와 NQ 의교점을 O 라하자. \ABC = ; \CDA = µ; \MON =!; \OMA = \OMB = Á; \ONB = \ONC = Ã 라하자. 사각형 OMDN 과사각형 ONBM 의내각의합은 360 ± 이므로 M φφ D δ R C ω S ψ ψ β N A P B! + Á + (360 ± µ)+ã =360 ± =(360 ±!)+Ã + + Ã 2! = + ± 사각형 ABCD 가내접사각형이려면 + µ = 180 ± 이어야하므로! = 90 ± 이다. 따라서 \NOR = \NOP =90 ±, \ONR = \ONP = Ã 이므로 ) 4NOP 4NOR (*! =90 ± ) 따라서 OP = OR, OQ = OS ( ) 대각선이서로수직이고,( ) 을만족하므로 PQRS 는마름모이다. O Q 74. \A =2\B 인모든삼각형 ABC 에대해, C 에서 A 까지의거리와 AB 의수직이등분선까지의거리의비가일정함을보여라. ( 체코슬로바키아 1991-4) 풀이 AB k CD 인등변사다리꼴 ABCD를그리고, 두대각선 AD와 BC의교점을P, AB와 CD의중점을각각 M, H라하자. 그럼 PAB, PCD는밑각이 \B 와같고P를꼭지점으로하는닮은이등변삼각형. 따라서, 삼각형 CAD와 DBC도그와닮은이등변삼각형이고, CA = CD =2CH. (CH가 C에서 AB의수직이등분선까지의거리임 )
182 기하 75. 원 O의외부의한점P를지나는직선이원 O와두점B, C에서만난다고하자 ( 단, PB < PC). 직선 PO와원O가만나는두점을 Q, D라하고 ( 단, PQ < PD) 점 Q를지나고직선 BC에수직인직선이원 O와만나는또다른점을 A라고하자. 이때, BD 2 = AD CP 이면직선 PA는원O의접선임을보여라. ( 한국 2006-J7) 증명 ( 서울중평중 3 학년송하일 ) \QAD =90 ± 이므로 PC k AD 가된다. 그러면사각형 ABCD 는원에내접하므로등변사다리꼴이고 BD = AC 가된다. 문제의조건으로부터 AC 2 = AD CP 가되는데, AD : AC = AC : CP 이고 \PCA = \CAD 이므로 4PCA»4CAD 가된다. \PAC = \ADC 가되고접현각의성질로부터 PA 는원의접선이된다. 76. AB와 CD가평행인사다리꼴 ABCD가있다. 이사다리꼴에서변 AD, BC 위에각각점 P, Q가있는데, \AP B = \CPD 이고 \AQB = \CQD 라고한다. 사다리꼴의두대각선의교점에서 P, Q까지의거리가같음을보여라. ( 러시아 1994 최종-y9-7) 증명 B, C의 AD에대한대칭점B 0, C 0 을잡으면P는세직선BC 0, B 0 C, AD가만나는점. BB 0, CC 0 이 AD와만나는점을 H, K라하자. 4BAH와 4CDK는 AA닮음. 4BPH와 4CPK도 AA닮음. R = AC \ BD 라하면BP : PC 0 = BP : PC = BH : CK = BA : CD = CR : RD 이므로 RP k DC 0. 즉 RP = DC 0 BR BD = CD AB AB+CD = AB CD. AB+CD 대칭적으로 RQ도같은식. 77. 삼각형 ABC 의변 BC, CA, AB 의바깥쪽에점 P, Q, R 을잡자. 이때, 4BPC»4ACQ»4RBA 이면이세삼각형의외심 P 0, Q 0, R 0 로이루어진삼각형 R 0 P 0 Q 0 도이세삼각형과닮았음을증명하여라. 증명 BPC 의외접원과 CQA 의외접원의교점을 F 라하자. 그럼 \AF B = 360 (180 \P +180 \Q) =\P + \Q =180 \R: 따라서, F 는 ARB 의외접원위에도있다. 선분 P 0 Q 0 은두원 P 0, Q 0 의공통현 (= CF) 과수직으로만난다. 이교점을 X 라하자. 선분 Q 0 R 0 과선분 R 0 P 0 의경우에도마찬가지이다. 이교점들을각각 Y, Z 라하자. \P 0 XF = \P 0 ZR =90 ± 이므로 P 0 ;X;F;Z 도한원위에있다. 따라서 \P 0 =180 \BFC = \P: 같은이유로 Q 0 ;X;F;Y 와 R 0 ;Y;F;Z 도각각한원위에있다. 또 \Q 0 = \Q, \R 0 = \R 이다. ) 4Q 0 P 0 R 0»4BPC»4ACQ»4RBA 78. ABCDEF 는서로마주보는변이평행한볼록육각형이다. 삼각형 ACE와 BDF의넓이가같음을보여라. ( 헝가리 1958-3)
3.2 기하고급문제 183 증명삼각형 ACE를제거했을때육각형에서남는세작은삼각형의넓이를각각두배로키운세평행사변형을육각형안쪽에그리면, 육각형안쪽에작은삼각형 XY Z가하나구성되는데그 XY Z의세변의길이는육각형의마주보는평행변의길이의차들임. 삼각형 BDF에대해서도마찬가지이고, 그럼이렇게구성된 XY Z와 X 0 Y 0 Z 0 은 SSS합동으로넓이가같음. 79. \ACB = 120 ± 인 4ABC 가있다. 이삼각형의외접원의중심을 O, 반지름을 R 이라하자. 또한 4ABC 의수심을 H 라하자. AOBH 는원에내접하는사각형이고그원의중심 O 0 은원 O 위에있음을증명하여라. ( 통신강좌 1998-16-14) 풀이 우선다음과같이그림을그리자. \ACB = 120 ± 이므로 \CAB = µ 라두면 \ABC =60 ± µ \ACK = \ABC + \CAB =60 ± \BCJ = \ABC + \CAB =60 ± 그런데 \AKC = \BJC =90 ± 이므로 \CAK = \CBJ =30 ± 4HAB에서 \AHB =180 ± \HAB \HBA =180 ± (\HAC + \CAB) (\JBC + \CBA) =180 ± (30 ± + µ) (30 ± +60 ± µ) =60 ± AOBH에서 \AHB + \AOB =180 ± ) AOBH는원에내접하는사각형 \AOB = 360 ± \AOB = 360 ± 2\ACB =120 ± ( 원 O에서 ) 4AOB는이등변삼각형이므로 \ABO = 180± 120 ± =30 ± 2 \ABO 는 _ AO 의원주각따라서 _ AO 의중심각 \AO 0 O =2\ABO =60 ± AO 0 = OO 0 이므로 4O 0 AO 는정삼각형따라서 O 0 A = AO = R ) O 0 O = O 0 A = R O 0 이 O 에서 R 만큼떨어져있으므로 O 0 은원 O 위에있다. 80. 4ABC 에서 D 를 A 에서 BC 로내린수선의발이라하고, E 를 B 에서 AC 로내린수선의발, F 를 C 에서 AB 로내린수선의발이라하자. 그리고 BE 의중점을 X, CF 의중점을 Y 라하면 4DXY 의외접원은이삼각형의수심을지남을증명하여라. 그리고 4DXY 는 4ABC 와닮음임을보여라. ( 통신강좌 1998-17-18) 증명 ( 중등부이지훈 ) A A F X H E Y E F H X Y B D M C (i) B D M C (ii) (1) 수심을 H; BC 의중점을 M 이라하자. i) X; Y 가 AD 에대해반대방향에있을때 ; BE =2BX; BC =2BM; \MBX 는공통이므로 4MBX v 4CBE )\MXB =90 ± 마찬가지로 4CMY v 4CBF 에서 \CY M =90 ±
184 기하 \HXM = \HDM =90 ± ) H; X; D; M은한원위에있다. \HXM + \MY H =180 ± ) H; X; M; Y 는한원위에있다. 따라서 H; X; D; Y; M은한원위에있다. ii) X; Y 가 AD 에대해같은방향에있을때 ; BE =2BX; BC =2BM; \MBX 는공통이므로 4MBX v 4CBE )\MXB =90 ± 마찬가지로 4CY M v 4CFB 에서 \CY M =90 ± \MXH = \MY H =90 ± ) M; Y; X; H 는한원위에있다. \MXH + \MDH =180 ± ) M; X; H; D 는한원위에있다. 따라서 H; X; D; Y; M 은한원위에있다. i), ii) 에서 4DXY 의외접원은수심 H 를지난다. (2) i) \BCF + \FBC = \BCF + \CHD =90 ± \FBC = \CHD = \YXD (* 원주각 ) \BCE + \CBE = \BHD + \CBE =90 ± \BCE = \BHD = \XY D (* 원주각 ) 따라서 4DXY v 4ABC(AA 닮음 ) ii) \FBC + \BCF = \DHC + \BCF =90 ± \FBC = \DHC = \DXY (* 원주각 ) \DCE + \DHX = \DY X + \DHX =180 ± )\DCE = \DY X 따라서 4DXY v 4ABC(AA 닮음 ) i), ii) 에서 4DXY v 4ABC 81. 평면위에삼각형 ABC 가있고, P 가 BC 의중점, Q 는 CA 위의 CQ=QA =2 를만족하는점, R 은 AB 위의 AR=RB =2 를만족하는점이다. 이삼각형 ABC 가지워지고세점 P, Q, R 만남아있을때, 원래의삼각형 ABC 를다시작도하는방법을설명하여라. ( 아일랜드 1995-9) 풀이 R에서 PQ의중점을이은직선과평행하게 P, R, Q를지나는등간격의세평행선을그리고, 또역시등간격으로 Q 다음의평행선을하나더그리고, PQ, PR이이마지막평행선과만나는점을각각 T, U라할때, TU를 1:4로내분하는점을 A라고하면됨. 82. 정삼각형 ABC 내부의점 M에서각변에내린수선의발을D, E, F 라하자. DEF가직각삼각형이되는점M의자취를구하여라. (Towns 1987봄 SA5) 풀이각변을현으로하여정삼각형내부에원주각이 150 ± 인원호를셋그리면됨. 83. 4ABC 에서 \A, \B 의이등분선이변 BC, AC 와만나는점을각각 D, E 라하자. 점 C 에서직선 AD, BE 에내린수선의발을각각 F, G 라할때, AB k FG 임을보여라. (1995 서울시 ) 증명 CFIG cyclic. \IGF =90 ± \FGC =90 ± \FIC =90 ± (\IAC + \ICA)=\IBA. 84. 직사각형 ABCD 에서 E 는 CD 의중점이고 P 는선분 BC 위를움직이는점이며 F 는 AE 와 BD 의교점이다. PA 2 + PF 2 + PE 2 이최소일때 BP : PC 를구하여라. (1998 교육청경시 ) 풀이 B를원점으로두고 C(6c; 0), A(0; 6a) 로좌표를두자. 그럼 E(6c; 3a) 이고, F 는 4ACD의무게중심이므로 F (4c; 4a) 이다. P (x; 0) 이라하면 (0 x 6), 준식 y = PA 2 + PF 2 + PE 2 =(x 2 + (6a) 2 )+((x 4c) 2 +(4a) 2 )+((6c x) 2 +(3a) 2 )=3x 2 20cx +61a 2 +52c 2. y가최소일때는 x = 10 10 c 3 일때, 즉 BP : PC = x :6c x = 3 c : 8 c =5:4 답 3 85. 삼각형 ABC에서 A 0, B 0, C 0 은각각변BC, CA, AB 위의점으로, AA 0, BB 0, CC 0 은한점O에서만 AO 난다. A 0 O + BO B 0 O + CO C 0 O =92일때, AO A 0 O BO B 0 O CO C 0 를구하여라. (AIME 1992-14) O 풀이 x + y + z =92, 1 x + 1 y + 1 =1 z 일때xyz를구하라는상황인듯.
3.2 기하고급문제 185 86. P 는어떤예각삼각형내부의임의의점이다. P 에서이삼각형의둘레위의점에이르는거리중에서가장긴것을 D, 가장짧은것을 d 라하자. (a) D 2d 임을증명하여라. (b) 등호가성립할조건을구하여라. ( 헝가리 1943-2) 풀이 (a) P 에서각꼭지점을잇고또각변에수선을내리자. 그럼주어진예각삼각형이 6개의직각삼각형영역으로분할된다. 이영역들의꼭지점쪽의각 ( 꼭지각이라하자 )6개를합하면내각의합이므로 180 ± 가되고, 따라서 30 ± 이하의꼭지각이존재한다. 그런꼭지각을갖는영역을직각삼각형 PAH라하자 (A가꼭지점, H가수선의발이다 ). D는가장긴거리이고d는가장짧은거리이므로 d PH = PAsin \PAH PAsin 30 ± = 1 2 PA 1 2 D (1) 로 D 2d 가성립함을알수있다. (b) 등호가성립한다면 (1) 식에서모두등호가성립해야한다. 즉,30 ± 보다작은꼭지각이존재하면안되고, 그럼합이 180 ± 이기위해서는모든꼭지각이정확히 30 ± 가되는수밖에없다. 따라서, 예각삼각형의모든내각이60 ± 이고, 즉정삼각형이된다. 정삼각형의경우중심을 P 로하면실제로D =2d 가성립한다. 별해 (a) 예각삼각형을 ABC 라하고, AP, BP, CP 가대변과만나는점을각각 E, F, G 라하자. PE AE + PF BF + PG CG = j4pbcj j4abcj + j4pcaj j4abcj + j4pabj j4abcj =1 이므로 PE AE, PF BF, PG CG 중에 1 PE 이하인것이존재한다. 일반성을잃지않고 3 AE 1 3 3PE AE = AP + PE 이므로 2PE AP, 이로부터 이라하자. 그럼 D AP 2PE 2d ( ) 즉, D 2d 임을확인할수있다. (b) 위의증명에서보았듯이 PE AE, PF BF, PG CG 중에 1 보다작은것이하나라도있으면 ( ) 에서등호가 3 성립하지않는다. 따라서, 모두 1 3 이상, 즉모두정확히 1 이어야한다. 또,( ) 에서모두등호가성립해 3 야함을생각하면 ² D = AP = BP = CP 여야하므로 P 가외심이어야하며, ² PE = PF = PG = d 여야하므로 P 는내심이기도하고, ² PE가그변에이르는최단거리여야하므로 E는 P 에서내린수선의발, 즉 AE 등이모두수선이므로 P 는수심이어야하며, ² 각변에서높이의 1 이되는자취직선들을그려보면이세직선이한점무게중심에서만나므로 P 는 3 무게중심일수밖에없다. 이중에원하는대로두어가지를골라잘말하면금방 ABC가정삼각형이어야함이확인된다. 87. 지름 AB 를갖는반원과이반원의원주위의한점 X A 나 B 와는다른 가주어져있다. t A, t B, t X 를각각 A, B, X 에서의이원의접선이라고하자. 직선 AX 가 t B 와만나는점을 Z 라하고, 직선 BX 가 t A 와만나는점을 Y 라하자. 이때, 세직선 YZ, t X, AB 가한점에서만나거나모두평행함을증명하여라. ( 캐나다 1974-5)
186 기하 증명 AB 와 YZ 가 P 에서만난다고하자. t X 가 t A, t B 와만나는점을각각 C, D 라하자. 접선의성질에의해 CA = CX, DB = DX 가되고, 그럼직각삼각형의성질에의해 C, D 는각각 4AXY, 4BXZ 의외심이다. 즉, C, D 는각각 AY, BZ 의중점. P 가 4PAY 와 4PBZ 의닮음의중심이므로 P, C, D 는일직선을이룬다. AB 와 YZ 가한점에서만나지않고평행할때는 ABZY 가직사각형 ( 정사각형 ) 이고 X 가그중심이므로 t X 도이들과평행하다. 88. 원 S 밖의점 P 에서 S 에그은두접선의접점을각각 A, B 라하자. AB 의중점을 M 이라하고, AM 의수직이등분선이 S 와삼각형 ABP 내부에서만나는점을 C 라하자. AC 와 PM 의교점을 G 라하고, PM 이 S 와삼각형 ABP 의바깥에서만나는점을 D 라하자. BD 와 AC 가평행하다면, G 가삼각형 ABP 의무게중심임을보여라. ( 중미 2007-6) 증명 AC = CG 임은당연하고, 약간의각계산으로 BAG와 BNC가합동인이등변삼각형임을알아내어 AG = CN 임을확인하면 (N은 AC가 BP와만나는점 ), G는 AN의 3등분점이므로 N은 BM의수직이등분선위에있을수밖에. 89. 변 AB 의길이 c, 내접원의반지름 r, 그리고변 BC 와 CA 의두연장선및변 AB 에접하는바깥원의반지름 r C 등이주어졌을때, 삼각형 ABC 를작도하여라. ( 헝가리 1900-2) 힌트 r 과 r C 가연관된닮은삼각형을찾을수있다. 90. 정사각형 ABCD 에서 \MDN =45 ± 가되도록변 AB, BC 위에각각점 M, N 을잡았다. MN 의중점을 R 이라하고, 대각선 AC 가선분 MD, ND 와만나는점을각각 P, Q 라할때, RP = RQ 임을증명하여라. ( 인도지역예선 1999-3) 증명 \PDN =45 ± = \PCN 이므로 PNCD는 cyclic. 내대각으로 \MPN = \NCD =90 ±. 고로, B;P; Q는 MN을지름으로하는원위에있고, 그원의중심은R. 91. BC k AD 인사다리꼴 ABCD에서 AC = BC + AD 이고두대각선의교각은 60 ± 라고한다. AB = CD 임을증명하여라. (Towns 1992봄 JO2) 증명두대각선의교점을P라하자. BC k AD 이므로 4PBC»4PDA 이고닮음비는 BC : AD 이다. 즉, PC : PA = BC : AD 인데, 문제에서 PC+PA = BC+AD 라했으므로 PC = BC, PA = AD 가된다. 따라서, 4PBC와 4PDA는밑각이60 ± 인이등변삼각형이므로 ( 밑각이 120 ± 일수는없으므로이경우는배제된다 ) 정삼각형이다. 그럼 4AP B와 4DPC는 SAS합동이되고, 따라서 AB = CD 이다.
3.2 기하고급문제 187 별증두대각선의교점을 P 라하고 AEBD 가평행사변형이되는점 E 를잡자. 그럼 AD = BE 이므로 EC = EB+BC = AD+BC = AC 로 4CAE 는 CA = CE 인이등변삼각형이다. AE k PB 이므로 4CPB 도 CP = CB 인이등변삼각형이고, 이등변삼각형의밑각은 90 ± 보다작아야하므로 \CPB =60 ± 이다 ( 문제의조건에서두대각선의교각이 60 ± 라고했는데 \CPB =120 ± 일경우를배제할수있다 ). 따라서, 4PBC 와 4PDA 는모두정삼각형이고, 그럼 4AP B 와 4DPC 는 SAS 합동이된다. 이로부터 AB = CD 이다. 92. 정사각형 ABCD 의변 AB, BC 위에각각점 E 와 F 를잡아 BE = BF 가되도록하였다. BN 을삼각형 BCE 의한수선이라하자. \DNF =90 ± 임을증명하여라. ( 오스트리아 - 폴란드 1979-1) 93. 원호 AC 와꺾인선 ABC 로둘러싸인그림이있다. 원호와꺾인선은 AC 에대해서로반대쪽에있다. 호 AC 의중점을지나이영역의넓이를이등분하는직선을작도하여라. (Towns 1987 가을 J4) 94. 삼각형 ABC에서직선 AC를각각직선AB와 BC에대해대칭시킨두직선이점 K에서만난다. 직선 BK가외심을지남을증명하여라. (Towns 1988봄 JA2) 95. 4ABC 내의점 M 에서각수선 ( 높이 ) 에수선의발을내렸다. 각꼭지점에서높이를따라앞에서내린수선의발까지잰길이가셋모두같다고한다. 이길이는내접원의지름과도같음을증명하여라. (Towns 1987 가을 S4) 96. 한정사각형이같은크기의정사각형과중심을공유하고 45 ± 회전되어겹쳐져있다. 각변은상대편정사각형에의해 a : b : a 의비로세부분으로분할된다. 임의의볼록사각형에대해, 각변을위의비 a : b : a 로세부분으로분할하고, 각꼭지점에서그점에이웃한두분할점을찾아그두점을연결하는직선을각각그린다. 이네직선으로결정되는사각형이원래의사각형과같은넓이를가짐을증명하여라. (Towns 1990 봄 JA2) 97. 사각형 ABCD 의네변 AB, BC, CD, DA 는각각사각형 A 0 B 0 C 0 D 0 의네변 A 0 B 0, B 0 C 0, C 0 D 0, D 0 A 0 과길이가같다. AB k CD 이고 B 0 C 0 k D 0 A 0 임을알때, 두사각형모두평행사변형임을증명하여라. (Towns 1990 가을 JA4) 98. 한원위에두점K, L이있다. 꼭지점 C와두중선AK, BL의교점이둘다이원위에있는삼각형 ABC를작도하여라. (Towns 1991봄 SO2) 99. 주어진각의한반직선위에한고정점A를잡자. A에접하고다른반직선과두점에서만나는모든원을생각하자. 그두교점을 B, C라할때, 삼각형 ABC의내심은원의크기에상관없이모두한직선위에있음을증명하여라. ( 러시아 1989 4차-y9-3) 증명각의꼭지점을 O라하고A의 ( 각의이등분선에대한 ) 대칭점을 A 0 이라하면 AA 0 은항상4ABC의각 A의이등분선이됨. 간단한각계산으로금방확인됨. 100. 세원 k 1, k 2, k 3 가서로다른세점에서각각서로외접한다. 서로다른두원위의 ( 접점과는다른 ) 점 A, B 에대해, 직선 AB 가그두원의접점을지날때점 A, B 가서로대응한다고하자. A 2 k 1, B 2 k 2, C 2 k 3, D 2 k 1 은 A 와 B, B 와 C, C 와 D 가서로대응하는점이다. AD 가 k 1 의지름임을보여라. ( 몰도바 1996 최종 -y9-7, 헝가리 1950-2 변형 ) 증명일반적으로서로접하는두원에대해 A와 A 0 이 B와 B 0 에대응한다면 AA 0 k BB 0 이고방향은서로반대이다. 또한, AA 0 이지름이면 BB 0 도지름이다. 즉, AA 0 이 k 1 의지름이라면 AA 0 k BB 0 k CC 0 k DD 0 이고 DD 0 은 AA 0 과방향이반대인지름이므로끝.
188 기하 101. 삼각형 ABC 의세변의길이를 BC = a, CA = b, AB = c 라하고, D 와 E 를변 AC 와 AB 의중점이라하자. 두중선 BD 와 CE 가직교할때, 또그때만 b 2 + c 2 =5a 2 임을증명하여라. ( 아일랜드 2001-2) 증명 PQ? RS () PR 2 + QS 2 = PS 2 + QR 2 을이용하면끝. 102. 주어진정삼각형 ABC 에대하여, 점 A 의반대편에 BC 를지름으로하는반원을그리자. 선분 BC 의삼등분점을 P, Q 라하고, 직선 AP 와 AQ 가반원의호와만나는점을각각 K, L 이라하자. 점 K, L 이반원의호를삼등분함을증명하여라. ( 호주 1984-2) 증명반원을원으로확장하면삼각형의옆변과중점에서만나므로... 2 : 1 닮음비를발견하여금방. 103. B 1, B 2, B 3 은각각 4A 1 A 2 A 3 의세변 A 2 A 3, A 3 A 1, A 1 A 2 위에위치하는, 꼭지점이아닌점들이다. 세선분 A i B i 의수직이등분선들은절대로서로일치하지않음을증명하여라. ( 오스트리아 - 폴란드 1980-5) 증명 A i B i 의수직이등분선은중점연결선 M j M k 와만난다. 따라서, 두개의이런수직이등분선이일치한다면그직선은두중점연결선의교점을지나고, 따라서어떤변의중점M j 를지나야한다. 그럼 B i 두개가꼭지점이될수밖에... 104. 두원S 1, S 2 가두점A, B에서만나고 S 1 의중심O가S 2 위에있다. S 1 의현AC가 S 2 와점D에서다시만난다. OD와 BC가서로수직임을보여라. ( 러시아 1990 4차-y9-7) 증명 BP 를 S 1 의지름이라하면 \PCD = \PBA = \ADO 이므로 OD k PC? BC 105. 아래그림에서 l 1 과 l 2 는평행선이고, AB 는이들에수직인직선이며, P, Q, R, S 는 AB 위의점 C 를중심으로하고 AB 보다긴지름을갖는원이 l 1, l 2 와만나는점이다. PR PS 의값은 AB 위의점 C 의위치와상관없이일정함을증명하여라. (IMTS R12-5) 증명피타고라스정리로열심히계산하면끝. 106. 볼록사각형 ABCD 가어떤원과 8 개의점에서만나원주를 8 개의호로나누고있다. 이사각형의내부에있는네호의길이를시계반대방향순으로 p, q, r, s 라할때, p + r = q + s 가성립한다고한다. ABCD 가원에내접하는사각형임을증명하여라. ( 독일 BW 2002 1 차 -3) 증명 ( 김지훈 ) 원과사각형 ABCD가만나는 8개의점을 E, F, G, H, I, J, K, L이라하자. 그러면 p + r = q + s 에의해 \EOF + \JOI = \LOK + \GOH 이다. 그러므로 \EOF부터시계반대방향으로각 8개를 2a, 2b, 2c, 2d, 2e, 2f, 2g, 2h라표현하자. 그러면삼각형 EOF, 삼각형 EOL이이등변삼각형이므로 \FEO, \OEL은각각90 ± a, 90 ± b 가된다. 따라서 \AEF = a + b. 같은방법으로 \AF E = a + h 이므로 \FAE =180 ± 2a b h 가되고반대쪽 \ICJ에서도마찬가지로 \ICJ =180 ± 2e d f. 따라서 \ICJ + \FAE 가 180 ± 임을보인다. \ICJ + \FAE =360 ± b h d f 2a 2e 이므로 QED. (2a +2b +2c + +2h =360 ± 이고 2a +2e =2c +2g) 107. AB = BC, AD = DC 를만족하는사각형 ABCD에내접하는원이이사각형의변 AB, BC, AD와각각점K, M, N에서접한다. 대각선 AC가 MN과점P에서만날때, 네점A, K, P, N이한원위에있음을보여라. ( 러시아 1992 4차-y9-3) 증명 \KAP = \BKM = \BMK = \MNK.( 차례로 : 좌우대칭평행, 이등변삼각형, 접현각 ) 108. 삼각형 ABC의변BC 위에 BB 1 = CC 1, \BAB 1 = \CAC 1 을만족하는점 B 1, C 1 을잡을수있었다. ABC가이등변삼각형임을보여라. ( 러시아 1992 4차-y9-7)
3.2 기하고급문제 189 증명두원 ABB 1 과 ACC 1 의크기가같다. 109. 삼각형 ABC의변AB와 BC 위에각각점 M, N이있다. 두선분AN, CM이점O에서만나고 AO = CO 라한다.(1)AM = CN 이면 ABC는이등변삼각형인가?(2)BM = BN 이면 ABC는이등변삼각형인가? ( 러시아 1993 4차-y9-3) 풀이 (1) 은반례가있음.(2) 는 YES 110. C 1 과 C 2 는두점A와 B에서만나는두원이다. 점 B를중심으로하고점 A를지나는원을 C 0 이라하자. C 0 과 C 1 의공통현이 C 2 에접할조건을말하고, 그것을증명하여라. (IMTS R15-5) 증명 C 1 과 C 2 의크기가같음. 각계산좀만해보면됨. 111. 삼각형 ABC 에서, 변 BC 와 AC 의수직이등분선이직선 AC, BC 와각각점 M, N 에서만난다. O 를삼각형 ABC 의외심이라할때, 다음을증명하여라. (a) 다섯점 A, B, M, N, O는한원k 위에있다. (b) k의반지름은삼각형 MNC의외접원의반지름과같다. ( 몰도바 1996 최종 -y10-7) 증명 (a) \OMC = \OBA 임을간단히확인.(b) 현 MN에대한원주각의크기가같으므로원의크기도같다. 112. 삼각형 ABC 의내심을 I 라하자. 변 AB 와 CA 의중점을각각 N, M 이라하고, 직선 BI 와 CI 가직선 NM 과각각점 K 와 L 에서만난다고할때, AI + BI + CI > BC + KL 임을증명하여라. ( 주니어발칸 1997-3) 증명 BI + CI > BC 이므로 AI > KL 만보이면충분하다. MLC, NBK가이등변삼각형이므로 KL = ML+ NK MN = MC + NB MN = 1 (AB + AC BC) 2 이고이것은 A에서내접원에그은접선의길이. 113. 예각삼각형 ABC 의외접원의중심을 O 라하고반지름을 R 이라하자. 직선 AO, BO, CO 는대변과각각점 D, E, F 에서만난다. 다음을증명하여라. ( 남미 1985-6) 1 AD + 1 BE + 1 CF = 2 R 증명 ( 임준혁 ) R : AD = AO : AD = j ABOCj : j4abcj. 같은식으로 R : BE = j BCOAj : j4abcj. R : CF = j CAOBj : j4abcj. j ABOCj + j BCOAj + j CAOBj =2 j4abcj 이므로 R AD + R BE + R =2 CF 이고여기에 R을나누면등식이성립한다. 114. 각 A와 C가같은사각형ABCD가있다. 각 B의이등분선이삼각형 BDC의외접원과점 C 1 6= D 에서만난다. 또각D의이등분선이삼각형 BDA의외접원과점 A 1 6= B 에서만난다. 사각형 A 1 BC 1 D가평행사변형임을보여라. ( 몰도바 1997 최종-y8-3) 증명 \DA 1 B = \A = \C = \DC 1 B, \A 1 DC 1 = \A 1 DC + \CDC 1 = \A 1 DA + \CBC 1 = \A 1 BA + \ABC 1. 문제에별도움은되지않지만, 참고로, 원주각이같으므로두원 CBD와 ABD의크기는같다. 115. AB = BC 이고 \BCD = \EAB =90 ± 인볼록오각형 ABCDE 가있다. 이오각형의내부에점 X 가존재하여 AX 는 BE 에수직이며, CX 는 BD 에수직이다. BX 는 DE 에수직임을보여라. ( 호주 1992-3) 증명 BMXN 은 cyclic. BM ME = BN BD 임에서 MNDE 도 cyclic. 이후각계산. 116. 주어진선분 AB를점B 방향으로점 D까지연장하자. 이때BD의길이는임의로정해진다. AD을지름으로하는반원을그리고그중심을 H라하자. \ABG가예각이되는반원위의한점 G를잡은뒤, EH ED = EZ 2 을만족하도록 BG에평행한선분 EZ를그리자 (E는 AD 위의점, Z는반원의호위의점이다 ). 또한 ZH와 TB가평행이되도록반원위에점 T 를잡자. \ABG =3\TBG 임을증명하여라. ( 호주 1992-5)
190 기하 증명각조금만쌓아보면금방... 117. 각 ACB 가둔각인삼각형 ABC 가있다. 점 C 에서 AB 에내린수선의발을 D 라하고, AB 의중점을 M 이라하자. 또 AC 의연장선위에 EM = BM 을만족하는점 E 를잡고, 두직선 BC 와 DE 의교점을 F 라하자. BE = BF 라면 \CBE =2\ABC 임을증명하여라. ( 호주 1993-1) 증명 M은 4ABE의외심. \AEB는직각. BECD cyclic. EF의중점을 N이라하면 BNE, BNF, BDC가모두닮음직각삼각형. 118. 8개의점 A 1, A 2, B 1, B 2, C 1, C 2, D 1, D 2 가한원의둘레위에차례로놓여있다. A 1 B 2 = B 1 C 2 = C 1 D 2 = D 1 A 2 이면, 네직선A 1 A 2, B 1 B 2, C 1 C 2, D 1 D 2 로이루어진사각형이원에내접하는사각형이됨을보여라. ( 러시아 1993 최종-y9-6) 증명위의독일BW 2002 1차-3 와비슷하게각계산을좀해보면... 일정한같은길이의두현이있을때두현의사이에놓이는두호의길이의합이일정함을이용함 ( 간단한불변량 ). 119. 볼록육각형 ABCDEF 의대각선 AD, BE, CF 는각각이육각형의넓이를이등분한다. 이세대각선은한점에서만남을증명하여라. ( 폴란드 1966 3 차 -5) 증명 ( 최재연 ) jabcdj = 1 jabcdef j = jbcdej, 2 즉 jabdj = jbdej 이므로 BD k EA. 마찬가지로 AC k DF, CE k FB. 그럼대응되는세변이모두평행하여 4ACE»4DFB. 이런사실들로부터 AD \ CF = P 라하면AP : PD = AC : DF = EA : BD 가되고, 그럼 4PAE»4PDB 가되어 (SAS) BPE는일직선이됨. 별해한점에서만나지않고교점들이서로다를때중앙에한영역이더생기고거기서넓이관계의등식등을열심히써보면... 120. 한직선위에세점 A, B, C 가이순서대로놓여있다. 세정삼각형 ABD, BCE, CAF 를그리는데, 직선 AC 에대해 D 와 E 는같은쪽에있고 F 는반대쪽에있도록한다. 이정삼각형들의무게중심을세꼭지점으로하는삼각형은정삼각형임을증명하여라. 또, 이새로운정삼각형의무게중심은직선 AC 위에놓임을증명하여라. ( 아일랜드 1994-2) 증명세정삼각형ABD, BCE, CAF의무게중심을각각 G, H, I라하고, AC에대한그대칭점들을 G 0, H 0, I 0 이라하자. 그럼 I 0 (G)A(G 0 )I(H 0 )C(H) 는마름모가되고 ( 괄호안은그변이지나는점을표기한것 ), I 0 GG 0 IH 0 H는건너뛴두묶음의세변의길이가각각같은등각육각형이다. 여기서 GHI나 G 0 H 0 I 0 이정삼각형임은자명 ( 등각육각형에서그삼각형을오려낸나머지세삼각형이합동 ). 또한, 이등각육각형의중심 O는당연히AC 위에있고, 정삼각형 GHI, G 0 H 0 I 0 들의중심도 O임은자명하다. 121. E는마름모 ABCD의대각선AC 위의 A, C와는다른임의의점이다. N, M은각각직선AB, BC 위의 A, C와는다른점으로, AE = NE, CE = ME 를만족한다. 직선 AM과 CN이점K에서만날때, 세점 K, E, D가한직선위에있음을보여라. ( 러시아 1993 4차-y9-7) 증명 4NEC 4AEM. ANKE, CMKE cyclic. \AKC =2\NAE = \BAD 이므로 AKCD도 cyclic. KE는 \AKC를이등분하므로 K 반대쪽의호 AC의중점D를지남. 별증 4EAN»4DAC, 이로부터 4CAN»4DAE (SAS). 그럼 \ANK = \AED 로내대각이되므로 KED는일직선. 122. AC > AB 인삼각형 ABC 의각 A 의이등분선이외접원 와점 D 에서다시만난다. D 를원 의중심 O 와이었더니직선 DO 가직선 AC 와점 E 에서만났다. BE 와 AD 가수직일때, AO 와 BD 가평행함을보여라. ( 인도지역예선 2000-5) 증명 BC의중점을 M, AD와 BC, BE의교점을각각 Z, X라하자. EBC, OAD가이등변삼각형이고 BDMX가 cyclic이므로, \ADB = \ECB = \EBC = \XDM = \OAD. 별증 ABE는이등변삼각형인데 \A = 2 \BOC = \BOD 1 이므로이등변삼각형 OBD와닮음. 그리고같은이유로내대각에서 ABOE cyclic. angleobd = \AEB = \AOB.
3.2 기하고급문제 191 123. D, E는각각직각삼각형ABC의두옆변AC, CB 위의점이다. C에서 DE, EA, AB, BD에내린수선의발들이한원위에있음을증명하여라. ( 러시아 1989 4차-y10-3) 증명수선의발을나열된차례로 P, Q, R, S 라하자. \DPS = \DCS = \DBC = \SRC, \EPQ = \ECQ = \EAC = \ERC 이므로 \SPQ + \SRQ =180 ± 124. 마름모 ABCD 안에내접하는원이있다. 이원의한접선이변 BC, CD 와각각점 M, N 에서만난다. 어떤접선을그렸느냐에상관없이삼각형 AMN 의넓이가일정함을보여라. ( 유고슬라비아 1980 고 2-2) 증명내접원이 MN, BC, CD 와접하는점을각각 X, Y, Z 라하고, 중심을 O 라하자. jomxj = jomy j, jonxj = jonzj, joamj = jocmj, joanj = jocn 으로부터 jamnj = jyozc. 125. 내접원의반지름과외접원의반지름이주어졌을때, 그에해당하는직각삼각형을작도하는방법을설명하여라. ( 이탈리아 1987-3) 풀이내심에서빗변을보는각이 135 ± 임을이용.[ 참고 ] R, r과한각의크기가주어지는것이삼각형의작도조건중하나이다. 126. 평행사변형 ABCD 의변 AB 위에점 E 가, 변 CD 위에점 F 가있다. AF 와 ED 의교점을 G, EC 와 FB 의교점을 H 라하자. 또 GH 의연장선이 AD, BC 와만나는점을각각 L, M 이라하자. DL = BM 임을증명하여라. ( 호주 1994-8) 증명 E, F 에서각각 AD 에평행하게그은직선이 GH 와만나는점을각각 J, K 라하자. JE=DL = GE=GD = GA=GF = LA=F K 이므로 DL LA = JE FK. 마찬가지로 BM MC = JE FK = DL LA. 별증대각선의교점 ( 평행사변형의중심 ) 을 O라할때, GOH가일직선임을메넬라우스로증명해도됨. 계산막노동이긴하지만. 127. 한직선과두동심원과의교점을순서대로 A, B, C, D 라하자. AE 와 BF 는각원에서하나씩택한, 서로평행한현이다. C 에서 BF 에내린수선의발을 G 라하고, D 에서 AE 에내린수선의발을 H 라하자. GF = HE 임을증명하여라. ( 호주 1995-3) 증명 AG 0 C 를 BH 0 D 로평행이동. 평행이며길이도같다는것을 로쓰기로할때, AB G 0 H 0 = HG EF 이므로 GF = HE. 128. ABCD 는볼록사각형이고 P 와 Q 는각각변 AD 와 BC 위의점으로 AP PD = BQ QC = AB CD 를만족한다. 직선 PQ 가두직선 AB, CD 와이루는각이같음을증명하여라. ( 남미 1987-6) 증명 ( 이성곤 )
192 기하 AP PD = BQ QC = CD AB = b Ã! a 라고두자. 이제 PQ와직선AB; CD의교점을각각 R; S라고하자. 이제문제는 \BRQ = \QSC를증명하는것과동치이다. AC위에 AX : XC = a : b인점x를잡으면, PX==CD;QX==AB이다. ) 4AP X 4ADC; 4CQX 4CAB 4AP X 4ADC에서 PX = a CD. 4CQX 4CAB a+b 에서 QX = b AB. a+b 그런데조건에서 AB : CD = a : b라고하였으므로 AB = ak, CD = bk라둔다면 PX = QX = abk a+b. ) 4PQX는 PX = QX인이등변삼각형이다. 한편, \PQX = \ARQ( 엇각 ), \QP X = \QSC( 동위각 ) 이므로 \BRQ = \PQX = \QP X = \QSC. 129. 볼록오각형 ABCDE에서 AE = AD, AB = AC 이고 \CAD는 \AEB와 \ABE의합과같다. 선분 CD는삼각형 ABE의중선AM의길이의두배임을증명하여라. (Towns 1984봄 JO2) 130. 볼록사각형 ABCD 의두대각선 AC 와 BD 가서로직교하고, 그교점을 E 라하자. E 를 AB, BC, CD, DA 에대해각각대칭시킨점들이모두한원위에있음을증명하여라. ( 미국 1993-2) 증명 1=2만큼축소하면각변에내린수선의발이한원위에있음을보이는문제가됨. 변환기하그담은대각의합이 180 ± 임을보여서. 131. 원 k 에내접하는예각삼각형 ABC 가있다. 점 A 에서그은접선이직선 BC 와점 P 에서만난다. 선분 AP 의중점을 M 이라하고, 원 k 와직선 BM 의두번째교점을 R 이라하자. 직선 PR 이원 k 와다시만나는점을 S 라할때, AP 와 CS 가서로평행함을증명하여라. ( 주니어발칸 2005-2) 증명 MP 2 = MA 2 = MB MR 이므로 4MPB»4MRP 이고 \MPR = \MBP = \RBC = 180 ± \RSC 132. 원에내접하는사각형에서각각변의중점에서대변에수선을내렸다. 이네수선이한점에서만남을보여라. ( 독일 BW 1977 1 차 -4) 증명중점사각형 ( 평행사변형 ) 의중심에대해 O 를대칭시킨점. anticenter 라고도한다는얘기있음. IMO 쇼트였다는얘기도. 133. 중심이각각 O 1, O 2 인두원이두점 A, B 에서서로만난다. 두원의중심은직선 AB 에대하여서로반대편에위치하고있다. 직선 BO 1 과 BO 2 는각기자신의원과 B 1, B 2 에서다시만난다. B 1 B 2 의중점을 M 이라하자. M 1, M 2 는각각원 O 1, O 2 위의점으로 \AO 1 M 1 = \AO 2 M 2 이라하자. 그리고, B 1 은작은호 AM 1 위에, B 는작은호 AM 2 위에있다고하자. \MM 1 B = \MM 2 B 임을보여라. ( 주니어발칸 2002-2) 증명 4AO 1 O 2 4MO 2 O 1 이므로 \MO 1 A = \MO 2 A. 그럼 \MO 1 M 1 = \MO 2 M 2. 또한 O 1 M 1 = O 1 A = O 2 M, O 1 M = O 2 A = O 2 M 2 이므로 4MO 1 M 1 4M 2 O 2 M. 고로 MM 1 = MM 2. 이제 B 가 M 1 M 2 위에있음만확인하면된다. 그건각계산좀해서 \M 1 BM 2 = \M 1 BB 1 +\O 1 BO 2 + \O 2 BM 2 =180 ± 임을확인하면됨. 134. 평행한두평면 와 가주어져있다. 구 ¾가 와점T에서접한다. T 를지나는서로다른두직선 a, b가 와각각C, D에서만나고, 구 ¾와는각각 A, B에서만난다. 네점A, B, C, D가한원위에있음을증명하여라. ( 몰도바 1998 최종-y11-3) 증명 a, b로결정되는평면위에서만생각하면쉬움. 이평면이 와만나는교선을 XY 라하면 (X가 C랑같은쪽 ), \BDC = \BTX = \BAT 등. 135. 삼각형 ABC에서변 AB, BC 위에각각점 D, E를잡자. 선분 DE의삼등분점을 K와 M이라하자. 직선 BK와 BM이변AC와만나는점을각각 T 와 P 라할때, TP 1 AC 3 임을보여라. ( 러시아 1990 4차-y11-6)
3.2 기하고급문제 193 증명 B 를중심으로 DE 를확대해도삼등분점은유지되므로,WLOGE = C 라하고, D 가 AB 를 x :1 로내분한다고하자. AT KC 순서라고하자. 간단한비례로 AT : TC = jkabj : jkbcj =1+x :2, AP : PC = jmabj : jmbcj =2+2x :1. 따라서, TP AC =1 1+x 3+x 1 3+2x =1 1+ x 3 + 2 3 x 3+x 1+ 2 3 x 2 3 x 3+2x = 1 3 2x 3 ( 1 3+x 1 3+2x ) 1 3. 136. 삼각형 ABC 의변 BC 의중점을 F 라하자. 변 AB 와 AC 의바깥쪽으로, D 와 E 에서의꼭지각이직각인이등변삼각형 ABD 와 ACE 를그렸다. DEF 도직각이등변삼각형임을증명하여라. ( 아일랜드 1996-4, Towns 1991 봄 JA4 변형 ) 증명 D는 BQ의중점, E는 CG의중점이라하자. 4AQC 4ABG. DF = 1 2 QC = 1 BG = FE. 2 AQBI는 cyclic. \QIB = \QAB = \R. 137. 볼록육각형 ABCDEF 에서 AB k CF; CD k BE; EF k AD 이다. 두삼각형 ACE 와 BDF 의넓이가같음을증명하여라. (Towns 1984 가을 SO1) 증명육각형에서각삼각형을제외하고남은삼각형조각들을등적변형으로비교. 138. 한선생님이학급학생들에게다음과같은실습을하도록하였다 : 반지름이각각 1 과 10 인두동심원을그리고, 작은원에서세접선을그려둘씩만나는교점 A, B, C 가모두큰원안에있도록한다. 4ABC 의넓이를 S, 부채꼴과비슷한세영역을각각 S 1, S 2, S 3 이라할때 ( 그림참조 ), S 1 + S 2 + S 3 S 를구해보아라. 모든학생의결과가똑같음을증명하여라. (Towns 1985 가을 J3) 증명 3 ( 작은반원 ) 3 ( 큰반원 )+ 원 =2 ( 준식 ) 139. 마름모가아닌평행사변형 ABCD 가있다. \BAD 의이등분선이직선 BC, CD 와각각점 K, L 에서만난다. 세점 C, K, L 을지나는원의중심은 B, C, D 를지나는원위에있음을보여라. (Towns 1986 봄 S5) 증명원의중심을 O 라하자. 4BKO 4DCO 임을보이면됨 (SAS). 140. 예각삼각형 ABC 에서 AB 와 AC 를지름으로하는두원을각각 C 1, C 2 라하자. C 2 는 AB 와 F 에서다시만나고, C 1 은 AC 와 E 에서다시만난다. 또한, 선분 BE 와원 C 2 의교점을 P, 선분 CF 와원 C 1 의교점을 Q 라하자. AP = AQ 임을증명하여라. ( 중미 2000-5) 증명 C 1 과 C 2 의 A 아닌교점을 D라하면AD? BC. 4ABC의세수선AD, BE, CF는한점 ( 수심 ) H에서만난다. FBDH, ECDH는 cyclic이므로, AF AB = AH AD = AE AC. 또, AC가현PEP 0 을수직이등분하므로 \AP H \AP P 0 = \ADP 로 4AHP»4AP D. 즉 AP 2 = AH AD = AQ 2.
194 기하 141. AD k BC 이고 AB? AD 이며내접원을갖는사다리꼴 ABCD 가있다. 내접원의반지름을 R 이라하고대각선의교점을 M 이라하자. 4DCM 의넓이는얼마인가? (Towns 1986 가을 S1) 풀이 j4abmj = j4cdmj 임을보이고, 그로부터열심히풀면넓이는 R 2 임을알아낼수있다. 좀더일반화하여, AB? AD 의조건이없어도이것은성립함을증명할수있다. 대각선의교점은항상내접원의세로지름위에있다. 142. \A =60 ± 인예각삼각형 ABC가있다. B와 C에서그은두수선의교각을이등분하는두직선중하나가외심을지남을증명하여라. (Towns 1987봄 JA4) 증명두수선의교점을 F. 여러개의 30 ± 들이발견됨. 각 F 의횡이등분선이 AB, AC와만나는점을 D, E라하면ADE는정삼각형. DE 위에서 F 와대칭이되는점을 O라하면O에서 AB, AC에내린수선의발이중점이되어외심임을확인할수있음. 143. 예각삼각형 ABC 에서 AD 와 BE 는두수선이다.[BDE] [DEA] [EAB] [ABD] 일때, 이삼각형은이등변삼각형임을보여라. 단,[XY Z] 는삼각형 XY Z 의넓이를나타낸다. ( 중미 2002-2) 증명 [BDE] [ADE] 이면 [BDX] [AEX] 이므로 [DAB] [EAB]. 반대쪽부등호도마찬가지. 따라서, 등호성립. 그럼 AB k DE. ABDE cyclic 이므로등변사다리꼴. 144. P 와 Q 는각각이등변삼각형 ABC 의두등변 AB 와 AC 위의점으로, AP = CQ 를만족한다. P, Q 는이삼각형의꼭지점이아니다. 삼각형 AP Q 의외접원이삼각형 ABC 의외심을지남을증명하여라. ( 아일랜드 2006-2) 증명 ABC의외심을 O라할때, AP OQ가 cyclic임을보이면됨. 4OPA 4OPQ (O를중심으로회전합동 ) 임에서금방. 145. P 는원 C 위의고정점이고 Q 는직선 l 위의고정점이다. R 은 P, Q, R 이일직선을이루지않는한에서원 C 위를움직이는점이다. P, Q, R 을지나는원이직선 l 과다시만나는점을 V 라고할때, 직선 VR 은항상한고정점을지남을보여라. ( 아일랜드 2007-3) 증명 PQ, RV 와원 C 의교점을각각 S, X 라하자. PQVR, PSXR 이모두 cyclic 이므로 \PSX = 180 ± \PRX = \PQV. 즉 X 가그고정점. 146. 어떤반원의지름 EF 가삼각형 ABC 의변 BC 위에놓여있고, 이반원이변 AB, AC 와각각점 P, Q 에서접한다. 직선 EP 와 FQ 의교점 K 가 A 에서 BC 로내린수선위에있음을증명하여라. ( 주니어발칸 2000-3, 알바니아출제 ) 증명 E와 P 가같은쪽에있을때를K, 반대쪽에있을때를 L이라하자. \PAQ =180 ± \POQ = 180 ± 2\PEQ =2(90 ± \PEQ)=2\PKQ 이므로 K는 A를중심으로하고 P, Q를지나는원위에있다. L도마찬가지이고 KL은지름이므로 A 2 KL. 그럼 L은 KEF의수심이므로끝. 147. 평면에평행사변형 ABCD 가있다. A 와 B 를지나면서반지름 R 인원을그리고, B 와 C 를지나면서반지름 R 인원을또그리자. 이두원의또하나의교점을 E 라하고, E 는평행사변형의꼭지점이아니라고하자. ADE 의외접원도반지름이 R 임을증명하여라. ( 북유럽 1987-2) 증명 BCE 를평행이동하여 ADF 가되도록하면 AEDF cyclic by 원주각 or 원주대각 148. 삼각형 ABC 에서각 C 의이등분선이변 AB 와만나는점을 D 라하자. 삼각형 ABC 의넓이를 F 라할때, 다음부등식을증명하여라. ( 오스트리아 - 폴란드 2004-2) 2F µ 1 AD 1 AB BD 증명 A, B를지나CD에평행한직선이각각직선 BC, AC와만나는점을 A 0, B 0 이라하자. 그럼 ABB 0 A 0 2F 은등변사다리꼴. 또, AD, 2F BD 는각각A0, B 0 에서직선 AB에이르는거리. 그두거리의차는직각삼각형의빗변 A 0 B 0 보다작다.
3.2 기하고급문제 195 149. 원 위에서로다른두점 A, B 가있으며, AB 는지름이아니다. P 가 위를자유롭게움직이는점일때 ( 단, P 6= A; B), 삼각형 ABP 의수심의자취를구하여라. ( 이탈리아 2006-3) 풀이원 와 P 를통째로직선 AB에대해선대칭시켜서원 0 과점P 0 이라하면, PP 0 \ 0 이 ABP 의수심이됨. 즉, 자취는 0. 150. E 는정사각형 ABCD 의변 AB 의중점이고, 점 F, G 는 EF 와 AG 가평행이되게하는선분 BC, CD 위의점이다. FG 는정사각형의내접원에접함을증명하여라. ( 헝가리 1960-3) 증명 BC, CD의중점을각각 H, K라하면HE + FK = EF 만증명하면충분함. AB =2,HF = a 라두고피타고라스와닮음을이용해걍계산하면됨. 좀더좋은방법이있을까나?
196 기하
제 4 장 조합 4.1 조합중급문제 1. 체스판 (8 8) 에서나이트는가로두칸과세로한칸, 혹은가로한칸과세로두칸이떨어진위치로움직일수있다. 체스판의한귀퉁이에서나이트가출발하여모든칸을정확히한번씩들르고대각선방향의귀퉁이칸에도착하는것이가능한가? ( 오클랜드 1997-7) 증명흑칸! 백칸! 흑칸으로이동하게되므로, 시작과끝이같은색칸이되려면이동은짝수번, 지나온칸은홀수개. 모순. 2. 1 에서 n 1 까지의자연수들중에서중복을허용하여 n 개의수를골랐는데그합이 2n 보다작았다. 이수들중에서몇개를잘고르면합이 n 이됨을증명하여라. ( 아일랜드 1988-6) 증명 a 1, a 1 + a 2, a 1 + a 2 + a 3, :::; a 1 + a 2 + + a n 들중n으로나눈나머지가같은것이있으면그둘의차이가역시 a i 들의부분합이면서 n의배수가된다. 나머지가모두다르다면이들은 n개의수이므로 n으로나눈나머지가각각하나씩대응되고, 그럼이중에 n으로나눈나머지가 0인것이있다. 3. 셈뛰기는 M 가지종목으로이루어진시합이다. A, B, C 세명만이참가하여셈뛰기시합이열렸다. 각종목에서 1 등에게는 p 1 점이수여되고,2 등에게는 p 2 점,3 등에게는 p 3 점이수여된다. p 1 >p 2 >p 3 > 0 이고 p 1, p 2, p 3 은정수이다. 최종점수는 A 가 22 점, B 가 9 점, 그리고 C 도 9 점이었다. B 는 100 미터달리기에서우승했다. M 은얼마이고또높이뛰기에서 2 등을한것은누구인가? ( 캐나다 1986-2) 풀이 s = p 1 + p 2 + p 3 이라하자. 그럼모든점수의합계 Ms =22+9+9=40이된다. s 6, M 2 이므로 (M; s) =(2; 20), (4; 10), (5; 8) 만가능하다. (i) M =2이면 ;22=A<2p 1 < 2B =18이므로모순. (ii) M =4이면 ; B p 1 +3, A<4p 1 이므로 p 1 =6이어야한다. 그럼 s =10,p 1 >p 2 >p 3 에서 p 2 =3,p 3 =1이다. 이때A 3p 1 + p 2 =21이므로모순. (iii) M =5이면 ; B p 1 +4, A<5p 1 이므로 p 1 =5. 그럼 p 2 =2,p 3 =1. 이때, A = 5+5+5+5+2, B =5+1+1+1+1이므로다음표와같다. (1) (2) (3) (4) (5) 계 A 2 5 5 5 5 22 B 5 1 1 1 1 9 C 1 2 2 2 2 9 따라서, 100 미터달리기이외의종목에서 2 등을한것은모두 C 답 별해 M 2 이고 B>p 1 이므로 p 1 8 이다. (i) p 1 =8이면 B =8+1이므로 M =2인데이는 22 = A<2p 1 에모순.
198 조합 (ii) p 1 =7이면 B 7=2이므로 M 3 이고, 그럼 A<3p 1 에역시모순. (iii) p 1 =6이면 B에서 M 4, A<Mp 1 에서 M =4. 그럼 B = 6+1+1+1 이고 p 3 =1. A =6+6+6+4 또는 6+6+5+5인데어느경우도 C가 9보다커져서모순. (iv) p 1 4 이면 B에서 M 6, A<Mp 1 에서 M =6. 그러나, M - 22+9+9 이므로 p 1 + p 2 + p 3 이정수가될수없어모순. (v) 따라서, p 1 =5이다. B에서 M 5, A<Mp 1 에서 M =5. 그럼 B =5+1+1+1+1,p 3 =1 이고, B 때문에 C는 3등을기껏해야 1번밖에안했으므로 C 1+2+2+2+2=9. 등호가성립할때이므로 p 2 =2이고, A =2+5+5+5+5 가된다. 높이뛰기 2등은 C이다. 4. n(a) 는 A 의원소의개수, kak 는 A 의부분집합의개수이다. kak +2kBk = ka [ Bk 가성립할때, n(a) = 2000 이면 n(a \ B) 는얼마인가? ( 셈본중등초급도전문제 1.1.1) 풀이 n(a), n(b), n(a[b), n(a\b) 를각각a, b, c, d라하자. 그럼문제의조건에서 2 a +2 b+1 =2 c, a = 2000 이다.2의거듭제곱에관한위의식이성립하려면 a = b +1이고 c가이들보다1만큼클때뿐이다 (2진법전개를생각해보면쉽다 ). 따라서, a = 2000, b = 1999, c =2001이고, d = a + b c = 1998 답 5. 어떤상자에 24 개의공이있다. 그중 2 개는빨간공이고나머지 22 개는파란공이다. 준경이가 1 에서 24 까지의자연수중하나를골라 `n 개 ' 라고선언하면, 정욱이는상자안에서보지않고 n 개의공을하나씩차례로꺼낸다. 그래서 n 번째꺼낸공이처음으로나온빨간공이면준경이가이기는것으로한다. 이길확률이최대가되도록하려면준경이는얼마를불러야하는가? ( 플란더즈예선 1990/1991) 풀이일종의 telescoping 답 1 6. 5 행 N 열의 5N 개의점이있다. 첫행의왼쪽부터 1; 2;:::;N, 그리고다음행의왼쪽부터 N +1;:::;2N, 이런식으로왼쪽에서오른쪽으로번호를붙이고다음행으로넘어가다시번호를붙이는방식을행우선번호라고하자. 비슷하게, 열우선번호라는것은첫열의맨위부터 1; 2; 3; 4; 5, 다음열의맨위부터 6; 7; 8; 9; 10, 이런식으로위에서아래로번호를붙인후다음열로넘어가는방식이다. i 번째행에서점 P i 를각각골라, 다섯점 P 1 ;P 2 ;:::;P 5 를뽑았다. 점 P i 의행우선번호를 x i, 열우선번호를 y i 라하면, x 1 = y 2, x 2 = y 1, x 3 = y 4, x 4 = y 5, x 5 = y 3 이된다는것을발견하였다. 이것이가능한 N 의최소값을구하여라. (AIME 2001 1 차 -11) 풀이 %EEE Suppose the row 1 point is in col a, the row 2 point in col b and so on. Then a = 5b-3, b+n = 5a-4, c+2n = 5d-1, d+3n = 5e, e+4n = 5c-2. First two equs give a = 5b-3, N = 24b-19. Last three give 124e = 89N+7. Hence 124e = 2136b-1684. Now 2136b-1684 = 28b+52 mod 124, and by trial smallest possible solution (for b and hence for N) is b = 7, giving N = 149, e = 107. Continuing, we get a = 32, d = 88, c = 141. So it is indeed a solution. 답 149 7. 집합 A 는 n(a) 1000, A ½f0; 1; 2; 3;:::;1997g 을만족한다. A 의원소중에 2 k (k 0 인정수 ) 꼴이있거나아니면 a + b 가 2 k (k 0 인정수 ) 꼴이되는 A 의서로다른두원소 a, b 가존재함을보여라. ( 아일랜드 1997-8) 풀이 0에서 1997까지의수를한집합안에 2 k 꼴인수만있거나, 집합에포함된두수를더하면 2 k 꼴이되도록적절히 999개묶음으로나누는방법을찾는다. 1997과더하여 2 k 꼴을만들수있는수는2048 1997 = 51이다.1996; 1995;::: 도짝을찾으면 (51; 1997); (52; 1996);:::(1023; 1025) 의 973개의묶음을만들수있다. 같은방법으로 50과짝이될수있는숫자는14이므로 (14; 50); (15; 49);:::(31; 33) 의 18개의묶음을만들수있다. 13의짝은3이므로 (3; 13);:::(7; 9) 의 5개묶음을만들수있다. 종합하면 0; 1;::: ;1997의집합을 (51; 1997); (52; 1996);::: ;(1023; 1025): 973개묶음 (1024): 1개. (14; 50); (15; 49);::: ;(31; 33): 18개.
4.1 조합중급문제 199 (32): 1 개. (3; 13);::: ;(7; 9): 5 개. (0; 1; 2; 4; 8): 1 개. 총 973+1+18+1+5+1=999 개의묶음으로나누었다. 즉어떻게 A 를고르더라도원소가하나인 2 k 꼴의묶음을선택하게되거나, 한묶음안에서두개의숫자를고르게되어그합이 2 k 꼴이된다. 마지막묶음에서둘이상의숫자가포함된경우 (0; 1; 2; 4; 8) 에서는 0 만 2 k 꼴이아니므로이경우에도 A 의원소중 2 k 꼴의원소가포함되게된다. 8. 1 부터 64 까지의수를가로, 세로각각 8 칸인바둑판에채워넣되, 아래형태의그림에든수의합은항상 5 로나누어지도록할수있는가?( 아래와방향만다른형태의그림도모두고려할것.) ( 통신강좌 1990-E3) 풀이 표 4.1: A F G (H) a b c f g B d C D E e O O O O O O O O 문제에서요구하는대로 64개의수를표에채워넣을수있다고한다면, 위그림에서 A + a + b + c 0 B + a + b + c (mod 5) 이므로, A B (mod 5) 이다. B + c + d + e 0 C + c + d + e (mod 5) 이고, 비슷한방법으로 B C D E (mod 5) 이다. F,G는 C,D를이용하여A와 5에대한합동임을알수있다. H A (mod 5) 도어렵지않게보일수있으나증명에서꼭필요하지않으므로생략한다. 결국 A에서 G7칸과,O로표시한8칸등총15개의칸이 5에대해합동이어야하는데 1에서 64까지의수중5로나눈나머지가같은수는최대 13개있으므로문제에서요구하는대로표에숫자를채우는것이불가능함을알수있다. 답 : 불가능하다. 9. 세자리의정수중 2 또는 3 의배수이고 5 의배수도 7 의배수도아닌것들은몇개가있는가? ( 셈본중등초급도전문제 1.1.3) 풀이집합적으로나타내어보자. 세자리정수전체의집합을 U라하고, 그중2의배수의집합을 A, 3의배수의집합을 B, 5의배수의집합을 C, 7의배수의집합을 D라하자. 그럼우리가원하는것은 (A [ B) (C [ D) 의원소의개수이다. 이것은다음과같이구할수있다. 단, E = C [ D 이고, ja \ Bj = jabj 와같이표현하도록하겠다. X = j(a [ B) Ej = ja [ Bj j(a [ B)Ej =(jaj + jbj jabj) (jaej + jbej jabej) = jaj + jbj jabj (jacj + jadj jacdj) (jbcj + jbdj jbcdj) +(jabcj + jabdj jabcdj) = jaj + jbj jabj jacj jadj jbcj jbdj + jabcj + jabdj + jacdj + jbcdj jabcdj
200 조합 세자리의정수중 n의배수인것은 f(n) =[ 999 n ] [ 99 ] n 개가되므로, X = f(2) + f(3) f(6) f(10) f(14) f(15) f(21) + f(30) + f(42) + f(70) + f(105) f(210) = 450 + 300 150 90 64 60 43+30+21+13+9 4 따라서, 답 412 주 jf Ej = jf [ Ej jej 로계산해도된다. 그럼 X = ja [ B [ C [ Dj jc [ Dj 가되고, 만일포함-배제의원리를알고있다면위의결과식이더쉽게이해될수있다. 10. 태우에게는서로다른 5 개의수가적힌카드가있다. 태우가이카드들을적당한순서로한장씩지용이에게보여줄텐데, 지용이는카드가보여지면그카드를가질것인가버릴것인가바로결정해야한다. 지용이가어떤카드를가지겠다고결정하면게임은끝나고, 지용이가택한카드의수가태우가가진카드중에서가장큰수이면지용이가이긴다. 지용이는처음두장의카드는그냥버리고, 세번째로보여지는카드부터지금까지나왔던카드보다큰수가나오면그것을갖는것으로전략을세웠다. 지용이가이길확률은얼마인가? ( 플란더즈 1992-2) 풀이최대수가 k 번째나와서지용이가가질확률을 a k 라하면, p = a 1 + a 2 + a 3 + a 4 + a 5 = 0+0+ 1 5 + 1 5 2 3 + 1 5 1 2 = 13 30 11. 여러사람이모인자리에서서로아는사람끼리만한번씩악수를하였다고하자. 각사람마다악수한횟수가모두같고전체악수의횟수가 2005 번일때, 모인사람의수가될수있는가장작은수를구하여라. ( 한국 2005 1 차 -J15) 풀이모인사람의수를 n이라하고, 각사람마다악수한 ( 일정한 ) 횟수를 k라하자. 각각의악수는두사람사이에이루어지므로각사람마다악수한횟수를모두합하면각각의악수가 2번씩세어져전체악수의횟수의 2배가된다. 즉, nk =4010이고,4010=2 5 401 로소인수분해되므로, n>k의조건을생각하면 n은최소401이되어야한다. 그리고 n =401,k =10의경우는다음과같이실제로구성될수있다 : 401명을원형으로앉혀놓고, 각자좌우로거리 5 이하인사람들끼리만아는것으로한다. 답 401 12. 10 명의사람이있는데, 각사람에게그사람을제외한나머지사람들의나이의합을쓰라고하였다. 그 10 개의합으로집합을만들었더니 9 개의원소를갖는집합 f82; 83; 84; 85; 87; 89; 90; 91; 92g 가되었다. 이 10 명각각의나이를모두구하여라. ( 인도지역예선 1993-7) 풀이 10명의나이를 a 1, a 2, :::; a 10 이라하고, 이들의나이의합을 S = a 1 + a 2 + + a 10 이라하자. 그럼 fs a 1 ;S a 2 ;:::;S a 10 g = f82; 83; 84; 85; 87; 89; 90; 91; 92g 로 10 개의수가서로다른 9 개의수로나타나야하므로우변의수들중에서어느하나는두번나타난다. 그것을 m 이라고하자. 그럼두집합의수의합이같아야하므로 (S a 1 )+(S a 2 )+ +(S a 10 )=82+83+84+85+87+89+90+91+92+m 10S (a 1 + a 2 + + a 10 )=9 87 + m 9S =9 87 + m 따라서, m 은 9 의배수이고, 주어진수들중에서 9 의배수인것은 90 뿐이므로 m =90 이다. 그럼 S = 97. 이로부터열명의나이는각각 5, 6, 7, 7, 8, 10, 12, 13, 14, 15 살임을알수있다. 13. 어떤풀리그토너먼트에 5 개의팀이참가했는데, 각팀은다른팀과꼭한번씩대결하고, 각대결에서는어느한팀의승리가반드시결정된다. 각팀이다른팀과의대결에서승리할확률은모두 1 2 로같다고하자. 각팀이승리한횟수가모두다를확률을구하여라. (AIME 1999-13 변형 )
4.1 조합중급문제 201 풀이 %EEE ( 원래문제풀이 ) Team totals must be 0, 1, 2,..., 39. So we must be able to order the teams as T_1, T_2,..., T_40, so that T_i loses to T_j for i < j. In other words, this order uniquely determines the result of every game. There are 40! such orders and 780 games, so 2780 possible outcomes for the games. Hence prob = 40!/2^780. 답 15=128? 14. 어떤반의학생들이급우들에게각자적어도한통씩의연하장을보냈다. 그럼 x 1! x 2!!x n! x 1 과같이꼬리를물며연하장을보낸몇명의학생이있음을보여라. ( 통신강좌 1991-2-29) 증명아무나한학생을골라 x 1 이라하자. x 1 이연하장을보낸친구중아무나한명을골라 x 2 라하자. 이렇게 x 1! x 2! x 3! 를계속잡을수있다. 그러다가처음으로이미위의목록에등장했던학생이다시등장하는경우가반드시생긴다 ( 비둘기집 ). 그학생이x i 에서처음으로등장하고 x j 에서다시등장했다면, x i! x i+1!!x j 1! x i 가우리가찾는연하장꼬리물기의한예가된다. 별증 ( 조철현 ) 대우를증명하자. D가기본회로를가지지않으면 D에는길이가가장긴기본통로가존재하므로그통로가지나는정점의집합을 X라하자. 이때D가방향그래프이므로 X에끝정점이존재하는데, 이정점에서 X 중의한정점으로나아간다면기본회로가생겨모순, X 이외의정점으로나아간다면가장긴통로라는조건에모순이되므로가장긴기본통로의끝정점의나가는차수는 1이상일수없다. 따라서, D의각정점에서나가는차수가모두 1이상일수없다. 즉, 방향그래프 D의각정점에서나가는차수가모두 1이상이면 D는기본회로를갖는다. 15. 1; 2; 3;:::;9 의 9 개의숫자로두수를만들어서곱했을때, 최소가되는두수를구하여라. 단, 아홉개의숫자를모두정확히한번씩사용해야한다. ( 셈본중등초급도전문제 3.3.2) 풀이 (1) 한자리수가있을경우 : 1 23456789 = 23456789 ( ) 2 13456789 = 26913578 이고나머지경우들은 (1 3) 10000000 = 30000000 보다크므로이들보다크다. 따라서이경우 ( ) 이최소이다. (2) 두수모두두자리이상의수일경우 : 두수 A, B 의맨앞두자리만생각해보면 A B (13 23) 100000 > 29000000 으로모두 ( ) 보다훨씬크다. 따라서, 최소의곱을만드는두수는 답 1, 23456789 16. 첫네항이1, 9, 8, 1로주어진수열이있다. 이후모든항은그바로앞의네항의합의일의자리숫자가된다. 이수열에 1, 2, 3, 4가연속하여나오는경우가있는가? ( 유고슬라비아 1981 고1-3a) 풀이홀홀짝홀홀꼴로주기 5 로반복됨. 불가능. 17. 12 개의꼭지점과 16 개의선분이아래그림과같이주어져있다. ² ² ² ² ² ² ² ² ² ² ² ² 꼭지점을지나지않는임의의곡선을그린다. 곡선이선분의한쪽에서다른쪽으로지날때 ` 교차한다 ' 고한다. 예를들어원은한개의선분을가지고있고이때접선은이선분과 ` 교차 ' 하지않는다. 모든선분을딱한번씩교차하는곡선을그릴수없음을증명하여라. ( 소련 1961-1)
202 조합 증명각각의영역마다내부에점을하나씩찍고, 각각의선분마다그선분을가로지르는곡선을그리는데양끝점이인접한두영역에그려진점이되도록한다.( 이렇게그려진그래프를원래그래프의쌍대 (dual) 그래프라고한다.) 그럼차수가홀수인점이 4개있으므로한붓그리기가안된다. 18. 어떤선거운동에서여러정당들에의해 p 가지서로다른공약들이선언되었다 (p >0). 여러정당이같은공약을선언할수있고, 어떤두정당도최소한하나의공통공약을갖는다. 그러나어떤두정당도그공약들의집합이완전히같지는않다. 정당의수는 2 p 1 을넘지않음을보여라. ( 캐나다 1972-8) 증명전체공약의집합을 T 라하자. T 의두부분집합A와 T A를하나로묶는식으로하면 T 의모든부분집합은둘씩묶여총 2 p =2=2 p 1 묶음을이룬다. 만일, 정당의수가 2 p 1 을넘으면, 비둘기집의원리에의해같은묶음에속하는두정당이있고, 그두정당은서로소이므로모순. 따라서, 정당의수는 2 p 1 을넘을수없다. 19. 대한중학교의학생수는 500 명이고동아리의수는 20 개이다. 이제학생들에게 1 부터 500 까지의고유번호를부여하고 20 개의동아리를 C 1 ;C 2 ;:::;C 20 으로나타낼때, 동아리 C i 에속한학생수는 (i +10) 이라고한다. 고유번호가 j 인학생이속한동아리의수를 d j 라고할때, 다음값은얼마인가? ( 단, 한학생이여러개의동아리에동시에가입할수도있고, 또는아무동아리에도가입하지않을수도있다.) d 1 + d 2 + + d 500 ( 한국 2003 1 차 -J9) 풀이구하는값이각학생이활동하고있는동아리의수를모두합친것인데이값은각동아리에등록된회원수를모두더한것과같다. 그러므로 d 1 + d 2 + + d 500 = C 1 + C 2 + + C 20 =11+12+ +30=410: 답 410 20. A 에서 B 까지최단경로로갈때가는방법은모두몇가지인가? (2000 대전. 충남영재수학교실 2 차평가 ) A B 답 42( 가지 ) 21. 지름이 1 인 100 개의공을 10 10 1 크기의상자에넣을수있음은명백하다. 같은상자에 105 개의공을넣으려면어떻게할까?106 개의공은어떻게넣을수있을까? ( 플란더즈 2005-2) 풀이 ( 과천고 2 학년홍성준 )(1)105=9 5+10 6 에서힌트를찾는다. 그림과같이 3 개의공이서로접하는방식으로 10, 9, 10, 9, 10, 9, 10, 9, 10, 9, 10 개의 11 줄로공을넣으면가로로는 10 이므로문제없고, 세로로는 5 p 3+1; 9:660 < 10
4.1 조합중급문제 203 이므로넣을수있다. (2) 또, 106 = 9 4+10 7 이므로 10개씩넣는줄을늘려본다. 즉, 10, 10, 9, 10, 9, 10, 9, 10, 9, 10, 10개로넣으면세로가 4 p 3+3; 9:928 < 10 이므로역시넣을수있다. 22. 7 7 크기의판의두칸을검은색으로칠하고나머지칸은흰색으로남겨둔다. 이렇게해서만들수있는판의모양은모두몇가지인가? 단, 판을돌려서같은모양이되는경우는한가지로본다. (AIME 1996-7) 풀이 %EEE There are 49 48/2 = 1176 ways of choosing 2 cells. In 24 cases the two cells are diametrically opposite. In the other 1152 they are not. The 1152 form groups of 4 related by rotation, so 288 distinct. The 24 form 12 groups of 2. So total 300. 답 300 23. 임의의자연수 n 에대해, n 의자릿수의합과 n +2 의자릿수의합의차를 t(n) 으로정의하자. 예를들어, t(199) = j19 3j =16 이다. t(n) 이가질수있는값중에 2000 보다작은것은모두몇개인가? (AIME 1999-5) 풀이 %EEE If there are k carries in going from n to n+2, then the digit sum of n+2 is 9k-2 smaller than that of n. So possible values of t(n) are 2 and 7+9k, for k = 0, 1, 2,.... Largest < 2000 is k = 221: 7 + 221 9 = 1996. Hence 223 values < 2000. 답 223 24. 다음과같이다섯개씩의 와 가있다. 이들을서로이웃한둘의자리를바꾸는움직임만으로, 들을모두오른쪽으로모으고, 들은모두왼쪽으로모으고싶다. 몇번의움직임이면되겠는가? 일반적으로두모양이각각 n 개씩있다고하면몇번의움직임이면되겠는가? 당신이구한답이최소임을증명하여라. ( 셈본중등초급도전문제 5.1.1) 풀이편의상 = X, = O 이라고하자. XOXOXOXOXO를 OOOOOXXXXX로만들면된다. 다음과같이옮기면된다. OXOXOXOXOX 5번 OOXOXOXOXX 4번 OOOXOXOXXX 3번 OOOOXOXXXX 2번 OOOOOXXXXX 1번그럼총 1+2+3+4+5=15번움직였다.O왼쪽에 X가모두k개있다면 O가이X들모두의자리를넘어가려면최소 k번의움직임이필요하다. 처음각각의 O에대해이런k들을계산하면 1, 2, 3, 4, 5이므로, 총 1+2+3+4+5=15번보다더적게움직여서는O를모두왼쪽으로모으는것이불가능하다. 25. 45 각형의모든꼭지점에 0; 1;:::;9 중하나씩의번호를매기는데, i 6= j 인모든쌍 fi; jg 에대해서, 양끝점이 i, j 로번호가붙은변이있도록할수있는가? ( 소련 1963-11) 풀이 i =0일때모든j =1;:::;9에대해양끝점이 i, j인변이있으려면 0으로이름붙은점은적어도 5개있어야한다. 다른번호를 i로생각해도모두마찬가지이므로, 각번호마다 5개씩, 적어도총 50개의점이필요한데꼭지점은 45개뿐이므로불가능하다. 주셈본에도비슷한문제가... 한붓그리기. 꼭지점이 50개라면딱된다 ( 가능하다 ). 10개의번호를꼭지점으로하여 K 10 을그리고, 두점씩짝을지어5개의변을추가로그린다. 그럼모든점의차수가짝수이므로출발점으로돌아오는한붓그리기가가능하고, 이한붓그리기를펼쳐그리면원하는 50각형이된다.
204 조합 26. 1972년부터몇년간달력을모으면모든해의요일을정확히표시할수있는달력을모두수집한것이될까? ( 호주수학경시 ) 풀이 1972년은윤년이고 4년마다윤년이반복되며다시 2000년은윤년이아니다. 또한윤년이아닌해는 365 = 7 52 + 1이므로대부분해가지나면요일이하루씩미뤄진다. 윤년인경우에는이틀씩미뤄지는데,1월1일부터 2월 28일까지는윤년에서다음해로넘어갈때요일이이틀미뤄지고,3월 1일부터 12월 31일까지는윤년인해에요일이이틀미뤄진다. 모든해의달력을정확히표시하려면모든윤달 (2월) 의요일을표시할수있어야한다. 윤년에서다음윤년이될때요일변화를살핀다. 1972년의 2월 1일이월요일이라고하면 1973년수요일,1974년목요일, 1975년금요일,1976년토요일로요일이이틀당겨지게되는데 7과 2는서로소이므로일곱번째윤년이돌아오면 2월 1일이월요일부터일요일까지모든요일이순환하게된다. 1972년부터 4 6+1=25년간, 즉 1996년까지달력을모으면윤년을모두모을수있다. 이때4년마다윤년인규칙에예외가없다. 25년간달력을모으면평년달력도요일별로모을수있음을보이자. 1973년 1월 1일이월요일이라면 4년뒤에는토요일인식으로월, 토, 목, 화, 일, 금 (1993년) 요일이있고,1월 1일이화요일인 1985년다음해인 1986년의 1월 1일은수요일이므로모든종류의평년달력을모을수있다. 답 :25년. 27. 음이아닌정수들의수열 fa k g 가모든 k 1 에대해 a k a 2k + a 2k+1 을만족한다. ( 가 ) 모든자연수 n에대해, 수열 fa k g에는 n개의연속한 0이항상있음을보여라. ( 나 ) 문제의조건을만족하면서 0이아닌항을무한히많이갖는수열의예를하나만찾아제시하여라. ( 루마니아 2005 지역예선 y9-4) 풀이 ( 가 ) a k 가 0이아니면a 2k 와 a 2k+1 중에적어도하나는 a k 보다작다. 즉,0이아닌항에대해서그보다작은항을항상찾을수있으므로, 음아닌정수들에서는더작은항을계속쫓아가다보면유한번안에0인항을만날수밖에없다. 그리고, a k =0이면 a 2k = a 2k+1 =0으로 2개의연속한 0이나타나게되고, 그럼다시 a 4k = a 4k+1 = a 4k+2 = a 4k+3 =0으로 4개의연속한 0이나타나게되고, 귀납적으로임의의자연수 m에대해2 m 개의연속한 0이항상나타나게된다. 따라서, 임의의자연수 n에대해 n개의연속한 0이항상있다. ( 나 ) 다음의수열이한예가된다. 1; 1; 0; 1; 0; 0; 0; 1; 0; 0; 0; 0; 0; 0; 0; 1; 0; 0; 0; 0; 0; 0; 0; 0;::: 즉, k =2 n 꼴일때 a k =1 이고그외에는모두 0 인수열.( 이외에도예는얼마든지많다.) 28. 8 학년학생들의장기토너먼트에두명의 7 학년학생이참가하도록승인되었다. 각경기자는다른참가자와꼭한번씩경기를하여, 이긴경우에 1 점, 비긴경우에 0.5 점, 진경우에 0 점씩을받았다. 두명의 7 학년학생은둘이합해모두 8 점을얻었고, 각 8 학년학생들은서로똑같은점수를얻었다고한다. 이장기대회에참가한 8 학년학생들은모두몇명인가? 답은유일한가? ( 캐나다 1976-3) 풀이 8학년참가학생수를 n명, 이들이얻은똑같은점수를 k점이라고하자. 매경기마다두사람이받는점수의합이 1로일정하므로참가자들이얻는총점은전체경기의수와같게된다. ³ n +2 (n +2)(n +1) kn +8= = 2 2 즉, n 2 +(3 2k)n =14이다. 좌변이 n의배수이므로 n j 14, 즉 n =1; 2; 7; 14 말고는불가능하다. n =1; 2 일때에는 k<0 이되어곤란. n =7이면 k =4이고, n =14이면 k =8이다.(n; k) =(7; 4) 는 9명의학생이모든경기에서비긴경우의예가있고,(n; k) =(14; 8) 은같은학년학생끼리는모두비기고 8학년은 7학년에게 1승 1무씩을거둔경우의예가있다. 따라서, n =7; 14 둘다가능하고답은유일하지않다. 29. 4 10 19 크기의합동인 94 개의벽돌이있다. 이벽돌들을한장씩계속쌓아올려서 ( 즉, 높이는 4 또는 10 또는 19 씩계속높아진다 ) 만든탑의높이는몇가지가생길수있는가? (AIME 1994-11)
4.1 조합중급문제 205 풀이 %EEE Suppose x bricks are oriented to add 4 to the height, y to add 19 and z to add 10. Since 5 10 = 3 4 + 2 19 we can take z = 0, 1, 2, 3, or 4. Also we must have y = 94-x-z, so height = 4x+10z+19(94-x-z) = 1786-15x-9z. If z=0, height = 1786-15x = 1 mod 5; if z=1, height = 1777-15x = 2 mod 5; if z=2, height = 1768-15x = 3 mod 5; if z=3, height = 1759-15x = 4 mod 5; if z=4, height = 1750-15x = 0 mod 5. So these values of x,z all give different heights. We can check that 1750 > 15 94 = 1510, so for z=0 there are 95 possible values of x (0, 1,..., 94), for z=1, 94 etc. Hence in total 95+94+93+92+91 = 465 possible heights. 답 465 30. 상식이는가위바위보를하는로보트를만들었다. 이로보트는가위, 바위, 보를낼확률p, q, r을주인이입력하면그확률대로결정하여가위바위보를한다. 근영이가친구들과가위바위보를하는것을관찰했더니가위를자주내는버릇이있었고, 그확률은가위가 1 2, 바위와보는각각 1 4 씩이었다. 상식이는가위바위보로보트를근영이와대결시키려고한다. 단판승부로할때 ( 비겨도다시가위바위보를하지않는다고할때 ), 이길확률이최대가되도록하는 p, q, r을입력시키고싶다. (1) q의값은얼마로해야하는가? (2) 그때, 이길확률은얼마가되는가? (IT꿈나무올림피아드 2006 1차 ) 풀이상식이의로보트가이길확률 X는 p + q + r =1임에서 X = 1 4 p + 1 2 q + 1 4 r = 1 4 + 1 4 q 이고, 이것은 q =1일때최대값 1 2 을갖는다. 답 (1) 1 (2) 1 2 31. 한학급에서 3 문제짜리수학시험을보았다.A 문제를푼학생은전체의 1=6, B 문제를푼학생은전체의 1=2, C 문제를푼학생은전체의 2=3 이었다.A 와 B 문제를동시에푼학생은 A 를푼학생의 2=3 이었고,A, B, C 를모두푼학생은전체의 1=12 이었다. 아무것도풀지못한학생은전체의 1=12 이었고,B 와 C 만을푼학생수는 A 와 C 를동시에푼학생수와같았다.A 문제만푼학생은없고,B 문제만푼학생이 9 명이었다면, 전체학생은모두몇명인가? (2000 대전. 충남영재수학교실 2 차평가 ) 풀이전체학생수를 x라고둔다. A만푼학생이없으므로 n(a) =n(a \ B)+n(A \ C) n(a \ B \ C) ) x 6 = x 9 + n(a \ C) x 12 ) n(a \ C) = 5x 36 : 다음 n(b \ C A) =n(a \ C) 를이용한다. n(b) =n(b \ A)+n(B \ C A)+n(B A C) = x 9 + 5x 36 +9= x 2 ) x =9) x =36: 4 답 36( 명 ) 32. 어떤아파트블록에는아이들이있는부부들만이살고있다. 이아파트의모든남자아이에게는여자형제가있고, 이아파트에는여자아이보다남자아이가더많다. 이아파트에아이들보다어른들이더많을수있는가? ( 뉴질랜드 2000-1) 풀이모든집에는여자아이가있다. 남자아이가여자아이보다더많다. 끝. 33. 평면은유한개의직선에의해서몇개의영역으로나뉜다. 이때같은경계선에접해있는영역이서로다른색깔로칠해지기위해서는 2 가지색깔만써도됨을보여라. ( 통신강좌 1991-2-33) 풀이 ( 서울과학고박종원 ) 직선의갯수를 n 이라할때수학적귀납법으로증명해보자. (1) n =1 일때는명백하다. (2) n = p (p 1) 일때성립한다고하고 n = p +1 일때를생각해보자.
206 조합 일단 p 개의직선만그으면가정에의해성립한다. 이제하나의직선 l 을추가하자. 그리고 l 을기준으로한쪽에있는영역들은색깔을반전시키자. l l 그러면 l 의한쪽부분씩만보면그것들은반전되어도같은경계선에접해있는영역이서로다른색깔로칠해져있음은명백하다. 이제문제가되는것은 l 을경계선으로하는영역들이다. 그런데 l 을경계로하는영역들은 l 이없을때는한영역이었으므로같은색이었는데 l 의한쪽을반전시켰으므로다른색이되었다. ) n = p +1 일때도성립. ) 수학적귀납법에의해모든 n( 1) 에대해성립. 34. 한변의길이가1인정사면체 ABCD의꼭지점 A에서한마리의개미가출발하여모서리를따라기어간다. 이개미는어느꼭지점에다다를때마다, 어느모서리를따라왔는지에대해서는잊어버리고, 세개의모서리중에서각각 1 3 의동등한확률로선택하여계속기어간다. 거리 7을기어간후도착한꼭지점이 A일확률은얼마인가? (AIME 1985-12) 풀이 %EEE Let p_n be the prob of return after distance n. Then p_{n+1 = (1-p_n)/3, because to return after n+1, it must have been at a different vertex after n, and then chosen the correct edge to return. Obv p_1 = 0, so p_2 = 1/3, p_3 = 2/9, p_4 = 7/27, p_5 = 20/81, p_6 = 61/243, p_7 = 182/729. 답 182=729 35. 다음은가로세로대각선의합이모두같은마방진이다. 이마방진을완성하여라. ( 플란더즈예선 1999/2000 1 차 ) 2 1 x 풀이한줄의합이 3x.... 셈 PS 에있는풀이를이용... 36. 4 4 크기의표의각칸에숫자 1 또는 2 를썼다. 이표의 3 3 부분영역에있는숫자들을모두합하면항상 4 의배수가되는데, 표전체의숫자의합은 4 의배수가아니라고한다. 이 16 개의숫자의합의가능한최대값과최소값을구하여라. ( 이탈리아 2005-3) 풀이 (KAIST 06학번김태우 ) 3 3 부분영역의수의합은최소 9, 최대 18이므로,4의배수가될수있는합은최소 12, 최대 16이다. 최소값 12가 9보다 3이큼에서 2가 3개이상있어야하고, 최대값 16이 18보다 2가작음에서 1이 2개이상있어야함을알수있다. 그리고,2가 3개있는경우나 1이 2개있는경우는아래와같이실제로가능하다. 1 1 1 1 1 1 2 1 1 2 2 1 1 1 1 1 2 2 2 2 2 1 2 2 2 2 1 2 2 2 2 2 따라서,16 개전체의숫자의합은최소 19, 최대 30 이다.
4.1 조합중급문제 207 37. 어떤상자에 900 장의카드가있는데, 각각 100 부터 999 까지의수가하나씩적혀있다. 영락이는이상자에서몇장의카드를보지않고꺼내어각카드에적힌수의자릿수의합을계산한다. 영락이가꺼낸카드중에자릿수의합이같은세장의카드가있다는것을확신할수있으려면꺼낸카드는최소몇장이되어야하는가? ( 플란더즈예선 1994/1995) 풀이세자리수의자릿수의합으로가능한것은최소 1(100인경우 ) 부터최대 27(999인경우 ) 까지이다.1에서 27까지중자릿수를더했을때그합이되는수가 3개이하인것은 1과 27( 각각한개씩만있음 ) 뿐이다. 그러므로 2에서 26까지의수중같은것이적어도세장있으려면25 2 + 1 = 51장이있어야하고, 100, 999가포함되었을경우를생각하면 51 + 2 = 53장이필요하다. 답 53장 주자릿수의합이 2인경우는2가지있으므로사실고려하지않아도상관없다.1가지뿐인 1과 27의경우만특별히취급하면충분하다. 38. 1 에서 1000000 까지의자연수중에서, 완전제곱수하나와완전세제곱수하나의합으로표현될수있는자연수 ( 예를들어 33 은 5 2 +2 3 과같으므로그런자연수 ) 와그렇지못한자연수는어느쪽이더많은가? ( 뉴질랜드 2001-4) 풀이완전제곱수는 1000개, 완전세제곱수는 100개가있다. 이둘이조합하여만들어지는수가모두다르다고하더라도최대 1000 100 = 100000개의수밖에만들어지지않는다. 즉, 그렇지못한수가 900000개이상이므로더많다. 39. 몇가지크기의공 65 개가두개의상자에나뉘어담겨있다. 각각의공의색깔은하양, 검정, 빨강, 노랑중한가지이다. 같은색의공으로다섯개를어떻게골라도그중어느두개의공은크기가같다. 같은상자안에담긴같은크기같은색의세개의공이있음을증명하여라. ( 인도지역예선 1990-1) 증명같은색의공으로다섯개를어떻게골라도그중어느두개의공은크기가같다고하였으므로, 각각의색의공은크기가네종류씩뿐이다. 어느한쪽상자에33개이상의공이있고, 그중어느한가지색의공은9개이상있고 ( 모두 8개이하이면 4 8=32< 33 으로모순 ), 그럼그중어느한크기의공은 3개이상있다 ( 모두 2개이하이면 4 2=8< 9 로모순 ). 40. 어떤토너먼트시합에서각각의두사람은서로꼭한번씩대결한다. 각대결에서이기면 1점을, 비기면 1 2 점을, 지면 0점을받는다. 가장낮은점수를받은 10명의집합을 S라하자. 모든선수가자기가받은점수의정확히절반을 S의선수와대결하여얻었음이확인되었다. 이토너먼트에참가한사람은모두몇명인가? (AIME 1985-14) 풀이 %EEE n+10 players. The 10 worst played 45 games amongst themselves. They must have got between them 45 points from these games and hence 90 points in total. The top n players played n(n-1)/2 games amongst themselves, giving them n(n-1)/2 points. Hence they got n(n-1) points in total (the rest from the 10 worst). So the total points scored by everyone were 90 + n(n-1). But total is (n+10)(n+9)/2. Hence n^2-21n + 90 = 0, n = 6 or 15. But the top n get n(n-1) points in total, an average of n-1 each, and the bottom 10 get an average of 9 each. Hence n 10. So n = 15. 답 25 41. 2 명의소년이정육면체의 12 모서리를빨강과파랑의색으로색칠한다. 먼저첫번째소년이세모서리를골라빨강으로색칠하고, 다음두번째소년이파랑으로다른세모서리를, 다시첫번째소년이또다른세모서리를골라빨강칠을하고, 마지막으로두번째소년이남은세모서리를파랑으로색칠한다. 첫번째소년이어느한면의네모서리가모두빨강칠이되도록할수있는가? ( 통신강좌 1991-3-32) 풀이면이모두 6개이므로변 3개를잘고르면, 어떤면에도그중한변이속하게할수있겠다 (1). 이제아래그림에서 a, b, :::, `의 12개의모서리를다음과같이네집합으로분할하자. fa; f; lg; fb; g; ig; fc; h; jg; fd; e; kg: 각집합의세원소는서로꼬인위치에있어 (1) 을만족시키게된다. 첫번째소년이어떻게세변을골라도두번째소년의차례에이네집합중빨강칠이되지않은세변을포함하는집합이적어도하나있
208 조합 다. 이집합의세변을모두파랑으로칠하면, 색칠이끝났을때빨강칠의모서리로둘러싸인면이없게된다. a b c e d h f g i j l k 그러므로, 두번째소년이최선의방해를한다면첫번째소년은결코빨강모서리의면을얻을수없다. 42. 서로다른실수들 x 1 ;x 2 ;x 3 ;:::;x 40 이주어져있다. 처음두항 x 1 과 x 2 를비교하여만일 x 2 <x 1 이면두항을서로바꾼다. 그다음, 두번째항과세번째항을비교하여뒷항이더작다면두항을서로바꾼다. 이런식으로 39 번째항과 40 번째항에이를때까지, 이웃한두항을비교하여뒷항이더작다면두항을바꾸는것을계속하여반복한다. 처음에수열의순서가임의로주어졌을때, x 20 이 30 번째항이되어끝날확률을구하여라. (AIME 1987-13) 풀이문제의조건은 x 1 ;x 2 ;:::;x 31 중에서 x 31 이가장크고x 20 이두번째로크다는것과같다. 따라서, 이럴확률은 1 30 31 = 1 930 이다. 43. 모든정수들에빨간색또는파란색을칠한다고하자. 어떤두빨간점도거리 d만큼떨어져있지않고, 어떤두파란점도거리 1만큼떨어져있지않도록하려면, d는어떤수가되어야하는가? (IMTS R13-3) 풀이빨간점이 n과 n +1에두번연속하여나타났다고하자. 그럼이들과거리 d만큼떨어진 n + d와 n + d +1은빨간점이될수없으므로둘다파란점이다. 그럼거리 1만큼떨어진파란점이생겼으므로모순. 따라서, ² 빨간점도파란점도같은색점이연속하여나타날수없다. 그렇다면빨간점다음은파란점, 파란점다음은빨간점,... 이렇게빨파빨파빨파빨파빨파... 로칠하는방법만이유일하다. 이때, 두빨간점사이의거리는모든짝수가가능하고홀수는나타나지않으므로, d는모든홀수가될수있다. 44. 어떤주사위에는 1, 2, 3 의눈만이각각 a, b, c 개의면에그려져있다. 이런주사위 2 개를동시에계속던지면두주사위의눈의합이 3 이될때가 3 회에 1 회꼴로나온다고한다. c 는얼마인가? (1999 교육청경시 ) 풀이합이 3이되려면한쪽이1, 한쪽이2여야하고, 이확률이 1=3이므로다음과같이계산한다. a 6 b 6 2= 1 3 ) a b =6: 이러한자연수 a; b는순서에상관없이 1; 6이거나 2; 3인데, a + b 6이어야하므로 2; 3이다. 따라서 c =6 a b =1: 답 :1: 45. 네쌍의부부가모인어느모임에서두사람씩조를이뤄네경기의체스게임을동시에했다. 여 B 씨는남 E 씨와뒀다. 여 A 씨는여 C 씨의남편과뒀다. 남 F 씨는남 G 씨의부인과뒀다. 여 D 씨는여 A 씨의남편과뒀다. 남 G 씨는남 E 씨의부인과뒀다. 남 H 씨는누구와결혼했는지알수있겠는가? 당신의답이옳음을증명하여라. ( 플란더즈 1995-1)
4.1 조합중급문제 209 풀이남E씨는여B씨와경기했으므로,E씨는여C씨의남편 ( 여A씨와경기한 ) 도아니고여A씨의남편 ( 여D씨와경기한 ) 도아니며또한 E씨의부인은남G씨와경기했으므로여B씨일수도없다. 따라서, E씨의부인은 D씨다. 그럼 G씨와경기한것도 D씨이다. 따라서,A씨의남편은 D씨와경기한 G씨이다. G씨의부인인 A씨는 F씨와경기하였고, 그럼 C씨의남편은 A씨와경기한 F씨이다. 따라서, 남은짝은 B씨와 H씨뿐이다. 답여B씨 46. 집합 N = f1; 2;:::;ng 에대해조건 X ½ Y ½ N 이만족되도록집합 X, Y 를구성할수있는모든경우의수를구하여라. 풀이 N의각각의원소에대해 (1) X에도포함되지않는것,(2)X에는포함되지만 Y 에는포함되지않는것,(3)Y까지모두포함되는것등 3가지를독립적으로결정할수있다. 따라서, 구하는경우의수는 3 n 답 47. 상자안에빨간구슬 m개와파란구슬 n개가있고, m + n 1991 이다. 두개의구슬을임의로꺼내면, 그두구슬이같은색일확률이 1 2 이라고한다. m의가능한최대값을구하여라. (AIME 1991-13) 풀이 %EEE Simplifying, (m-n)^2 = m+n. Put N = m+n. Then larger of m, n is (N + N)/2, so we also want N to be as large as possible. Hence N must be largest square <= 1991, namely 44^2 = 1936, giving 990, 946. 답 990 48. 다음의 4 4 크기의숫자판에는각칸마다숫자가하나씩적혀있고, 가로혹은세로로연속한세칸의숫자의합은모두같다고한다. 33 23 0 17 ( 가 ) ( 나 ) ( 가 ) 와 ( 나 ) 에들어갈숫자를차례로써라. (IT 꿈나무올림피아드 2006 1 차 ) 풀이연속한네칸의수를a, b, c, d라하면a + b + c = b + c + d 이므로 a = d 이다. 즉, 가로나세로로세칸이동한곳에서로위치하는두수는같다. 그럼우선다음을알수있다. 33 33 23 23 0 17 ( 가 ) 0 33 33 그럼연속한세칸의합은 33 + 23 + 0 = 56 이다. 따라서,0+17+( 가 )=56이므로 답 ( 가 )39 ( 나 )33 49. 반지름 1 인원에내접하는임의의사각형에대해, 가장짧은변의길이가 p 2 보다작거나같음을보여라. ( 캐나다 1969-9) 증명원의중심을 O 라하고, 사각형의네꼭지점을차례대로 A, B, C, D 라하자. \AOB, \BOC, \COD, \DOA 등네각의합은 360 ± =4 90 ± 이므로, 비둘기집의원리에의해이네각중에는 90 ± 보다작거나같은것이있다. 일반성을잃지않고 \AOB 90 ± 라하면, A 와 B 는반지름 1 인 4 분원에포함되고, 따라서 AB p2 이다.
210 조합 50. 어떤다면체가있는데, 임의의꼭지점에서출발해서변을따라움직이다가처음의꼭지점으로돌아오면어떻게움직여도그동안변을따라꼭지점을이동한횟수가항상짝수번이된다고한다. 이다면체의꼭지점을두가지색으로칠하는데, 변으로이웃한점은항상서로다른색이되도록할수있음을보여라. ( 통신강좌 1996-12-19) 증명먼저주어진그래프 G가연결된그래프 G 1 ;G 2 ; ;G r 로분리된다고하자. 그리고각선의길이를 1이라고하고, 임의의점 P (2 G i ) 를기준으로통로의길이가 n만큼떨어진점에 ( 1) n 이라는색을칠하자.(n은 0 또는자연수 ) 그렇다면이때임의의 G i 에서점 X(2 G i ) 에서 Y (2 G i ) 로가는통로가2개이상있고, 그중한통로 I에대해 I의길이가 라하자. 이제 X의색이 ( 1 또는 1) 라하면Y의색은 ( 1) 이다. 그런데다른통로 II에대해II의길이가 이고 6 (mod 2) 라면 X! I Y! II X라는회로의길이 + 는홀수가되어모순이다. 따라서 (mod 2) 이고임의의점 X에서 Y 로가는모든통로의길이는모두짝수이거나홀수이다. 그러므로각점의색깔은유일하게 1 또는 1로정해지고인접한점은서로다른색으로칠해지게된다. 51. 1 이상 126 이하의자연수중에서 8 개의서로다른자연수를임의로택하였다. 그중에서다음조건을만족하는두수 x, y 를항상찾을수있음을보여라. 1 < y 2 ( 통신강좌 1996-13-26) x 풀이비둘기집의원리를이용한다. 한묶음에서임의의두수를꺼냈을때 x<y 2x의관계가성립하도록 1에서 126까지의수를나눌수있을까? 한묶음에서가장작은수를 x라고하면같은묶음의모든수는 2x 이하여야한다. (1; 2) (3; 4) (5;::: ;8) (9;::: ;2 4 ) ::: (2 6 +1;::: ;126) 위와같이 7개의묶음으로나누면 8개의숫자를골랐을때적어도한묶음에두수가있어그두수가 x<y 2x이다. 52. G 는 11 개의꼭지점을갖는다면체이다. G 의어떤꼭지점도다른모든꼭지점과변으로연결되어있는경우는없다. G 의변을모두지우고 G 에서서로연결되어있지않았던두점을모두새로운변으로이어그래프 G 를만들자. G 와같은연결구조를갖는다면체는존재하지않음을보여라. ( 통신강좌 1997-14-19) 증명 G 의변의수를 e, G 의변의수를 e 0 이라하면 e + e 0 = 11 2 =55 이다. 그런데다면체 ( 혹은평면그래프 ) 는 ( 변의수 ) 5 3 ( 꼭지점의수 ) 6 가되므로 ( 셈본중급참조 ), e; e 0 3 11 6=27 이된다. 여기서 55 = e + e 0 54 이므로모순. 53. 집합 A = fa; b; cg 에서의함수 f : A! A 중에서, f(f(x)) = f(x) 를항상만족하는것은모두몇개인가? ( 플란더즈예선 1996/1997 1 차 ) 풀이먼저 f(x) =x; x = a; b; c인함수는 f(f(x)) = f(x) =x를항상만족한다. f(x) = y(x 6= y) 라고가정한다. 이때f(f(x)) = f(x) ) f(y) = y 이므로, f(x) = y이면 f(y) = y이다. 이때x, y와다른z가있을때, f(z) =x이면 f(x) =x여야하므로모순이고, f(z) =y, f(z) =z인경우에는모순이생기지않는다. 즉,(f(x);f(y);f(z)) = (x;y;z); (y;y;y); (y;y;z) 가가능하다. x; y; z에 a; b; c를다르게대입할수있으므로이러한함수 f는 1+3+3 2=10개있다. 답 10개 54. S 는 6 개의원소로이루어진집합이다. 합집합이 S 가되도록 S 의두부분집합을택하는 ( 두부분집합은서로같을수도있다 ) 방법은모두몇가지인가? (AIME 1993-8)
4.1 조합중급문제 211 풀이 %EEE Let the subsets be A and B. For each element we have three choices (A, B or both). That gives each pair of subsets twice except for the case A = B = S. Hence (3^6 + 1)/2. 답 365 55. 각면이삼각형또는오각형인 32 개의면을갖는다면체가있다. V 개의전체꼭지점각각에는 T 개의삼각형과 P 개의오각형이만난다.100P +10T + V 의값은얼마인가? 오일러의공식 V + F = E +2 를이용해도좋다 ( 단, F 는면의개수, E 는모서리의개수이다 ). (AIME 1993-10) 풀이 %EEE V + 30 = E (Euler). VT + VP = 2E (counting edges). So V(T+P-2) = 60. Also, there are VT/3 triangles and VP/5 pentagons, so V(T/3 + P/5) = 32. Hence 32V(T+P-2) = 32 60 = 60V(T/3 + P/5). Hence 3T + 5P = 16. Hence P + T = 2. Hence V = 30. 답 250 56. 1 부터 100 까지숫자가하나씩적힌구슬 100 개가일렬로배열되어있다. 우리에게는특별한정리기계가있는데, 이정리기계는우리가 100 개중에서 50 개를적당히골라주면, 그기계는이 50 개의구슬만을오름차순으로정리하여원래의 50 개의위치에순서대로둔다. 예를들어,6 개중에서 3 개를정리해주는기계라면,132465 에서 326 을골라그기계에게정리시키면 123465 가된다. 다음은이정리기계를다섯번사용하여 1 부터 100 까지오름차순으로항상정리할수있음을증명하여라. (IT 꿈나무올림피아드 2006 1 차 ) 증명 100개의구슬의위치를왼쪽부터 25개씩묶어 A, B, C, D그룹이라고하고, 두그룹X, Y에있는 50개의구슬을정리시키는것을 (X,Y) 로나타내기로하자. 그럼 (A,B), (C,D), (A,C), (B,D), (B,C) 순으로정리시키면된다. 처음두번의시행을거치면 1부터 25까지의수는 A그룹또는 C그룹에있게되고,76부터 100까지의수는 B 또는 D그룹에있게된다. 그다음두번의시행을거치면A그룹은 1부터 25까지오름차순으로정리되고 D그룹은 76부터 100까지오름차순으로정리된다. 마지막시행을거치면모두오름차순으로정리된다. 57. 어떤나라는몇개의 ` 주 ' 로지역이분할되어있다. 각주는하나의폐곡선으로경계지어져있다 ( 한주가여러덩이로분할되어여기저기존재하거나하지않는다 ). 이나라의지도를만들고자하는데, 변으로인접한두주는항상서로다른색으로나타내고자한다. 이나라에서는둘씩서로이웃한네주를찾을수없지만, 지도를만들기위해서는네가지색이필요하다고한다. 이런지도도있음을예를들어서보여라. ( 통신강좌 1991-2-34) 풀이 ( 서울과학고박종원 ) 다음과같은그래프를생각해보자. A E D C F D; E; F 는같은색일수없다. 따라서 (G) 3. (G) =3 이라고하자. D 는 1 색, E 는 2 색, F 는 3 색이라고하면 A 는 3 색, B 는 2 색일수밖에없고, C 는 1 색,2 색,3 색인점과모두연결되어있으므로결국 3 가지색으로칠하는것은불가능하다. 따라서 (G) =4 이다.( 이때 C 는 4 색으로하면된다.) 또 G 의어떤부분그래프도 K 4 를포함하지않는다. B 58. (1) 어떤나라에는 2 개의대도시와 n 개의소도시가있다. 이나라에통신망을구성하는데, 하나의통신선은두도시를연결하며,2n 1 개의통신선만으로모든도시를직간접적으로연결하고자한다. 단, 대도시끼리혹은소도시끼리통신선이연결되어서는안된다. 가능한통신망의연결구조는모두몇가지인가? (2) n 개의대도시가있다고하고같은문제를풀어라. ( 통신강좌 1997-14-21)
212 조합 풀이 (1) 각통신선은대도시와소도시사이만을연결해야한다. 대도시와소도시사이를연결하는통신선은 2n종류가있는데, 그중에딱하나만을제외해야하는것이므로방법은 2n가지. n =1일때는이것은불가능하고, n 2 일때는항상모두연결되어있다.(2)2n개의모든도시가연결되기위해서는적어도 2n 1개의선이있어야하는데, 딱그개수만큼만선을쓸수있으므로연결구조는나무가된다. 즉, 두대도시a와 b를연결하는경로가있어야하므로 a, b에동시에연결된소도시 x가있다. α β ξ 만일 x 이외에다른소도시가 a, b 와연결되어있다면, 꼭지점 4 개로구성된회로가존재하게되어나무임에모순이다. 따라서,x 가될소도시를하나택하고, 나머지소도시들은 a 에연결할지 b 에연결할지를각각선택하면된다. 즉, 총경우의수는 n 1 2 n 1 = n 2 n 1 이다. 59. 어떤파티에 20 명의남자와 20 명의여자가있다. 각각의남자는 x 명의여자를좋아하고각각의여자는 y 명의남자를좋아한다. 둘이서로좋아하는남녀는한쌍도없다면, x + y 의최대값은얼마인가? (IT 꿈나무올림피아드 2006 1 차 ) 풀이 ( 남자A, 여자B) 쌍을모으는데,A가 B를좋아하거나 B가 A를좋아하는쌍을모두모은다고하자. 서로좋아하는경우는없으므로, 그럼이런쌍은모두 20x +20y개가된다. 그런데, 가능한 ( 남자, 여자 ) 쌍은총 20 20개이므로 20(x + y) 20 2, 즉 x + y 20 이다. 실제 x =0,y =20일때등호가성립하는경우가있음을쉽게알수있다. 답 20 60. 네개의동전이있는데세개는똑같은무게의진짜동전이지만, 나머지하나는가짜로무게가다르다. 저울을이용해가짜를찾고싶은데, 이저울은동전을하나만올려놓으면저울의측정범위보다가벼워서무게를측정하지못하고,2 개이상올려놓아야정확한무게를수치로나타내준다고한다. 이저울을 4 번만사용하여가짜도찾고그것이진짜동전보다가벼운지무거운지까지알아내는방법을찾아라. ( 오클랜드 2001-5) 풀이 1 네동전을 a, b, c, d 라하고 a + b + c; a + b + d; a + c + d; b + c + d 의무게를각각재자. 그럼가짜가포함되지않은한경우만무게가다르고 (X 라하자 ), 나머지세경우는가짜가하나포함되어무게가모두같다 (Y 라하자 ). 즉, X 에포함되지않은동전이가짜이고, X>Y 이면가짜가가벼운경우, 반대는가짜가무거운경우이다. 주위의풀이에서네번잰무게를모두합한후 3으로나누어주면전체무게의합 a + b + c + d가된다. 여기서위에서잰각각의무게를빼어주면 a, b, c, d의무게를알수있고, 그로부터결론을얻어도된다. 풀이 2 네동전을 a, b, c, d 라하고 a + b; a + c; a + d 를각각재자. 만일이세번의무게가모두같으면 (Y 라하자 ) b, c, d 는모두같은무게이고따라서 a 가가짜이다. 그럼진짜동전 2 개의무게 X = b + c 를재어 X>Y 이면가짜가가볍고그반대는가짜가무거운경우가된다. 만일 a 가진짜이면가짜는 b, c, d 중에하나이므로세번중진짜만으로잰두번은무게가같고 (X 라하자 ) 가짜가섞인한번은무게가다르다 (Y 라하자 ). 역시 X, Y 의무게비교에따라같은방식으로결론을내리면된다 ( 이때는저울을세번만사용하게된다 ). 풀이3 P = a + b + c + d, Q = b + c + d, R = c + d, S = b + d 를각각재자. 그럼 a = P Q, b = Q R, c = Q S, d = R + S Q 로각각직접계산해낼수있다. 그럼진짜와가짜가분명히드러나고어느것이무거운지도확인된다. 주이외에도많은방법이가능하다.
4.1 조합중급문제 213 61. 양팔저울과몇개의추를이용해 1g, 2g, :::; ng 의질량을모두측정할수있게하고싶다. 예를들어, 1g 의추와 3g 의추가있으면 n =4 까지가능하다. 이때,2g 의질량은 1g 의추와 3g 의추를각각서로다른팔에올리는방법으로측정할수있다.(1)3 개의추로가능한최대의 n 을구하여라.(2)4 개의추로가능한최대의 n 을구하여라. (IT 꿈나무올림피아드 2006 1 차 ) 풀이 (1) 숫자 0, 1, 2 대신숫자 1, 0, 1을이용한3진법에연관시킬수있다. 그럼 13부터 13까지측정가능하고이 27가지를측정하는경우가최대임을확실히알수있다.( 왼쪽접시와오른쪽접시의추를모두서로바꾸어놓는경우를생각하면 k의측정이항상쌍으로함께존재해야한다.) 실제로 1g, 3g, 9g의추로최대13 g까지측정할수있다. (2) 마찬가지로 1g, 3g, 9g, 27g의추로최대40 g까지측정할수있다. 답 (1) 13 (2) 40 62. 다음과같이주어진 3 3 크기의부호판이있다. + + + + + 여기서한가로줄혹은한세로줄을마음대로택해그줄에있는세칸의부호를모두바꾸어놓을수있 + + + 다. 예를들어가운데세로줄을택하면 와같이바뀐다. 이런작업만을반복할수있을때, 다 + + + 음중만들어낼수있는부호판을모두골라라. (IT 꿈나무올림피아드 2006 1 차 ) (1) + + + + (2) + + + + + + + + + (3) + + + + (4) + (5) + + + + + 풀이 (1), (2), (5) 는쉽게만들수있다. (3), (4) 는만들수없는데,2 2 부분영역을택했을때어떻 게작업해도그부분영역안의 + 부호의개수의홀짝이변하지않아야함을관찰하면증명된다. 답 63. 한변의길이가 1 인정삼각형타일과정사각형타일이각각 30 개,10 개가있다. 이것들을이용해정사면체나정육면체, 사각뿔등여러가지다면체를여러개만들수있다. 최대몇개의다면체를만들수있는가?( 같은모양의다면체를여러개만들어도되고, 몇종류인지를세는것이아니라몇개인지를세는것이다.) (IT 꿈나무올림피아드 2006 1 차 ) 풀이면의개수가가장적은다면체는사면체뿐이고, 따라서하나의다면체를만들때각각적어도 4개의타일이필요하다. 타일이모두 40개이므로많아야 10개의다면체를만들수있다. 그런데,10개를만들수있다면모두사면체로만들어야하는데사각형타일도있으므로이것은안된다.9개를만드는것은예를들어다음과같이가능하다 : 정사면체 6개와삼각기둥 3개. 답 9개 64. 정육면체의각면에서로다른자연수를하나씩써넣는다. 또, 각꼭지점에는그점에서만나는세면의수의합을써넣는다. 그리고, 여덟꼭지점에부여된이합들을다시모두합하여 S 라고한다. 각면에여섯개의자연수를처음에어떻게써넣었더라도이 S 는항상 a 의배수가된다. a 의최대값을구하여라. (IT 꿈나무올림피아드 2006 1 차 ) 풀이각면의수는그면과접한 4 개의꼭지점에포함된다. 즉,6 면의수를 u; v; w; x; y; z 라하면 S = 4(u + v + w + x + y + z) 가된다. 답 4 65. 볼록다각형에모든대각선을그었다. 이도형의변이나대각선에각각방향성을적당히잘줘서루프가생기지않게할수있음을보여라. (A! B! C! A 와같이선의방향성대로따라가서원래자리로돌아오는것을루프라고한다.) ( 독일 BW 1974 1 차 -4) 증명꼭지점에순서를줘서항상순서가빠른점에서늦은점을향하는방향이되도록하면됨.
214 조합 66. 정육면체의각면을 4개의작은정사각형으로 4등분하고, 각각의작은정사각형을 3가지색중에서골라색칠한다. 공통변을갖는인접한두정사각형은서로다른색이어야한다. 각각의색이꼭 8번씩쓰였음을증명하여라. (Towns 1988가을 JO4) 증명정육면체의각꼭지점 X에대해, X에서만나는작은세정사각형의모임을생각. 모두 8개의모임이있으므로한가지색이 9번이상쓰이면비둘기집의원리에의해같은모임에그색이두번쓰였고그럼인접한정사각형이생기므로모순. 67. 다면체의각꼭지점마다자연수를하나씩붙이는데, 한모서리의양끝점의두수는서로소가아니고이웃하지않은두점의수는서로소가되도록할수있음을증명하여라. ( 소수는무한히많음을가정해도좋다.) (Towns 1988가을 SO3) 증명각변마다소수하나씩주면끝. 68. (a) 정18각형의꼭지점들을번갈아흑백으로색칠하였고, 여기서두사람이게임을한다. 두사람은번갈아서, 같은색의두꼭지점을잇는대각선을하나씩그린다. 이대각선들은서로도중에교차할수없다. 마지막으로대각선을그린사람이이기는것으로할때, 두사람모두최선의전략으로게임을한다면누가이길까? (b) 정20각형에서같은게임을하면결과가어떻게될까? (Towns 1988봄 JO3 변형 ) 69. 컴퓨터화면에수 123이나타나있다. 매분마다컴퓨터는화면의수에 102를더한다. 컴퓨터전문가미샤는그가원할때마다화면에나타난수의자릿수의순서를마음대로바꿀수있다. 화면에네자리의수가나타나지않도록할수있는가? (Towns 1991봄 JO4) 풀이할수있다. 예를들어 123 : 312! 414! 516! 618! 720 : 027! 129! 231 : 312! 로반복시킬수있다. 70. 모든 10자리수들의집합을다음의두부분집합으로분할할수있다 : M은두5자리수의곱으로나타낼수있는모든10자리수들의집합이고, N은나머지10자리수들의집합이다. M과 N 중에어느집합이큰가? (Towns 1988봄 SO3) 풀이두 5 자리의수를 10 4 a b<10 5 이라하면이런쌍의개수는 a =10 4 ; 10 4 +1;:::;10 5 1 일때를차례로생각하면 (10 5 10 4 )+ +3+2+1= 1 2 (105 10 4 )(10 5 10 4 +1) jmj. 즉, 2jMj < 10 10 10 9 (= jmj + jnj) 임을확인할수있어서 jmj < jnj. 71. 1; 2;:::;n의재배열중에서, 첫항이아닌각각의 k에대해 (1 k n) 그보다왼쪽에 k 1, k +1중에적어도하나가항상있는것은모두몇개인가? (Towns 1988가을 SA3) 풀이 1:::n 의배열의처음에수하나를더끼워넣는것을생각하면 p n+1 =2p n. 즉 p n =2 n 1. 72. 1989개의수로된집합이있는데, 여기서 10개의수를임의로골라합하면항상양수가된다. 이집합의모든원소의합도양수임을증명하여라. (Towns 1989가을 JO3) 증명모든가능한 10개세트를모두합하면각수는동일한횟수 (a라하자. a는그한수를제외한나머지 1988개의수에서그수와세트를이룰 9개의수를고르는경우의수임 ) 씩더해지므로, as > 0 이됨. 즉, S>0. 73. 10명의친구들이서로에게연하장을보냈다. 각자 5통씩보냈다고한다. 그럼서로연하장을보낸두명이있음을증명하여라. (Towns 1989가을 SO1) 증명두친구의쌍은 1 10 9=45 2 가지. 연하장은 5 10 = 50통으로친구쌍보다더많으므로어떤두연하장은같은쌍사이에오갔음 ( 비둘기집 ). 74. 9 9 판의각칸에파리가한마리씩앉아있다. 어떤신호가떨어지자, 각파리가대각방향으로이웃한칸으로동시에움직였다. 그럼한칸에여러마리의파리가있거나아무파리가없을수도있게된다. 빈칸은최소몇개인가? ( 러시아 1989 4차-y9-2)
4.1 조합중급문제 215 풀이흑백으로판을칠하면 ( 코너가흑색 ) 백색칸끼리는짝을이룰수있으므로 OK. 흑색칸들은다시대각방향으로인접한것이서로다른색이되도록흑청으로칠하면 ( 코너가흑색 ) 흑색칸이청색칸보다 9개가많아서빈칸은최소 9개... 75. \ 날으는룩 " 은보통의체스룩 ( 차 ) 처럼가로또는세로방향으로몇칸이든움직이지만인접한칸으로는바로갈수없다.4 4 체스판에서날으는룩이 16번움직여모든칸을들러원위치로돌아오는것이가능한가? (Towns 1991가을 JO2) 풀이가능한것으로기억함. 76. 어떤왕국에 32 명의기사가있다. 이중몇몇은다른누군가의가신이다. 한명의가신은한명의주인만섬기며, 주인은누구나그의가신보다부자이다.4 명이상의가신을둔기사를바론이라한다. 바론은최대몇명인가? 단, 이왕국의법에의하면, 내가신의가신은내가신으로보지않는다. (Towns 1991 가을 JA1) 풀이가신은최대 31 명이므로바론은최대 7 명.7 명일때실제가능함이쉽게확인됨. 77. 1 에서 12 까지의수가원주위에임의의순서로배열되어있다. 이웃한두수의차가 1 보다크면그두수의자리를바꿀수있다. 이런과정을유한번반복하여이수들을크기순서대로배열할수있음을보여라. ( 러시아 1989 4 차 -y8-3) 증명 1을 2에이웃할때까지옮기고,(12) 를 3에이웃할때까지옮기고,(123) 을 4에이웃할때까지옮기고... 하면됨. 78. 같은크기의한세트의책 20권이책꽂이에임의의순서로일렬로꽂혀있다. 도서관사서는이를제1권부터제20권까지왼쪽에서오른쪽으로순서대로재배열하고싶다. 사서는제위치에있지않은책중한권을그책의자리에있는책과바꿔꽂는것만할수있다. 이렇게재배열을완료할때까지필요한교환횟수는사서의작업순서에상관없이일정함을증명하여라. ( 러시아 1989 4차-y8-7) 증명무조건 20 - (cycle 의개수 ) 번작업하게됨 ( 고정점도길이 1 짜리 cycle 로간주 ). 79. 정육각형 ABCDEF 의각꼭지점을중심 O 와선분으로이었다. 이 7 개의점중 A, C, E 는파란색으로, 나머지점들은모두흰색으로색칠되어있다. 우리는이 7 개의점중하나를택해그점과그점에이웃한점들의색을모두바꾸는조작을할수있다 ( 파란색은흰색으로, 흰색은파란색으로 ). 이런조작을유한번시행하여 (a) 점 B, D, F 만파란색이고나머지는흰색이되도록할수있음을보여라. (b) 점 B, D 만파란색이고나머지는흰색이되도록할수없음을보여라. ( 러시아 1989 4 차 -y9-5) 증명 (a) A, B, C, D, E, F 를한번씩조작하면됨.(b) 항상짝수개의칸이동시에변하므로파란색점의개수의홀짝이불변. 80. 1에서 n 2 까지의자연수를 n n 체스판의각칸에임의로적었다. 두수의차가n +1보다작지않은이웃한 ( 꼭지점또는변을공유하는 ) 두칸이존재함을증명하여라. (Towns 1990가을 SO1) 증명귀류법.1 에서 n 2 까지건너가는거리가항상 n 1 번이하이면충분한데, 그럼차이가 (n 1)n 이하라야해서모순. 81. 똑같이생긴 25개의동전중에 3개는가짜이다. 모든진짜동전의무게는서로같고, 모든가짜동전의무게도서로같지만가짜동전은진짜동전보다가볍다. 천칭을두번만사용해서 6개의진짜동전을가려내고싶다. 어떻게하면될까? ( 러시아 1990 4차-y9-1) 풀이 12개씩달면무겁거나평형인쪽에가짜동전은 1개이하. 그 12개를 6개씩나눠달면무겁거나평형인쪽에가짜동전이 0개.
216 조합 82. 가방에 1993 개의빨간공과 1993 개의까만공이들어있다. 우리는반복적으로매시간마다공을 2 개씩꺼내는데, (1) 2개의공이같은색이면, 모두버린다. (2) 2개의공이다른색이면, 까만공은버리고, 빨간공은가방에다시집어넣는다. 이과정을모두마쳤을때, 가방안에빨간공하나만남게될확률은얼마인가? (IMTS R10-4) 풀이빨간공은항상홀수개. 확률은 1 83. f1; 2;:::;10g 을두집합 fa 1 ;a 2 ;:::;a 5 g 와 fb 1 ;b 2 ;:::;b 5 g 로분할하였다. a 1 <a 2 <a 3 <a 4 <a 5 이고 b 1 >b 2 >b 3 >b 4 >b 5 일때, (1) 각각의쌍 fa j ;b j g (1 j 5) 에서둘중에큰것은항상 6 이상임을보여라. (2) 5X ja i b i j =25가항상성립함을보여라. ( 인도지역예선 2002-4) i=1 증명어디서본듯... 통신강좌에서였나... (1) 을보이면 (2) 는바로딸려나옴. 84. 원위에몇개의점을잡아다음과같이번호를매긴다. 먼저, 한지름의양끝점을잡아둘다 1 로번호를매긴다. 그다음부터는그전단계에서생긴호의중점을찾아양끝두점의번호를더한값으로번호를매긴다. 이과정을 n 번시행했을때원위의점의번호들의합은얼마인가? 예를들어,4 번의시행뒤에는 (1,4,3,5,2,5,3,4,1,4,3,5,2,5,3,4) 와같이번호가매겨진다. ( 소련 1963-13) 풀이 n 단계의합을 a n 으로하면우선 a 0 =2 이고, 전단계의각수는두곳에더해지므로 a n+1 = a n +2a n =3a n. 따라서, a n =2 3 n. 85. n 이 5 보다큰양의정수라하자. 집합 fn +1;n+2;:::;n+30g 에서소수는많아야 8 개임을증명하여라. (IMTS R11-2) 증명 mod 30 으로 1, 7, 11, 13, 17, 19, 23, 29 인수들만소수일가능성이있고나머지는모두합성수. 86. 3 3 3 루빅큐브의표면에폐곡선을그리는데, 꼭지점은지나지않게하면서각각의작은사각형을딱한번씩지나게그릴수있는가? (Towns 1989봄 JO4) 풀이쉽게할수있다. 각면을차근차근지나가기만해도. 87. 99 99 판에두명이번갈아가면서비어있는칸을골라체커를놓는다. 단, 이웃한칸이모두비어있거나이웃한칸들중딱한칸에만상대편의체커가있을때만그칸에체커를놓을수있다. 이웃한칸이란변을공유하는상하좌우의네칸을말한다. 더이상체커를놓을수없는사람이진다고할때, 먼저하는사람과나중에하는사람둘중에어느쪽이필승법을갖게되는가? ( 러시아 1992 4차-y9-2) 풀이천원에놓은후대칭플레이 88. 공간상에어느네점도한평면위에있지않은다섯점이주어져있다. 이중두점을잇는각선분을흰색, 초록, 또는빨강으로칠하는데, 모든색이다사용되어야하고같은색의세선분이삼각형을구성하는경우는없어야한다. 이다섯점중에세가지색이모두만나는점이있음을증명하여라. ( 불가리아 1981 4 차 -1) 증명귀류법. 한점에서같은색세변이있을수없음을보이고, 모든점에서 2:2이면전체에두가지색만사용됨을확인. 89. 1997개의연속된양의짝수들을생각하자. 이들중 41개를택해만든부분집합에는차가 100보다작은두수가언제나존재함을증명하여라. ( 몰도바 1997 최종-y7-5)
4.1 조합중급문제 217 증명 40개의집합 fa; :::; a +98g, fa + 100; :::; a + 198g,..., fa +3900;:::;a+3992g 을두고비둘기집. 또는 41개의수를 b 1 <b 2 < <b 41 이라할때, b n+1 b n +100임을이용. 90. 한직선이 8 8 체스판을가로지르고있다. 직선이어떤칸의내부의점을지날때, 그직선이그칸을가른다고말한다. 이직선은 64 개의칸중최대몇개의칸을가를수있는가? ( 헝가리 1930-2) 풀이 WLOG, 기울기는 0 이상이라할수있고, 위로혹은오른쪽으로인접한칸을하나씩추가하는방법으로생각하면최대 1+7+7=15개. 91. 11명의수학자들과그남편들이함께회의에참석했다. 때때로남편들은회의장에서서로를지나치는데, 한번서로지나친짝은다시는서로지나치지않는다. 서로지나칠때에는, 한사람만다른사람을알아보거나, 서로알아보거나, 아니면서로아예못알아본다. 한사람이알아보는경우를 `목격 ' 이라하고, 서로알아보는경우에는 `잡담 ' 이라고하자. 잡담일경우에는서로잠깐서서얘기를나눈다. 잡담은두번의목격으로도세어진다.61번의목격이있었다면, 남편중한명은적어도 2번의잡담을나누었다는것을증명하여라. (IMTS R20-3) 증명목격이면 A! B를, 잡담이면 A! B 와 B! A 를둘다그리는 graph를생각하자. 그럼이런 directed edge가 61개. 잡담이 1번이하인남자는나가는변의개수와들어오는변의개수의합이 11개이하. 그럼변은모두 11 11 =60:5 2 개이하. 모순. 92. 홀에는 100 명의사람들이있고, 이들각각은적어도 66 명의다른사람들을알고있다. 서로아는네명이반드시존재한다고확신할수없음을증명하여라.[A 가 B 를알면 B 도 A 를안다.] ( 폴란드 1967 3 차 -2) 증명 33, 33, 34 명의세그룹으로분할한후, 서로다른그룹끼리만항상아는것으로하면됨. 93. 홀에는 100 명의사람들이있고, 이들각각은적어도 67 명의다른사람들을알고있다. 서로를모두알고있는 4 명이존재함을보여라. ( 폴란드 1967 3 차 -4) 증명각자가모르는사람은많아야 32명. a가아는사람이적어도 67명, 이집합을 B. B의한사람 b는 B 안에서많아야 32명을모르므로 B 안에서적어도 34명을안다. 이 34명의집합을 C. C의한사람 c는 C 안에서적어도한명 D를아니까,ABCD가그네사람. 94. 현재미국에서는 1억 2천만대의전화가사용되고있다고가정하자. 모든전화기에 10자리의전화번호 ( 각자릿수는 0부터 9까지사용 ) 를배정하는데, 한자리만을실수로틀리게걸었을때는항상이것을알아내고수정할수있도록할수있는가?( 예를들어, 812-877-2917 번호의전화기로전화를거는데, 812-872- 2917이라고잘못걸었을때, 812-872-2917과한자리만다른번호는 812-877-2917 이외에존재하지않도록배정해야한다.) (IMTS R22-3) 증명 ABC-DEF-GHIJ가사용하는번호일때 xbc-def-ghij, AxC-DEF-GHIJ,... 등은모두사용할수없고또ABC-DEF-GHIJ에만소속되어야한다. 즉, 사용번호하나당사용될수없는번호 90개가함께배정되고, 그럼필요한번호는 1억2천만 91개인데이것은 10자리번호의개수 10 1 0을넘으므로모순. 95. 1999 1999 표에서몇개의칸을칠했는데, 모든행과열마다색칠된칸이딱하나있다. 1000 1000 크기의부분영역에는색칠된칸이항상적어도하나포함됨을증명하여라. ( 몰도바 1999 최종-y8/9-6) 증명그영역에색칠된칸이없다면그영역과같은행의나머지칸들을모은 1000 999 의각행마다색칠된칸이하나씩있어야하는데그 1000개의칸이 999개의열에있을수없... 96. 한반에16명의학생이있다. 매월선생님은학생들을두그룹으로나눈다. 선생님이항상같은그룹에있었던두학생이없도록하고싶다면최소몇달이지나야하는가? ( 러시아 1994 4차-y9-8) 풀이 1달후어느8명은같은그룹. 2달째후그 8명중어느4명은아직여전히계속같은그룹. 3달째후그4명중어느2명은아직여전히계속같은그룹. 따라서, 적어도 4달은필요. 실제 4달이면충분함 : 각학생의번호를 2진법으로쓴후 k달째에 2 k 1 의자리가같은학생들끼리한그룹이되도록하면됨. 답 4달
218 조합 97. A 는집합 f1; 2;:::;100g 에서 50 개의수를골라만든부분집합이다. A 의어떤두원소도합이 100 이아니라면 A 는완전제곱수를적어도하나포함함을증명하여라. ( 인도지역예선 1996-7) 증명 36, 64 의짝과 100 이핵심. 비둘기집약간비틀기ㅎ 98. 탁자위에성냥개비세더미가있다. 한더미는성냥100개, 다른한더미는성냥 200개, 또다른한더미는성냥300개로되어있다. 두명이번갈아가면서한더미를골라서그더미를두개의더미로나누는게임을한다 ( 나눈더미는비어있을수없다 ). 더이상나눌수있는더미가없는사람이진다. 먼저하는사람과나중에하는사람중누가필승법을갖겠는가? ( 러시아 1994 최종-y9-3) 풀이먼저하는사람이 300 개더미를 100 개,200 개로나눈후대칭플레이. 99. 이웃한두자연수의비가항상소수가되도록, 서로다른 1995개의자연수를한원의둘레위에적을수있는가? ( 러시아 1995 4차-y9-2) 풀이소인수하나를추가하거나제외하며원을돈후자기자신으로돌아와야하는데,1995가홀수라서곤란함. 홀짝성. 혹은곱셈을먼저다하고나눗셈을나중으로몰아서해도되는데, 그럼곱셈횟수와나눗셈횟수가같아야하니까. 100. N N 개의칸배열에서둘레의 4(N 1) 개의칸을생각하자. 이칸들에 4(N 1) 개의연속한 ( 꼭양수일필요는없는 ) 정수들을하나씩써넣으려고한다. 대각선에평행한변을갖는직사각형에네꼭지점의위치에있는수를합하면그값은항상일정해야한다. 대각선자신도양끝점에있는두수의합이역시같은값이어야한다. 이것이가능한지다음의경우에대해답하여라. (a) N =3 (b) N =4 (c) N =5 (Towns 1984 봄 JO3) 풀이 (a) 와 (c) 는가능함.(a) 는 1...8 을,(c) 는 0...15 를이용하면됨.(b) 는불가능함.(n 5) + + (n +6)=12n +6 은 4 의배수가아니라서. 101. n 2 명의학생으로구성된어떤학급이있다. 이학급에서는매주마다한번씩모든학생들이참가하는퀴즈대회가열린다. 담임선생님은학생들을 n 명씩 n 개팀으로편성하는데, 어느한주에같은팀에있었던어떤두학생도다른주에다시같은팀에속하지않는방식으로가능한한오랜기간이되도록편성하려고한다. 이기간이 n +2 주보다짧음을증명하여라. ( 아일랜드 1995-1) 증명 n2 1 n 1 = n +1<n+2. 102. 한변의길이가 n 인정사각형꼴의종이를 n 2 개의단위칸으로분할하였다. 일부이웃한두칸의사이에는단위길이의벽들을그려미로를만드는데, 임의의한칸에서출발하여벽을가로지르지않고이미로를따라임의의다른칸으로항상갈수있도록한다. 가능한벽의총길이는최대얼마인가? ( 이탈리아 1987-7) 풀이나무문제네. connected graph가되려면벽을가로지로는변의개수는최소몇개냐하는.2n(n 1) (n 2 1) = (n 1) 2 개가최대. 103. N 은 n 자리의자연수이고,(a) 모든자릿수가다다르며,(b) 연속한세자릿수의합은항상 5 의배수가된다. n 6 임을증명하여라. 또한, 최고자리의숫자를무엇으로시작하더라도항상이런 6 자리수를찾을수있음을증명하여라. ( 인도지역예선 1996-4) 증명 a 1 a 4 a 7 (mod 5) 이므로 a 7 까지존재할수없음. 항상찾을수있다는것은 (0; 1; 4) 짝과 (0; 2; 3) 짝을첫자리가 0, 1, 2, 3, 4 중무엇이냐에따라적당히돌려쓰면됨 104. (a) 0에서 9까지의 10개의자연수를한원주위에배열하는데인접한두자연수의차가항상 3, 4, 5 중하나가되게할수있는가?( 예를들어서 0에서 6까지의수는 0, 3, 6, 2, 5, 1, 4 순서로배열하면가능하다.) (b) 0에서 13까지의 14개의수로한다면어떤가? ( 소련 1967-4)
4.1 조합중급문제 219 풀이 (a) A = f0; 1; 2g, B = f3; 4; 5; 6g, C = f7; 8; 9g 로두자. 집합의 A의수옆에는항상 B의수. 따라서, AB꼴의변이 6개. CB꼴의변도 6개. 그러나 B는변에8번만사용될수있으므로모순.(b) 가능하다.251403610139128117 105. N N 체스판에 N 2 개의말 (` 마 (knight)' 로생각한다 ) 이놓여있다. 서로잡히는위치에있었던말들이항상서로이웃한 ( 적어도한점에서만나는 ) 위치의칸에있게되도록말을재배열할수있는가? (a) N =3일때풀어라. (b) N =8일때풀어라. (Towns 1983가을 J5) 풀이 (a) 가능함. 123/456/789 를 167/852/349 로재배열하면됨.(b) 불가능함. 먼저순서대로번호를매겼을때,C3과서로잡는위치는8곳( 그집합을A라하자 ) 이므로그 8곳이 C3의이웃으로옮겨져야함 ( 게다가한칸의이웃을최대 8칸이므로 A를제외한녀석이 C3의이웃으로올수없다 ). 그 A 중 E4를생각하면,E4와서로잡는위치에있었던 8곳도모두 E4의이웃으로옮겨져야함. 그럼 C3와 E4의새로운공통이웃이 ( 적어도 2개 ) 생기는데, 옮기기전에 C3와 E4의공통잡는위치는없었으므로모순. 106. 단위정사각형칸으로분할된무한히펼쳐진종이가있다. 두칸사이의`거리 ' 란가로세로로움직여두칸사이를연결하는경로의최단길이 ( 칸의중심에서중심까지 ) 를말한다. 거리가 6인칸들은항상서로다른색이되도록모든칸을색칠한다면, 필요한색의개수는최소몇개인가? 그개수로색칠하는예를들고, 더적은개수로는불가능함을증명하여라. (Towns 1984봄 JO5) 증명한칸을기준으로상하좌우 3의거리에있는4개의칸을생각하면, 이 4개의칸은서로거리 6에있는칸들이므로 K 4 가구성되어모두색이다름. 즉 4색이상이필요함.4색으로는실제로간단히구성가능함. 107. (1) 차가 2이거나 5인두자연수는항상서로다른집합에속하도록, 자연수전체를세부분집합으로분할할수있음을증명하여라. (2) 차가 2, 3, 또는 5인두자연수는항상서로다른집합에속하도록, 자연수전체를네부분집합으로분할할수있음을증명하여라. 한편, 세부분집합으로분할하는것은곤란함을보여라. ( 아일랜드 1998-8) 증명 (1) mod 3으로분할 (2a) mod 4로분할.(2b) 세부분집합으로곤란하다는것은 fa; a +2;a+ 3;a+5g 를고려해보면 a +2와 a +3은항상같은집합에있어야함을말할수있어서... 108. n 4 인볼록n각형을생각하자. 이다각형의꼭지점들로이루어진삼각형들로이다각형을분할하였다. 원래다각형의변을두개갖는삼각형들은검은색으로칠하고, 하나만갖는삼각형들은붉은색으로칠하며, 다각형의변을하나도갖지않는삼각형들은흰색으로칠하였다. 하얀삼각형보다검은삼각형이두개더많음을증명하여라. ( 주니어발칸 2004-4) 증명삼각형은총 n 2 개. 검정, 빨강, 하양삼각형의개수를각각 a, b, c 라하면 a + b + c = n 2. 각삼각형이갖는 n 각형의변의개수를모두합하면 2a + b = n. 변변빼면 a c =2. 109. 5 명이상의사람이원탁에둘러앉아있을때, 모든사람이두명의새로운사람과이웃하여앉도록재배열할수있음을보여라. ( 폴란드 1968 3 차 -2) 증명일단 n 이홀수일때는 1; 3; 5;:::;n;2; 4;:::;n 1 로끝남. n 이짝수일때는 1; 3; 5;:::;n 1; 2;n;n 2;:::;4 로끝남. 별증 n명이라하자.2 a n 2 인 n과서로소인수 a가존재하면번호를 a만큼씩증가하며배열하면끝. 이런 a가존재하지않을때는 Á(n) =2일때이고, 이것을소인수분해와부등식으로적당히풀면 5 이상에서는 n =6뿐. n =6일때는123456을152463등으로재배열하면됨. 110. 쥐가한변의길이가 3 인정육면체모양의치즈를갉아먹고있다. 이치즈는단위정육면체 27 개로나누어져있고, 이쥐는다음의규칙에따라치즈를갉아먹는다 : 한코너에있는단위조각부터먹기시작해서, 그조각을다먹은후방금먹은단위조각과면으로이웃한다른단위조각을먹기시작한다. 이런과정을통해서모든치즈를다먹는데, 중심에있는단위조각치즈를마지막으로먹는것이가능한가? ( 유고슬라비아 1981 고 1-4)
220 조합 풀이이웃한칸을흑백이번갈아되도록색칠하면 ( 코너가흑 ), 흑칸이하나더많은데흑칸에서시작하여백칸으로끝날수있는지를묻는것이니. 111. 10000 개의칸으로이루어진 100 100 크기의마룻바닥을 1 3 크기의타일로깔려고한다. (1) 이마룻바닥의중앙에 2 2 크기의타일을하나깔면나머지바닥을1 3 타일들로잘깔수있음을증명하여라. (2) 마룻바닥의한코너에 2 2 크기의타일을하나깔면나머지바닥을잘깔수없음을증명하여라. ( 아일랜드 1999-4) 증명 (1) 중앙타일을제외한나머지영역을바람개비형태로 4개의합동인직사각형영역으로나누어보면끝. (2) ABCABC... 색칠이용 112. n 개의변을갖는다면체가존재하는자연수 n 을모두구하여라. ( 폴란드 1969 3 차 -6) 풀이 (1) k각뿔은 2k개의변을가지므로 6 이상의짝수는모두가능함. 한삼각형면에사면체를새로쌓으면 3개의변이늘어나므로,9이상의홀수도모두가능함.(2) 오일러의공식 e = v+f 2 4+4 2 =6 이므로 1, 2, 3, 4, 5는불가능. 이제 e =7일때만확인하면되는데, v 또는 f가 4가되어사면체밖에안되고그럼또수가맞지않으므로불가능. 113. 집합 f1; 2;:::;Ng 의공집합이아닌모든부분집합을생각하자. 각부분집합에서원소들의역수의곱을구해, 이것을모두합하면얼마가되는가? (Towns 1986 가을 J4) 풀이 (1 + 1 2 )(1 + 1 3 ) (1 + 1 ) N 으로인수분해에다 telescoping... 114. 돌세더미가있다. 그중한더미에서다른두더미의돌의수의합만큼새로돌을제거하거나추가할수있다. 예를들어, [12,3,5] 에서 12 + 5 = 17 을두번째더미에추가하여 [12,20,5] 로만들수도있고, 3+5=8 을첫번째더미에서제거하여 [4,3,5] 로만들수도있다. [1993,199,19] 로부터시작하여이런시행만으로빈더미가하나생기도록할수있는가? (Towns 1993봄 SO4) 풀이계속셋다홀수라서불가능. 115. 한항공사가한나라의64개의마을을연결하는 2000개의양방향항로를운항한다. 어떤두도시사이에도이항공사의항로를통하여오갈수있음을증명하여라. ( 몰도바 2000 최종-y8-5) 증명가능한모든변의개수는 64 63 2 = 2016 이므로 K 64 에서오직 16 개의변만빠졌다. 그런데 64 점의그래프에 cut 이존재하려면 ( 두도시 A 와 B 가연결되지않았다면, 도시 A 에연결된모든도시의집합과, 나머지도시들의집합간에변이없어야하므로 )mink(64 k) =63 개이상의변이제거되어야한다. 116. 서로다른무게의 68개의동전이있다. 천칭을 100번만써서가장무거운동전과가장가벼운동전을찾아내는방법을구하여라. (Towns 1985봄 JO2) 풀이 34쌍으로분할하여각쌍을잰후 (34번) 무거울가능성의그룹 (H) 과가벼울가능성의그룹 (L) 으로 34:34로나눈다.H에있는34개들로승자승토너먼트를벌여가장무거운것을찾고 (33번), L에서도마찬가지로가장가벼운것을찾으면 (33번) 됨. 117. 한변의길이가l인정사각형꼴로생긴마을이있는데,3 3 형태의 9블록으로구성되어있다. 각블록은아스팔트도로로경계지어져있다. 이마을의한쪽구석에서출발하여아스팔트도로를따라마을을돌다처음의위치로돌아오려고한다. 모든아스팔트도로구간을각각한번이상지나가고자한다면움직여야하는최단거리는얼마가되는가? (Towns 1984가을 JO2) 풀이이순환로에서코너의각점은 1번들르면되고, 다른 12점은적어도두번씩은들러야하므로방문횟수는 1 4+2 12 = 28번이상. 즉, 달리는거리도 28 이상이고, 실제로이만큼만달리는길을구성할수있다. 답 28
4.1 조합중급문제 221 118. 먼바다의어떤섬에서사용되는언어의낱말은문자 a, b, c, d, e, f, g 로만구성된다고한다. 한낱말을다음과같은변형을통해다른낱말로만들수있을때, 그두낱말을동의어라고말하기로하자. (i) 다음과같은규칙으로한문자를두문자로대체시킬수있다 : a! bc; b! cd; c! de; d! ef; e! fg; f! ga; g! ab (ii) 한문자가다른똑같은두문자사이에끼어있으면그두문자는제거할수있다. 예를들면, df d! f. 이언어의모든낱말은동의어임을보여라. ( 중미 2007-4) 증명 a! bc! cdc! d 로대체되므로, 같은방법으로 a! d! g! c! f! b! e! a 로모든문자들은서로대체될수있다. 119. 15 n 크기의판을다음두가지모양의타일로잘깔수있는자연수 n 을모두구하여라. ( 중미 2000-2) 풀이 5 3atom이있으므로 5 j n 또는 3 j n 이면잘깔수있음. n =5A +3B꼴일때도잘깔림. 이런꼴이아닌수들은n =1; 2; 4; 7뿐인데, 첫행부터차근히따져보면이것들은다불가능. 120. 8 개의축구팀이풀리그를벌인다. 비기는경우는없고, 각두팀은꼭한번씩경기를한다. 대회가끝났을때,A 가 B, C, D 를이기고,B 가 C, D 를이기고,C 가 D 를이긴네팀 A, B, C, D 를항상찾을수있음을보여라. (Towns 1985 가을 J1) 증명 4팀이상을이긴팀 A가있다.A에게진 B, C, D, E를생각하면 B, C, D, E들끼리의경기에서 2팀이상을이긴 B가있다.B에게진 C, D를생각하면끝. 121. 한여행객이어떤나라를여행하고있다. 우선수도 A 에서출발하여비행기를타고거기서가장먼도시인 B 로간다. 다음 B 에서역시비행기를타고거기서가장먼도시인 C 로간다 (C 는 A 가될수없다고한다 ). 이런식으로계속여행한다면이여행객은다시수도로돌아오지는않게됨을증명하여라. ( 오클랜드 2001-7) 증명 A에서 B로갈때의이동거리를 d 1, B에서 C로갈때의이동거리를 d 2,... 이렇게다음도시로이동하는거리를차례로 d 1 ;d 2 ;d 3 ;::: 이라하자. B에서 A보다 C가더먼거리이므로 BA > BC, 즉 d 1 <d 2 이다. C에서다시가장먼도시로이동하므로 d 3 CB = d 2 이다. 이런식으로계속가장먼도시로이동하므로 d 1 <d 2 d 3 d 4 가된다. 따라서, 만일다시수도 A 로돌아오게된다면 d k >d 1 인데, 이것은 A 에서가장먼도시 B 보다더먼도시가있다는것이므로모순이다. 그러므로, 이여행객은수도로는되돌아오지않게된다. 122. 6 10 크기의직사각형꼴초콜렛을가지고두명이서게임을한다. 이초콜렛은금을따라서만잘린다. 먼저하는사람이한직선을따라자르고, 자른한쪽을먹어버린다. 두번째사람이남은조각을역시직선으로자르고한쪽을먹어버린다. 이런식으로번갈아계속하는데, 마지막한조각을남기는사람이승리한다. 완벽한게임이라면둘중에누가이길까? (Towns 1986 가을 J3) 풀이먼저하는사람이계속정사각형을만들면이김. 대칭성의게임. 123. 산책은동서남북중어느한방향일수있는길이 1 인걸음들의연속으로이루어져있다. 같은지점을두번지나지않는산책을 ` 자기회피적 ' 이라고한다. 원점에서출발하여 n 걸음동안자기회피적으로산책하는방법의수를 f(n) 이라고한다. f(1), f(2), f(3), f(4) 를구하고, 다음부등식을보여라. ( 캐나다 1979-5) 2 n <f(n) 4 3 n 1
222 조합 증명첫번째걸음은 4가지방법이있고, 두번째걸음부터는방금지나온지점으로다시돌아갈수없으므로많아야 3가지방법만있다. 그러므로 f(n) 4 3 n 1 의오른쪽부등식이확인된다. 그리고, 처음부터북쪽과동쪽만선택해서움직인다면같은지점을다시지나지않는 2 n 가지의산택을얻을수있다. 이외에서쪽으로계속가는산책도있으므로 f(n) > 2 n 의왼쪽부등식도확인된다. 124. 서로다른 1 n(n +1) 2 개의수들이아래와같은삼각형꼴로임의로배열되었다 :.... 위로부터 k 번째행에서가장큰수를 M k 라하자. M 1 <M 2 <M 3 < <M n 일확률을구하여라. ( 캐나다 1990-2) 풀이구하는확률을 P n 이라하자. 마지막행에가장큰수가들어갈확률은 n n(n+1) = 2 n+1 이고, 마 2 지막행에가장큰수를넣었으면그다음은 M 1 <M 2 < <M n 1 만만족하면되므로, P n = 2 n +1 P n 1 = 2 n +1 2 n P n 2 = = 2 n 1 (n +1)n 3 P 1 = 2 n (n +1)! 답 위와같은점화식풀이가가능하다. 125. 아홉개의정수 1; 2;:::;9 중에서균일한확률로임의의하나를고를수있는선택기가있다. 수를 n(> 1) 번고른후, 이 n 개의수의곱이 10 으로나누어떨어질확률을구하여라. ( 미국 1972-3) 풀이포함배제.1 ( 8 9 )n ( 5 9 )n +( 4 9 )n 126. A, B 와다른 2001 명의사람이원형으로둘러서있다.A 와 B 는서로이웃해있지않다.A 로부터시작하여둘이번갈아가며옆사람중한명을터치한다. 터치당한사람은그즉시대형에서떠나야한다. 상대방을터치한사람이승자가된다. 둘중한명에게필승의전략이있음을보이고그전략을구하여라. ( 중미 2001-1) 풀이 A 의전략 :A 와 B 사이 ( 좌우각각 ) 에홀수명씩있도록만든다. 대칭성 or 불변량. 127. 8 8 체스판의남서쪽구석에 9개의졸이 3 3꼴로놓여있다. 가로, 세로, 대각선의방향으로졸이다른졸하나를뛰어넘으며이동하는것이가능하다. 이졸들을 (a) 북서쪽구석으로 3 3꼴로옮겨놓을수있는가?(b) 북동쪽구석으로는가능한가? (Towns 1987봄 JA3) 풀이각졸의위치는x-좌표와 y-좌표모두홀짝이항상처음그대로유지된다. 좌상귀든우상귀든개수가맞지않아서불가능. 128. 똑같이생긴 n 개의양초가촛대에끼워져있다. i =1; 2;:::;n 에대해, i 번째날에는 i 개의양초를딱한시간동안불을밝힌다. n 번째날이지났을때 n 개의양초가모두다소모되었다고한다. 가능한 n 을모두구하여라. 그리고, 각각의 n 에대해, i 번째날에불을밝히는규칙을제시하여라. ( 스웨덴 1961-3) 풀이양초가탄총시간은 1+2+ + n = n(n+1) 2 이므로, 한초당 n+1 2 시간씩탔다. 따라서 n은홀수. 실제로 n이홀수일때,1+k번째날과 n k번째날을짝지어한날처럼불을밝히면됨. 129. 임의로주어진자연수 n에대해, 1 a + 1 b + 1 c = 1 을만족하는자연수해 (a; b; c) 는유한개뿐임을보여라. n ( 스웨덴 1967-3 변형 )
4.1 조합중급문제 223 증명 WLOG a b c 라할때, 1 n = 1 a + 1 b + 1 c 1 a + 1 a + 1 a = 3, a 즉 a 3n 이므로우선 a는유한가지. 각각의 a에대해서b도유한가지씩. 130. 2003 개의돌이한더미를이루고있다. 두사람이번갈아서이더미에있는돌의개수의어떤약수만큼돌을가져간다. 마지막돌을가져가는사람이지는것으로할때, 둘중한사람의필승의전략을구하여라. ( 중미 2003-1) 풀이을이항상홀수를만들어서넘기면이긴다. 131. 3 n 격자판이다음과같이채워져있다. 첫번째행에는 1 에서 n 까지의수가오름차순으로있고, 두번째행은첫행을적당한 i 에대해 i 번회전시킨 i +1;:::;n;1; 2;:::;i 1;i 꼴로되어있다. 세번째행에도 1 부터 n 까지의수가적당한순서로들어있는데, 각각의열의세수를합한값은모두같다고한다. 이런식으로수를배치하는것이가능한 n 은어떤값들인가? 그런모든 n 에대해, 가능한서로다른배치방법의수를각각구하여라. ( 아일랜드 2002-6) 풀이첫열부터둘째줄에 n이쓰인열까지의처음 n i개열을앞부분, 그이후의 i개열을뒷부분이라부르자. 앞부분의제3행에쓰인수들끼리는모두서로홀짝이같다. 뒷부분의제3행에쓰인수들끼리도모두서로홀짝이같다. 따라서, 제3행에서앞부분, 뒷부분이홀수전체와짝수전체를나눠가져야하며, 그럼 n이짝수이면 i = n n 1, n 2 이홀수이면 i = 2 일수밖에없다. 각각조사해보면 n일홀수일때의 2가지만가능하다. 132. 남북으로 m 개의길과동서로 n 개의길이난직사각형꼴의마을이있다. 어떤교차로에서출발하여이마을의길을따라다른모든교차로를딱한번씩지나처음의위치로돌아올수있었다. 이것이가능한 m; n > 1 을모두구하여라. ( 스웨덴 1972-2) 풀이 m 또는 n이짝수이면쉽게가능.( 회로는하나만잡으면그회로가어떤점을출발점으로하든다커버함 ) m, n이모두홀수이면체스판처럼흑백을번갈아칠하면항상흑백을번갈아지나야하는데, 회로라면흑백의칸수가같아야해서모순. 133. 한정육면체의여덟꼭지점을자신의꼭지점으로하는, 스스로와만나지않는공간팔각형이있다. 이공간팔각형이정육면체의모서리를적어도하나포함함을증명하여라. ( 레닌그라드 1987-16) 증명스스로와만나지않으므로주대각선은많아야하나, 각면의면대각선도많아야하나씩만있다. 그럼선분은최대 7 개뿐. 134. 흑백이번갈아칠해진 8 8 체스판이있다. 이체스판의두열을맞바꾸거나두행을맞바꿀수있다. 유한번의이런조작들만으로, 체스판의왼쪽절반은모두검은칸, 오른쪽절반은모두흰칸이되도록할수있는가? ( 레닌그라드 1987-25) 풀이각줄의검은칸의개수는항상 4 개로불변. 135. 두사람 A 와 B 가숫자 1, 2, 3, 4, 5 를사용하여 2005 자리의수 N 을만드는게임을한다. A 가먼저첫번째자릿수를택하고, 다음 B 가두번째자릿수를택하고, 그다음은 A 가, 그다음은 B 가, 이런식으로번갈아한다. N 이 9 의배수가되면 A 가이기는것으로한다. 두사람모두최선의경기를한다면누가이기겠는가? ( 아일랜드 2005-7) 풀이 B 가계속둘의두턴의합이 6 이되도록하면이김. 2004 자리까지딱 9 의배수가되기때문에, 한자리더하면 9 의배수가안됨. 136. 10 10 크기의판에서빨강과파랑, 두사람이번갈아다음과같은게임을한다. 파랑이먼저시작하고, 각자의차례에서는 ( 아직선택된적이없는 ) 한행이나열을택해그줄의모든칸을자신의색으로칠한다. 그줄에서이미색칠된칸들도새로운색으로덧칠된다.20 번의차례가지나모든행과열이한번씩선택되어색칠된후에는게임이끝난다. 빨간칸의개수가파란칸의개수보다 10 개이상많으면빨강이이기는것으로하고, 그렇지않으면파랑이이긴다. 두사람중누구에게필승의전략이있는지결정하고, 그전략을서술하여라. ( 중미 2005-4)
224 조합 풀이한칸은딱두번씩색칠됨. 한대각선위의칸을빨강이모두먹는다는전략 (i번째행or열을상대방이먹으면 i번째열or행을먹는다는전략 ) 으로하면빨강승리. 우상삼각형과좌하삼각형영역은서로대칭적으로반전된형태가됨. 137. 3 3 표에서 2 2 부분영역을택해그영역에속하는네수를 1 씩증가시킬수있다. 처음에 9 개의수가모두 0 이었다면, 이런조작을유한번하여다음을만들수있는가? ( 레닌그라드 1988-1) 4 9 5 10 18 12 6 13 7 풀이중앙은네코너칸의합과같아야함. 불변량. 별해체스판처럼흑백을번갈아칠하면흑칸의합과백칸의합이항상같아야함. 138. 8 8 크기의판에각행과각열에정확히 4 개씩의검은칸이있도록흰색과검은색으로모든칸을색칠하였다. 서로붙어있는흰칸쌍의개수와서로붙어있는검은칸쌍의개수가같음을보여라 ( 단, 서로붙어있다는것은한변을공유한다는것을뜻한다 ). ( 이탈리아 1991-4) 증명흰흰쌍 x개, 검검쌍 y개, 흰검쌍 z개라하자. 그럼흰칸과변이만나는횟수는 2x+y =4 32 4 4 이고검은칸과변이만나는횟수는 y +2z =4 32 4 4 로같다. 즉, x = z. ( 서로다른두가지관점에서세기 ) 139. A 와 B 는 jaj + jbj >n 1 을만족하는 f1; 2;:::;n 1g 의두부분집합이다. a + b = n 을만족하는 a 2 A 와 b 2 B 를찾을수있음을증명하여라. ( 헝가리 1953-1) 증명귀류법 : 한집합에수하나넣으면반대쪽집합에서는다른수하나가못들어감 ( 혹은 k, n k는 A, B 통틀어 2개밖에못들어감 ). 그럼 jaj + jbj n 1. 140. 단위정육면체의각면에다른단위정육면체를하나씩붙여만든,7 개의정육면체들로만들어진블록을 ` 크로스 ' 라고부르자. 무한히많은크로스블록으로공간전체를빈틈없이가득채울수있음을보여라. ( 체코슬로바키아 1986-3) 증명그냥착착맞물려쌓으면다음의타일을차곡차곡쌓은것으로볼수있음. OO OOO OO 이타일은평면을규칙적으로잘깔수있으니까끝 141. 어떤정수 N 이존재하여, n N 인모든정수 n 에대해한정사각형을더작은정사각형 n 개로항상쪼갤수있음을보여라. 증명 ( 이탈리아 1994-1) n 이되면 n +3 도되므로, n =6; 7; 8 일때하면끝. 142. fa; b; cg 로구성된길이 n 의낱말중에서 a 를짝수개포함하는것의개수를구하여라. ( 이탈리아 1996-4) 풀이 x n+1 =2x n +(3 n x n )=x n +3 n, x 1 =2 의점화식. 143. 공간에있는 n 개의점을모두빨간색혹은파란색선분으로연결하였다고할때, 자신으로부터그어져나간빨간색선분의개수가같은두점이존재함을보여라. 단, n 2 이고어떤세점도같은직선위에있지않다고한다. ( 한국 1992-4) 증명유명한악수문제그자체네. 악수한횟수가다다르다면0부터 n 1까지다한명씩가져야하는데,0인녀석과 n 1 녀석이서로모순. 144. n 개의서로다른양의실수의집합 fa 1 ;a 2 ;:::;a ng 이있다. 공집합이아닌모든가능한부분집합에대해그원소의합을계산한다. 적어도 n(n +1)=2 개의서로다른값 ( 합 ) 이있음을증명하여라. ( 소련 1963-8)
4.2 조합고급문제 225 증명 a 1 <a 2 < <a n 라할때, a 1 <a 2 < <a n (n sums), a 1 + a n <a 2 + a n < <a n 1 + a n (n 1sums), a 1 + a n 1 + a n <a 2 + a n 1 + a n < <a n 2 + a n 1 + a n :::; a 1 + a 2 + + a n (1 sum). (n 2sums), 145. 1부터 100까지의자연수들이한원의둘레에나열되어있다. 이웃한두수의합들중에서차이가 2보다큰것이반드시존재하는지증명하거나반증하여라. ( 몰도바 1999 최종-y10-3) 증명반례가있음. 100 2 98 4 96... 46 54 48 52 50; 51 49 53 47... 97 3 99 1 로하면이웃한두수의합은항상100, 101, 102 중하나. 146. 반지름 r인 7개의원판이반지름이 1인원판을덮고있다. r 1 2 임을증명하여라. (Towns 1987봄 SA2) 증명정육각형의여섯꼭지점과중심까지 7개의점을생각하면, 반지름이 1 2 보다작은원은두점을동시에덮을수없음. 147. 정육면체의각꼭지점에 1 또는 1 이부여되어있다. 또한각면에는, 그면의네꼭지점에부여되어있는수들의곱이부여된다. 이렇게부여된 14 개의수전부의가능한합을모두구하여라. ( 남미 1991-1) 풀이꼭지점의부호하나를바꾸면네수의부호가바뀌므로 mod 4로일정함. 모두 1일때최대의경우로 14이고, 14, 10, 6, 2, 2, 6, 10은모두가능함. 14는모두 1이어야하는데이것은불가능. 148. a 1 ;a 2 ;a 3 ;a 4 ;a 5 ;a 6 ;a 7 과 b 1 ;b 2 ;b 3 ;b 4 ;b 5 ;b 6 ;b 7 은각각 1, 2, 3, 4, 5, 6, 7 을재배열한것이다. ja 1 b 1 j, ja 2 b 2 j, ja 3 b 3 j, ja 4 b 4 j, ja 5 b 5 j, ja 6 b 6 j, ja 7 b 7 j 들이모두서로다를수는없음을보여라. ( 스웨덴 1980-2) 증명 ja 1 b 1 j + + ja 7 b 7 j (a 1 + + a 7 )+(b 1 + + b 7 ) 0(mod2). 그런데이들이모두다르다면 0부터 6까지딱한번씩나와야하므로 0+1+ +6=21은홀수가되어모순. 149. 어떤농구대회에서모든두참가팀사이에두번씩의경기가있다. 경기에서이기면 2 점을얻고, 지면 0 점을얻으며, 비기는일은없다. 우승팀이 26 점을얻었고, 꼴찌팀은딱두팀이공동으로 20 점을얻었다고할때, 참가팀의수를구하여라. ( 이탈리아 2001-2) 풀이나머지팀의점수는 22 또는 24. 26 + 20 2+22(n 3) 2n(n 1) 26 + 20 2+24(n 3). 정리하면 12n n 2 13n 3. 답 12 150. kn 명의사람이있다 (k, n 은자연수 ). 각각의사람은자기이외에 (k 1)n 명넘게안다. 서로모두아는 k +1 명이존재함을증명하여라.( 단, A 가 B 를알면 B 도 A 를안다.) ( 폴란드 1995/1996 1 차 -3) 증명각자는모르는사람이많아야 n 2명. A 1 이모르는 n 2명을제외시키고남은집합을 X 1. X 1 에서 A 2 를골라, A 2 가모르는n 2명을제외시키고남은집합을 X 2. 이런식으로계속하여서로아는 A 1 ;A 2 ;:::;A k 를찾을수있고, 그때까지 k(n 2) 명이제외되었고 k명이선택되었으므로 k(n 2) + k<kn이라서 X k 에남은인원이있으므로그중아무나 A k+1 로하면된다. 4.2 조합고급문제 1. (a) 4 7 체스판의각각의칸을검은색이나흰색으로칠하자. 어떻게칠해도네코너의칸이모두같은색인직사각형 ( 이체스판의가로선과세로선으로이루어진 ) 이항상생기게됨을증명하여라. (b) 4 6 체스판에서는그런직사각형이하나도없도록칠할수있음을보여라. ( 미국 1976-1)
226 조합 증명 (a) 귀류법으로풀자.WLOG첫행에검은칸이넷이상이라하자. 그검은칸을갖는네열만생각할때, 제2행, 제3행각각에흰칸이셋이상있다 ( 검은칸이둘이상있으면검은직사각형생김 ). 그럼이제2행의세흰칸과제3행의세흰칸중에서로겹치는 ( 같은열에있는 ) 짝이둘이상있다. 그게흰직사각형을이루므로끝 ( 사실상 7 3 만으로도충분함 ). (b) 위와같은예가있다. 2. A 가임의의 n 자리자연수이면적당한연속된몇자릿수의곱이완전제곱수가되게할수있다고한다 ( 예 : 7323658 에서라면 2 3 6=6 2 ). 최소의 n 을구하여라. (1994 겨울학교모의고사 ) 풀이최소의 n은 16이다. 우선 15일때의반례는232523272325232 등이있다. 이수의경우만일7을포함하면 7 2 의배수는아니므로곤란하고,7을포함하지않으면양쪽중어느한쪽 2325232의부분이므로비슷한논리로 5를포함하면안되고, 그럼 232의부분이므로 3을포함하면안되고, 그럼 2라서결 ky 국모두안된다. n 16 일때는, p k = a i =2 x k 3 y k 5 z k 7 w k 로정의할때,(x k ;y k ;z k ;w k ) 는 mod i=1 2로 16종류이므로 (0; 0; 0; 0) 인것이있거나아니면같은종류인둘이있다. 전자이면그자체의 p k 가완전제곱수이고, 후자이면 (x k ;y k ;z k ;w k ) (x j ;y j ;z j ;w j )(mod2) 라할때 (k <j) p jy j = a i 가 p k i=k+1 완전제곱수이다. 3. A 와 B 가자연수가하나씩적힌카드 2n 장을가지고게임을한다. 이카드를섞어숫자가위로보이도록한행으로늘어놓는다.A 가먼저시작하여두사람이번갈아차례를갖는데, 자기차례에서는행의양끝중어느하나의카드를빼내어갖는다.B 가마지막카드를집으면게임이끝나고, 두사람은자기가가진카드의숫자를합해점수를계산한다.A 가최선을다하면 A 의점수가언제나 B 의점수이상이됨을증명하여라. ( 영국 2003/2004 1 차 -3) 증명 ( 대원외국어고 3학년최일규 ) 줄에카드들을왼쪽부터 1, 2, 1, 2, ::: 이런식으로번호를매긴다.1끼리더하고 2끼리더하여큰것의카드들을잡는방법이있으면언제나이긴다.1들의합이 2들의합보다크다고가정하자. 1, 2, 1, 2, :::; 1, 2 의상태에서1을취한다. 그러면 2, 1, 2, :::; 1, 2 의상태가되는데상대방은 2을취할수밖에없다. 그러면 1, 2, :::; 1, 2 혹은그거꾸로의상태가되는데나는다시 1을취한다. 이런식으로하면나는 1들만취하게되고, 상대는 2들만취하게된다.1들의합이 2들의합보다크므로내가이겼다. 4. n 2 명의선수가참가하고 k 일동안계속되는시합이열렸다. 매일선수들은 1; 2; 3;:::;n 중하나의점수를받는데어느두선수도같은점수를받지않는다. k 일이모두지나서보니모든선수가똑같이총 26 점씩을받았다. 이것이가능한모든순서쌍 (n; k) 를구하여라. ( 캐나다 1990-1) 풀이매일 n(n+1) 2 의점수가나간다. k일이지나서 n명의모든선수가 26점씩받았다고하였으므로 n(n +1) k =26n 2 즉, k(n +1)=52 이다. k와 n +1은양의정수이고, 문제의조건에서 n +1 3, 또시합이하루뿐이었다면모두같은점수를받을수없으므로, 가능한약수의쌍은 (k; n +1)=(2; 26); (4; 13); (13; 4) 뿐이다. 즉,(n; k) =(25; 2); (12; 4); (3; 13) 이고, (n; k) =(25; 2) 1일째 : 1 2 24 25 2일째 : 25 24 2 1 (n; k) =(12; 4) 1; 2일째 : 1 2 11 12 3; 4일째 : 12 11 2 1 (n; k) =(3; 13) 1일째 : 1 2 3 2일째 : 2 3 1 3일째 : 3 1 2 4일째 : 1 2 3 5일째 : 3 2 1.
4.2 조합고급문제 227 이들각각은실제로조건을만족하는점수배정이위와같이가능하다. 답 (25; 2); (12; 4); (3; 13) 5. 6 명의학생이있고각각은서로알거나서로모른다 (A 가 B 를아는데 B 는 A 를모르는경우는없다 ). 그럼서로서로다알거나서로서로다모르는 3 명이항상있게된다. 이사실이맞다는가정하에서,6 명중에서로다알거나서로다모르는 3 명을찾을수있는방법이사실은항상적어도 2 가지씩있음을보여라. 그리고,3 가지를찾는것은불가능할수있음을보여라. ( 셈본중등고급도전문제 1.1.2) 증명 (1) 앞의문제에의해, 일반성을잃지않고, A, B, C, D, E, F 의 6명중에세명A, B, C가서로안다고하자. 귀류법으로, 서로다알거나서로다모르는 3명의관계가더존재하지않는다고해보자. D, E, F 가서로다알지는못하므로그중에, 일반성을잃지않고, 두명D, E는서로모른다. 만일 A가 D와 E를모두모르면서로모르는 3명이생기므로, A는 D와 E 중적어도한명을안다. 마찬가지로 B와 C도각각D와 E 중적어도한명을안다. A, B, C의 3명이각각 D, E 둘중적어도한명을알고있으므로비둘기집의원리에의해 D와 E 둘중어느한명은A, B, C 중적어도 2명을안다. 그럼그 3명이서로아는관계가되어모순. 따라서, 서로다알거나서로다모르는 3명의관계는항상적어도 2가지씩은있다. (2) A, B, C가서로다알고, D, E, F 가서로다알고, 두그룹fA; B; Cg와 fd; E; F g 사이에는서로다모른다고하자. 그럼서로다알거나서로다모르는 3명의관계가정확히 2개뿐임을확인할수있다. 따라서, 그런관계를 3가지찾는것은불가능할수있다. 6. 임의의정수 11개가주어지면, 그중6개를뽑아그합이 6의배수가되게할수있음을보여라. 단, 주어진 11개의정수가모두서로다를필요는없다. ( 한국 2003-J4) 증명 ( 대전유성중 3년김도연 ) 일단 5개의원소중 3개를뽑으면그합이3의배수임을증명하자. 집합을 3으로나눈나머지가같은것끼리 3개로나누면 A = f3; 6; 9; 12; 15; 18; 21;:::g B = f1; 4; 7; 10; 13; 16; 19;:::g C = f2; 5; 8; 11; 14; 17; 20;:::g 이다. 비둘기집의원리로적어도한집합에는두개의원소가존재 1) 두원소가한집합에속해있고각각집합에서 1개씩 3개의원소를뽑으면합이 3의배수가된다. 2) 세집합중한집합에세개의원소가속해있어도합은3의배수가된다.: 증명끝! 그렇다면 5개의원소중 3개를뽑으면그합이3의배수가된다. 정수 11개가주어지는데, 이정수를a; b; c; d; e; f; g; h; i; j; k라하자. 이중5개인 a; b; c; d; e를뽑아그합이 3의배수가되는수를 a; b; c라하자.( 그래도일반성을잃지는않는다.) a + b + c =3p (p 는정수 ) (1) 그리고, 나머지중에 5 수를뽑아 d; e; f; g; h 라하고, 그들중에서도합이 3 의배수가되는것을, d; e; f 라하면, d + e + f =3q (q 는정수 ) (2) 또, 다섯수를뽑아그들을 g; h; i; j; k 라하면, 그들중에서도합이 3 의배수가되는것이존재. 그것들을 g; h; i 라하면, g + h + i =3w (w 는정수 ) (3) 그런데, p; q; w 중비둘기집의원리로합이 2의배수인것이적어도한쌍존재! p + q =2t. (1) 과 (2) 를더하면, 그것을 p; q 라하면 a + b + c + d + e + f =3p +3q =3(p + q) =3(2t) =6t 결국임의의정수 11 개가주어지면그중 6 개를뽑아합이 6 의배수가될수있게할수있다. 7. 가로, 세로격자선이일정하게그려진무한한격자판의모든칸을검은색또는흰색으로칠하는데,6 개의칸으로구성된직사각형 (3 2 또는 2 3) 에는항상 2 개의검은칸이들어있도록하였다.9 11 직사각형에는몇개의검은칸이들어있을수있는가? ( 오클랜드 1998-7)
228 조합 풀이연속한 3칸 ( 일반성을잃지않고가로방향이라하자 ) 에 2개이상검은칸이면 ( 윗변에붙어있는경우에는그밑3칸은비어있고다시그밑 3칸은다시 2개검은칸이므로일반성을잃지않고변에붙어있지않은경우로생각할수있다 ) 그위아래의 6칸은모두비어있고, 그럼세로방향의 3 2 직사각형들을생각하면원래의 3칸이모두검은칸이어야해서모순이발생. 연속한 3칸이모두비어있을때도그아래 3칸은위와같으므로마찬가지. 따라서, 연속한 3칸에는늘검은칸이 1개씩. 그럼모두 33개. 8. 한원위의서로다른 9 점을연결하는 36 개의선분각각을빨강이나파랑으로색칠하였다. 이 9 점중임의의 3 점으로이루어지는삼각형각각이적어도하나의빨간변을가진다고하자. 그럼어떤네점이있어서그들사이를잇는 6 개의선분이모두빨강이됨을증명하여라. ( 캐나다 1976-8) 증명귀류법으로, 파란 K 3 도없고빨간K 4 도없다고하고모순을찾아보자.9점중한점을A라하자. A는파란변3개와빨간변 5개를갖는다는것을보이자. 만일파란변을 4개이상가지면, 파란 K 3 이없기위해이4점들은모두빨간변으로연결되어야하는데, 그럼빨간 K 4 가생겨서모순이다. 한편, A에파란변이 2개이하이면빨간변이 6개이상이고, 잘알려진대로이 6개의점중에빨간K 3 이있거나 ( 그래서 A와함께빨간K 4 를이루거나 ) 파란 K 3 이있고, 어느경우도곤란하므로역시모순이다. 그럼 A는정확히 5개의점과빨간변으로연결되고, 대칭적으로모든점의빨간변의차수가 5가된다. 그럼홀수개의점이홀수차수를가지므로빨간변과점이만나는것은모두홀수번세어지고, 이것은빨간변의개수의 2배여야하는데짝수가아니므로모순. 따라서, 문제의명제가참이다. 9. 지름이 5cm 인원안에 10 개의점이있다면어떤두점간의거리는 2cm 보다작음을보여라. ( 통신강좌 1991-2-41) 풀이원의중심부에지름 2 cm의작은원을하나그리고, 남은도넛꼴테두리부분을 8등분하자. 그럼모두 9개의영역이생기고,10개의점이있으니비둘기집의원리에의해어느한영역에두점이포함된다. 그두점사이의거리는 2 cm 이하이다. 10. 2m 개보다적은개수의원소를갖고있는집합인데, m 개의원소를아무렇게나택하면그합이나머지수들의합보다항상클때, 이집합을 m- 큰집합이라고부르자. (1) 원소의개수가 2m 1개인 m-큰집합의원소는모두양수임을보여라. (2) 일반적으로, 원소의개수가 n개인 m-큰집합에최대로속할수있는양이아닌수의개수를 k m (n) 이라고하자. k m (n) 을구하여라.( 단, m n<2m) (ML프로포절 144-1, 레닌그라드 1991-11 변형 ) (1) 의풀이 ( 성재중졸업예창완 ) 원소의개수가 2m 1 개인임의의 m- 큰집합의원소들을 a 1 a 2 a 3 a 2m 1 이라고하자.2m 1 개의원소를가지므로 m 개의원소를택하면나머지원소의개수는 m 1 개이다. 문제의조건은가장작은 m 개의원소들을택했을때그합이나머지큰원소들의합보다크다는얘기로바꿔도무방함을알수있다. 따라서, a 1 + a 2 + + a m >a m+1 + a m+2 + + a 2m 1 좌변에서 a 1 을제외한모든원소들을우변으로이항시키면 a 1 > (a m+1 a 2 )+(a m+2 a 3 )+ +(a 2m 1 a m) 에서우변의각항들이모두 0 이상이므로 a 1 은반드시양수이다. 비슷한방법으로, 혹은 a 1 이상의수들이므로, a 2 ;:::;a m 도모두양수임을알수있다. (2) 의풀이 ( 동국대한의대 05학번김규현 ) 앞에서 k m (2m 1) = 0 임을보였고, k m (m) =m 1 인것도쉽게알수있다. 여기서 k m(n) =2m 1 n 임을추측할수있다. 이제이것을증명해보자. 원소가 2m 2 개인 m-큰집합에서다음이성립한다. a 1 + a 2 > (a m+1 a 3 )+(a m+2 a 4 )+ +(a 2m 2 a m ) 0
4.2 조합고급문제 229 만일 a 1 0 이면 a 2 > 0 이므로양이아닌수의개수는최대 1 개이다. 일반화시켜생각해보면 2m k 개의원소를갖는 m- 큰집합에서 (1 k m) a 1 + a 2 + + a k > (a m+1 a k+1 )+ +(a 2m k a m ) 0 이므로 a 1 ;a 2 ;:::;a k 1 0 이면 a k > 0 이어서양이아닌수는기껏해야 k 1 개가된다. 또한실제로, k 1 개의 0 과 2m 2k +1 개의 1 로이루어진집합은, m 개의수의합이최소 m k +1 이고나머지수의합이최대 m k 이므로 m- 큰집합임을알수있다. 따라서, k 1 개가정확한최대값이다. 즉, k m (n) =2m 1 n 이성립한다. 별해 ( 미국 Cornell 대학교수학과김현규 ) (1) 양이아닌수가있다고하고, 최소의원소를 m이라하자 (m 0). 나머지 2m 2 개의원소를둘로나눠서한쪽의합을A, 다른쪽의합을 B라하자. A +( m) >B, B +( m) >A가성립하므로, 두식을연립하면m<0 으로모순이된다. (2) n =2m p 라하자 (1 p m). 가장큰 m p개의원소를택해그합을 S, 그다음큰m p개의원소를택해그합을 T, 나머지 p개의원소의합을 U라하자. m-큰집합이므로 T + U>S, 즉 U>S T 0 이므로, U의모든원소가0 이하일수는없고양수가적어도하나는있다. 따라서, k m (n) p 1. 그런데 0이 p 1개있고나머지2m 2p +1개가모두 1인집합에대해계산을해보면 m-큰집합이됨을확인할수있으므로, 등호가성립함을알수있다. 즉, k m (n) =2m n 1. 11. 어떤금속막대기를 1001등분하는 1000개의각점에빨간구슬을박고, 1002등분하는 1001개의각점에파란구슬을박은다음, 이막대기를 2003등분하여잘라내었다. 이렇게만들어진 2003개의토막중구슬이박혀있는토막의개수를구하여라. 단, 구슬은부피가없는점으로간주한다. ( 한국 2002-J8) 풀이 막대기의길이를 2003 이라하자.2003 개의구간 I n =[n 1;n] (n =1; 2;:::;2003) 이잘린막대기에해당한다. 빨간구슬, 파란구슬은각각 2003 2003 i, j 의위치에해당하는데, 1001 1002 2003i 1001 =2i + i 1001 2 I 2i+1; 2003j 1002 =2j j 1002 2 I 2j 이다. 즉, 빨간구슬은 I 3 ;I 5 ; ;I 2001 의홀수구간에하나씩, 파란구슬은 I 2 ;I 4 ; ;I 2002 의짝수구간에하나씩박혀있다. 따라서, 구슬이박혀있는토막은 I 1 과 I 2003 을제외한 2001 개이다. 12. N 이양의정수들의집합일때, 다음세조건을만족하는함수 f: N! N 가존재할까? 있다면그중하나를찾고, 없다면그것을증명하여라. (1) f(2000) = 1999 이고, (2) 임의의소수 p에대해, 수열 f(p);f(2p);f(3p);:::;f(kp);::: 에는모든소수가각각무한히많이나타나며, (3) 임의의양의정수 n에대해f(f(n)) = 1 이항상성립한다. (1998 대전. 충남영재수학교실중급반 / 고급반평가시험 ) 풀이합성수는소수에대응시키고, 소수는 1에대응시키는함수라면조건 (3) 은해결된다. 이런함수중에서 (2) 를만족시키는것을찾을수있으면,(1) 의조건을추가하는것은간단하다. 서로다른소수 p; q와 2 이상의정수 t에대해f(pq t )=q 로정하고, 나머지수들에대해서는 f(2000) = 1999 를제외하고는모두 f(n) =1로하자. 그럼세조건이모두만족함을쉽게알수있다. 13. 다섯점을둘씩모두이은완전그래프 K 5 는평면에는그릴수없지만도너츠모양의면위에서는그릴수있다. 그렇다면평면에는그릴수없지만구면에는그릴수있는그래프가존재하는가? ( 셈본중등중급도전문제 5.3.2) 풀이존재하지않는다. 즉, 구면에그릴수있는그래프는모두평면위에그릴수있다. 구면에어떤그래프를그렸다하면그그래프는최소한개의면을가지고있다. 그러면그면내부의점을하나잡고그점에구멍을뚫은후그구멍을벌려서평면을만들수있다. 과정에서그래프의연결상태에는전혀변화를주지않았으므로구면에그릴수있는모든그래프는평면에도그릴수있다.
230 조합 14. 3 n 3 n 체스판을다음두모양의타일로덮으려고한다 (n 1). Α 형 Β 형 (a) 딱한칸만제외하고완전히덮는것이가능함을보여라. (b) 위와같이깔때,B형타일은최소몇개를써야하는가? ( 통신강좌 1995-10-46) 풀이 (a) 그림 1 일단 n =1 일경우성립함을보자. 그림 2 그림 3 위의그림과같이 n =1 일때는성립한다. 그리고, n =2 일때는그림 1, 2, 3 을참고하면아래처럼가능하다. 1 1 1 1 1 1 1 1 2 1 1 빈칸이들어있는칸은 n =1 일때처럼채우면된다. n 3 일때는다음을생각한다. 빈칸이들어있는 9 9 부분을제외하고는각 9 9 를그림처럼채운다. 그리고빈칸이들어있는부분은 n 1 일때성립하였으므로채울수있다. 즉모든 n 1 에대해완전히덮는것이가능하다. 99 3 2 9 1
4.2 조합고급문제 231 (b) 채울칸수는 (3 n ) 2 1 개이다. A 형으로만채워지려면채울칸수가 3 의배수여야하는데그렇지않다. 따라서 B 형은적어도 1 개는사용되어야한다. 한편 (a) 에서나타낸방법을쓰면 B 형은 1 개만사용된다. 따라서 B 형을사용하는갯수의최소값은 1 이다. 15. 1 에서 9 까지의자연수의카드가있다. 두사람이번갈아가면서카드를하나씩가져간다. 카드세장으로 15 를먼저만드는사람이이긴다. 첫사람이최선의방법을이용할때두번째사람은결코이길수없음을보여라. (ML 프로포절 136-4) 증명 1 에서 9 까지중세개의수의합으로 15 를만드는경우는다음의 3 3 마방진에모두나온다. 8 1 6 3 5 7 4 9 2 따라서, 위의문제는이 3 3 마방진에서두사람이 O, X 를번갈아그리며 Tic-Tac-Toe 게임을하는것과같다. 그리고, 잘알려져있다시피, 이 Tic-Tac-Toe 게임은두사람이최선의방법을사용하면서로비기게되어있다. 16. 정육면체의각꼭지점에 1부터 8까지의숫자를하나씩쓰고, 각각의모서리에는양끝꼭지점의숫자를더한값을적는다. 모서리의수들이모두다르도록할수있는가? ( 플란더즈 2002-J3) 풀이 1;:::;8에서둘을합하여만들수있는수는 3;:::;15의 13가지가있다. 정육면체에는모두 12개의모서리가있으므로, 모서리의수들이모두다르다면 3;:::;15 중딱하나를제외한모든합이다나타나야한다 ( ). 3, 4, 5, 6이모두나타난다고하자. 그럼 3=1+2, 4=1+3 이므로 1점은 2점,3점과모두이웃한다. 그럼 2점과 3점은서로이웃하지않으므로 5=1+4로나타나야하고,1점은 4점과도이웃한다. 그럼 2점과 4점도서로이웃하지않으므로 6=1+5로나타나야하는데,1점은이미 2, 3, 4점과이웃하여더이상다른점과이웃할수없으므로모순. 따라서,3,4,5,6중모서리에나타나지않는수가적어도하나있다. 대칭적으로, 12, 13, 14, 15 중에도모서리에나타나지않는수가적어도하나있고, 그럼적어도 2개의수가모서리에나타나지않아야하므로 ( ) 에모순이다. 따라서, 모서리의수들이모두다르도록할수없다. 주 한꼭지점은 3 개의모서리와이웃하므로, 모든모서리의수를다합하면 (3 + +15) a =3(1+ +8) 즉, 모서리에나타나지않아야하는수는 a =9 라는것도알아낼수는있다. 17. 4 개이상의짝수개의구슬로이루어진목걸이가있다. 각구슬은빨강, 파랑, 혹은녹색이고파랑구슬과녹색구슬의수는같다. 이목걸이에서두곳을잘라두줄로나누는데, 두줄의구슬이각각짝수개이고각각같은수의파랑구슬과녹색구슬을갖도록할수는없다고한다. 이목걸이의빨강구슬의개수로가능한값을모두찾아라. ( 호주 2004-7) 풀이 ( 대전과학고 2학년황윤하 ) 빨강, 파랑, 녹색의구슬을각각 R, B, G로표시하고, 그개수가각각 2m, n, n개라고하자.b와g가연속해있는경우가있으면이 BG를잘라내면두줄각각같은수의 B와 G를갖게된다. 또,RR인경우가있어도이것을잘라내면된다. 따라서,BG도RR도없어야한다 ( ). 두구슬X와Y 사이에 R들뿐이면 X와 Y가이웃하다고말하기로하자. n =0이면 RR이반드시있으므로 n 1 이다. 어느 B에서출발하여 G를만날때까지계속간다고생각하면 B와 G가이웃한경우가반드시있다. 그럼이이웃한 B와 G 사이에는 ( ) 에의해R이꼭하나있어야한다. 즉,BRG가반드시있다. 이양옆어딘가에 R이있으면그것까지잘라내면또두줄각각같은수의 B와 G를갖게된다. 따라서, 이양옆에 R이올수없고, 또 BG가생겨서도안되므로 BBRGG꼴이어야한다. 이렇게계속하면가능한경우는 BB B R R GG G 과같은경우만가능하다. 따라서, 빨강구슬의개수는 2개이다.
232 조합 18. 원탁에 10 명의손님자리가명찰과함께놓여있다.10 명의손님의명찰을확인하지않고아무렇게나않았더니제자리에앉은손님이한사람도없었다. 이때, 명찰이놓인원탁을적당히회전시키면적어도 2 명의손님이제자리에앉게됨을증명하여라. ( 한국 1989-1) 증명처음앉은자리에서원탁을한자리씩돌리면 9번돌리는동안에꼭한번만자기자리의명찰이돌아오고 10번째돌리면처음자리에되돌아온다. 처음에는아무도제자리에앉은사람이없고,9번돌리는동안제자리에앉게되는경우는 10번있으므로적어도한번은두사람이제자리에앉게되는경우가생긴다. 19. 양의실수로이루어진주어진순서쌍 (a; b; c; d) 를 (ab; bc; cd; da) 로변환한다. 이를임의로여러번반복한다. a = b = c = d =1 이아닐경우에절대로원래의순서쌍과같아지지않음을증명하여라. ( 소련 1961-5a) 증명 (a; b; c; d) 에서 (ab;bc;cd;da) 로변환할때, 네수의곱은abcd에서 (abcd) 2 으로변화한다. 따라서, 이곱은 (abcd) 2n 꼴이고, 원래의순서쌍과같아지기위해서는 abcd =1이어야한다. d =1=abc로두자. 그러면,(a; b; c; 1=abc)! (ab; bc; 1=ab; 1=bc) 가된다.(x; y; 1=x; 1=y) 꼴은변환하면 (xy; y=x; 1=xy; x=y) 로다시 (x 0 ;y 0 ; 1=x 0 ; 1=y 0 ) 꼴이되므로, 원래의순서쌍으로돌아오기위해서는 ac =1이어야한다. 즉, 순서쌍은 (a; b; 1=a; 1=b) 꼴이어야한다. 이제네수의합을살펴보자.(a; b; 1=a; 1=b) 는 (ab; b=a; 1=ab; a=b) 가되므로합은 a + b +1=a +1=b에서 ab+b=a+1=ab+a=b꼴로변화한다. 이식을정리하면 (a+1=a)(b+1=b) 꼴이된다. 그런데 a+1=a 2이고 b +1=b 2이므로 (a + 1 a )+(b + 1 b ) (a + 1 a )(b + 1 b ) 가성립한다 ( 등호는 a = b =1 일때만성립 ). 따라서 a = b =1 일때만같고그외에는항상합이증가한다. 즉, 원래의순서쌍과같아지기위해서는 a = b = c = d =1 이어야만한다. ( 부등식증명 ) x; y 2 이면 xy x y +1=(x 1)(y 1) 1 이므로 xy x + y 이다 ( 등호는 x = y =2 일때만성립한다 ). 20. n 개의서로다른소수가주어져있다. 그소수들로만소인수분해되는 n +1 개의정수가있을때, 이중적당히몇개를곱하여완전제곱수를만들수있음을보여라. ( 통신강좌 1996-13-27) 개요적당히몇개를곱해만들수있는수는2 n+1 1개. 각소인수의지수가홀이냐짝이냐로분류하면 2 n 가지. 비둘기집으로지수의홀짝이같은것이있고, 둘을곱한후중복된 ( 그래서제곱이된 ) 항을제거하면됨. 21. 한정육면체의모든꼭지점에 1부터 8까지의자연수를아무렇게나매겼다. 주대각선으로마주보는두꼭지점을정육면체의세변을이용해연결하는경로중에서, 이경로위에있는네꼭지점의수의합이 21 이상이되는경우가항상있음을증명하여라. ( 루마니아 2005 지역예선 y8-4) 증명 ( 대전대덕중 3 학년이선재 ) 7 과 8 의위치에따라경우를나눠보자. 위그림에서 `²' 이 7 과 8 이놓인위치이다. (1) 7 과 8 이이웃해있을때 : a 와 A 중큰것을택한다. 대칭적이므로, 일반성을잃지않고 A 가크다고하자. 다음 b 와 B 중큰것을택한다. 일반성을잃지않고 B 가크다고하자. 그럼 8+7+A + B 8+7+4+2=21 이된다.(A 와 B 중작은것은그보다더작은것이있으므로 2 이상이고, A 와 B 중큰것은네수 a, A, b, B 중가장큰수이므로 4 이상이다. 따라서 A + B 4+2 이다.)
4.2 조합고급문제 233 (2) 7 과 8 이면대각선의두끝점일때 : 역시 a 와 A 중큰것을택하고, b 와 B 중큰것을택하면 (1) 과같은결론. (3) 7 과 8 이주대각선의두끝점일때 : 나머지 6 개의점이모두대칭적이므로일반성을잃지않고 A 가가장크다고하면 A =6 이고이점을택해연결하면 8+7+6+x>21 이된다. 따라서, 어느경우에나주대각선의양끝점을잇는경로의네꼭지점의수의합이 21 이상이되는경우가항상있다. 22. (a) 어느해연말에, 어떤학급의전체 20명의학생이각각급우 10명에게연하장을보냈다. 그럼서로연하장을주고받은 2명의학생이반드시있음을증명하여라. (b) 위의문제를일반화하여, 학급의학생수를 n명이라하고, 각학생은각각급우 m명에게연하장을보낸다고하자. 서로연하장을주고받은 2명의학생이반드시있음을확신할수있으려면 n과 m은어떤값이라야하는가? ( 플란더즈 1993-1) 풀이 m n=2 이면비둘기집원리로됨. m<n=2 일때의반례는, n명의학생을원형으로배열해두고, 각학생마다자기왼쪽으로 m명에게연하장을보내는것으로하면됨. 23. 어느시험에서채점을한결과, 어떠한두개의문제를선택하여도이두문제중한문제만을푼학생은항상 6명이상이었다. 시험을본학생이 11명이었다면출제된문제의개수가 12 이하임을보여라.( 단, 등호조건을보일필요는없음.) ( 한국 2005-J4) 증명 ( 성남수내중 3 학년임동혁 ) 문제가 n 개이고,1; 2;:::;n 번문제를푼학생들의집합을각각 A 1 ;A 2 ;:::;A n 이라하자. 문제의조건에서임의의 i 6= j 에대해 ja i [ A j j ja i \ A j j 6, 즉 ja i j + ja j j 2jA i \ A j j 6 이다. 이것을모든 i; j 에대해다더해보자 (1 i<j n). ³ n S =(n 1)(jA 1 j + + ja n j) 2(jA 1 \ A 2 j + + ja n 1 \ A n j) 6 =3n(n 1) 2 이된다. 각학생이푼문제의개수를 k 1 ;k 2 ;:::;k 11 이라하면 µ ³k1 ³ k11 S =(n 1)(k 1 + + k 11 ) 2 + + 3n(n 1) 2 2 S = n(k 1 + + k 11 ) (k1 2 + + k11) 2 3n(n 1) 이된다. S 를완전제곱식으로고치면 11X ³ S = k i n 2 11n 2 + 11n2 2 4 4 i=1 이므로 11n2 4 3n(n 1) 이다. 즉, n 12 이다. 24. knight's tour 란, 체스판에서기사 ( 장기의 ` 마 ' 의움직임과같다 ) 가어느한칸도두번들르지않고꼭한번씩지나가는방법을말한다.4 4 체스판에서는 knight's tour 가존재하지않음을보여라. ( 통신강좌 1997-14-12) 증명 Α Β 2 Β 1 Α Β 1 Χ Χ Β 2 Β 2 Χ Χ Β 1 Α Β 1 Β 2 Α 체스판을다음과같은세종류로나누어보자.( 각칸은꼭지점,` 마의움직임가능 ' 은선분과같은역할을한다. 그냥참고삼아 )
234 조합 이제살펴보면, A 에 knight 가있으면다음움직일수있는곳은 C 뿐이다. B i 에서는또다른 B i 또는 C 로이동이가능하다.(i =1; 2) C 에서는 B 1 ;B 2 또는 A 로이동이가능하다. 이제,knight'stour 가존재한다면,knight 가이동한곳의기호를한줄로나열하면, 위에서구한성질에의해 C! AorB i 들 (B i 여러개도가능 )! C! AorB i 들! ( 물론 (A or B i 들 )! C 로시작될수도있다.) 그런데, C 는 4 개인데, C 사이에있어야할 ( 앞이나뒤도가능 ) A; B i 가 6 가지이상이므로 (A4 개, B 1 ;B 2 로 B 가 2 가지 ) 이러한배열은불가능하다. ) 모순 ) knight's tour 가존재하지않는다. 25. 연결상태가구와같은다면체에서는오일러의공식 v e + f =2 가성립함을안다.( 단, v, e, f 는각각꼭지점, 모서리, 면의개수이다.) 연결상태가도너츠와같은, 즉구멍이하나뚫린다면체에서는언제나 v e + f =0 이성립함을보여라. 단, 이다면체의각면은구멍없는다각형이어야한다. 예를들어다음과같이구멍이있는면이그려진경우에는그면을몇개의면으로더쪼개야한다. (KAIST 대전. 충남영재수학교실중급반 2000) 풀이도넛형태의다면체는반으로잘라서구와연결상태가같은두개의다면체로만들수있다. 이때단면은 2개가나오는데그단면 2개는각각다각형이된다. 두면의v; e를 v 0 ;e 0 이라하면 v 0 = e 0 이다. 한편, 나누어진다면체각각의 v; e; f를 v i ;e i ;f i (i =1; 2) 라하면v i e i + f i =2이고원래도넛형태의다면체에서 v = v 1 + v 2 v 0 ;e = e 1 + e 2 e 0 ;f =(f 1 2) + (f 2 2) 이므로 v e + f = (v 1 + v 2 v 0 ) (e 1 + e 2 e 0 )+(f 1 + f 2 4) = (v 1 e 1 + f 1 )+(v 2 e 2 + f 2 )+(e 0 v 0 ) 4 = 2+2+0 4=0 따라서 v e + f =0 이다. 26. 주어진 n 개의정수의합이 94 이다. 한원의원주위에시계뱡향으로이정수들을늘어놓는데, 각각의정수는그다음두정수의차와같도록하였다. 가능한 n 을모두구하여라. ( 뉴질랜드 2000-7) 풀이늘어놓은수가모두똑같을수는없다. 극값원리로가장큰것을택해논하면 aa0 의반복꼴뿐임이금방됨. 27. 양의정수들을 3 3 으로배열하는데각행의수의합, 각열의수의합, 두주대각선의수의합이모두 m으로같도록만든것을합방진이라하자. m이 3의배수임을보이고, 이배열의수들중가장큰수는최대 2m 1 3 임을보여라. ( 아일랜드 1989-2) 증명합방진의정가운데에오는수를 x라고하자. 가운데를지나는모든줄 ( 행, 열, 대각선 ) 을합하면모든수를더하고가운데원소를3번더더한것이므로 3m +3x가되고, 각줄은합이m이므로 3m +3x =4m이다. 따라서 m =3x로 3의배수이다. 가운데가 x이고각줄의합이m =3x이므로가운데를지나는임의의줄에서나머지원소는 x + y; x y(y 0) 로표현할수있다. x y>0이므로 x>y이고 x + y<2x = 2m 3 이다. 그러므로배열의수들은기껏해야 2m 3 1이하이다.
4.2 조합고급문제 235 28. n은짝수이다. 자연수 1; 2;:::;n 중에서 a + c = b + d 를만족하는서로다른네수의집합fa; b; c; dg를 n(n 2)(2n 5) 고르려고한다. 이렇게수를고르는경우의수가가지임을보여라. ( 호주 1989-4) 24 풀이귀납법.ora<b<c인세수를고르면d는저절로결정.2b = a + c 인경우 (b = d 인경우 ) 배제하고 a, b, c 등의순서가섞이는것을고려해주면... 29. 체스판에서나이트 (knight) 는날일자 ( 日 ) 의대각선방향으로이동한다. 즉, 체스판이충분히넓다면나이트는한점에서 8 군데로이동할수있는것이다.7 7 체스판을나이트가각칸을정확히한번씩들러출발지로되돌아올수있는가? ( 통신강좌 1997-14-20) 풀이체스판을흑백을교대로칠하면, 나이트는흑인칸에서백인칸, 혹은백인칸에서흑인칸으로만이동할수있다. 체스판에흑은 25개, 백은 24개이다. 한편모든칸을돌고오는방법 ( 해밀턴회로 ) 이있다면, 흑의수와백의수가같아야하므로모순, 따라서불가능. 30. k 개의변을가지는연결된그래프 G 의각변에 1; 2; 3;:::;k 의번호를모두붙이는데,2 개이상의변에공통으로속하는 G 의정점에서는번호들의최대공약수가 1 이되게붙일수있음을보여라. ( 통신강좌 1997-15-22) 증명 ( 과기원수학과 96학번신희성 ) 주어진점에이웃한변의총수를그정점에서의차수라하자. 모든점에서의차수의총합은변의개수의 2배이므로짝수이다. 따라서차수가홀수인점은짝수개이다. 그래프 G가연결되어있으므로, 이그래프의임의의두홀수점을잇는통로를만들수있다. 따라서우리는그래프 G를홀수점을양끝점으로하는몇개의통로와몇개의회로로분해할수있다. 그리고회로는다른회로및통로와합칠수있으므로연결된그래프 G가 2m개의홀수점을가지면 m개의통로로, 홀수점이없으면오일러회로로분해된다. 각통로나회로에순서대로 1, 2, 3, :::, k까지번호를매기면차수가 2이상인점에연결된두선분은숫자가연속이므로그점에이웃한변에붙여진숫자들의최대공약수는 1이다. 31. m n 크기의판의각칸에한자리숫자를하나씩써넣는데, 어느위치에서가로, 세로적당한크기의직사각형꼴의부분영역을잡아도그영역안의숫자전체의합이 5 의배수가되지않게하고싶다. mn 의최대값을구하여라. (IT 꿈나무올림피아드 2006 1 차 ) 풀이한줄에5개의수 a, b, c, d, e가연속으로있다고하고직사각형영역의다섯가지합 a, a + b, a + b + c, a + b + c + d, a + b + c + d + e 를생각하자. 이중에5로나눈나머지가같은것이있다면그둘의차또한직사각형영역의합이되고 5의배수가되므로곤란하다. 따라서, 이다섯합이모두나머지가서로다른데, 그럼이중에 5의배수 ( 나머지가 0인수 ) 가있으므로역시곤란하다. 그러므로, 한줄에 5개의수가있으면안되고, m, n 모두 4 이하이다. 그리고, m = n =4일때는모든칸에1을넣으면조건을만족한다. 따라서, mn의최대값은 4 4 = 16. 32. 집합 f1; 2; 3; 4; 5; 6; 7; 8g 에서 3 개의원소를갖는부분집합을몇개만드는데, 어느두부분집합도공통원소가많아야하나뿐이도록하고자한다. 이런성질을만족하는부분집합들을최대몇개까지만들수있는가? ( 뉴질랜드 2003-6) 풀이 ( 광주동성고 3 학년방재혁 ) 우선부분집합의개수가 9 개이상이면불가능함을보이자. 부분집합의개수가 9 개이상이면원소의총개수는 27 개이상이다. 그러면비둘기집의원리에의해 1,..., 8 중적어도하나는 4 개이상의부분집합에속한다. 그수를 a 라하면 a 를포함하는네집합 fa; x 1 ;x 2 g; fa; x 3 ;x 4 g; fa; x 5 ;x 6 g; fa; x 7 ;x 8 g 에서 x 1 ;:::;x 8 은모두달라야한다. 그런데 x 1 ;:::;x 8 은 a 를제외한 7 개의수중에서골라야하므로이
236 조합 것은모순이다.8 개의부분집합은다음과같이만들수있다. A = f1; 4; 6g B = f1; 2; 3g C = f3; 6; 8g D = f3; 4; 5g E = f2; 5; 8g F = f5; 6; 7g G = f2; 4; 7g H = f1; 7; 8g 따라서, 최대 8 개까지만들수있다. 33. X 는집합 f1; 2; 3;:::;1989g 의부분집합이다. X 의어떤두원소의차도 4 나 7 이되지않는다고할때, X 는최대몇개의원소를가질수있는가? (AIME 1989-13) 풀이 %EEE (Thanks to Bugz Podder for this.) We show first that we can choose at most 5 numbers from {1, 2,..., 11 such that no two have difference 4 or 7. Translating if necessary, we make take the smallest number to be 1. That rules out 5,8. Now we can take at most one from each of the pairs: 2,9; 3,7; 4,11; 6,10. 1989 = 180 11+9, so that means we can pick at most 5 181 = 905. If we pick 1, 3, 4, 6, 9, then that works and moreover it allows us also to pick 11+1, 11+3, 11+4, 11+6, 11+9. So 905 is also possible. 답 905 34. 평면위에 n 개의다각형을그렸는데, 임의의두다각형은항상서로만나고, 접하는점이나세다각형이모두지나는점은없다고한다. 이렇게해서생긴교점의수를 v 라하고, 다각형들에의해나뉜평면의영역 ( 제일바깥의무한영역은제외 ) 의수를 f 라할때, f = v +1 이성립함을증명하여라. (KAIST 대전. 충남영재수학교실주말교육 2000) 증명귀류법으로, 성립하지않는경우가있다고하자. n =1일때1=0+1로성립하므로 n 2 인경우이다. 그런경우들중에가장작은 n을택하자. 그려진다각형중어느하나를택해, 그다각형이다른다각형과만나는교점을 2k개라하자. 그럼그다각형은 2k개의둘레조각으로분할됨. 각각의둘레조각은원래하나였던영역을둘로나누고있음. 따라서, 이다각형을들어내면 f 0 = f 2k, v 0 = v 2k 가됨. n의최소성에대해더작은 n 1일때는성립하므로 f 0 = v 0 +1이고, 그럼대입하면 f = v +1 도성립하게되어모순. 별증오일러의평면그래프공식 v e + f =1에 4v =2e ( 각교점마다 4개의변이연결되므로 ) 를대입하면바로나옴. 35. 어떤왕국에 N 명의기사가살고있다. 각기사들은서로간에친구이거나적이다. 그리고각기사들은정확히 3 명의적을가지고있다. 만약이왕국이 \ 친구의적은나의적이다." 라는법에의해지배된다면가능한 N 의값은? ( 통신강좌 1998-16-28) 풀이기사 A와 B가서로적이라고하자. 조건에의해다른기사들은 A; B중의하나이상과는적이다. 이에따라 N 7이면 A; B 중적어도하나는넷이상의적을가진다. 그러므로 4 N 6이다. 각 N명의기사에게적이세명이므로 3N 쌍의적이존재한다. 즉, N은짝수 2 이다. 그러므로 N=4, 6만이가능하다. 또한, 서로모두적인경우 N =4가가능하고,3:3의두팀이서로적대하는상황이면 N =6도가능하다.
4.2 조합고급문제 237 36. 1 m<n 40 인 780 종류의도미노 [m; n] 을모두모은집합을 D 라하자. 이도미노들중에서몇개를일렬로늘어놓는데, 인접한두도미노 [a; b] [c; d] 는 b = c 를만족해야한다. 각각의도미노 [m; n] 은 [m; n] 또는 [n; m] 과같이늘어놓을수있다. 최대몇개의도미노를늘어놓을수있을까? (AIME 1998-15) 풀이 %EEE There are 39 elements [m, 40]. Unless 39 appears as the first or last number in the line, it must occur an even number of times. Similarly for the other numbers. So 38 numbers and hence 19 dominos [m, n] cannot appear. So the sequence cannot be longer than 780-19 = 761. We claim that a sequence of 1 + (4 + 8 +... + 4n-4) with first number 1 and last number 2 is possible for the set of dominos with largest number 2n. Induction. n = 1. [1,2] gives 1. [1 2n-1][2n-1 2][2 2n][2n 3][3 2n-1][2n-1 4]... [2n-2 2n][2n 1] prefixed to the line for 2n-2 gives 4n-4 more dominos than the line for 2n-2. (The extra dominos are the pairs [2n-1 a] and [2n a] for a = 1, 2,..., 2n-2.) Putting n = 40 shows that 761 is realised. 답 761 37. A와 B 두사람이32개의공기돌을두고돌가져가기게임을한다. 먼저 A가시작한후차례를번갈아돌을가져가는데, 자기차례에 1개혹은적당한소수개의돌을가져갈수있다. 마지막돌을가져가는사람이이기는것으로할때, 두사람모두최선의전략을따른다면누가이기겠는가? (IMTS R3-5) 풀이 B 가이긴다.A 는 4 의배수를만들수없고,B 는항상 4 의배수를만들면된다. 38. 1 부터 16 까지의숫자를 4 4 숫자판에적당히배치하였다. 각행과각열의숫자의합을구했더니이 8 개의합이모두서로다르고또모두 n 의배수가되었다. n 의최대값은얼마인가? (IT 꿈나무올림피아드 2006 1 차 ) 풀이 1+2+ +16=8 17 이므로 n은 8 17의약수이다. 만일 n이 17의배수이면, 각행과열의합 8개가모두다르다고했으므로그중가장큰값은 8 17 이상이된다. 이는가장큰 4개의수의합 13 + 14 + 15 + 16 보다훨씬크므로불가능하다. 따라서, n은 8의약수이다. n =8일때도앞과마찬가지로 8개의합중가장큰것은 8 8 이상이고이것은 13 + 14 + 15 + 16 보다크므로불가능하다. n =4 일때는다음과같이가능한경우가있다. 1 8 3 4 16 5 6 7 2 20 9 10 11 14 44 13 16 15 12 56 28 40 36 32 합 답 4 39. 가로 5, 세로 2 인칠판이벽에고정되어있다. 칠판을여러개의사각형들만으로분할한것을 ` 그림 ' 이라하자. 각각의분할된사각형이한변의길이가 1 인정사각형이거나가로 1, 세로 2 인직사각형이거나또는가로 2, 세로 1 인직사각형일때그그림을 ` 멋진그림 ' 이라하자. 멋진그림은모두몇개인가? ( 한국 2006 1 차 -J20) 풀이 ( 인천광성중 2 학년이원열 ) n 2 크기의칠판에서만들수있는 ` 멋진그림 ' 의수를 f(n) 이라하고, 아래그림과같이 (n 1) 2 에한개의정사각형을덧붙인칠판에서만들수있는 ` 멋진그림 ' 의수를 g(n) 이라하자.
238 조합 f(n) 은 (i) a 를정사각형으로분할할경우 : 총 g(n) 가지,(ii) ac 를묶어분할할경우 : 총 f(n 1) 가지, (iii) ab 로묶어분할할경우는 c 를정사각형으로분할할경우 : g(n 1) 가지와 cd 를묶어분할할경우 : f(n 2) 가지가있다. 따라서, f(n) =g(n)+f(n 1) + g(n 1) + f(n 2) (n 3) 가된다. 비슷하게 g(n) 에대해서도 e 를기준으로생각하면 g(n) =f(n 1) + g(n 1) (n 2) 이것을이용해 f(n) 을간단히하면 f(n) =2g(n)+f(n 2) f(0) = 1, g(1) = 1, f(1) = 2 로볼수있으므로이점화식으로작은 n 에대해 g, f 의값을구해나가면 g(2) = 1 + 2 = 3 g(3) = 3 + 7 = 10 g(4) = 10 + 22 = 32 g(5) = 32 + 71 = 103 f(2) = 2 3+1=7 f(3) = 2 10 + 2 = 22 f(4) = 2 32 + 7 = 71 f(5) = 2 103 + 22 = 228 문제에서원하는것이 f(5) 이므로 답 228 40. 어떤그룹에 29 명의학생이있다. 각학생은항상진실만을말하거나항상거짓말을한다. 어느날모든학생들이원형으로모여앉았다. 그리고각학생들은 \ 내옆의두학생은모두거짓말쟁이다." 라고얘기했다고한다. 이그룹에는진실을말하는학생이 10 명이상임을보여라. 또, 정확히 10 명일수있겠는가? ( 통신강좌 1998-16-30) 풀이 L L T L L T L L T L T L L L T T L L L L T L L T L L T L L T 만약 10 명이하의학생이진실을얘기한다면최소한세거짓말쟁이는이웃하여앉게된다. 이경우세거짓말쟁이중가운데사람은진실을얘기하고있는것이므로모순이발생한다. 정확히 10 명의학생이진실을얘기하는예는다음과같다.(T 는진실을얘기하는학생,L 은거짓말쟁이학생이다.) 41. 3 종류의색을사용해서평면위의모든점에색을칠한다. 이때반드시그거리가 1 이되는 2 개의똑같은색의점이있음을증명하여라. ( 셈본중등초급도전문제 1.2.2) 증명귀류법을이용하여, 거리가 1이되는두점은항상서로다른색이라고가정하고모순을찾아보자. 한변의길이가 1인정삼각형을이루는세꼭지점 A, B, C는항상서로다른세가지색의점이된다. 선분 BC에대한A의대칭점A 0 도 B, C와정삼각형을이루므로, A와 A 0 은같은색의점이된다. A와 A 0 사이의거리는 p 3이고, 거리가 p 3이되는임의의두점은항상같은색의점이됨을알수있다 ( 그런두점사이에는항상두정삼각형을이루는 B, C와같은점을잡을수있으므로 ). 따라서, A를중심으로반지름 p 3인원을그리면이원위의점은모두A와같은색이고, 이원위에거리1인두점이있으므로모순이다. 따라서, 거리가 1이되는2개의같은색의점이반드시있다.
4.2 조합고급문제 239 42. 혼자서하는다음과같은퍼즐이있다. 퍼즐판에는말을끼워넣을수있는구멍여러개가둥글게원형으로배열되어있다. 한번의움직임이란, 어떤구멍에끼워져있는말하나를없애고그양옆의구멍의상태를바꿔놓는것이다 ( 구멍의상태를바꿔놓는다는것은, 거기에말이있으면그말을없애는것을, 말이없으면새로운말을끼워넣는것을뜻한다 ). 처음에퍼즐판에딱하나의말이놓여있는것으로시작하고, 퍼즐판에말이하나도남지않도록하는것을목표로한다. 퍼즐판의구멍이 12 개라면이것이가능한가? 구멍이 10 개이면어떤가? 일반적으로, 구멍의개수가어떤경우에이것이가능한가? ( 오스트리아 - 폴란드 1981-4) 풀이 ( 부산장전중 3학년안현태, 수정됨 ) 퍼즐판의구멍의개수를 n이라하자. (i) n =3k +1꼴일때 : 말하나에서시작하여다음과같은과정을차례로거치자. : : O : : : : : : : : O : O : : : : : : : O O : O : : : : : : O O O : O : : : :. : O O O O O O O : O O O O O O O O O O : 그럼 3k개의연속한말을얻을수있다. 이것을 3개씩묶어없애면모든말을다없앨수있으므로가능하다. (ii) n =3k +2꼴일때 :(i) 의방법을응용하여먼저다음의꼴을만들자. : O O O O O O O O : O O O O O O O O O O O : 여기서제일앞의말을택해움직이면다음의꼴이된다. : : O O O O O O O O O 그럼역시연속한 3k개의말이남으므로모두없앨수있다. (iii) n =3k꼴일때 : 이때는불가능함을보이자. 퍼즐판의각위치에차례로 1부터 3k까지번호를붙이자.1; 4; 7;:::;3k 2 위치의말의개수를 p개, 2; 5; 8;:::;3k 1 위치의말의개수를 q개, 3; 6; 9;:::;3k 위치의말의개수를 r개라하자. 어느위치의말을택해움직이든, 항상 p, q, r의말을하나씩상태를바꾸게되므로 p, q, r의홀짝이모두한꺼번에변한다. 즉, 맨처음에 (p; q; r) = ( 홀, 짝, 짝 ) 으로시작하므로,( 홀, 짝, 짝 ) 과 ( 짝, 홀, 홀 ) 만을번갈아갖게된다. 말을모두없앴을때는 ( 짝, 짝, 짝 ) 이므로이것은불가능하다. 따라서, n =3k +1혹은 3k +2꼴일때만가능하고 3k꼴일때는불가능하다. 43. 한자리수, 즉 0; 1; 2;:::;9 로이루어진수열 a 1 ;a 2 ;:::;a 99 가다음성질을만족한다. a n =1 이면 a n+1 6=2; a n =3 이면 a n+1 6=4 이때, a k = a l, a k+1 = a l+1 인서로다른두수 k; l 2f1; 2; ; 98g 이존재함을보여라. ( 폴란드 1996 2 차 -4) 증명순서쌍 (a k ;a k+1 ) 들을생각하자. 문제는이런 98개 (k =1; 2; :::; 98) 의순서쌍중에같은것이있음을보이라는것이다.10개의숫자로순서쌍을구성하므로모두 100가지가가능한데, 그중 (1; 2) 와 (3; 4) 를제외해야하므로가능한순서쌍은 98가지. 따라서, 두번사용된순서쌍이없으려면모든가능한순서쌍이딱한번씩사용되어야한다. 그런데, 숫자 1은앞자리로 9번, 뒷자리로 10번사용되었으므로맨마지막에사용되어야한다. 이것은숫자 3도마찬가지이므로모순. 44. n(> 1) 개의점이주어져있다. 어떤두점들은선분으로이어져있다 ( 같은점에서같은점을연결하는선분은없다 ). 어떤두점이주어져도한점에서다른점까지선분을따라도달하는길이꼭하나씩있다고하자. 이때 n 1 개의선분이있음을증명하여라. ( 소련 1961-8)
240 조합 증명점의숫자에대해수학적귀납법을사용하자. 먼저 n =1 일때, 선분의개수는 0 이므로 n 1 개의선분이있다. 이제 n k 개의점이있을때선분의개수가항상 n 1 개라고하자. 이때 k +1 개의점이있는조건을만족시키는그래프를생각하자. 이그래프에서한선분을없앤다면어떻게될까? 제거하는선분의양끝점을각각 a; b 라고하자. 만일임의의점 c 에대해 c 에서 a 로가는경로가이선분을포함하지않는다면, c 에서 b 로가는경로는반드시이선분을포함해야만한다 (b 에서 c 로가는경로가유일하기때문이다 ). 마찬가지로, c 에서 a 로가는경로가이선분을포함한다면 c 에서 b 로가는경로는이선분을포함하지않아야한다. 즉, 이선분을제거하면그래프가 a 를포함하는쪽과 b 를포함하는쪽 2 개로나눠지며경로의유일성이보존된다. 이제양쪽의그래프의점의개수를 n 1 ;n 2 라하면 n 1 ;n 2 k 이고 k +1=n 1 + n 2 이다. 따라서총선분의개수는 (n 1 1) + (n 2 1) + 1 = (n 1 + n 2 ) 1 = n 1 이다. 45. 한변의길이가정수 cm 인각크기의정육면체블록들이무제한으로주어져있다. 정확히 2001 개의블록을이용해 ( 빈틈없이쌓아 ) 정육면체를만들때필요한서로다른크기의블록은최소몇종류인가? ( 뉴질랜드 2001-6b) 풀이 2001a 3 이세제곱수가되는경우는없으므로한종류로는안된다.2 3 과 1 3 두종류만이용하여만들수있음을보이자.2 3 블록을 n개사용한다면 8n + (2001 n) =m 3 을만족하는경우를먼저찾아야한다. 즉, m 3 =2001+7n 1(mod7) 이고, 이런 m은 7k +2; 3; 6꼴이면항상가능하다. n이작을수록실제쌓는것이유리하므로 2001보다큰 m 3 중에서가장작은것을찾으면 m =13 3 = 2197. 이때,2 3 블록 28개와 1 3 블록 1973개를사용하면실제로쉽게만들수있다. 46. 종이위에 p 2 개의점을찍고게임을시작한후, 두명의경기자가번갈아다음과같이변을그린다 : 한번의차례에서단하나의변을그리는데, 이미그려진그림에서두점을연결하는곡선을그리거나, 한점을자기자신에게연결하는곡선을그릴수있다. 단, 새로그리는변은이미그려진다른변이나점과교차할수없고, 각점의차수는최대 3까지만을허락한다. 그리고새로운변을그린후에는그변의중앙에새로운한점을추가한다. 이런시행을반복하다가더이상변을추가할수없을때, 마지막변을그린사람이이기는것으로한다. 이게임은항상유한번의시행을한후끝나게됨을보이고, 이시행횟수의최대값을 p에대해구하여라. ( 새싹놀이, 통신강좌 1999-18-5) 시작 1 번 2 번 3 번 4 번 풀이모든점의가능한차수의합을생각하자. 처음에 p개의점이있으니가능한차수의합은 3p에서시작한다. 만일어떤두점이연결되면그두점이차수를하나씩잃고가능한차수가 1인새로운점이하나생겨나므로전체가능한차수의합은 1이줄어든다. 처음에 3p에서시작하며가능한차수의합이 1보다작아질수없으므로 ( 가능한차수의합이 1이면한점이남았다는것인데스스로를연결하는곡선은차수를2 줄이므로불가능하다 ) 이게임은항상유한번의시행후에끝난다. 최대한게임이오래가는방법은차수가 3p에서시작하여 1씩줄어들어최후에 1이남는경우이다. 즉시행횟수는 3p 1이최대라고예상할수있다. p에대한귀납법으로 3p 1번의시행이가능함을보이자. p =1일때는처음점에자기자신으로의곡선을긋고 (1번) 그곡선위에생긴점과처음점을연결하는곡선을그으면 (2번) 된다. p = k일때 3k 1번의시행이가능하다고하자. p = k+1개의점이있다면먼저 k개의점에대해 3k 1번의시행을한후, 나머지 1개의점에 2번의시행을하고, 마지막으로양쪽에서가능한차수가 1인최후의점들을연결해주면된다 ( 최후의점들이바깥쪽에드러나도록항상할수있으므로가능하다 ). 따라서 3p 1번의시행이최대이다. 47. 음이아닌정수들의수열 fa k g k 1 이모든 k 1 에대해 a k a 2k + a 2k+1 을만족한다.
4.2 조합고급문제 241 ( 가 ) 모든양의정수 n 1 에대해, 수열 fa k g에는 n개의연속한 0이항상있음을보여라. ( 나 ) 문제의조건을만족하면서 0이아닌항을무한히많이갖는수열의예를하나만찾아제시하여라. ( 루마니아 2005 지역예선 y9-4) 풀이 ( 가 ) a k 가 0이아니면a 2k 와 a 2k+1 중에적어도하나는 a k 보다작다. 즉,0이아닌항에대해서그보다작은항을항상찾을수있으므로, 음아닌정수들에서는더작은항을계속쫓아가다보면유한번안에0인항을만날수밖에없다. 그리고, a k =0이면 a 2k = a 2k+1 =0으로 2개의연속한 0이나타나게되고, 그럼다시 a 4k = a 4k+1 = a 4k+2 = a 4k+3 =0으로 4개의연속한 0이나타나게되고, 귀납적으로임의의자연수 m에대해2 m 개의연속한 0이항상나타나게된다. 따라서, 임의의자연수 n에대해 n개의연속한 0이항상있다. ( 나 ) 다음의수열이한예가된다. 1; 1; 0; 1; 0; 0; 0; 1; 0; 0; 0; 0; 0; 0; 0; 1; 0; 0; 0; 0; 0; 0; 0; 0;::: 즉, k =2 n 꼴일때 a k =1 이고그외에는모두 0 인수열.( 이외에도예는얼마든지많다.) 48. A 는 16 개의자연수를모은집합이다. A 에서서로다른임의의두원소를곱하면항상 1994 를넘지않는다고한다. A 에서서로소가아닌두원소를찾을수있음을증명하여라. ( 인도지역예선 1994-5) 증명1 귀류법으로풀자. A의모든수가둘씩서로소라하자. 그럼 A에속하는 1이아닌수x는반드시소인수 p를하나는갖는데, p를 A의다른수와공유할수없으므로 x마다서로다른 p를하나씩갖는다. A에속하는 1이아닌수를x 1 ;x 2 ;:::;x 15 라하고, p 1 ;p 2 ;:::;p 15 들을각각의한소인수라하자. p i 들은모두서로다르므로일반성을잃지않고 p 1 <p 2 < <p 15 라할수있다. 소수를작은것부터차례로 15개를나열하면 2, 3, 5, 7, 11, 13, 17, 19, 23, 29, 31, 37, 41, 43, 47이므로 p 14 43, p 15 47 이다. 그럼 x 14 x 15 p 14 p 15 43 47 = 2021 > 1994 가되어모순이다. 따라서, A 의모든수가다둘씩서로소일수는없다. 증명2 역시귀류법으로풀자. a 2 A 가합성수이고 p가 a의한소인수라면 A에서 a를제거하고대신 p를넣자. 그래도 A의수들은여전히둘씩서로소이고또두수를곱하면1994 이하이다. 이런과정을반복하여 A에합성수가하나도없도록할수있고, 그럼 1과소수들만남는다. 이때, A의가장큰두수 a 14, a 15 는각각14번째와 15번째로작은소수인 43, 47 이상이므로, a 14 a 15 43 47 > 1994 가되어모순이다. 따라서, 귀류법의가정이틀렸고, A 에서서로소가아닌두원소를찾을수있다. 49. 수직선위의점1, 1 2, 1 3, 1 4, 1, ::: 들을생각하자. 길이가 k인 5개의닫힌구간으로이점들을모두덮을 5 수있을때, k의최소값을구하여라. ( 플란더즈 1996-3) 풀이답은 0.1이다. k =0:1 일때는다섯개의구간으로 [0; 1 10 ], [ 1 9 ; 1 5 ], [ 1 4 ; 1 3 ], 1,1 2 을각각덮으면된다. k<0:1 이면 0:1 k 보다작은 1 n 을하나택할수있고, 그럼 1 n, 1 10, 1 5, 1 3, 1,1 2 을각각따로덮어야하므로불가능. 답 0.1 50. 유한한넓이를가진정사각형모양의평면이있다. 갑과을이이평면위를임의의크기의볼록다각형타일로채우는데, 도형이서로겹치게되거나, 평면의테두리를넘게되는사람이진다. 단, 도형의크기는바로전에놓인도형의크기이상이어야하고, 맨처음에놓이는도형의크기는최대평면의넓이의 1/100 이다. 갑이처음시작할때갑의필승전략을구하여라. (ML 프로포절 165-1) 풀이갑은처음에평면의중심 O를점대칭이되도록덮는다. 그후에는을의플레이를 O에대해점대칭이되도록따라하면된다. 그럼을이타일을둘수있다면갑도항상타일을둘수있고 ( 점대칭이유지되므로을이둔타일을제외하면을이둔곳의점대칭인곳은둘수있고, 타일은볼록다각형이므로바로전에을이둔타일과새로갑이둔타일이어떤점에서만난다면그와점대칭인점에서도만나고그럼그두교점을연결한선분위에있는 O도지나므로모순 ), 넓이의유한성에의해게임은항상유한번안에끝나므로갑이이긴다.
242 조합 51. 2, 3, :::; 102 를적당한순서로늘어놓은것을 a 1 ; a 2 ; :::; a 101 이라하자. 이런수열중에서모든 k 에대해 a k 가 k 의배수가되는것들을모두찾아라. (Towns 1980 J3) 풀이 a i 6= i 인 i에대해생각해보자. a i = j 라하면j는 i가아닌, 즉 i보다큰 i의배수이다. j는이미 a i 가차지했으므로, a j = k 라하면k도 j보다큰 j의배수가되고, 이런식으로 (a n 를함수a(n) 으로생각하면 ) i! j! k! (! y! z) ( ) 계속자신보다큰배수로대응되는 fa ng 의부분수열이생긴다.102 이하의수만사용하므로이것이무한히계속될수없고, 그럼 a y = z 에서끝난다면 a z 는더이상정의되지않는항이어야하므로 z =102 여야한다. a 1 > 1 이므로부분수열 ( ) 은 1 부터시작하여 102 로끝나는것하나뿐이고, fa n g 의나머지항들은모두 a n = n 의자기자신으로대응되는항이어야한다. 즉,102=2 3 17 = p 1 p 2 p 3 의약수들 1; p 1 ; p 2 ; p 3 ; p 2 p 3 ; p 3 p 1 ; p 1 p 2 ; p 1 p 2 p 3 (= 102) 들중에서 ( ) 을만족하는부분수열을하나정할때마다수열 fa ng 이하나씩결정된다. ² 1! 102 : 1 개 ² 1! pq! 102 꼴 :3 개 ² 1! p! 102 꼴 :3 개 ² 1! p! pq! 102꼴 :3 2=6개의 13개가구하는모든수열이된다. 52. ABCD 는사면체이다. 네점 A, B, C, D 로부터모두같은거리에있는평면은몇개인가? ( 플란더즈예선 1989/1990) 풀이평면은항상공간을둘로나누므로네점으로부터같은거리에있는평면은점 A; B; C; D를두그룹으로나눈다. 만일 A; B; C; D가모두한쪽에있다면A; B; C; D는한평면에있고 4면체를이루지못한다. 따라서한쪽그룹이원소를항상 1개이상가진다. A; B; C; D를한쪽이 1개이상이되도록분할하는개수는 7개 (= 2 4 =2 1) 이므로, 평면의개수는총 7개이다. 53. X 는집합 f1; 2; 3;:::;15g 의부분집합이고, X 의서로다른어떤두부분집합도원소의합이같지않다고한다. X 의원소의합의최대값을구하여라. (AIME 1986-12) 풀이 %EEE The greedy algorithm gives X = {15, 14, 13, 11, 8, sum 61, but can we do better? If there are 6 elements, then there are 26-1 = 63 sums (excluding the empty subset). One of these has 6 elements and six have 5 elements. The other 56 have 4 or less elements and hence sum at most 15 + 14 + 13 + 12 = 54, contradiction (there are only 54 possible distinct sums). It is now messy. If we do not include 15, 14, 13, then... If we do, then we cannot include 12. If we do not include 11 either, then... etc. 답 61 54. 삼각형의세변의길이가될수있는서로다른세수를골라낼수있는집합을삼각성을갖는다고말하기로하자. 집합 f4; 5; 6;:::;ng 의 10 개의원소를갖는부분집합이모두삼각성을가질때, n 13 의최대값을구하여라. (AIME 2001 2 차 -5) 풀이 %EEE Clearly (4, 5, 9, 14, 23, 37, 60, 97, 157, 254) does not have the triangle property (each element from 9 on is the sum of the previous two). But if a_1 < a_2 <... < a_10 does not have the property, then a_10 a_9 + a_8 2a_8 + a_7 3a_7 + 2a_6... 34a_2 + 21a_1 34 5 + 21 4 = 254. 답 253
4.2 조합고급문제 243 55. 다음그림과같이도너츠모양의별이있다. 이별에서완전그래프 K 7 을그릴수있는가? 완전그래프 K n 이란 n 개의도시가각각둘씩모두직행도로 ( 도중에다른도로와만나지않는도로 ) 로연결되어있는구조를말한다. ( 통신강좌 1992-4-21) 풀이 A F J K I H A G C B D E 7 6 2 1 3 5 4 G C B D E A F J K I H A 듀얼그릴수있다. 직사각형위에그림과같은그래프를그린후에윗변과아랫변을 E와 E, F와 F가만나도록붙이면원통형의모양이된다. 그원통을다시 B 와 B, C 와 C, D 와 D 가만나도록맞붙이면도너츠모양이되며육각형꼴의각각의영역은모두다른색으로색칠되어야한다. 56. 한변의길이가1인정사각형안에그길이의합이 18인유한개의선분이있다 ( 정사각형은경계를포함하고선분도양끝점을포함하는것으로하자 ). 각선분은정사각형의어느변에평행해야하고, 선분끼리서로교차할수있다. 이선분들로나뉘어진정사각형의영역들중에적어도하나는 0.01보다적지않은넓이를가짐을증명하여라. (Towns 1980 J5/S5) 증명 ( 광주고 1 학년서준영, 수정됨 ) 주어진유한개의선분들이정사각형을몇개의영역 A 1 ;A 2 ;:::;A n 으로나눈다고하고, 임의의영역 A 에대해 A 의넓이를 S(A), 그영역의내부및둘레의선분의길이의합을 d(a) 로나타내기로하자. 각
244 조합 d(a) 를모두합하면정사각형둘레에있는것들은한영역의경계로쓰이고, 정사각형내부에있는것들많아야두영역의경계로쓰이므로, X d(ai ) 2 18 + 4 = 40 이된다. 임의의영역 A 에대해, A 를포함하는가장작은 ( 정사각형의변에평행한변으로된 ) 직사각형을 A 0 으로나타내자. 그럼 S(A) S(A 0 ); d(a) d(a 0 ) 이성립한다. 또, A 0 의두변의길이를 x, y 라하면산술 - 기하평균부등식으로 d(a 0 )=2x +2y 4 p xy =4 p S(A 0 ) 이성립한다. 따라서, 위의식들을종합하면 X p S(Ai ) X qs(a 0 i ) 1 X d(a 0 4 i ) 1 X d(ai ) 1 40 = 10 4 4 그런데, 모든영역의넓이의합은정사각형과같아 X S(Ai )=1 이다. 즉, b i =10 p S(A i ) 으로치환하면 X b 2 i =100; X bi 100 의상황이된다. 귀류법을쓰자. 모든영역의넓이가 0.01 보다작다면 b i < 1 이된다. 그럼 b 2 i <bi 이다. 이것은 100 = X b 2 i < X b i 100 으로 100 < 100 이되므로모순. 따라서, 넓이가 0.01 이상인영역이적어도하나있다. 57. n 명의사람이둥근탁자주위에앉아있다. 바로오른쪽에앉은사람이자신과성별이같은사람의수와바로오른쪽에앉은사람이자신과성별이다른사람의수가같다고한다. n 이 4 의배수임을증명하여라. ( 독일 BW 1973 1 차 -4) 증명이웃한두사람의짝을모두고려하자. 한사람을지정해그사람으로부터출발하여한바퀴를돌아자기자신으로돌아올때까지성별이짝수번바뀌어야원래의성별로돌아올수있으므로, 성별이다른짝의수는짝수개 (2a개라하자 ). 또한문제에서성별이같은짝의수와성별이다른짝의수가같다고했으므로, 짝은모두 2a +2a =4a개. 짝의수는사람의수와같으므로 n =4a. 별증성별을 1 로나타내면 a i a i+1 =1 인경우와 1 인경우의 i 의개수가같다. 즉, S = P a i a i+1 = 0. 여기서한 a k 의부호를바꾸면 S 의항중둘의부호가바뀌는데, f 1; 1g!f1; 1g 이거나 f 1; 1g! f1; 1g 이거나 f1; 1g!f 1; 1g 과같이바뀌고, 그럼 S 는각각 4 증가, 불변,4 감소이므로, S 가 mod 4 로불변임을알수있다. 이렇게한항씩부호를바꾸는일을반복하여모든항을 +1 로바꾸면그때의 S 의값은 n 이고이것이처음의 S 의값 0 과 mod 4 로같으므로 n 은 4 의배수. 58. 갑과을은각각 n 장의카드를갖고있다. 각자 1 부터 2n 까지의자연수를자기카드의각면에하나씩마음대로적어넣었다. 두사람의카드 2n 장을모아탁자에늘어놓는데,1 부터 2n 까지의모든수가윗면에보이도록할수있음을보여라. ( 아벨콘테스트 1996 결선 3) 59. n 과 m 은고정된자연수들이다. 꼭지점이 A =(0; 0), B =(n; 0), C =(n; m), D =(n 1;m), E = (n 1; 1), F =(0; 1) 로주어진육각형 ABCDEF 를 n + m 1 개의단위정사각형으로분할하였다. 이그림의격자선을따라각점을중복하여들르는일없이 A 에서 C 로가는경로의수를구하여라. ( 오스트리아 - 폴란드 1979-7) 60. p 는음이아닌정수이고 n =2 p 이다. 집합 f1; 2;:::;ng 의모든부분집합들중에서 x 2 A 인모든 x 에대해 2x =2 A 인부분집합 A 는최대몇개의원소를갖는지구하여라. ( 프랑스 1991-4)
4.2 조합고급문제 245 61. P 와 Q 는 n n 크기의체스판에서가로로이웃한두칸으로, P 가 Q 의왼쪽칸이다. 말하나를 P 에서출발시켜체스판을돌아다니게하려고한다. 허용된움직임은오른쪽이나윗쪽, 혹은좌하쪽으로한칸을이동하는것이다.( 예를들어,E5 에있었다면 E6 이나 F5, 또는 D4 로이동할수있다.) 어떤 n 에서도이말이체스판의모든칸을딱한번씩들러 Q 에도착할수는없음을증명하여라. ( 독일 BW 1971 1 차 -4) 62. 2000 개의사과가몇개의양동이에담겨있다. 여기서몇개의양동이를없애고다른몇몇양동이들의사과일부를없앨수있다. 또는몇개의양동이만없애거나몇몇양동이에서사과일부만없앨수도있다. 사과가 100 개이상남아있으면서남아있는모든양동이에같은수의사과가담겨있도록할수있음을증명하여라. (Towns 1987 가을 J6) 증명각양동이에담긴사과의개수를 a 1 a 2 a m 이라하자 (m 2000). a 1 100 이면바로끝. a 2 50 이어도바로끝. 이런식으로 a i 100 i 인 i =1; 2;:::;m 이있으면바로끝. 따 라서, a i < 100 i 일때만보면됨. 그럼 2000 = a 1 + a 2 + + a m < 100(1 + 1 2 + 1 3 + + 1 m ) 100(1 + ( 1 2 + 1 2 )+(1 4 + 1 4 + 1 4 + 1 4 )+(1 8 + + 1 8 )+ +( 1 2 10 + + 1 2 10 )=100 11 로모순. 63. 주어진 n 2 N 에대해, A 를 f1; 2;:::;ng 의부분집합들을일부모은집합이라고하자. 모든두집합 B;C 2A 에대해서집합 (B [ C) n (B \ C) 가짝수개의원소를가진다고할때, A 의원소는최대몇개인가? 단, X n Y 는차집합을뜻한다. ( 체코슬로바키아 1986-1) 풀이 jbj + jcj 가항상짝수라는것과같고, 이것은다시 jbj들의홀짝이모두같다는것. 원소가홀수개 ( 혹은짝수개 ) 인부분집합의개수는항상 2 n 1 개임이잘알려진...( 일대일대응으로증명가능 ) 64. 직교좌표계의제 1 사분면이격자선들에의해무한개의단위정사각형칸으로분할되어있다. 각각의칸에자연수를하나씩써넣는데, 각행과각열마다모든자연수가꼭하나씩있도록할수있는가? (Towns 1988 봄 JA6) 풀이할수있다. 원점에이웃한칸에먼저 1을쓰고, 귀납적으로다음과같이하면된다 : 원점에이웃한 2 n 2 n 정사각형의꼴을 A라하면, B =2 n + A (A의각항에2 n 을더해얻은꼴 ) 이라할때원점에이웃한2 n+1 2 n+1 정사각형은 B A 꼴이되도록하면됨. A B 65. A<B 인모든자연수순서쌍 (A; B) 들의집합을생각하자. 이쌍들중일부를 ` 검은색 ' 으로표시하고나머지는 ` 흰색 ' 으로표시하였다. A, A + D, A +2D 꼴의임의의세자연수에대해, 세쌍 (A; A + D), (A; A +2D), (A + D; A +2D) 중에항상두가지색이모두있도록색칠할수있는가? (Towns 1987 가을 S5) 풀이할수있다. Y X 의소인수분해에서 2의지수가짝수이면 (X; Y ) 를흰색, 홀수이면검은색으로하면됨. 66. 체스판의어떤칸에말이하나놓여있다. 두명이번갈아이말을다른칸으로옮기는게임을한다. 규칙은바로전에움직인거리보다항상더먼거리로움직여야한다는것이다 ( 말은항상각칸의중심에만놓을수있다 ). 더움직이지못하는사람이진다. 두사람모두최선의전략으로게임을한다고할때, 누가이기겠는가? (Towns 1989봄 JA2) 풀이대칭성의게임 67. 연습장한페이지를 ( 충분히많은 ) 작은정사각형칸들로분할하여 23가지의색으로칠하였다 (23가지의색을모두사용해야한다 ). 서로다른색의두칸이이웃하면그두색을 `좋은쌍 ' 이라부르자. 좋은쌍은최소몇개인가? (Towns 1988가을 SO4) 풀이그래프가연결되어야하므로변은최소 22개. 지층처럼나누어 23개의색을차례로쓰거나, 하나의색을바탕색으로하고나머지색을군데군데한칸씩쓰면됨. 68. 3 3 3 루빅큐브의표면에있는각각의작은정사각형칸마다대각선을하나씩그려, 스스로와만나지않는한꺾은선경로를얻을수있는가? (Towns 1989봄 SO4) 69. y = ax 2 + bx + c 꼴의이차함수 100개의그래프로좌표평면을분할할때, 가능한영역의개수의최대값은얼마인가? (Towns 1990봄 JA1)
246 조합 풀이두이차함수는많아야두점에서만난다. k개의이차함수를그렸을때분할된영역의수를 a k 라하면, 우선 a 1 =2이고, k +1번째이차함수를그리면이이차함수의그래프가앞서그려진 k개의이차함수와각각두점씩최대2k개의점에서만나고, 그럼그교점들에의해 k +1번째곡선이 2k +1개의조각으로나뉘어지는데, 각조각은원래하나였던영역을둘로나누게되므로영역의개수는최대 2k +1개늘어난다. 즉, a k+1 a k +(2k +1). 이점화식을풀면 a k k 2 +1 이된다. 등호가성립하는경우인모든이차함수가서로두점씩만나고모든교점이서로다를때는쉽게구현할수있다. n =1; 2;:::;100 에대해p n : y =100 n x 2 n 정도로하면충분하다. 답 100 2 +1 70. 신발 30짝이일렬로놓여있다.15짝은오른발용이고 15짝은왼발용이다. 왼쪽 5짝과오른쪽 5짝으로된연속한 10짝을찾을수있음을증명하여라. (Towns 1991봄 JO3) 증명 i번째신발이오른발용이면 a i =0, 왼발용이면 a i =1로나타내자. s i = a i +a i+1 + +a i+9 로정의할때, s 1 ;s 2 ;:::;s 21 중에서딱 5인것이있음을증명하는문제가된다. 귀류법으로, 딱 5가되는것이없다고하자. s i 는 i가증가함에따라연속적으로변화하므로 (a i 와 a i+10 이같으면 s i = s i+1 이고다르면 js i+1 s i j =1), 딱 5가되는것이없다면모두다4 이하이거나모두다 6 이상이다. 모두 6 이상인것은 0과 1의정의를뒤바꾸면마찬가지이므로 4 이하일때만풀면충분하다. 그럼 X := s 1 + s 2 + + s 21 21 4 =84. 그런데 X =10(a 1 +a 2 + +a 30 ) (9a 1 +8a 2 + +a 9 ) (a 22 +2a 23 + +9a 30 ) 이다. 이것을 X =150 L R이라놓으면, 먼저 a 1 ;:::;a 9 중에는 1인것이4개이하이므로 L 9+8+7+6 = 30, 마찬가지로 R 30. 따라서, X =150 L R 90 으로앞에서얻은 X 84 와모순이다. 71. 주어진자연수 N 에대해, a + b + c = N 을만족하는서로다른자연수해 (a; b; c) 를생각하자. 이런순서쌍몇개를고르는데, 어떤두쌍에도같은수가없도록한다. 이렇게고를수있는최대의개수를 K(N) 이라할때, 다음을증명하여라. (a) K(N) > N 6 1 (b) K(N) < 2N 9 (Towns 1989 가을 JA4) 72. 평면의각점을정확히 1988 개의원이지나도록하면서평면을원들로가득덮을수있을까? (Towns 1988 봄 JA5) 풀이 1988 대신 2로바꿔놓았을때성공하면충분하다 ( 그작업을 994번반복하면되므로 ). 그리고이것은, 무한개의평행선을그린후각각의이웃한두평행선사이에서그두직선에동시에접하는모든원을다모아놓으면된다. 73. 8 8 체스판을다음과같은조건으로만들때최소몇개의칸을칠해야하는가? (a) 색칠된두칸은접할수 ( 변이나점을공유할수 ) 없고, (b) 색칠되지않은칸은어떤색칠된칸과접해야한다. (Towns 1988 가을 SA1) 74. 100개의계단이있다. 롱다리 K군이이계단을내려가려하는데,6계단혹은 7, 8계단점프 (k계단점프란 k 1개의계단을넘어 k번째계단에도착하는것을말한다 ) 로만오르락과내리락을한번씩번갈아하면서같은계단을두번밟지않고내려갈수있는가? (Towns 1989봄 JA1) 75. A 와 B 가 N 개의성냥개비를가지고게임을하였다. 먼저 A 가성냥개비들을각더미가두개이상이되도록두더미로나눈다. 다음 B 가두더미를각각둘로나누는데모두 1 개이상이되도록나눈다. 그리고 B 가먼저 4 개의더미중정해진규칙에따라 2 개를갖고 A 가나머지를갖는다.A,B 는모두자기가최대한많은성냥개비를갖도록더미를나누고선택한다. 규칙은다음의 3 가지중어느하나로이루어져있다. (1) B는가장큰더미 ( 둘이상일경우엔그중하나 ) 와가장작은더미 ( 둘이상일경우엔그중하나 ) 를갖는다. (2) B는가운데의두더미 ( 경우 1일때A가갖게되는두더미 ) 를갖는다. (3) B는가장큰더미와가장작은더미를갖거나, 두개의가운데더미를갖는것에대한선택권이있다. 각각의규칙에대해 A, B 모두최선의경기를했을때 A 가갖게되는성냥개비는몇개일까? ( 소련 1961-10)
4.2 조합고급문제 247 풀이 v = b N 2 c, w = d N e 2 을나타내는것으로하자. v + w = N 임에유의하자. (1) 답은 v이다.a가나눈두더미를x y 라하면B는 x를 x 1과 1로나눠x를다가져갈수있다. 따라서,B는항상w 이상가져갈수있다. 반대방향을보자.A가x y =0또는 1이되도록나누면,B가한쪽에서두더미를다가져가는경우는 B가 w 이하만가져가게되므로, 양쪽에서한더미씩가져가는경우만살펴보면된다. x = a + b, y = c + d 이고최대더미와최소더미가 a, d라면, a + d a + b = x. 최대더미와최소더미가 c, b일때도c + b c + d = y. 어느경우나 B는 w 이하만가져갈수있다. 즉, 둘다최선을다한다면 B는 w만큼, 그리고 A는 v만큼가져가게된다. (2) 답은 w이다.a가n 2와 2로두더미를나누면 1인더미가무조건둘생기므로중간의두더미는기껏해야 b N 2 c +1=v 2 개이다. 한편,A가x y 로나누면B는양쪽더미를각각최대한절반에가깝게나누면중간의두더미의합은 v 이상이됨을확인할수있다. 따라서,B는v개를가져가고,A는w개를가져가게된다. (3) 답은 v이다.b는(1) 의전략을취한다면당연히 (1) 에서의결과인 w 이상을가져가는것이보장된다.A가 x y =0또는 1이되도록나누면,B가한쪽에서두더미를다가져가는경우는 (1) 과마찬가지이므로, 역시양쪽에서한더미씩가져가는경우만살피면된다. 또한 (1) 에서살펴보지않은,B가중간두더미를선택하는경우만살피면된다. 그경우에도 (1) 에서와비슷하게할수있는데, 최대더미와최소더미가 a, d라면 b + c b + a = x, c, b라면 a + d c + d = y 여서역시중간더미의합도 w 이하이다. 76. 총합이같은양수들의집합이두개있다. 첫번째집합은m개의수를가지고있고두번째집합은 n개의수를가지고있다. 이때m n 크기의행렬에 m + n개보다적은수를써넣어각행의합이첫번째집합과같고각열의합이두번째집합과같도록할수있음을증명하여라. 예예를들어 n(a) =3,n(B) =2,A = f1; 5; 3g, B = f2; 7g 일때다음과같이채울수있다. ( 소련 1962-14) 2 3 5 4 15 2 1 증명그냥첫행 ( 최초의수만제외하고 ) 의수를각칸마다그칸이속한열의합 ( 두번째집합의수들 ) 과같은수로하고마찬가지로첫열 ( 최초의수만제외하고 ) 의수를각칸마다그칸이속한행의합 ( 첫번째집합의수들 ) 과같은수로하면첫행첫열에해당하는칸의수는행합이든열합이든어느한쪽으로정하면나머지한쪽은두집합의총합이같다고했으므로저절로성립함. 77. 정육각형 A 1 A 2 A 3 A 4 A 5 A 6 의모든변과대각선을파란색또는빨간색으로칠했다. 단, 어떠한삼각형 A j A k A m (1 j<k<m 6) 도세변이모두파란색인경우는없다. 각각의 k =1; 2;:::;6 에대해빨간선분 A k A j (j 6= k) 의개수를 R k 라할때, 다음을보여라. ( 호주 1984-4) 6X (2R k 7) 2 54 k=1 증명 m =min(r k ) 라두면, m 2 일때는 (2R k 7) 2 3 2 으로바로끝. m =0,m =1일때만별도로경우를잘확인해주면됨. 각각 A 49+1+1+1+1+1=54, A 25+1+1+1+1+25=54 의꼴이됨. 78. Patera 라는나라에는 20 개의도시와이들도시간의교통을제공하는두항공사 Green Planes 와 Red Planes 가있다. 비행노선은다음과같이주어져있다. (i) 임의의두도시간의비행노선은오직한항공사에의해서만운영되고, 노선이없는경우는존재하지않는다. 이때비행기는두방향모두운행하며중간에멈추는일은없다. (ii) Patera에는 Red Planes만을이용해서는 ( 중간에다른도시를경유하더라도 ) 오갈수없는두도시 S, B가존재한다. Patera 의어떠한두도시에대해서도, 중간에다른도시를최대한곳만경유하면 Green Planes 만을이용하여이두도시를오고갈수있음을보여라. ( 호주 1987-3) 증명 X, Y 를임의의두도시라하자. XY 가 G면끝이므로 XY 가 R일때만생각. XSY 나 XBY 가 GG이면역시끝이므로 R이각각하나이상있고그럼S, B 사이에 R만의경로가생기게되어모순.
248 조합 79. (a) 14 개의톱니를가진두개의똑같은톱니바퀴가있다. 하나가다른하나의위에톱니가정확히포개어지도록얹어놓고, 대응하는네쌍의톱니를임의로골라서잘랐다. 윗바퀴를잘돌려서두바퀴의그림자가완전한하나의톱니바퀴가되도록항상할수있는가? ( 톱니바퀴하나를뒤집어서포개는것은허락되지않는다.) (b) 13 개의톱니로하고 (a) 를다시풀어라. (Towns 1988 가을 JA4) 풀이 ( 이윤한 ) (a) 두톱니바퀴가포개어져있을때위에있는잘린 4 개의톱니를순서대로 a, b, c, d 라하고아래있는잘린 4 개의톱니는위에 a, b, c, d 와대응하는대로 a 0, b 0, c 0, d 0 이라하자. 그림자가완벽한톱니바퀴모양을하지않을때는처음 a, b, c, d 모두 a 0, b 0, c 0, d 0 와겹치는때 1 가지, a 가 b 0, c 0, d 0 과포개지는 3 가지, b 가 a 0, c 0, d 0 과포개지는 3 가지, c 가 a 0, b 0, d 0 과포개지는 3 가지, d 가 a 0, b 0, c 0 과포개지는 3 가지, 총 13 가지경우이상이될수없다. 따라서톱니가 14 개일때는가능하다. (b) 두톱니모두에서 1, 2, 4, 10 을자르면된다. 80. 모든변의길이가 1 이고마주보는변끼리모두평행한 2n 각형을 \rhombic" 이라한다. n =4 일때의아래의예시그림처럼, 이런 rhombic 2n 각형은한변의길이가 1 인마름모들로서로다른몇가지방법으로분할할수있다. 666 개의마름모로분할할수있는 rhombic 2n- 각형이존재하는 n 은어떤값들인가? (IMTS R11-3) 풀이 ( 송지용 )2n각형이 666개의마름모들로나누어진그림을생각하자. 이그림에서전체선분들의개수는 (666 4+2n) =36 37 + n 2 개이다 ( ). 2n각형위에서임의로한변 ( 기준꼭지점에서시계방향으로 k번째변 ) 을잡아이것과평행한선분들을갖는모든마름모들의집합을 P k, k =1; 2;:::;n이라하자. 그러면 P k 는자기자신을제외한나머지 n 1개의 P m, m 2f1;:::;ng fkg들과꼭하나의마름모를공통원소로갖는다. 즉, 각각의 P k 에는 n 1개의마름모가있고, k번째변 ( 임의의변 ) 과평행한선분은각각 n개씩있다. 따라서여기서알수있는전체선분들의개수는 n 2 개다 ( ). ( ) 과 ( ) 로부터다음과같은결론을얻을수있다. n 2 = n +36 37 ) n =37. 81. n n 체스판이주어져있다 (n 3). 凸모양을이루는어느네칸의색을동시에바꾸는 ( 흰색을검은색으로, 검은색은흰색으로 ) 조작이허용된다. 이런조작을유한번반복하여체스판의모든칸의색을원래의색과다르게바꾸는것이가능한 n을모두구하여라. ( 러시아 1990 4차-y9-3) 풀이홀수일땐불가. 짝수일땐항상가능.4의배수일땐너무쉽고, 이웃한두칸뒤집기가항상다음의 3combo로되기때문에. XXA OBCC OOC 위와같은그림일때 [XXAB]+[ACCC]+[BCCC] 콤보로해결. 사실 [ACCC]+[BCCC] 만으로도꽤의미가있는아이템 ( 대각두칸뒤집기 ) 이지만코너에서좀귀찮아져서 ; 82. 한변의길이가12인정사각형의내부에 1990개의점이주어져있다. 그중498개이상의점을내부에포함하는, 한변의길이가11인정삼각형을그릴수있음을증명하여라. ( 러시아 1990 4차-y9-4) 증명밑변에서 1+p 3 2 만큼밖으로미끄러뜨린정삼각형을깔면중심을포함하는한 4등분된사각형 을덮게됨.
4.2 조합고급문제 249 83. n 장의카드가한곳에쌓여있는데, 일부는위를향하고있고일부는아래를향하고있다. 위에서부터몇장의카드한묶음을집어그묶음을뒤집은후다시카드더미의맨위에놓는작업을할수있다. k 번작업하면초기상태에상관없이항상모든카드가아래를향하게할수있다면, k 의최소값은얼마인가? ( 러시아 1993 4 차 -y9-4) 풀이맨밑의, 아래로이미잘되어있는카드들은고려에서제외하고, 그이외의카드들을같은방향으로연속하여놓여있는것들끼리한덩어리라고부르자. 한번작업할때마다덩어리의수는최대 1개감소한다. 따라서, 구하는 k는이웃한카드가항상서로다른방향이고맨밑카드가위를향했을때의n번. IT에서출제했던 ABCBABCB...ABCBA 문제랑비슷한듯. 84. 8 8 체스판에서 16 개의칸을고르는데, 각행마다 2 칸씩, 각열마다 2 칸씩되도록택했다. 이 16 개의칸에 8 개의흰졸 ( 말 ) 과 8 개의검은졸을놓는데, 체스판의각행과열마다 1 개씩놓이도록할수있음을증명하여라. ( 헝가리 1933-2) 증명같은행, 같은열의칸마다 edge 로이으면몇개의짝수길이 cycle 이됨. 85. 좌표평면에무한개의직사각형을모은집합을갖고있는데, 이직사각형들은모두적당한자연수 m, n 에대해 (0; 0), (0;m), (n; 0), (n; m) 을네꼭지점으로갖는꼴들이다. 이집합의직사각형들중에하나가다른것에포함되는경우가있음을증명하여라. ( 헝가리 1934-3) 증명 (n; m) 직사각형을하나잡으면이것을포함하는녀석이없으려면나머지직사각형들은모두 x-좌표가 n보다작거나 y-좌표가 m보다작다. 그럼 x-좌표나 y-좌표가같은두녀석이있다. 86. 삼각프리즘 ( 삼각기둥 ) 의각꼭지점마다실수가하나씩배정되었는데, 각꼭지점의수는변으로이웃한세수의평균이된다고한다. 여섯개의수가모두같음을증명하여라. ( 헝가리 1935-3) 증명여섯개의수중가장작은수 x가있다. x가그에이웃한세수 ( 모두 x 이상 ) 의평균이되려면이웃한수도모두x일수밖에없다 ( 아니면평균이 x보다커지므로 ). 같은논리를이웃한수로옮겨가서계속하다보면여섯개의수가모두같을수밖에. 87. 평면위에어느세점도한직선위에있지않은 n 개의점이있다. 이점들에임의로 A 1 ;A 2 ;:::;A n 로어떻게번호를붙여도, 꺾인선 A 1 A 2 A n 은자기자신과만나지않는다고한다. 이것이가능한 n 의최대값을구하여라. ( 주니어발칸 2003-2) 풀이볼록사각형을이루는네점이존재하면안됨. 점 5개면항상볼록사각형을이루는네점이존재하므로 n =4가최대. 88. 평면위의격자점 ( 정수좌표의점 ) 들에대해생각하자. 유리수기울기를갖는직선들에대해다음을보여라 : (i) 이런직선은격자점을전혀지나지않거나무한히많은격자점을지난다. (ii) 각각의직선에대해다음을만족하는양수 d가존재한다 : 이직선위의점들을제외하고는어떤격자점도이직선에이르는거리가 d보다가깝지않다. (Putnam 1951-A5) 증명 ( 박경태 )(i) (1) 이직선이격자점을지나지않을때! 지나지않는다. (2) 이직선이한번이라도격자점을지날때귀류법으로이직선이 p번만격자점을만난다고가정하자. 기울기 q p 에대해서이직선이점 (a; b) 를지난다고할때 ( 이때의점은가장오른쪽상단에있거나오른쪽하단에있는점이라하자 )(a + p; b + q) 도이직선을지난다. 따라서우리의가정이틀렸고이직선이한번이라도격자점을지나면무한히많은격자점을지난다. (ii) (1) 이직선이격자점을지나는경우 : 기울기 q p 일때가로 p, 세로 q 직사각형을그려 1단위로다자른뒤에대각선을하나그린다. 그리고직선주위의격자점에서직선에수선의발을내려길이가가장짧은것을택한다. 그것이 d의최대값이다.
250 조합 (2) 이직선이격자점을지나지않는경우 : 기울기 q p 일때가로 p, 세로 q 직사각형을그려 1단위로자르되, 이직선이 x축과만나는점에서왼쪽격자점까지의거리를직사각형그림에덧붙이고오른쪽에는그길이만큼감하여대각선을그린뒤 (1) 과같이한다. 89. 어느시골의한초등학교에스무명의아이들이다닌다. 그중어떤두아이도할아버지 ( 외할아버지와친할아버지 ) 중에같은분이있다. 이학교에적어도 14명의손자손녀가있는할아버지가적어도한명있음을보여라. (Towns 1994가을 JO4) 증명 2명의학생의할아버지쌍이 (A; B), (A; C) 라하면 (A; D) 꼴이또있으면그다음부터모순이므로할아버지는 A, B, C의 3명뿐. 비둘기집. 90. 체스판의각행, 열, 대각선마다짝수개씩의졸이있게하려면졸은최대몇개까지놓을수있는가? ( 러시아 1993 최종 -y9-7) 풀이필요조건 : 홀수길이의대각선마다하나씩은빼야함. 충분조건 : 두주대각선만싹비운그림이 48개의한예가됨. 91. 거짓말쟁이와참말쟁이들의국제회의에 32명이참가하여, 각줄에8명씩네줄로앉아있다. 쉬는시간동안각각의참가자들은모두자신과인접한 ( 앞뒤혹은좌우로 ) 자리에거짓말쟁이도있고참말쟁이도있다고주장을했다. 거짓말쟁이는최소몇명인가? ( 러시아 1994 4차-y9-4) 풀이 BbCEEeGG BCCcEF-g a-cdfffh AAdDDFhH 위와같이 A, B, C, D, E, F, G, H 8 개영역으로분할하면각영역마다거짓말쟁이가적어도한명은있어야하고,8 명을실제배치하는경우는위의소문자위치에배치하는경우가있음. 92. 집합 M = f1; 2;:::;2ng (n 2) 을서로소인 k 개의집합 M 1 ;M 2 ;:::;M k 로분할하였다. 단, k 3 +1 n 이다. k +1 개의짝수 2j 1 ; 2j 2 ;:::;2j k+1 이한집합 M j 에속하고,2j 1 1;:::;2j k+1 1 도한집합 M r 에속하는 (1 j; r k) 경우가있음을증명하여라. ( 불가리아 1979 4 차 -6) 증명비둘기집. k 2 +1개이상의짝수를갖는집합이있고, 이짝수들에서 1을뺀홀수들중k +1개이상을갖는집합이또있다. 93. 1 보다큰정수들로이루어진무한수열 fa n g 의항들이모두서로다르다고한다. a k >k 를만족하는 k 를 100 개이상찾을수있음을보여라. (Towns 1982 봄 J3) 증명무한히많이찾을수있겠지. 저런 k개유한개라고가정하고모순을이끌어내면됨. 저런 k가모두 N 이하라하고, a 1 ;:::;a N 의최대값을 M이라하면 (M >N임은자명 ), a 1 ;:::;a M 은모두M 이하의수들이되어모순. 별해 a k >a k 1 ;:::;a 1 인 k 가무한히많음을이용할수도. 94. 모든항이 0 또는 1 인 m n 크기의행렬들의집합을생각하자. 이런행렬들중에서각각의행과열에속한 1 의개수가모두짝수인행렬의개수를구하여라. ( 아일랜드 1994-4) 풀이 m 1 개의행을결정하면나머지한행도자동적으로결정됨.2 (m 1)(n 1) 개 95. a 와 b 두문자로만이루어진어떤이상한언어가있다. 문자 a 는하나의낱말이라고한다. 또한, 다른모든낱말들은다음과같은규칙을따라만들어진다. 1. 임의의낱말로부터맨오른쪽에 b 를추가하여새로운낱말을만들수있다.
4.2 조합고급문제 251 2. aaa라는문자열을포함하는임의의낱말로부터그하나의 aaa를 b로바꾸어새로운낱말을만들수있다. 3. bbb라는문자열을포함하는임의의낱말로부터그하나의 bbb를제거하여새로운낱말을만들수있다. 4. 임의의낱말로부터그낱말을두번이어써서새로운낱말을만들수있다. 예를들어,4에의해aa는낱말이고, 다시 4에의해aaaa는낱말이다. 그리고,2에의해ba는낱말이고, 다시 1에의해bab 또한낱말이다. 다시, 1에의해babb는낱말이고, 4에의해babbbabb는낱말이다. 마지막으로 3에의해baabb는낱말이다. baabaabaa는낱말이아님을증명하여라. (IMTS R44-1) 증명 a 의개수가 3 의배수인것은나오지않는다. 96. 아래의상황에서한변의길이가 7 인정사각형 K 를한변의길이가 3 인정사각형여덟개로덮는것이가능한가? (a) 작은정사각형의모든변이항상큰정사각형의변과평행해야한다. (b) 작은정사각형의변이큰정사각형의변과꼭평행할필요는없다. ( 유고슬라비아 1980 고2-3) 풀이 (a) 3.5 간격으로 3 3개의점을찍으면한정사각형이두점을덮을수없음.(b)4개는코너에덮고, 남은 4개는 45도기울여서남은십자영역을 1/4씩덮으면됨. 97. 11 11 수판에 1; 2; 3;:::;121의수를일대일로써넣는다. 연속하는두수는항상서로이웃한칸에들어가게하면서모든완전제곱수가한열에있도록할수있는가? ( 러시아 1995 4차-y9-8) 풀이 1부터 121까지차례로여행한다고생각하자. 홀수는검은칸, 짝수는흰칸. 제곱수열에는제곱수말고다른수는들어갈수없으므로 3 4 5, 8 9 10 등은항상행으로연속하게놓여야하고, 그럼제곱수열은홀수번째열 ( 제곱수에는홀수가하나더많으므로 ) 이면서양끝열일수없으니제3, 5, 7, 9열중의하나이다. 제곱수에서는제곱수열을항상가로질러가야하므로 2-3, 10-15, 26-35, 50-63, 82-99 등 50개의수가제곱수열로나뉜한쪽영역에있어야한다. 그런데 50개는 11개의배수가아니므로모순. 98. P 를볼록n각형이라고하자. P 의각각의꼭지점으로부터그점에이웃하지않은 P 의 n 2개의변 ( 혹은그연장선 ) 에수선을내렸다. 이수선들중적어도하나는 P 안에완벽하게들어가있거나혹은 P 의한변임을증명하여라. (IMTS R24-5) 증명 ( 신한솔 ) AB를가장긴변이라하고 A, B를지나며 AB와수직인직선을 L1, L2라하자. L1과 L2사이에점이존재하지않는다고하자. 그럼직선 L1바깥쪽 (L1에대해B와반대쪽) 의점X, L2바깥쪽의점 Y 가있고XY 가 P 의한변이되는점X, Y 가존재하고그럼 XY 가 AB보다크므로모순. 따라서 L1와 L2사이에 ( 직선포함 ) 점K가존재한다. 그럼각KAB, KBA가예각또는직각이므로K에서 AB에내린수선은P내부에있다. P.S: 볼록다각형이므로삼각형 KAB는 P 내부에있음 99. 유리수들을뿌리나무 ( 시작점이있는수형도 ) 꼴로나열하는데, 각각의유리수 a 마다 (a와 b는서로소 ) 두 b 개의자손 a a + b 와 b 가있도록하려고한다.0<r<1 인모든유리수r이딱한번씩나열되게하려 a + b 면뿌리 ( 시작점 ) 에는어떤유리수를놓아야하는가? 그리고, 주어진유리수 p 가뿌리로부터몇번째단 q 계 ( 층 ) 에있는지알아내는절차를설명하여라. ( 독일BW 1976 1차-3) 풀이 a=(a + b) < 1=2, b=(a + b) > 1=2 임에주목. 시작점에는 1=2. p=q > 1=2 이면그위에는 (q p)=p 가, p=q < 1=2 이면그위에는 p=(q p) 가부모. 이렇게거슬러올라가면서 1=2 이될때까지의횟수를세면됨. 100. 대회에참가한사람들중어느네명을골라도그중어느한명은나머지세명을모두안다고하자. 이대회에참가한다른모두를아는참가자가있음을보여라. 단,A 가 B 를알면 B 도 A 를아는것으로한다. ( 몰도바 1997 최종 -y9-7)
252 조합 증명우선참가자수는 4명이상임이자명하고,4명일때는성립함이자명. 성립하지않는경우가있다고하고, 그런가장적은인원을 n명이라하자 (n 5). 이 n명중가장많은사람들을아는사람을 A라하자.A를제외한나머지 n 1명만생각하면그안에서도문제의조건이잘만족되고 n 1은성립하는수이므로, 이 n 1명중다른모두를아는사람 B가존재한다. 그리고, n은성립하지않는인원이므로 AB가서로모르고,A의최대성에서 A는 B가아는모두를안다.ABCD를택하면 A와 B는나머지세명을모두아는사람이아니므로 CD는서로알고,C,D는임의였으므로 A와 B를제외한나머지모두가서로안다. 그럼나머지모두가다른모두를아는참가자가되어끝. 귀납법 + 유한극값 101. 스스로와교차하지않는길이가 1000 보다큰꺾인선이주어진단위정사각형안에있다. 이단위정사각형의변들중하나와평행하면서이꺾인선과적어도 501 개의점에서만나는직선이항상존재함을보여라. ( 독일 BW 1971 2 차 -4) 증명다음과같이 AB가있을때이것을지우고대신CD와 EF로대체한다고생각하자. 그래도변에평행한직선이주어진선분들과만나는점의수는변함이없다. 또한 AB CD + EF. 꺾인선들은 P AB 의합이므로 1000 X AB X CD + X EF 일반성을잃지않고,500< P CD 라하면 CD 가속해있는변중 501 개의선분이자신위에있는점이존재한다. 102. K 는자연수 M 의자릿수들을재배열하여얻은자연수이다. 다음을증명하여라. (a) 2M의자릿수의합과 2K의자릿수의합은같다. (b) M과 K가둘다짝수이면, M=2의자릿수의합과 K=2의자릿수의합은같다. (c) 5M의자릿수의합과 5K의자릿수의합은같다. (Towns 1983봄 JO4) 증명 (a) 각자릿수를 2배했을때십의자릿수는기껏해야 1, 일의자릿수는최대 8이므로, 서로다른자리에있던수의 2배가중첩되어 carry가발생하지않으니까, 그냥각각의자릿수를그자리에서 2배한것의자릿수의합을각각구한후더한것이나마찬가지라서. (b) 홀수인자릿수는그아랫자리에 1을꿔준다음생각하면 (a) 에서생각한것과마찬가지. (c) (a) 랑비슷하게, 각각의자릿수별로분해해서더해도그만. 자릿수의 5배는십의자리가최대 4, 일의자리가최대 5이므로중첩되어 carry가발생하는일이없음. 103. 어떤증가하는정수수열이홀수항으로시작하고짝수항, 홀수항, 짝수항, 이렇게짝수와홀수가교대되며나타날때, 이수열을교대적이라고한다. 항을하나도갖지않는공수열도교대적인것으로인정하기로하자. 집합 f1; 2;:::;ng 에있는수들만이허용된교대적수열들의개수를 A(n) 이라하자. A(1) = 2 이고 A(2) = 3 임을보여라. A(20) 의값을구하고, 그것이옳음을증명하여라. ( 영국 1994 1 차 -5) 증명 ( 이종민 + 김규완 ) 짝수부터시작하는연속된 n개의자연수중에서숫자들을뽑아수열을만들때증가하는수열이고짝수항으로시작하고홀수항, 짝수항이렇게짝수와홀수가교대되게만드는수열을 B(n) 이라하자 (n은 0 이상 ). 집합f1; 2; 3; 4; 5g에서 `교대적 ' 수열을만들때수열의초항이 1일때,3일때,5일때로나눌수있다. 수열의초항이1일때뽑을수있는나머지남는수는 f2; 3; 4; 5g인데이들을뽑는경우의수는B(4) 이다. 마찬가지방법으로수열의초항이 3일때뽑을수있는수는 f4; 5g이고이들을뽑는경우의수는 B(2). 초항이 5일때는 B(0). 따라서 A(5) = B(4) + B(2) + B(0) 으로나타낼수있다. 이를일반화하면, A(n) =B(n 1) + B(n 3) + B(n 5) +
4.2 조합고급문제 253 ( 단, B(0) = 1). 그런데, 집합 f1; 2;:::;ng 의교대적수열에 +1 한수열은집합 f2; 3;:::;n +1g 에서짝수, 홀수순서대로뽑은것과일대일대응이므로 A(n) =B(n) 이성립한다. 따라서 A(n) =A(n 1) + A(n 3) + ( 단, A(0) = 1 이고괄호안의숫자가음수일때는생각하지않는다 ). A(n 1) = A(n 2)+A(n 4)+ 이므로대입하면 A(n) =A(n 2)+A(n 3)+A(n 4)+ +A(2)+A(1)+A(0). 이로부터 A(n) =A(n 1) + A(n 2) 를얻는다 ( 피보나치수열 ).23581321345589144233377 610 987 1597 2584 4181 6765 10946 17711 = A(20). 104. 평면위의 3 개이상의점으로이루어진, 다음을만족하는집합이존재하는가? (i) 이집합의어떤세점도한직선위에있지않다 ; (ii) 이집합의임의의세점에대해, 그세점을지나는원의중심이다시이집합에속한다. ( 브라질 1996-2) 풀이1 ( 강성경 ) 이점들의볼록포 K중의한변AB를택하자. 그러면직선 AB를기준으로한쪽평면에만나머지점들이존재하게된다. i) 만약 A, B외의점C가있어서 \ACB가둔각이라면문제는자명해진다.( 증명생략 )[ 외심이직선 AB의바깥쪽에있어야하므로, 한쪽평면에만점들이있다는것에모순 ] 만약 \ACB가직각이면그외심 O와 A, B는한직선위에있데되어모순. 만약모든 A, B가아닌C에대해 \ACB가예각이라고하면, ii) 만약 4ABC 가둔각삼각형이면,WLOG\CAB > 90. 이때 4ABC 의외심을 O 라하면, 삼각형 4AOB 에서 \ABO, \BAO < 90 인것은당연하다. 따라서 \AOB 가둔각이면 i) 의경우, 예각이면 iii) 의경우로넘어가게되어, 이경우는해결할필요가없어지게된다. iii) 4ABC 가예각삼각형이면, 4ABC 의외심 O 1 역시 4ABC 안에있다. 그리고 \ACB = µ 면 \AO 1 B =2µ 다. 이런작업을 n 번하여 \AO n B 가최초로둔각이되게하자. 이경우는 i) 번의경우와같으므로해결할필요가없다. 풀이2 ( 정명준 ) 이런집합 S가존재한다고하자. 처음, Sfa 1 ;a 2 ;a 3 g에는세점이속하고, 이세점은삼각형을이룬다. 이제, 네번째원소`a 4 ' 는삼각형의외심이므로, a 1 a 2 의수직이등분선위에있다. 또, 세점a 1 ;a 2 ;a 4 이이루는삼각형의외심`a 5 ' 은역시a 1 a 2 의수직이등분선위에있다. 이제세점a 1 ;a 2 ;a 5 이이루는삼각형의외심`a 6 ' 도 a 1 a 2 의수직이등분선위에있으므로, S의세원소 a 4 ;a 5 ;a 6 은한직선위에있다. 그럼 (i) 에의해이세점이모두다른점일수없다. 외심은원래삼각형의꼭지점일수없으므로, a 4 = a 6 만이유일하게가능하다. 각 a 1 a 4 a 2 = x라할때, 약간의중심각-원주각계산을통해 360도 2(360도 2x) =x의식을얻을수있고, 따라서 x =120도. a 2 a 3, a 3 a 1 도마찬가지로 a 1 a 2 처럼한점을더선택한후외심이수직이등분선위에있음을이용할수있으므로, 우선 S는 \a 1 a 4 a 2 = \a 2 a 4 a 3 = \a 3 a 4 a 1 =120 ± 인경우만빼고는존재할수없다 ( 즉, a 4 와 a 6 들이일치해서일직선위의세점 a 4 ;a 5 ;a 6 이나오지못하는경우임 ). 그러면, 4a 1 a 2 a 3 이정삼각형인경우만따로보자. 우선위와같이 a 5 까지를정의하자. 그럼 a 3 ;a 4 ;a 5 가한직선위의서로다른세점이되어모순. 즉처음세점이정삼각형을이룰때도 S가존재하지않는다. 즉이런집합S는존재할수없다. 105. 길을따라 (n 1) 2 개의단위블록으로구획된, 한변의길이가 n 1 인정사각형꼴의도시가있다. 왕복버스노선을정하려고하는데, 각노선은최대한번방향을바꿀수있고 ( 우회전또는좌회전 ), 임의의두교차로사이를많아도한번의환승이면갈수있도록하려고한다. 최소몇개의노선이필요한가? ( 러시아 1992 4 차 -y9-4) 풀이우하꼭지점위치를 A라하자. A와 i번째열을포함하는노선을 r i 로정하자. 그럼이 n개의노선으로우선충분함 ( 항상 A에서갈아타면됨 ). 만일노선이 n개보다적다면, 어떤노선도세로방향으로는지나지않는열이적어도하나있고, 그럼그열안의 n개의교차점은각노선이가로방향으로한번씩지나가야하므로적어도 n개의노선이필요함.
254 조합 106. 정12각형의모든변과대각선을, 주어진 12가지색깔중에서하나씩골라칠한다. 모든세가지색깔에대해서, 적당한세꼭지점이존재해서, 그세꼭지점으로이루어지는삼각형이그세가지색깔의변을모두갖도록할수있는가? ( 러시아 1995 4차-y9-4) 풀이 12 개의꼭지점에서세점으로된삼각형의개수 12 3 과 12 가지색깔에서세변을이룰세가지색을고르는방법의수 12 3 이똑같으므로, 각각의서로다른 3 색구성의삼각형이정확히하나씩존재함. 변 ( 및대각선 ) 의수는 12 2 =66> 5 12 이므로같은색의 6 개의변이존재하고, 같은색의두변을갖는삼각형이없으므로이 6 개의변은어떤둘도한점에서만나는법이없다. 즉, 이변을한변으로하는삼각형은각각 10 개씩총 60 개이상이되는데,3 색구성에서그색의변을갖는삼각형은 11 2 =55 개이므로모순. 비둘기집. 107. 어떤무용수업에서 15 명의남학생과 15 명의여학생이각각일렬로줄을서서 15 쌍이짝지어지도록하였다. 각각의짝에서남학생과여학생의키는기껏해야 10 cm 밖에차이가나지않았다고한다. 남학생과여학생을각각다시큰키순서로다시줄을세웠을때, 새로운짝들도남학생과여학생의키차이가항상 10 cm 이하가됨을증명하여라. (Towns 1984 봄 JA2) 증명 a 1 <a 2 < <a n, b 1 <b 2 < <b n 으로줄을세웠다고하자.WLOG,b i a i > 10 이라면 a 1 ;:::;a i ; b i ;:::;b n 의 n +1명을생각하면이중에짝이었던둘 a j, b k 가반드시있고 b k a j b i a i > 10 이므로모순. [ 주 ]2열횡대로섰는데항상앞사람이키가컸을때각행을키순으로배열하고나서도항상앞사람이키가크다는유명한문제의응용변형임. 108. 1 부터 9 까지의숫자를꼭한번씩써서만들어지는모든아홉자리의수들을생각하자. 이런두수의합이 987654321 이될때이두수를 ` 조건적인쌍 ' 이라고부른다. (a) 조건적인쌍이적어도둘있음을보여라. (b) 조건적인쌍은홀수개임을보여라. (Towns 1983가을 S2) 증명일의자리의합과십의자리의합이각각 11이므로, 일의자리와십의자리를둘다교환해도역시조건적인쌍. 109. (a) 20 20 크기의종이의각각의격자칸마다말이하나씩놓여있다. 바냐가수 d를고르고, 페탸는각각의말을동시에거리 ( 칸과칸사이의중심거리 ) d만큼의위치로이동시킨다. 바냐가수 d를고르고, 페탸는각각의말을동시에거리 ( 칸과칸사이의중심거리 ) 가 d 이상떨어진위치로이동시킨다.( 최대의 d를구하고, 그 d 이하에서는모두가능함을증명하고, 그 d를넘는값에서는모두불가능함을보여라.) (b) 21 21 크기에서위의문제를다시풀어라. (Towns 1984봄 SO5) 증명 (a) 중앙의네칸중한칸을생각하면그녀석은최대거리 10 p 2 까지만움직일수있고,( 제 1, 제 3), ( 제 2, 제 4) 사분면끼리평행이동하는것을생각하면 10 p 2 가실제로가능하다.(b) 역시중앙의칸을생각하면 10 p 2 까지가한계. 이거리의이동을실제로구현할수있다.20 20 은 (a) 처럼옮기고나머지 L 자영역만잘옮기면됨. 110. 주어진 2 이상의자연수 n에대해, 집합 A = f1; 2;:::;ng 를생각하자. M은 A의부분집합이고, M의어떤두원소의합도 42의배수가아니다. 이런 M이가질수있는원소의최대개수를 S(n) 으로나타내다. S(n) = 1997 이면 n은얼마인가? ( 몰도바 1997 최종-y11-4) 풀이 42로나눈나머지에따라분류한그룹을둘씩짝짓기.0과21의집합은스스로와짝이되므로별도. 1997 = 20m +1+1+a 에서 m =99,a =15. 답 4173 111. (a) 원주를등간격의호로나누는 10개의점이있다. 이들을둘씩연결하여5개의현을얻었다. 이현의길이가모두다를수있을까? (b) 원주를등간격의호로나누는 20개의점이있다. 이들을둘씩연결하여얻은 10개의현중에는같은길이의두현이반드시존재함을증명하여라. (Towns 1982가을 J4) 증명 (a) 모두다를수있다.(b) 두가지색으로색칠.
4.2 조합고급문제 255 112. 5 9(5 행 9 열 ) 크기의직사각체스판에서다음과같은게임을한다. 처음에몇개의돌이이체스판의임의의칸들에놓여있었다. 단, 한칸에는돌이하나만놓일수있다. 완전한움직임이란이체스판의모든돌을다음과같은규칙에따라동시에움직이는것을말한다. (i) 각각의돌은위, 아래, 왼쪽, 오른쪽중의한방향으로한칸을이동할수있다. (ii) 위나아래로움직였던돌은다음번완전한움직임때는왼쪽이나오른쪽으로움직여야한다. (iii) 왼쪽이나오른쪽으로움직였던돌은다음번완전한움직임때는위나아래로움직여야한다. (iv) 완전히움직인후에도각칸에는두개이상의돌이있을수없다. 더이상완전한움직임을행할수없으면이게임은끝난다. 처음에이체스판위에 33 개의돌이있었다면이게임은유한번안에끝남을증명하여라. 또한, 이게임이영원히계속되도록처음에 32 개의돌을잘배치하고게임을진행하는것이가능함을증명하여라. ( 아일랜드 1996-10) 증명 32개가가능하다는것은 4개씩 cycle로묶어4 8 에배치하면끝. 33개이상일때불가능하다 는것 : 체스판을다음과같이색칠하면모든말은 ABCDABCD.. 나 ADCBADCB.. 순으로움직이게됨. ABABABABA A가 15개, B가 12개, C가 8개, D가 10개. DCDCDCDCD A의위치에말이 9개이상있으면두번진행했을때모두 C의위치에가게되므로곤란. ABABABABA B와 D를합해 17개이상있으면다음에 A와 C를합해 17개이상있게되므로역시곤란. DCDCDCDCD 따라서, $A + C + (B+D) \leq 8 + 8 + 16 = 32$. ABABABABA 113. N 은 1415 개의점들로이루어지고둘레가 2001 인볼록다각형이다. N 의꼭지점들중에서넓이가 1 보다작은삼각형을이루는세점을찾을수있음을증명하여라. ( 주니어발칸 2001-4) 증명귀류법으로모든삼각형의넓이가 1 이상이라하자. 임의의한변을 a라하고, 거기서부터각변을차례로 a 1 ;b 1 ;a 2 ;b 2 ;:::;a 707 ;b 707 이라하자. a i, b i 를두변으로갖는삼각형의넓이는 p p 2 2Si pa i b i ai+b i. 2 이식에2를곱한후모든i에대해합하고또 a도합하면우변이둘레의길이가되므로 a +1414 p 2 2001, 즉 a 2001 1414 p 2. a는임의였으므로다시둘레의길이는 2001 1415(2001 1414 p 2), 즉 1415 p 2 2001 인데이것은성립하지않음.[ 주 ] 처음부터 a를최대인것으로택하고시작하면귀류법아닌비둘기집원리등으로쓸수도있음. [ 주 ]1415는 p 2의소수전개와관련이있음. 114. 정수 n>2 이주어져있을때, 다음성질을만족하는서로다른 n 개의수 a 1 ;:::;a n 의예를찾아라 : 임의의두원소의합 a i + a j (i 6= j) 를모두모은집합의 ( 서로다른 ) 원소의개수가최소이다. 한편, 이집합의원소의개수가최대인경우도예를찾아라. ( 폴란드 1968 3 차 -3) 풀이 (1) a 1 <a 2 < <a n 이라할때, a 1 + a 2 <a 1 + a 3 < <a 1 + a n <a 2 + a n < < a n 1 + a n 의 2n 3 쌍이모두서로다르고실제로등차수열일때이렇게딱 2n 3 개의합이생김.(2) a k =2 k 등으로하면모든합이다서로다름. 115. 평면위에 n>3 개의점이주어져있고, 어느세점도한직선위에있지않다. n 보다작은자연수 k 에대해다음을증명하여라. (a) k n 2 일때, 각각의점은어느세선분도삼각형을이루지않도록적어도 k개의다른점들과선분으로연결될수있다. (b) k> n 2 이고각각의점은적어도 k개의다른점들과선분으로연결되어있을때, 삼각형을이루는세선분이반드시존재한다. ( 폴란드 1968 3차-6) 증명 (a) 완전이분그래프 (b) 귀류법으로삼각형이없다고하자. A가 A 1 ;A 2 ;:::;A k 와연결되어있을때, 삼각형이없으므로 A 1 은 A 2 ;:::;A k 와모두연결되지않는다. 그럼 A 1 과연결될수있는것은최대 (n 1) (k 1) = n k<k개이므로모순. 116. S 는집합 f1; 2; 3;:::;1000g 의부분집합으로, S 의임의의서로다른두원소 m, n 에대해 m + n 은항상 S 에없다고한다. S 는최대몇개의원소를갖는가? ( 중미 1999-6)
256 조합 풀이일단최대의경우는 S = f500; 501;:::;1000g 일것으로쉽게추측됨. 즉 501 개. 증명이관건. 귀류법으로 502 개이상의 S 가존재한다고가정. 그럼 S 에는 500 미만의수들이존재. 그수들을 a 1 <a 2 < <a k 라하자. S 에는 500 이상의수도틀림없이존재하고, 그중가장큰수를 M 이라하자. 우선 M +1;:::;1000 의 1000 M 개의수는 S 에없음 (1). M a 1 >M a 2 > >M a k (1000 M) 은모두 S 에있을수없는 500 이상 M 미만의서로다른수들. 이게 k (1000 M) 개이므로 (1) 과함께 500 이상의수가적어도 k 개는 S 에있을수없음. 그럼 jsj 501 이되어모순. 117. 다음을만족하는최대의정수 A 를구하여라 :100 이하의모든자연수들의임의의재배열에대하여, 합이 A 이상이되는연속한 10 개의수가항상존재한다. ( 폴란드 1971 3 차 -5) 풀이한재배열 ¼ 안에나타나는연속한 10항의합의최대값을 M(¼) 라쓸때, A는 M(¼) 의최소값을말한다. 수열 ¼ = 100; 1; 99; 2; 98; 3;:::;51; 50 에서는연속한 10항의합은 505 또는 500이므로 M(¼) = 505. 즉 A 505. 100항을 10항씩 10부분으로분할할때 100항의총합은 5050 10M(¼) 이므로항상 M(¼) 505. 답 505 118. 두기사가바둑을두는데제한시간시계를이용하고있다. 각자 40 수씩끝냈을때각자소요한시간이 2 시간 30 분씩으로똑같았다. 두사람이사용한시간의차이가정확히 1 분 51 초였던순간이있었음을보여라. 그리고, 정확히 2 분의차이였던순간은반드시있었을까? (Towns 1985 가을 J4) 풀이그런순간이없었다고하면, a 1 < 111, b 1 <a 1 +111 < 2 111, a 1 +a 2 <b 1 +b 2 +111 < 3 111,... b 1 + + b 40 <a 1 + + a 40 +111< 80 111 = 8880 < 9000 = 2 시간 30 분으로모순. 119. 어떤마을의도로는세방향으로이루어져있는데, 이마을을같은크기의정삼각형블록들로분할하고있다. 교차로에진입한차량은직진하거나 120 ± 만큼좌로혹은우로회전할수있다. 교차로가아닌곳에서방향을바꿀수는없다. 한교차로를막지난지점에서차한대가출발하였고, 이차가다음교차로에들어서는순간두번째차가같은출발점에서그를향해출발하였다. 두대모두같은속력으로계속등속운동을한다. 이두대가언젠가서로만날수있는가? (Towns 1986 봄 J3) 풀이 A B C A B C C A B C A B B C A B C A 위와같은육방격자색칠을생각하면항상 A! B! C 로옮겨가게되므로. 변도 AB! BC! CA 위를움직이고. 120. (\ 시지푸스노동자 ") 언덕을오르는 1001 개의계단이있다. 그중몇몇계단에는바위가있고, 한계단에는바위가최대 1 개있을수있다. 시지푸스는바위하나를골라그윗쪽의가장가까운빈계단으로옮길수있다 ( 즉, 바위가연이어있으면한꺼번에여러단을올릴수도있다 ). 그럼훼방꾼은바로아랫단이비어있는바위하나를골라한계단밀어내린다. 시지푸스와훼방꾼은이렇게번갈아움직인다. 처음에가장낮은 500 개의계단에 500 개의바위가있었다. 시지푸스의목표는정상에바위하나를올리는것이다. 훼방꾼이시지푸스가목표를이루지못하도록할수있을까? (Towns 1986 봄 J6) 풀이훼방할수있다. 연속한바위들의묶음과묶음사이에빈계단이둘이상연속하여있을수없도록 ( 또한, 맨밑계단은항상비어있지않도록 ) 만들수있음을보이면충분. 121. n n 체스판의한대각선의칸을 1로채우고나머지칸을모두 0으로채웠다. 스스로교차하지않는임의의닫힌 `룩-회로 '( 체스판의모서리에평행한선분들로이루어진닫힌경로 ) 의모든칸의수들을1씩증가시킬수있다. 이런시행으로모든칸의수가똑같아지도록할수있는가? (Towns 1992가을 JA1)
4.2 조합고급문제 257 풀이칸에흑백을번갈아색칠하면흑칸과백칸의수가똑같이늘어나는데, 처음에 1은모두흑칸에만있어서흑칸의수의합이항상 n만큼큼. 색칠. 122. a 1 ;a 2 ;:::;a 100 은 1901 부터 2000 까지의정수들을재배열한것이다. 이로부터다음과같이부분합수열을만든다. s 1 = a 1 ; s 2 = a 1 + a 2 ; s 3 = a 1 + a 2 + a 3 ; :::; s 100 = a 1 + a 2 + + a 100 s 1 ;:::;s 100 중에 3 으로나누어지는것이없도록하는수열 a 1 ;:::;a 100 은모두몇개인가? ( 캐나다 2000-2) 풀이정수수열 a 1 ;a 2 ;:::;a k 의부분합중에 3의배수인것이없으면, 이수열의맨앞을제외한사이사이에 3의배수인항을추가하거나혹은 3의배수인항을제거해서만든수열 a 0 1 ;a0 2 ;:::;a0 k 도부분합중에 3의배수인것이없다. 따라서,1901; 1902;:::;2000 중에서 3의배수인것은제외하고만든후, 나중에 3의배수들을끼워넣기로하자. 이 100개의수중에서 mod 3으로 0, 1, 2인것은각각33, 33, 34개있다. 수열에 3의배수가없다면, 부분합 s i 가 mod 3으로 1, 2일때각각다음항은 mod 3으로 1, 2인것만올수있다. 즉,mod3으로가능한수열은 1; 1; 2; 1; 2; 1; 2; 1;::: 과 2; 2; 1; 2; 1; 2; 1; 2;::: 두가지뿐인데,2가하나더많으므로 2; 2; 1; 2; 1; 2;:::;1꼴이유일하다. 이제 mod 3으로 1인것과2인것의위치는결정되었으므로여기서 mod 3으로같은것끼리순서를바꾸는경우의수 33! 34! 만큼나오고, 다음이 67개의수에서맨앞을제외한사이사이에 33개의 3의배수들을작은것부터차례로끼워넣 33! 34! 99! 는경우의수는 67 68 69 99가된다. 따라서, 구하는순열의개수는 답 66! 123. n 3 인양의정수 n 에대하여 S n = f1; 2;:::;ng 이라하자. 원소의개수가모두짝수인 S n 의부분집합 A 1 ;A 2 ;:::;A n 이주어져있다. 그러면 A i1 4 A i2 4 4A it =? 를만족시키는공집합이아닌 S n 의부분집합 fi 1 ;i 2 ;:::;i t g가존재함을보여라. ( 단, 집합 A, B에대하여 A 4 B =(A [ B) (A \ B) 이다.) ( 한국 2004-J2) 증명 임의의 I = fi 1 ;i 2 ;:::;i tg 에대해 t I = 4 A ik = A i1 4 A i2 4 4A it k=1 와같이쓰기로하자. 우선집합연산 4에대한다음의성질을확인해두자 ( 증명은뒤에 ). (1) 4은교환법칙과결합법칙이성립한다 ( 따라서위와같이쓸수있다 ). (2) A 4 B =? () A = B. (3) I 는 fa i g i2i 들중홀수개에포함되는원소들만을모은집합이다. (4) J 4 K = J4K. I(6=?) ½ S n 인모든첨자집합 I들의모임을 G라하자. 이때문제는 I =? 인 I 2 G 가존재함을보여라. 라는것이다. 주어진 I 2 G 에대해, k 2 A i 인 i 2 I 의개수를 a k (I) 로나타내기로하자. A i 들은모두짝수개의원소를가지므로 a 1 (I)+a 2 (I)+ + a n(i) = X ja i j = 짝수 i2i 가된다. 따라서, a k (I) 중홀수인것은짝수개이고,(3) 에의해그런 k 들만 I 의원소이므로, I 는항상짝수개의원소를갖는다. S n 의부분집합중에서짝수개의원소를갖는것은 2 n 1 개있다 (1; 2;:::;n 1 의포함여부를알면 n 의포함여부가저절로결정되기때문에 ). 따라서, I 의종류는많아야 2 n 1 개있다. 그런데, G 는 S n 의부분집합중공집합을제외한것들이므로 2 n 1 개있고, 이것은 n 3 일때 2 n 1 보다크므로, 비둘기집의원리에의해 J = K
258 조합 인서로다른두 J; K 2 G 가있다. 즉, I = J 4 K 라하면 (4) 와 (2) 에의해 I = J 4 K =? 이된다. 그리고 J 6= K 이므로 (2) 에의해 I 6=?, 즉 I 2 G 이므로원하던 I 를찾았다. 위의 4 의성질들은아래와같이증명된다. (1) 교환법칙 : A 4 B =(A [ B) (A \ B) =(B [ A) (B \ A) =B 4 A 결합법칙 : 교집합기호는생략하고여집합 X c 는 ¹ X 로대신쓰겠다. 벤다이어그램을그려보면 (A 4 B) 4 C 는다음과같음을확인할수있다. (A 4 B) 4 C = A ¹ B ¹ C [ ¹ AB ¹ C [ ¹ A ¹ BC [ ABC 이결과는대칭적인식이므로, A, B, C 의순서를적당히뒤바꾼 (B 4 C) 4 A 도똑같은집합이되고, 교환법칙이성립함을이용하면 (A 4 B) 4 C =(B 4 C) 4 A = A 4 (B 4 C) 임을알수있다. (2) 벤다이어그램등을이용하면 A 4 B =(A B) [ (B A) 이됨을금방알수있다. 이것을이용하자. A 4 B =? () (A B) [ (B A) =? () A B = B A =? () A ½ B ½ A () A = B (3) jij 에대한수학적귀납법으로증명하자 : jij =1 일때는 I = A i 이므로자명. jij = m 일때성립한다고가정하자. jij = m +1 일때, I 에서한원소 x 를골라 I 0 = I fxg 라하자. fa i g i2i 중홀수개에포함되는원소는 (i) A x 에포함되고 fa i g i2i 0 중짝수개에포함되거나 (ii) A x 에포함되지않고 fa i g i2i 0 중홀수개에포함된다. 즉, 귀납법의가정에의해, (i) A x 에포함되고 I 0 에포함되지않거나 (ii) A x 에포함되지않고 I 0 에포함된다. 따라서, 이런원소는 I = A x 4 I 0 에포함된다. 비슷하게, fa i g i2i 중짝수개에포함되는원소는 A x 와 I 0 에동시에포함되거나동시에포함되지않아서, I 에포함되지않는다는것도확인할수있다. 즉, jij = m +1 일때도성립. 따라서, 수학적귀납법에의해증명이되었다. (4) J 4 K 에서는 J 와 K 에동시에나타나는첨자들만 (2) 에의해소거되고 J 와 K 의어느한쪽에포함되는것들은남는다. 즉, a m(j 4 K) =a m(j)+a m(k) 2a m(j \ K) a m (J)+a m (K) (mod 2) 이다. 즉임의의m 2 S n 에대해, A j (j 2 J) 들과 A k (k 2 K) 들중 m을포함하는것들을모두센개수가 A i (i 2 J 4 K) 들중 m을포함하는것들의개수와홀짝이같다. 따라서,(3) 에의해성립.
4.2 조합고급문제 259 124. 한원위의 10000 개의점에시계방향으로 1 부터 10000 까지의번호를붙였다. 이점들을둘씩짝지어 5000 개의선분으로이었는데, 각각의선분은다른딱하나의선분과만난다. 5000 개의선분각각에그양끝점의번호의곱을적었다. 선분에적힌수를모두합하면 4 의배수가됨을보여라. ( 중미 2001-6) 증명연결될만한현 PQ를잡고, P 에서 Q까지시계방향으로도는호를, 반대방향으로도는호를 라하자. PQ를가로지르는선분은딱하나이므로 안에서 ( 안에서도 ) 딱한점을제외하면나머지점들은 PQ 영역안에서네점씩묶인다. 따라서, 에있는점의개수는4k +1꼴. 즉, P 의수를n이라하면 P Q의수는n +4k +2에해당하고, n의홀짝에따라 n(n +4k +2) 1 또는 0(mod4). 그럼 n(n +4k +2) 1 2500 0(mod4). 125. 수 1; 2;:::;n 을한원의둘레에배열하는데, 각각의수는 ( 시계방향으로 ) 그다음두수의합을나누도록한다. 가능한 n>2 은어떤수들인가? ( 중미 2002-1) 풀이홀짝성. 짝수다음또짝수이면모두짝수가되어모순. 그럼짝홀홀짝홀홀... 이라서역시 n 5 이면모순. 126. 집합 f1; 2;:::;1985g의부분집합중에서다음을만족하는가장큰집합을구하여라 : 이부분집합의어떤두수의차도소수가아니다.(1은소수가아님에주의할것!) (Towns 1985봄 SO2) 풀이연속한두수를뽑는경우가있으면, 그두수의앞뒤로적어도 7개씩의수는부분집합에속할수없음. 혹은연속한 8개의수중에서많아야 2개만뽑을수있음. 따라서,4k +1꼴들만모두뽑은경우가가능하므로이것이최선. 127. 64 개의단위정육면체블록들이 8 8 정사각형을이루도록쌓여있다. 이들을 4 4 4 크기의정육면체꼴로다시쌓는데, 면으로이웃했던두블록은항상다시면으로이웃해있도록할수있는가? ( 레닌그라드 1987-24) 풀이 4 4 4 정육면체의꼭지점에놓이는 8개의블록중적어도하나는 8 8 정사각형의변에있었음. 그주변을차근차근탐문해가면 4 4 4 의한모서리의길이가너무짧음을알게됨. 128. 자연수들로이루어진집합 fr 1 ;r 2 ;:::;r k g의원소들은자연수 m으로나누었을때나머지가모두서로다 르다. k> m 2 이라면, r i + r j 가 m의배수가되는경우가존재함을증명하여라. 단, i, j가서로다를필 요는없다. ( 폴란드 1979 3차-1) 증명 r 1 ;:::;r k 들중에는m으로나눈나머지가 0인것이있을수없고, f1; 1g, f2; 2g,..., fb m 2 c; b m 2 cg 의각집합에서하나씩만뽑을수있음. 그럼 k b m 2 m 2 인데, 이것은모순. 간단한비둘기집. 129. 한천문학자가어떤 50 개의별을관찰하여각별들사이의거리를모두합하였더니 S 가되었다. 갑자기구름이나타나 25 개의별을가렸다. 남은 25 개의별들사이의거리를모두합한값은 S=2 보다작음을증명하여라. ( 레닌그라드 1987-30) 증명가려진별들을 A 1 ;A 2 ;:::, 남은별들을 B 1 ;B 2 ;::: 라하면, B i B j B i A k + A k B j 의삼각부등식을모두합했을때 25 P B i B j 24 P B i A k < 24( P B i A k + P A i A j ). 즉,25T<24(S T ). 정리하면 T< 24 49 S. 130. 올림피아라는나라에는 n 개의섬이있다. 그중주민이가장많은섬은파나센테르라는곳이고, 각섬에는서로다른수의주민이산다. 이들섬사이에양방향으로오갈수있는다리를몇개건설하려고하는데, 다음과같은조건을만족해야한다. (a) 각두섬을잇는다리는많아야하나씩이다. (b) 파나센테르로부터이들다리를통해모든섬에갈수있다. (c) 파나센테르로부터이들다리를각각최대한번씩만이용해다른섬에간다면, 도중에거치는섬의주민의수는항상점점감소한다. 이렇게다리를건설하는방법의수를구하여라. ( 중미 2006-5)
260 조합 풀이 (b) 는연결된그래프임을,(c) 는회로가없음을의미. 따라서, 나무. 주민이많은섬부터차례로연결시키는과정을생각해보면, 답 (n 1)!. 131. 갑항공사와을항공사는 6 개도시를연결하는비행직항로를운항하고있다. 각각의두도시쌍마다갑과을중한회사가 ( 양방향의 ) 직항로를운항한다. 한회사만을이용하여순환여행할수있는네도시가있음을증명하여라.( 단, 네도시 P, Q, R, S 의순환여행이라함은 P! Q! R! S! P 와같이돌아오는경로를뜻한다.) ( 아일랜드 2007-4) 증명1 A에서 B, C, D, E로갑항로가있다고하자. 그럼 B, C, D, E 사이에는한도시에서두도시를연결한갑항로가있어선안되고, 그럼 BC가갑항로라면 B나 C에선더이상갑항로가없으므로기껏해야갑항로는DE만추가될뿐. 그럼 B, C, D, E 사이를훑는을항로사각형존재. 따라서, 한도시에서네도시이상으로같은회사의항로가있어선안된다. 이번엔 A에서 B, C, D로갑항로가,E,F로는을항로가있다고하자.B에서 E, F에모두을항로가있어선안되므로B에서 E나 F로갑항로존재.C,D에서도마찬가지로 E나 F로갑항로존재. 그럼 B, C, D 중에어느둘은같은 E 또는 F로갑항로가있고그럼갑사각형존재. 증명2 삼각형 2개가있다는것은잘알려진바, 갑항로로삼각형 ABC가있다고하자.D에서 A, B, C 중두곳으로갑항로가있으면안되므로 D에서 A, B, C 중적어도두곳으로을항로존재.E,F에서도마찬가지로 A, B, C 중각각적어도두곳으로을항로존재. 두을항로집합이 D, E, F에서모두달라야하므로 ( 같으면을사각형존재 )ABC와 DEF를왔다갔다하는을육각형존재. 여기서뚝딱뚝딱그림마무리하면됨. 132. 어떤행성의주민들이두문자 A와 O로만이루어진언어를사용한다. 실수를피하기위해서같은길이의두단어에서는항상철자가적어도세자리는다르다. 길이 n짜리단어는 2n 개이하임을보여라. n +1 ( 독일BW 1971 2차-2) 증명각단어와많아야한자리만다른문자열들을모두모으면모두서로다른문자열임. 133. 1 1 칸들로구획된 M N 직사각형판이있다.1 2 의도미노도많이있다. 각도미노가두칸을차지하도록판에몇개의도미노를올려놓았다. 판이가득차지는않았지만, 어떤도미노도움직일수없다 ( 판의선을따라서옆칸으로미끄러져갈수없다 ). 판의가장자리에는틀이있어서도미노는판밖으로삐져나올수도없다. 빈칸의수는 1 MN 5 보다작음을증명하여라. (Towns 1989가을 JA5) 증명도미노를하나이상사용했으므로일단 MN 2. 빈칸이있으면그빈칸의주면은항상다음과같이구획되어야함을증명할수있음. 114 2X4 233 ( 증명 : 빈칸이있으면그빈칸을포함하는빈칸들의연결된영역중에서가장바깥부분 ( 즉, 도미노와만나는부분 ) 의칸이있을것이고, 그빈칸을 X, 도미노가있는칸 (WLOG, X의윗칸 ) 을 1이라하자.1을까는도미노는 X를향하지않는방향으로깔려있을것이고, 그럼 WLOG 위와같이 11 밑에 X. X 왼쪽의 2도빈칸일순없으므로이곳은도미노가있고역시 X를향하는방향이아닐테니그림처럼 11, 22, X. 계속비슷하게 33, 44도마찬가지.) 만일,X옆칸( 상하좌우의칸 ) 이다른빈칸과이웃하면위의그림이잘그려지지않으므로모순. 즉,X의옆칸4곳은 X만의것. 즉, 빈칸마다그옆네칸을포함한다섯칸이고유의영역으로확보되어야하므로 MN 5k. 134. 30 쌍의부부가둥근탁자에둘러앉아있다. 앉은자리들이정다각형을이룬다고할때, 부부간의거리가같은두부부가존재함을보여라. ( 이탈리아 1989-2) 증명자리를흑백으로번갈아칠하면흑색 30석, 백색 30석. 부부간거리는 ( 짧은쪽으로 )1부터 30까지 30종류뿐이므로, 거리가같은두부부가없다면 30종류의거리가모두나타나야함. 그럼 15가지의홀수거리에앉은부부는서로다른색의자리에앉으므로이제흑색 15석과백색 15석이남고, 남은부부들은서로같은색자리에앉아야하는데한가지색이홀수개씩남아있으므로모순. 135. 집합 f1; 2;:::;3ng 에서 n +2 개의수를뽑으면, 그중차이가 n 보다크고 2n 보다작은두수를찾을수있음을증명하여라. ( 헝가리 1952-2)
4.2 조합고급문제 261 증명 0 a<b<n인 a, b에대해fa; n + bg, fn + a; 2n + bg, fb; 2n + ag 의각집합에서하나의수만뽑을수있으므로 mod n으로두종류 a와 b인수들에서많아야3개만뽑을수있음. n +2개이상의수를뽑으면두종류에서 4개이상을뽑게되는경우가생김. 비둘기집. 136. 공간의모든점을빨간색혹은파란색으로칠했다. 세꼭지점이빨간색이거나네꼭지점이모두파란색인단위정사각형이존재함을보여라. ( 독일 BW 1976 2 차 -4) 증명귀류법. 단위거리의두빨간점 P, Q가있다고하자. 정사각형 PQRS를생각하면 R, S는항상파란색. 그럼 PQ를축으로 RS를 60도회전하여생긴 TU도파란색이고 RST U가파란색단위정사각형이됨. 단위거리의두빨간점이없다면, 빨간한점으로부터단위거리의점은모두파란점이므로그구면위의네점으로된단위정사각형을찾으면끝. 빨간점이전혀없으면그것도그냥끝. 별증역시귀류법. 정사각형 PQRS를잡는데, 파란점은 2개 or 3개. 파란점이 2개이면, P 와 R이면정팔면체 A-PQRS-B를생각하면끝. 이제이런두빨간점과두파란점이대각선으로마주보는관계도없다고가정. 파란점이 2개이고 P 와 Q가파란점이면정육면체 PQRS-TUVW를생각하면끝. 이제빨간점둘과파란점둘도없다고가정. 파란점이 3개이면또정팔면체 A-PQRS-B를생각하면끝. 137. 모든꼭지점이변으로연결되어있는다면체가있다. 즉, 대각선을갖지않는다. 이다면체는사면체임을증명하여라. ( 헝가리 1948-2) 증명 K 5 는평면에그릴수없음. 별증점이 5개이상이면, 임의로세점을잡았을때, 다른점들은이세점을지나는평면에대해항상반대쪽영역에있어야함. 그럼일단점은 5개뿐이고, 이때처음에잡았던세점은임의이므로어떤세점도면을구성할수없고내부를가르는평면이되어야함. 이것은다면체에면이없다는것이므로모순. 138. 어떤고대법정에서는재판관들을위한 23 개의좌석이한줄로배열되어있었다. 재판이길어질경우, 몇몇재판관은법정을떠나기도하고다른재판관들이들어오기도한다. 법정으로들어오는경우판사는혼자들어올수도있고두명이짝을지어들어올수도있는데, 만약두명이짝을지어들어오는경우법정을떠날때도함께나가야하며법정을나간뒤에는짝의구분이없어진다. 또한법정에는관리인이존재하여만약두재판관이짝을지어들어오는경우연속된두좌석에앉도록안내한다. 만약재판이진행되는동안의어느시점에서도법정에앉아있는재판관이항상 16 명이하라면, 관리인이이작업을항상해낼수있음을보여라. ( 호주 1988-5) 증명 23개의자리를 3인실 7개와 2인실 1개로나눈다. 즉,[ABC][DEF][GHI][JKL][MNO][PQR] [STU] [VW] 로분할한다. 규칙은 `각 3인실의중앙석에는솔로부대는앉을수없다 ' 뿐으로하면된다. 그럼솔로부대가앉을수있는자리는 (7개의중앙석을제외하고 ) 총 16석이므로솔로부대는언제든지앉을수있다. 또한, 커플이들어올때 3인실에들어갈수없다면각 3인실은이미두명이상이앉아있는상태이고, 그럼 3인실에이미총 14명이앉아있으므로, 그커플은비어있는 2인실에앉으면된다. 139. 각행과열에자연수번호를붙인무한한크기의체스판을생각하자. 각칸에는최대 1 개의동전을놓을수있다. 임의의두자연수열 (a n ), (b n ) 에대해 n 번째행에는 a n 개의동전이있고 m 번째열에는 b m 개의동전이있도록동전을놓을수있음을보여라. 단, 동전은무한히제공된다. ( 이탈리아 1993-3) 증명먼저각열에는 1개씩의동전만있도록하고 ( 예를들어, 수열 a n 이처음몇항이2, 4, 3이라면아래와같이함 ) 제1행 : OO 제2행 :..OOOO 제3행 :...OOO 그다음각각의돌은같은행위에서만움직이는데, 첫번째열부터 b i 들의개수에맞도록오른쪽에서돌들을그열로끌어오면됨. 140. 한변의길이가 n 인정사각형을아래타일로덮을수있으려면 n 은어떤값이어야하는가? ( 이탈리아 1995-1)
262 조합 풀이우선넓이를고려할때 9 j n 2 이므로 3 j n. 체스판의흑백을칠하듯하면위의타일은항상두가지색을 6:3으로깔게되므로깔린두색의차이는항상 3의배수. 그런데 n이홀수라면두색의칸의개수차이는 1이므로곤란. 즉 2 j n. 따라서 6 j n 이고,6 6 이되므로이것은항상잘됨. 141. 마을에주점 A, B, C, D 가있고, A 와 D 사이를제외하고는어떤둘을택해도서로연결되어있다. 한취객이 A 에서시작해서, 술을한잔마시고연결된다른주점중아무곳으로나같은확률로이동하는행동을반복한다. (a) 다섯번째잔을마실때 C에있을확률은얼마인가? (b) n>5 일때, n번째잔을마실때이취객이있을확률이가장높은주점은어디인가? ( 이탈리아 1995-3) 풀이 n 번째잔을마실때 A 나 D 에있을확률을 p n 이라하면, B 나 C 에있을확률은 q n =1 p n. p 1 =1 이고, p n+1 = 2 3 qn = 2 3 2 3 pn. p n+1 2 5 = 2 3 (pn 2 5 ). 즉, pn = 2 5 + 3 5 ( 2 3 )n 1. 142. 20 명의학생이필기와구두로수학시험을쳤다. 이학생들중어떤두명을택해도필기와구두점수가모두같지는않다. A 의필기, 구두점수가둘다 B 의점수이상이라면 A 가 B 보다낫다고말한다. 각시험점수가 1 이상 10 이하의정수라고할때다음물음에답하여라. (a) A가 B보다낫고, B가 C보다나은학생 A, B, C가있음을증명하여라. (b) 학생이 19명이어도 (a) 가성립할까? ( 이탈리아 1995-2) 증명한학생이받을수있는모든점수쌍들을 20 20 의표로나타내어보자. 그럼각학생은어느칸엔가속하고, 한칸에두학생이동시에속할수없다.(a) 귀류법으로풀면, 한행혹은한열에세학생이있을수없다. 그럼모든행과모든열마다둘씩있다. 그럼제일왼쪽 ( 필기점수가제일작은 ) 열의두학생을택하면그중필기점수가높은학생과같은행에또한명이있으므로그래도세학생을찾게됨.(b) 위의표에서 / 방향의주대각선과그바로위의대각선을택하면 19명이그런세학생없이잘구성됨. 143. 한원의내부에 100개의점이있다. 어떤점도원의중심에있지않고, 어떤두점도같은반지름위에놓여있지않다. (a) 정확히 10개의점을갖는, 중심각이 2¼ 인부채꼴이존재함을보여라. 11 (b) 정확히 11개의점을갖는중심각 2¼ 인부채꼴도항상존재하는가? ( 러시아 1990 4차-y10-2) 11 증명 (a) 원판을 11등분했을때비둘기집의원리에의해 9 < 100 < 10 11 이므로 9개이하의점을갖는부채꼴도있고 10개이상의점을갖는부채꼴도있음. 그두부채꼴사이를연속적으로움직이는동안그움직이는부채꼴에포함되는점의개수도연속적으로변하므로딱 10개의점을갖는부채꼴존재. (b) 원판을 10등분하고각각의부채꼴의경계마다 10개씩의점을거의같은반지름위에위치시키면불가능한경우가됨. 144. 8 8 체스판의각칸에수 1; 2; 3;:::;64 를하나씩넣었다 ( 모든수가한번씩사용됨 ). 수의합이 100 보다큰 2 2 영역이적어도네곳있음을보여라. ( 스웨덴 1977-5) 증명 1+2+3+ +64= 64 65 2 = 2080. 2 2 영역 16 개로분할. 귀류법으로, 이중에서 13 개이상의영역에서수의합이 100 이하이면, 전체수의합은 S 13 100+(53+54+ + 64) = 1300 + 702 = 2002 < 2080 으로모순. 145. Antonio 와 Bernado 가게임을한다. 각각 m 개와 n 개의칩이있는두칩더미가있다.Antonio 가먼저시작하고다음 (i){(iii) 중하나를골라행동한다. (i) 칩을한더미에서하나가져오거나, (ii) 칩을양더미에서하나씩가져오거나, (iii) 칩을한더미에서다른더미로하나옮긴다. 더이상이런작업을할수없는사람이진다. 둘중어느쪽이든한명의필승전략이존재하는지를 m, n 에관한함수로표현하고, 존재할때의필승전략을서술하여라. ( 이탈리아 2004-4)
4.2 조합고급문제 263 풀이 ( 짝 ; 짝 ) 2 LS. 불변량게임. 146. (1) 한변의길이가 1인정사각형의내부에 288개의점이있다. AB와평행하고변 AD와 BC를잇는길이 1의선분들의집합 S를그리는데,288개의점각각에서 AD와평행하게 S의선에연결되는선분을그렸을때, 이런선분전체의길이의합이 24보다작도록할수있음을보여라. (2) (1) 에서, 선분전체의길이의합이 18 이하가되도록할수도있음을보여라. ( 스웨덴 1961-4 변형 ) 증명 (1) n 개의열로등분할한후각열의중앙선을그리면가장가까운중앙선까지는기껏해야 1 2n 1 이므로, 길이합은최대 n +288 2n = n + 144 n 이고, 이것은 n =12일때최소24. (2) AB부터 CD까지등간격으로 2n개의세로선을그리고, 짝수번째선들과홀수번째선들의집합을분리. 각점에서짝 수번째선들에이르는최단거리의합과홀수번째선들에이르는최단거리의합은각점에대해세로선간격만큼기여되므로 288. 따라서, 두집합중총합이더적은쪽을택하면선분의길이의합은 1 2n 1 최대 g n = 1 288 (2n + 2 2n 1 ). g n의최소값을구하면 g n = 1 2 + 1 288 ((2n 1) + 2 2n 1 ) 1 2 + p 288 인데 n은자연수라야하므로등호조건 2n 1= p 288 ; 17 에가장가까운n =9를택하면 g n g 9 = 1 2 (18 + 288 17 )=17+ 8 17. 147. mn 명의학생이있는데, 이들의키는모두다르다. 이학생들을 m 개의행과 n 개의열을이루도록세운다 (m; n > 1). 각각의행에서가장작은학생을뽑고, 그중가장큰학생의키를 A 라하자. 그리고각각의열에서가장큰학생을뽑고, 그중가장작은학생의키를 B 라하자. 다음중어떤것이가능할까? ( 헝가리 1963-1) A<B; A= B; A > B 풀이그냥 15퍼즐처럼순서대로세우면 A = B. 거기서 9와 13의위치를바꾸면 B는불변인데 A는줄어들어 A<B. A>B는항상불가능함. 왜냐하면 A>B라하면A = a i, B = b j 일때i행의학생들은모두 j열의학생들보다크니까 i행 j열의학생은오도가도못함. 148. 8개의단위정육면체가주어져있다. 이정육면체들에서 24개의면은흰색으로, 나머지 24개의면은검은색으로칠해져있다. 이정육면체를잘붙여서, 겉면에서나타난흰색과검은색의넓이가똑같은한변의길이 2인정육면체를만들수있음을보여라. ( 러시아 1990 4차-y11-3) 증명한블록의서로마주보는면을짝으로생각할때, 임의의블록의임의의짝의두면중에서어느면이드러나고어느면이숨도록할지를독립적으로선택할수있음. 한짝의두면이같은색이면상관할거없고, 두면이서로다른짝 ( 이런짝은짝수개 ) 들만적당히설정해주면되므로그냥끝. 149. 파티에있는사람중임의로네명을뽑으면, 그중에는나머지세명을알고있는한명이반드시포함되어있다고한다. 파티에있는사람중파티장에있는모든사람을알고있는사람이있음을증명하여라. ( 헝가리 1960-1) 증명가장많이아는사람 A가그사람. A가모르는사람 B가있다고하자. A는적어도3명을아니까 A가아는사람의그룹 X의크기는 3 이상. X에서로모르는쌍 C, D가있다면, fa; B; C; Dg가문제에모순. 따라서, X의사람들은모두서로안다. 그럼 A의최대성에서 X의사람들은더이상다른누구, 즉 B를모른다. 그럼그경우에도 X에서 C, D를아무나골라 fa; B; C; Dg가문제에모순. 150. 평면위에어느세점도한직선위에있지않은 3n 개의점이주어져있다. 이점들을꼭지점으로하는 n 개의속이꽉찬삼각형을구성하는데, 어떤두삼각형도서로만나지않도록항상구성할수있는가? ( 스웨덴 1969-6) 풀이모든점의 x-좌표가다르도록하는 x-축을하나잡고, x-좌표값이가장작은것부터 3개씩묶어구성하면됨.
264 조합
제 5 장 종합문제 5.1 종합중급문제 1. 어떤원의지름을 AB, 그리고이지름위의A와 B 사이의어떤고정점을 C라하고, Q를이원의둘레위를움직이는점이라고하자. Q와 C를잇는직선위에 AC CB = QC 를만족하는점 P 를잡자. 점 P 의 CP 자취를서술하고그를증명하여라. ( 캐나다 1976-4) 풀이 P 는 Q에따라결정되는점이므로P (Q) 의함수꼴로표현하자. 그럼함수 P 는 C를닮음의중심으로하여 AC AC CB 의비를갖도록점을옮기는축소 / 확대변환이다. 문제에서제시된비례식 CB = QC CP 에나타난선분에방향성이있다고하면 ( 즉, P 가 C에대해Q와반대쪽에있는점이라하면 ), 직선 AB 위의점D = P (B) 에대해, P 의자취는 BD를지름으로하는원이된다. 만일앞의비례식에나타난선분에방향성을고려하지않는다면, P 가 C 에대해 Q 와같은방향에나타나는자취가하나더있고, 이것은앞에서구한자취를점 C 에대해점대칭이동한원이다. 2. 눈금이없는세개의물통이있다. 각각 m 리터와 n 리터의용량을가진두물통은비어있고, m + n 리터의용량을가진나머지한물통은물로가득차있다. m 과 n 은서로소인자연수이다. 임의의 k =1; 2;:::;m+ n 1 에대하여, 한물통의물을다른관으로적당히옮기는작업을반복하여용량 m + n 리터의통에정확히 k 리터의물이남도록할수있음을보여라. ( 폴란드 1994 3 차 -4) 증명 m + n! m! n! m + n 으로만이동하는것으로한다면 n에는늘 km mod n 이들어가게됨. m, n이서로소니까 km은 n에대한완전잉여계를구성하게되어서... 3. 화폐수집가프레드는동전을몇개가지고있다. 각동전의지름은 10 cm 이하이고, 모든동전은 30 cm 70 cm 크기의단층 ( 동전들이서로겹쳐놓이지않는 ) 상자에들어있다. 그는지름이 25 cm 인새동전을선물받았다. 그의모든동전을 55 cm 55 cm 크기의단층상자에넣을수있음을증명하여라. (Towns 1992 봄 JO3) 증명 30 70 에서오른쪽 R =30 25 에완전히포함되는동전들만따로떼어냄. 그럼남은동전들은폭10을범퍼로하여모두 L =30 55 안에있음. 이제 L 좌하에 R을 90도회전시켜서놓고, 우하에 25 크기의동전을놓으면 55 55 에잘들어감.
266 종합문제 4. Randy: \ 안녕 Rachel, 재미있는이차방정식을썼구나? 근이어떻게되지?" Rachel: \ 근은두개의양의정수야. 한근은내나이와같고다른근은내동생 Jimmy의나이야." Randy: \ 와, 멋진데! 그럼내가너와 Jimmy의나이를알아낼수있나두고봐. 네방정식의계수들이모두정수니까그다지어렵진않을것같아. 그런데, 이세계수들을모두합하니까소수가되었어." Rachel: \ 재미있군. 이제내가몇살인지알아맞혀봐." Randy: \ 잠깐, 내가네나이를추측해서그걸이이차방정식의 x에대입해볼게... 이런,0이아니라 55가나왔네." Rachel: \ 맙소사, 저리가!" (1) Jimmy가두살임을증명하여라. (2) Rachel의나이를알아내어라. ( 캐나다 2001-1) 풀이 Rachel과 Jimmy의나이를a>b라하자. 그럼이차방정식은 m(x a)(x b) =0의전개인 mx 2 m(a+b)x+mab =0이된다. 세계수의합은 x =1을대입한값과같으므로 m(a 1)(b 1) = p ( 소수 ) 이다. (m; a 1;b 1) = (1;p;1); 즉 (m; a; b) =(1;p+1; 2) 일수밖에없다. 다음,Randy 가추측한 Rachel 의나이를 c 라하면 m(c a)(c b) = 55, 즉 (c p 1)(c 2) = 55 이다.55=5 11 이고 c p 1 <c 2 이므로,(c p 1;c 2) = ( 5; 11) 또는 ( 11; 5) 이다. 어느경우에나 p 1=(c 2) (c p 1) = 16 이므로, p =17. 따라서,Jimmy 의나이는두살이틀림없고,Rachel 의나이는 18 살이다. 5. 자연수 n 을최소공배수로갖는두개의자연수의순서쌍 (a; b) 의개수는 n 2 의양의약수의개수와같음을보여라. ( 헝가리 1962-1) 증명각소인수에대해그지수의종류를세어비교하면됨. p e k n 이면 (a; b) 의 p 의지수쌍은 (0;e); (1;e);:::;(e; e);:::;(e; 1); (e; 0) 의 2e +1 가지. 6. 세변의길이가 a, b, c 인삼각형의넓이를 S 라하자. 다음부등식을증명하여라. S p 3 12 (a2 + b 2 + c 2 ) 증명 양변제곱하고헤론의공식대입하여정리하면 (a 2 b 2 ) 2 +(b 2 c 2 ) 2 +(c 2 a 2 ) 2 0 과동 치. 7. 한원을이웃한두호의중심각의합이항상 103 ± 이하가되도록 7개의호로분할하였다. 이 7개의호의중심각이모두 A ± 이상임을말할수있는 A의최대값을구하여라. 그리고, 이값이정확한최대값임을증명하여라. (Towns 1992봄 JO4) 풀이 51, 52, 51, 52, 51, 52, 51 로하면조건을만족하며모든각은 51도이상. 만약 51도미만의각 이존재한다면그각과103도이하의각셋 ( 두중심각의합들 ) 을더하면360도보다작으므로모순. 답 51도 8. 다항식 x 4 +3x 3 +6x 2 +9x +12 를정수계수이차식두개의곱으로나타낼수없음을보여라. ( 플란더즈 1988-1) 증명 (x 2 + ax + b)(x 2 + cx + d) 를전개하여비교.WLOG3jb라하면! 3 j a, 3j c! 3 j d 에서 모순. 9. 어떤 IMO에서,34개국의단장과부단장들이심판관회의에참석하였다. 회의전에, 몇몇사람들은서로악수를하였지만같은나라의단장과부단장끼리는악수를하지않았다. 나중에한국팀단장이나머지 67명에게각자가악수한사람들의수를물어보았더니모두다른대답을하였다. 한국팀의단장과부단장이악수한사람들은각각모두몇명인가? ( 통신강좌 1989-D8)
5.1 종합중급문제 267 풀이각각의참석자들은자기자신과자기나라사람과는악수하지않으므로, 그들은 0»66명의사람들과악수할수있다. 한국팀단장의질문에대한대답이모두달랐으므로, i명과악수한사람을 P i 라나타내면우리는한국팀단장을제외한참가자들을 P 0 ; ::: ; P 66 로표시할수있다. P 66 는자기자신과자기나라사람을제외한모든참석자들과악수해야한다. 그는 P 0 와는악수하지않았으므로, 그가악수한사람은 P 1 ; P 2 ; ::: ; P 65 그리고한국팀단장이다. 그러므로 P 0 와 P 66 은같은나라사람이다. 마찬가지로 1 이상 65 이하인 i에대해, P i 는 P 1 ; P 2 ; ::: ; P 65 중 i 1명과한국팀단장과악수하였다. 같은방법으로,32이하의 j에대해p j 와 P 66 j 는같은나라사람이고 j < i < n j라면 P j 는 P j+1 ; ::: ; P n j 1 중 i j 1명과한국팀단장과악수하였음이귀납적으로증명된다. 그러므로한국팀부단장은 P 33 이고한국팀단장과부단장은각각 P 34 ; P 35 ; ::: ; P 66 과악수하였음을알수있다. 10. 1 보다작은양의유리수들중에서십진법으로전개했을때 10 개의서로다른숫자로이루어진길이 10 의순순환마디를갖는것들을모두모은집합을 M 이라하자. ( 가 ) M의모든원소들의산술평균을구하여라. ( 나 ) 다음을만족하는양의정수 n (1 <n<10 10 ) 이존재함을증명하여라 : 모든 a 2 M 에대해n a a 이음이아닌정수이다. ( 루마니아 2005 지역예선 y8-1) 풀이 ( 가 ) M 의임의의원소 a = 0: _a 1 a 2 _a 10 는, b i = 9 a i 로정해지는또다른 M 의원소 b =0: _ b 1 b 2 _ b 10 와항상짝을지을수있다. 그리고이두수 a 와 b 는 a + b =0:_999999999_9 =1 이므로평균이 1 2 이다. 따라서, M 의모든원소의평균도역시 1 2 이다. ( 나 ) M 의임의의원소 a 는 a =0: _a 1 a 2 _a 10 = a 1 1a 2 a 10 9999999999 = 9 a 1a 2 a 10 1111111111 의꼴이된다. 여기서 a 1 + + a 10 =0+1+ +9=45 로 9의배수이므로 a 1 a 2 a 10 도 9의배수, 따라서분자가정수이다. 즉, 1111111111a는언제나음이아닌정수이므로 n = 1111111112 로택하면된다. 11. n 을 2 보다큰정수라고하자. 직각삼각형의빗변의길이를 n 번거듭제곱한것은다른두변의 n 제곱을합한것보다더큼을증명하여라. ( 헝가리 1908-2) 증명빗변의길이를 c, 나머지두변의길이를 a, b 라할때, 다음의부등식이면충분하다. c n = c n 2 c 2 = c n 2 (a 2 + b 2 )=c n 2 a 2 + c n 2 b 2 >a n 2 a 2 + b n 2 b 2 = a n + b n 12. 세자연수 a, b, c 가다음방정식을만족한다. 2abc +2ab +2ac +3bc +2a +3b +3c =998 c 의최대값을구하여라. (IT 꿈나무올림피아드 2006 1 차 ) 풀이양변에 3을더하면 (2a +3)(b +1)(c + 1) = 1001 = 7 11 13 으로인수분해된다. 좌변의인수들은모두1보다크므로각각 7, 11, 13과하나씩대응하게된다. c가최대일때를구하는것이니 c +1 = 13 일때c =12. 13. T 목게임은오목게임과비슷한것인데, 오목을만드는대신, 돌네개가 T 자모양 ( 혹은이것을 90 ± 씩회전한모양 ) 으로늘어서도록만드는것을목표로한다.A 와 B 가게임을하고,A 가흑을쥐고먼저두는것으로할때, 두사람이최선의전략으로둔다면 A 와 B 중에누가이기고, 게임은몇수이내에끝날까? 풀이 A가 7수만에이긴다. 먼저 A가중앙에두고그곳은원점으로잡자. 일반성을잃지않고 B가 2번째수를제3사분면에둔다고하고, 그럼 A는 3번째수를 (1; 1) 에둔다. 다음, 역시일반성을잃지않고 B가 x y 인점 (x; y) 에둔다고하고, 그럼 A는 5번째수를 (0; 1) 에둔다. 그다음에는 (0; 2) 와 ( 1; 1) 이맞보기이므로 7번째수에서 A가이긴다. 14. 어떤사다리꼴을중단평행선 ( 윗변과밑변에평행하고같은거리에있는직선 ) 으로잘랐더니두영역의넓이의비가 1:k 가되었다. k 가가질수있는값의범위를구하여라. ( 플란더즈예선 1993-1994)
268 종합문제 풀이윗변의길이를 a, 아랫변의길이를 b라고할때중단평행선의길이는 a+b 2 이다. 이때중단평행선에잘려생긴두사다리꼴의높이는같으므로그넓이비는 a + a+b : a+b + b =3a + b : a +3b 2 2 이다. 넓이비 a+3b 3a+b 는 a또는 b 값이매우작게변함에따라서 3 또는 1 3 에가까워지므로 1 <k<3 3 을증명하자. a+3b = k ) a +3b =3ak + bk ) (3 k)b =(3k 1)a 3a+b 이때 1 1 <k<3 3 라면, 임의의양수 a에대해위식을만족하는b를찾을수있고, k 3 또는 k 3일때에는 a, b 중하나가0 이하가되어불가능하다. 즉, k의범위는 1 3 <k<3: 15. 최대공약수가 1 인서로다른 3 개의수가있는데이중어느두수도서로소가아니라고한다. 이 3 개의수의최소공배수 L 의최소값을구하여라. (IT 꿈나무올림피아드 2006 1 차 ) 풀이 3개의수를 a, b, c라하자. 어떤두수도서로소가아니므로공통소인수를갖는다. b와 c의한공통소인수를 p라하고, c와 a의한공통소인수를 q, a와 b의한공통소인수를 r이라하자. a, b, c의최대공약수는 1이므로, 그럼 p는 a의약수일수없고, q는 b의약수일수없고, r은 c의약수일수없다. 따라서, p, q, r은모두서로다르다. 그럼 L pqr 2 3 5=30. 실제로, 세수가6, 10, 15이면 L =30인경우가있으므로 30이최소값이다. 16. ABCD는볼록사각형으로 \CBD =2\ADB; \ABD =2\CDB; AB = CB 를만족한다. AD = CD 임을증명하여라. ( 캐나다 2000-4) 증명 BD의 B쪽연장선위에 BA = BC = BE 인점E를잡자. 그럼 B 는 4CAK 의외심이므로원주각의성질에의해 \BKC = 1 \CBD = \ADB 2 \BKA = 1 \ABD = \CDB 2 따라서, AECD 는평행사변형이다. AC 와 BD 의교점을 O 라하면평행사변형은 O 에대해대칭적이고, 그럼 O 에대한 B 의대칭점 F 를생각하면 ABCF 는마름모가된다. 즉, AC? BF 이고 AECD 도마름모이다. 그러므로, AD = CD. 17. 어떤양의실수를입력하면그수의소수부분을버리고정수부분만취해서얻어진정수와원래수를곱해서다시그수의정수부분만취해서돌려주는프로그램이있다.( 즉,[x[x]] 를돌려준다.) 이프로그램에 a 를입력했더니 10 이나왔고 2a 를입력했더니 51 이나왔다.3a 를입력하면얼마가나오겠는가? (IT 꿈나무올림피아드 2006 1 차 ) 풀이우선 [a[a]] = 10 을살펴보자.[a] 2 이면 a<3 이므로 a[a] < 6 이되어곤란하고,[a] 4 이면 a 4 이므로 a[a] 16 이되어역시곤란하다. 따라서,[a] =3일수밖에없다.[a[a]] = 10 은그럼 [3a] =10이되고, 이것은 10 3a <11, 즉 10 3 a< 11 3 을얻는다 (1). 다음 [2a[2a]] = 51 을살펴보자.[2a] 6 이면 2a[2a] < 42 이므로곤란,[2a] 8 이면 2a[2a] 64 이므로곤란하여,[2a] =7이다. 따라서 [14a] =51, 즉 51 14a <52, 51 52 a< 14 14 를얻는다 (2). (1) 과 (2) 의공통부분은 51 11 153 a< 14 3 이다. 그럼 (= 10:9 ) 3a <11 14 이므로 [3a] =10이고, [3a[3a]] = [30a] = 109 가된다.
5.1 종합중급문제 269 18. x, y, z 는다음을만족시키는자연수이다. 7x 2 3y 2 +4z 2 =8 16x 2 7y 2 +9z 2 = 3 이때, x 2 + y 2 + z 2 의값은얼마인가? (IMTS R10-1) 풀이두식을차례로A, B라하자.7A 3B 하면 x 2 + z 2 =65. 여기서 x 2 =1; 16; 49; 64 만가능함을알수있다.9A 4B 하면 x 2 + y 2 =84이고, 앞선 x 2 값들을대입해 y 2 이가능한지살펴보면 x 2 =16만가능함을확인할수있다. 그럼 y 2 =100,z 2 =49. 답 165 19. 검은말과흰말로 A 와 B 가 3 3 체스판에서게임을한다. 규칙은다음과같다 : 가. 두사람은번갈아게임을한다. 나. 자기차례에서는말하나를체스판의빈칸중한곳에놓는다. 다. 말을놓을때는검은색이나흰색말중어느것을선택해도되고, 항상같은색의말을놓지않아도된다. 라. 판이가득찼을때, A는각행, 각열, 각대각선마다검은말이짝수개있으면 1점씩얻는다. B는각행, 각열, 각대각선마다검은말이홀수개있으면 1점씩얻는다. 마. 전체 8점중5점이상을얻은사람이이긴다. (1) 4 : 4 무승부가가능한가? 설명하여라. (2) 처음하는사람이반드시이기는방법을서술하여라. ( 캐나다 1978-5) 풀이 (1) 가능하다. 예를들어다음과같이게임이끝난다면. ± ² ± ² ± ² ± ² ± (2) 먼저하는사람이 A 라면처음에검은말을중앙에놓는다. 그후에는항상 B 가놓는곳의대칭에되는위치에 B 가놓은말과다른색의말을놓는다. 그럼게임이끝났을때 A 는중앙을지나는 4 개의선에는모두짝수개의검은말이있어서우선 4 점을얻고, 좌변과우변이서로색이대칭으로뒤집혀있으므로둘중한곳에서 1 점, 또상변과하변중한곳에서 1 점을얻어총 6 점을얻게된다. 먼저하는사람이 B 라면처음에흰말을중앙에놓고같은방식으로하면된다. 20. 한공무원이인구조사를위해어느집을방문하였다. 그집에는몇명의여자들이함께살고있었는데, 이들은자신의나이를밝히기를꺼려하였고, 대신에 \ 당신이우리들중두사람을지목하면그두명의나이의합은기꺼이대답할게요." 라고말하였다. 그러자공무원은 \ 그렇다면가능한모든쌍의나이의합을제게다알려주세요." 라고하였고, 30, 33, 41, 58, 66, 69이라는답을받았다. 이로써공무원은계산을끝내즐겁게돌아갈수있었다. 그는각사람의나이를어떻게계산했을까? ( 인도지역예선 1990-7) 풀이 n 2 =6 이므로 n =4 명이다. 네사람의나이를 a b c d 라하자. 그럼 a + b a + c b + c; a + d b + d c + d 이므로 (b + c 와 a + d 사이의크기관계만불확실하다 ), a + b =30; a+ c =33; b+ d =66; c+ d =69 이다. 여기서 a와 b의홀짝이같음을알수있고, 따라서 a + c 와 b + c 도홀짝이같아서 b + c 도홀수여야한다. 그럼 b + c =41 이고, a + d =58 이다. 이제 a = (a+b)+(a+c) (b+c) = 30+33 41 =11 2 2 살이고, 나머지도비슷하게구하면각각 19, 22, 47살이다. 답 11, 19, 22, 47살
270 종합문제 21. 원 O의지름AB와현CD가점H에서직교한다. AB의길이는두자리의자연수이고, AB의자릿수의순서를바꾸어놓으면 CD의길이가된다고한다. OH의길이가양의유리수일때, AB의길이를구하여라. (AIME 1983-12) 풀이 %EEE Put AB = 10m+n, then OH = 3/2 (11(m+n)(m-n)), so 11 divides m+n and m-n is square. Hence m=6, n=5. 답 65 22. n 은 7 의배수가아니지만자리수의합은 7 의배수가되는자연수이다. n 의자리수들사이에 0 을몇개적당히끼워넣어 7 의배수로만들수있음을보여라. (KAIST Cyber 영재교육 2005 가을 ) 증명 n 에는 0 과 7 이아닌자릿수가 2 개이상있다.( 그런자릿수가하나도없으면 n 은 7 의배수이고, 딱하나있을때는자릿수의합이 7 의배수가될수없다.) n 의자릿수전체를앞쪽 (A) 과중간 (B), 그리고뒷쪽 (C) 의세부분으로나누는데 ( 즉, n = ABC = A 10 x + B 10 y + C 꼴 ), A 와 C 에는각각 0 과 7 이아닌자릿수가딱하나포함되도록하자. 그럼 A 와 C 는 7 의배수가아니다. BC 가 7 의배수가아니면그대로두고,7 의배수이면사이에 0 을끼워넣어 B0C 로만들자 ( 그럼 7 의배수가아니다 ). 이제 A, 10A, 100A, 1000A, 10000A, 100000A 는 7 로나눈나머지가모두다르므로 ( 그리고 7 의배수가아니므로 ), BC( 혹은 B0C) 의앞에적절한것을붙이면 7 의배수가된다. 별증 n = a m a 1 a 0 이라하면각자릿수사이마다 0 을 5 개씩끼워넣은수 a m00000a m 1 00 00a 1 00000a 0 이 7 의배수이다. 왜냐하면,10 6 1(mod7) 이므로,10 6m a m + +10 6 a 1 +a 0 a m + +a 1 +a 0 0 (mod 7) 이기때문이다. 23. 현이아버지는어린아들현이에게숫자교육을시키기위해, 세자리짜리숫자카드세트를샀다 (000{999). 그러나집에와서카드의개수를세어보았더니 1000 장이안되었다. 그래도똑똑한현이는이숫자카드세트만가지고도 000 부터 999 까지모두만들어내서잘갖고놀았다. 그러면이카드세트에는최소몇장의카드가있었을까? 주 0, 1, 6, 8, 9 는뒤집어서보면각각 0, 1, 9, 8, 6 이다. ( 통신강좌 1993-7-30) 풀이서로다른숫자카드 A, B가각각뒤집어서, B, A가된다면둘중에하나는없어도된다. 그런쌍의개수를세어보자. 일단 2, 3, 4, 5, 7 중어느하나가있는카드는뒤집어서숫자가되지않으므로생각할필요가없다. 이제 0, 1, 6, 8, 9로만이루어진 125개의카드에대해서생각하자. 뒤집어서자기자신이되는카드의개수 =5( 첫째자리와그에대응하는셋째자리 ) 3( 가운데자리숫자로올수있는것의개수 ) = 15. 따라서 125 15 = 110 은 55개의쌍으로나눌수있고둘중의하나는없어도된다. 답 945 24. ( 이문제가치러진대회는 2005년이다.) 전설에따르면, 때가되면깨어나이문제를풀고있는모든사람을잡아먹고다시그년도의자릿수의합에해당하는해동안잠이들게되는몬스터가있다. 이몬스터는서기 234년에처음으로이대회를습격하였다. 그러나여러분, 동요할필요는없다. 여러분의희망처럼이번해의여러분들은, 그리고앞으로도 10년동안은, 안전함을증명하여라. ( 플란더즈 2005-J1) 증명 ( 제주대기고 1 학년오재성 ) 처음왔던해인 234 는 9 의배수이고 9 의배수의자릿수의합은 9 의배수이다.9 의배수해만큼잠들었다다시나타나면역시그해도 9 의배수이고, 따라서, 몬스터가나타나는해는항상 9 의배수이다. 그런데, 2020 년까지각자릿수의합은 28 이최대이다. 그럼몬스터가잠드는기간은 9 년,18 년, 혹은 27 년이고, 따라서연속된세 9 의배수인해중적어도한번은몬스터가찾아온다. 즉,1980 년,1989 년, 1998 년중적어도한번은몬스터가찾아온다. 1980! 1998; 1989! 2016; 1998! 2025 따라서, 2007 년에는찾아올리없고,2015 년까지앞으로 10 년은안전하다. 25. \B = \C =90 ± 이고 AB =12 인사다리꼴 ABCD 가있다. BC 의중점을 M 이라할때, AM? DM 이라고한다. 이사다리꼴의높이가정수가되도록하는 CD 의길이를모두구하여라. ( 플란더즈예선 1995/1996 1 차 ) 풀이 CD = x 라하면 AD = x +12. (x +12) 2 (x 12) 2 = h 2, 즉 48x = h 2...
5.1 종합중급문제 271 26. 12 개의정사각형,8 개의정육각형,6 개의정팔각형으로구성된 26 개의면을갖는볼록다면체가있다. 각각의꼭지점에서는세면이만나는데, 정사각형, 정육각형, 정팔각형인면이꼭하나씩만난다. 두꼭지점을잇는다면체내부의 ( 모서리나표면위의선이아닌 ) 대각선은모두몇개인가? (AIME 1988-10) 풀이 %EEE There are (12 4 + 8 6 + 6 8)/3 = 48 vertices. From each vertex there are 3 edges and 1 + 3 + 5 = 9 diagonals. Hence 47-3 - 9 = 35 internal segments. Hence 48 35/2 in total. 답 840 27. n n 숫자배열이주어져있다. n 은홀수이고이배열의각수는 1 이거나 1 이다. 이때 1 을홀수개포함하고있는행의개수와열의개수의합이 n 이될수없음을증명하여라. ( 소련 1962-5) 증명한행혹은한열의수들을모두곱한것을생각하자. 그럼그결과는그줄에 1이짝수개이면 1, 홀수개이면 1이된다. 각줄에대해이것을계산한것을다시모두곱한것은각각의칸에배정된수에대해서는그수를포함하는행과열에서 2번포함되므로제곱한것이되어 1이된다. 따라서, 각줄의결과가 1인것이홀수개일수없다. 28. x 2 y 2 =2000 2 을만족하는정수해 (x; y) 는몇개인가? (AIME 2000 2 차 -2) 풀이 %EEE (x+y)(x-y)=2^8 5^6. There are two solutions with y = 0. None with x = 0. Suppose x, y are positive. Then x+y and x-y have same parity, so must both be even. The no. of factors of 2000/4 is (6+1)(6+1) = 49. There is one factor 2^4 5^3 and 24 pairs (2a,2b) with a, b unequal and 4ab = 2000. Each of these gives a lattice point (x,y). But each of these gives 4 lattice points (±x,±y). So 2 + 4 24=98 in all. 답 98개 29. 다음의계산을관찰하고그일반적인규칙을증명하여라. ( 셈본중등초급도전문제 2.4.2) 2 2 +3 2 +4 2 +14 2 = 15 2 4 2 +5 2 +6 2 +38 2 = 39 2 6 2 +7 2 +8 2 +74 2 = 75 2 8 2 +9 2 +10 2 +122 2 = 123 2 풀이 일반적인규칙은다음과같다. (2n) 2 +(2n +1) 2 +(2n +2) 2 +(6n 2 +6n +2) 2 =(6n 2 +6n +3) 2 이등식의확인은다음과같이할수있다. (6n 2 +6n +3) 2 (6n 2 +6n +2) 2 =(6n 2 +6n +3+6n 2 +6n +2)(6n 2 +6n +3 6n 2 6n 2) =12n 2 +12n +5 =(4n 2 )+(4n 2 +4n +1)+(4n 2 +8n +4) 이식이 (2n) 2 +(2n +1) 2 +(2n +2) 2 과같으므로확인되었다. 30. 볼록사각형 ABCD 의 6 개의거리 AB, AC, AD, BC, BD, CD 중에서최대의것을 g, 최소의것을 h 라하자. g h p 2 임을증명하여라. ( 호주 1991-1)
272 종합문제 증명 90 ± 이상의내각에대한대각선 ( 둔각삼각형의빗변 ) 을 g 0 이라하면끝. 31. 4 개의문자 A, B, C, D 에서중복을허용하여고른길이 n 의문자열을생각하자. 이렇게만든임의의문자열에서각문자를 0; 1; 2; 3 의숫자로하나씩대응하여적당히바꾸어주면최고자리가 0 이아니고 3 의배수인 4 진법의 n 자리의수를항상얻을수있음을보여라. ( 호주 2003-8 변형 ) 증명일반성을잃지않고, ( 왼쪽에서부터 ) 첫째자리를 A라고하자. A, B, C, D의개수를각각 a; b; c; d라고하면문자열의값은 Aa + Bb + Cc + Dd ( mod 3) 일때 3의배수가된다. a; b; c; d 중적어도 2개는 mod 3에대해같은값을가진다. 같은값을가지는두문자에 1, 2를대입하고나머지에 3, 0을대입하면충분하다. 예를들어, 두문자를 A; B라고하자. 그러면 A =1;B =2;C =3;D =0이라하면 a +2b +3c 0( mod 3) 으로 3의배수가된다. 만일두문자가 B; C라면 A =3;B =1;C =2;D =0이라두면 3의배수가된다. 32. x 가소수이고, y, z 는 x 2 + y 2 = z 2 을만족하는음이아닌정수들이다. y, z 를 x 에관한식으로구하여라. ( 플란더즈예선 2000/2001 1 차 ) 풀이 x 2 =(z + y)(z y) 이고 z + y 0;z y 이므로 x가소수일때 x 2 을나누어갖는방법은 (z + y; z y) =(x 2 ; 1) 뿐이다. 답 y = x2 1, z = x2 +1 2 2 33. 1; 2;:::;99; 100 에서 55 개의수를어떻게택해도그중에항상차이가 9 가되는두수가있음을증명하여라. ( 플란더즈 1989-1) 풀이 f1; 10; 19; :::; 100g, f2; 11; 20; :::; 92g,..., f9; 18; 27; :::; 99g 등으로비둘기집구분. 한집합의원소의개수가 11{12개이므로각각최대 6개까지만뽑을수있음. 그럼 54개가최대. 34. 서로다른 2000 개의자연수가있는데, 홀수와짝수의개수가같고, 총합은 3000000 보다작다. 여기에 3 의배수가있음을보여라. ( 독일 BW 1977 1 차 -1) 증명대우증명.3 의배수가없으면가장작은 2000 개의수로우선만들더라도총합이 3000 3001 2 3 1000 1001 2 혹은그냥 1000 3000 이되어 3000000 을넘는다. 35. 볼록사각형 ABCD의대각선 AC와 BD가점O에서만난다. 삼각형 AOB와 COD의넓이를각각 S 1 과 S 2 라하고사각형ABCD의넓이를 S라할때, 다음부등식을증명하여라. p S1 + p S 2 ps 또, 등호가성립할조건은 AB 와 CD 가평행할때임을보여라. ( 스웨덴 1986-2) 증명 OA, OB, OC, OD 를각각 a, b, c, d 라하면 j4aobj : j4bocj : j4codj : j4doaj = ab : bc : cd : da. 따라서, 준식은 p ab + p cd p(a + c)(b + d) 와동치이고, 이것은양변을제곱해서정리하면 ( p ad p bc ) 2 0. 이것은틀림없이성립하고, 등호는 ad = bc 일때, 즉 a : b = c : d 일때이므로 AB k CD 일때. 36. 사장은 1; 2;:::;9 의번호가붙은 9 통의편지를차례대로쓰고있다. 그는편지한통을다쓰면서류받침의편지들맨위에올려놓는다. 비서는서류받침에놓인편지들중에서제일위에있는것을뽑아들고와서타이프를치고있다. 편지한통을다타이프하면서류받침의제일위에있는편지를다시들고와서타이프한다. 비서는점심식사를하려고잠시자리를비우면서 `8 번편지를막끝냈음 ' 이라고메모를하고나갔다. 아직타이프하지않은편지들의가능한순서는몇가지나되는가?( 예를들어,1,7,8 번편지를이미입력하였고 6, 5, 9, 4, 3, 2 의편지가이순서대로남아있다면, 순서 6, 5, 9, 4, 3, 2 가한가지가능성이된다.) (AIME 1988-15)
5.1 종합중급문제 273 풀이 %EEE 8 has been typed, so 1, 2,..., 7 have already been put into the tray. Any left must be typed in decreasing order. 9 can come anywhere in the order or not at all (if already typed). So for a subset of {1, 2,..., 7 with k elements, there are k+2 possibilities for 9. Hence 1 2 + 7 3 + 21 4 + 35 5 + 35 6 + 21 7 + 7 8 + 1 9 = 2 + 21 + 84 + 175 + 210 + 147 + 56 + 9 = 704. 답 704 37. 자연수들로이루어진어떤무한등차수열이제곱수를포함한다. 이수열이무한히많은제곱수를포함함을증명하여라. 단, 등차수열이란, 바로앞의항에일정한수를더하여다음항을계속만들어가는수열, 즉 a; a + d; a +2d; a +3d;::: 꼴의수열을뜻한다. ( 소련 1963-10) 증명제곱수인항을 n 2, 공차를 d 라하면, n 2 + dk 꼴의수들은모두이수열의항이된다. 이때, (n + md) 2 = n 2 + d(2mn + m 2 d) 이므로이것은항상이수열의항이된다. 38. 세변의길이가연속하는세정수인삼각형이있다. 두번째로긴변에내린수선이그변을길이의차가 4 인두선분으로분할함을증명하여라. (Towns 1989 가을 JO2) 증명 AB = n +1, BC = n, CA = n 1 이고 A 에서 BC 에내린수선의발을 D, 그리고 BD = b, CD = c 로두자 (b + c = n). 피타고라스의정리로 (n +1) 2 b 2 = AD 2 =(n 1) 2 c 2. 넘겨서정리하면 4n = b 2 c 2 =(b + c)(b c) =n(b c). 따라서, b c =4. 39. f가상수함수가아닌실수함수라하자. 모든실수 x에대해, f(x +1)+f(x 1) = p 3 f(x) 를만족한다면 f는주기함수임을증명하여라. 또한, 이런모든 f에대해, 모든 x에대해f(x + p) =f(x) 가되도록하는가장작은양수 p는무엇인가? (IMTS R5-4) 풀이 f(x +1)= p 3 f(x) f(x 1) 의점화식으로보자. f(r) =a, f(r +1)=b 로두고이점화식에의해차례로구하면다음과같다. x r r +1 r +2 r +3 r +4 r +5 r +6 p p p p f(x) a b 3 b a 2b 3 a 3 b 2a b 3 a a 따라서, f(x +6) = f(x), 그럼 f(x + 12) = f(x) 가된다. a 와 b 는임의로정할수있으므로, p 가 12 보다작은자연수일수는없다. x 와 y 의차가정수일때 x 와 y 가같은클래스에있다고말하기로하자. 문제의조건식은같은클래스에속하는수들끼리만관련되도록함을알수있으므로, 서로다른클래스의수들마다독립적으로 f 를정의할수있다. 따라서, 주기 p 는정수일때만의미가있고, 최소값은위에서본것처럼 12 이다. 40. 8 원짜리와 15 원짜리우표들만이용할수있고또무제한으로공급된다. 그럼어떤우편요금 예를들어 18 원과 29 원 은정확히만들어질수없다. 얻어낼수없는가장큰우편요금 즉이를 n 이라할때, n 보다더큰요금들은모두얻어낼수있고이요금은얻어낼수없는그런요금 은얼마인가? 그리고그이유를설명하여라. ( 캐나다 1974-6) 풀이 8과 15는서로소이므로 15k (k =0; 1;:::;7) 들은 8로나눈나머지가모두다르다. 그리고,8로나눈나머지가같은각각의우표값들중에서가장작은값들이 15k들이다. 따라서,8로나눈나머지가같은각각의요금중정확히만들어질수없는가장큰요금은 15k 8꼴이고, 이중에서가장큰값은 15 7 8=97원 답 41. 다음중음이아닌정수해를갖는방정식은몇개인가? 또, 자연수해를갖는방정식은몇개인가?
274 종합문제 ( 플란더즈예선 1997/1998 1 차 ) p x = p x p x p x = p x + p x p x p x p x = p x + p x + p x p x p x p x p x = p x + p x + p x + p x p x p x p x p x p x = p x + p x + p x + p x + p x 풀이모든식이 x =0을근으로가지므로모든식이정수해를갖는다. 한편, 자연수해를갖지않는식은네번째와다섯번째식뿐이다. 답 5개,3개 42. 벽에똑같은시계가 2개있다. 하나는현재의모스크바의시각을, 다른하나는현재의그지역의시각을나타낸다. 두시침끝사이의가장가까울때의거리는 m, 가장멀때의거리는 M이다. 두시계의중심사이의거리는얼마인가? (Towns 1990가을 SO3) 43. 15 15 크기의정사각형표의각칸을빨강, 파랑, 혹은녹색으로칠했다. 그럼적어도어느한가지색에대해서는, 그색이칠해진칸의수가서로같은두행이있음을증명하여라. ( 뉴질랜드 2000-9) 증명귀류법. 각행마다각각의색에대해쓰인횟수가 0;:::;15 중하나만빼고모두나와야함. 빠진수를각각 x, y, z라하면, 다합하면3(1 + +15) (x + y + z) =15 15. 여기서모순. 44. z>y>x 3 일때, 1 x + 1 y = 1 2 + 1 z 를만족하는세정수 x, y, z 의값을구하여라. (1995 교육청경시 ) 풀이우선주어진조건에의해 1 z < 1 y < 1 x 1 3 이다. ² x =3 왜냐하면 x 4라면, 1 y + 1 x < 1 4 2= 1 2 < 1 2 + 1 z 로모순이다. ² y<6 만일 y 6이라면, 1 x + 1 y 1 3 + 1 6 = 1 2 < 1 2 + 1 z : 즉, y =4또는 5이므로 (x; y; z) =(3; 4; 12) 또는 (3; 5; 30) 45. a 가자연수일때, 부정방정식 의양의정수해의수는 x 1 +2x 2 +3x 3 + + nx n = a y 1 +2y 2 +3y 3 + + ny n = a n(n +1) 2 의음이아닌정수해의수와같음을보여라. 단, 여러변수가있는방정식의해란, 그식을만족시키는수들의순서쌍 (x 1 ;x 2 ;:::;x n) 을의미한다. ( 헝가리 1904-2) 증명두방정식의정수해의집합사이의대응 x i = y i +1이양방향으로다되는대응, 즉역함수가있는대응이므로일대일대응이다. 두집합의해가유한개임은자명하고, 이대응이두해집합의조건 " 양의정수 " 와 " 음아닌정수 " 에도잘부합하므로이대응은정의도잘된다. 46. n 은십진법으로전개했을때 7 가지숫자가나타나고, 그숫자들은모두 n 의약수이다. n 의자릿수로나타날수없는숫자를모두찾아라. ( 플란더즈 2000-1)
5.1 종합중급문제 275 풀이 0과 5는나타날수없다. 나머지는모두가능하다.7의배수,8의배수,9의배수가되도록하면충분한데, aaa:::aa ( 같은숫자가 18개 ) 이면항상 7의배수와 9의배수를만족하고, 그중에서888..88 을제일나중에쓰면8의배수도된다. 짧은수의예를찾으라면, 1369872같은것은 4를제외하고모두잘나타나는수이다... 47. 8 8 체스판에서흑색칸 1개, 백색칸 1개를지운것은 31개의도미노 ( ; ) 로겹치지않고채울수있 음을보여라. ( 통신강좌 1997-14-7) 증명우선, 가로나세로둘중하나가짝수인직사각형꼴의 Board 는도미노로채울수있다.( 이렇게 )( 단,0 도짝수에포함 )...... 이제, 지워진두개의칸이하나는흑색, 하나는백색이므로, 이두칸을대각의양끝으로하는직사각형은가로, 세로중하나는홀수, 하나는짝수이다. 일반성을잃지않고가로가짝수라고하면,ChessBoard 를다음과같이분할한다. 2 3 1 5 홀 4 짝 1 ; 3 은세로의칸수가짝수 ) 채움이가능하다. 2 ; 4 는가로의칸수가짝수 ) 채움이가능하다. 5 는 홀 5. 2 5 1 5. 3 짝 5 2 와 5 3 은세로가짝수 ) 가능 5 1 은가로가짝수 ) 가능 ) 모든각영역들이독립적으로도미노로겹치지않고채움이가능이때 Chess Board 의칸수는 62 개이므로도미노는 31 개가필요하였다. ) 증명끝 48. (a) 12개의합동인삼각형으로분할되는삼각형의예를하나찾아라. (b) 5개의합동인삼각형으로분할되는삼각형의예를하나찾아라. ( 러시아 1989 4차-y8-6) 풀이 (a) 정삼각형을 4등분하고각각을중심3등분함.(b)1:2직각삼각형을 4+1개쌓으면됨.(b) 의일반화로 m 2 + n 2 개가항상됨... APMO 2005 참조. 49. 세개의주머니에들어있는구슬은모두 a 개이며, 첫째주머니에 187 개, 둘째주머니에 a=5 개, 셋째주머니에 a=13 의배수개가들어있다. 이때, a 의값은얼마인가? (KMC 예선 2002 전기고 1) 풀이셋째주머니에든구슬수를 ak=13 이라고하자.(k 는자연수 ) 187 + a 5 + ak 13 = a ) 11 17 = 52 5k 65 a: 처음식에서 a 는 65 의배수여야한다. 즉 52 5k 는 1; 11; 17 중에하나여야하고이때 52 5k =17 이다. 11 17 = 17 65=a ) a =65 11 = 715 답
276 종합문제 50. 어떤삼각형의세변의길이 a, b, c 가두등식 a 2b + c =0; a 2 + b 2 +3c 2 +2ab 4bc 4ca =0 을만족한다. a, b, c 는모두자연수이고, 이중에어느하나는 5 이다. a + b + c 는얼마인가? (1999 KAIST 대전. 충남영재수학교실 2 차평가 ) 풀이 a 2 + b 2 +3c 2 +2ab 4bc 4ca =0) (a + b 2c) 2 = c 2 이때 a + b 2c = c 라면 a + b = c 로삼각형의세변의길이라는데모순되므로 a + b 2c = c 이다. (a 2b + c) (a + b 3c) =0=4c 3b ) 4c =3b ) a =2b c = 5 3 c 세수가자연수이므로 a =5;b=4;c=3 이유일한값이다. 답 12 51. 주연이는파란구슬, 빨간구슬, 노란구슬을가지고있다. 주연이와승오의대화를읽고주연이가가진구슬의개수를색깔별로각각구하면? 단, 주연이와승오는매우똑똑하다. (ML 프로포절 165-2) 주연 : 내가가진각색깔별구슬의개수를모두곱하면 240 이돼. 승오 : 그것으로는모르겠어. 주연 : 내가가진구슬의개수는네가가진구슬의개수와같아. 승오 : 아직도모르겠어. 주연 : 나는노란구슬보다파란구슬을 11 개더많이갖고있어. 승오 : 아! 알겠어. 풀이 ( 서울경희고 1 학년이승후 ) 240 을세인수의곱으로나타내는방법은여러가지이므로첫번째질문에서모르는것은당연하다. 세번째질문에서 240 을세인수로분해했을때그중어느두수의차가 11 이어야하는데, 240 의약수 12345681012151620243040486080120240 중에서차가 11 인쌍은 (1; 12), (4; 15), (5; 16) 뿐이다. 그리고, 각각 240 = 1 12 20 = 4 4 15 = 3 5 16 으로 240 을나타낼수있다. 그런데,(1; 12; 20) 과 (4; 4; 15) 은인수의합이각각 33, 23 이되는것으로 240 을나타나는방법이유일하므로 ( 막노동 ) 이것중에답이있었다면두번째질문에서승오가알아내었을것이다.(3; 5; 16) 는 (2; 10; 12) 와인수의합이같으므로이것만이두번째질문에서 ` 모른다 ' 고한것에적합하다. 답 16, 3, 5 개 52. a; b > 0 의조화평균은 2ab a + b 를말한다. 조화평균이 620 이되는두수m<n의쌍은모두몇개인가? (AIME 1996-8) 풀이 %EEE We have mn = 2^19 3^20 m + 2^19 3^20 n, or (m - 2^19 3^20)(n - 2^19 3^20) = 2^38 3^40. Now 2^38 3^40 has 39 41 factors. Leaving aside the square root 2^19 3^20 which does not give a solution, the factors form 799 pairs (one < square root and one >). Each pair gives a solution m, n. 답 799 53. 네개의접시 A, B, C, D 가있고,1000 개의콩이있다.2 a 3 b 5 c 꼴의수 (a; b; c =0; 1;:::; 9) 가각각의콩에하나씩적혀있다. 어떤콩을옮긴다는것은, 그콩이 A 에있었으면 B 로, B 에있었으면 C 로, C 에있었으면 D 로, D 에있었으면 A 로옮기는것을말한다. 처음에모든콩은접시 A 에있었고, 이제콩하나를택해그콩에적힌수의약수를갖는콩 ( 물론그럼자기자신도포함된다 ) 들을모두옮긴다. 또다른콩하나를택해그약수에해당하는콩들을모두옮기고, 이런식으로모든콩에대해한번씩작업한다 ( 작업한콩에는표시를해놓고표시가안된콩중에하나를계속택한다고생각하면되겠다 ). 이작업이모두끝난후, 접시 A 에있는콩은모두몇개인가? (AIME 1999-7)
5.1 종합중급문제 277 풀이 %EEE 2^a 3^b 5^c divides 2^A 3^B 5^C iff a A, b B and c C. So there are (10-a)(10-b)(10-c) multiples of 2^a 3^b 5^c. Thus it ends up in position A iff 4 divides (10-a)(10-b)(10-c). So we have: a = 2,6, any b, c, 200 poss; a = 0,4,8 b,c not both odd 3(100-25) = 225 poss; a odd, b = 2,5, any c, 100 poss; a odd, b odd, c 2 or 6, 50 poss; or a odd, b 0,4,8, c even 75 poss. Total 650. 답 650 54. 4ABC의내부에한점 P 를잡았을때 j4pabj j4pbcj 2j4PCAj 일확률을구하여라. 단, jxj는도형 X의넓이를나타낸다. (1995 학교경시 ) 55. 1보다큰자연수 n이주어져있다. p, q는 n 1 < p n q < n 을만족하는자연수일때, q의최소값을구 n +1 하여라. ( 플란더즈예선 2003 1차변형 ) 풀이 1에서각항을뺀것으로부등식을다시쓴다. 1 n > q p > 1 q n+1 : 이때q p는양수이고 q p =1인경우n +1>q>n이되어모순이므로q p 2여야한다. 또 q>n (q p) 이므로 q p가커질수록 q값이커진다. 따라서 q의최소값은 q p =2일때일것이라고짐작할수있다. q p =2) 2(n +1)>q>2n ) q =2n +1;p =2n 1: 이때q<n 3이므로 q p 3일때의q 값은항상 2n +1보다크다. 답 2n +1 56. 다음방정식의자연수해는모두몇개인가? h x i h x i = +1 10 11 단,[t] 는 t 를넘지않는가장큰정수를나타낸다. (Towns 1989 가을 JA1) 57. I 0 = f 1; 1g 로주어져있고, y 가 I n 1 의모든원소를취할수있을때방정식 x 2 2xy + y 2 4 n =0 의모든해 x 를모은집합을 I n 으로정의하자. 모든음아닌정수 n 에대해 I n 들의합집합을구하여라. ( 이탈리아 1987-4) 풀이 x y = 2 n. I n =(I n 1 +2 n )[(I n 1 2 n )=f (2 n+1 1);:::; 3; 1; 1; 3;:::;2 n+1 1g. 즉합집합은모든홀수들의집합. 58. 2001 을 1979 개의양의완전제곱수의합으로표현하는방법을모두구하여라. 단, 더하는순서는무시한다. ( 유고슬라비아 1979 고 2-3) 풀이 2 2 1, 3 2 1, 4 2 1, 즉 3, 8, 15의합으로 22를나타내는방법. mod 3으로생각하면경우가 3+3+8+8뿐임이금방. 59. 이등변삼각형 ABC에서 AD는밑변BC에내린수선이고, M은변AC 위의한점이다. DB DM < AB AM 임을증명하여라. ( 몰도바 1996 최종-y7-3) 증명 AD 에대한 M 의대칭점을 N 이라하면준식은 DB DN < BN 이고이것은삼각부등식. 60. 쌍곡선 2xy 5x + y =55 위에있는 x, y 가정수인점 (x; y) 의개수를구하여라. (IMTS R20-1) 61. 모양의타일은이한종류만으로다음과같이무한히기다란띠를만들수있다.
278 종합문제 이처럼주어진한종류의타일들만으로띠를만들수있는타일을모두골라라. (IT 꿈나무올림피아드 2006 1 차 ) (1) (2) (3) (4) (5) 풀이 (1) 은두장을맞붙여직사각형을만들수있고그것을계속이어붙여띠를만들수있다.(3), (4) 는두장을맞붙여꼴을만들수있고, 이꼴을일렬로계속이어붙이면띠를만들수있 다.(2) 는오목한부분을어떻게채울지따져보면 와같이계단형의띠가무한히이 어지게되는데이계단형띠는어떤가로선도존재할수없게하므로일자형띠는생길수없다.(5) 는오목한부분을채우는방법을생각해보면깔수있는방법은유일하고그경우띠를이룰수있는직선이생길수없음을관찰할수있다. 답 62. 볼록사각형의두대각선이이사각형을네영역으로나누는데, 각영역의넓이가정수이다. 이네영역의넓이의곱이완전제곱수가됨을보여라. ( 유고슬라비아 1981 고 2-2) 증명 대각선이교점에의해각각 a : b, c : d 로내분되었다면네영역의넓이가 ac; bc; ad; bd꼴이되겠 군. 63. m 과 n 은 m 2 + mn + n 2 이 9 의배수가되게하는정수이다. m, n 은 3 의배수임을증명하여라. ( 헝가리 1958-2) 증명 9 j (m n) 2 +3mn 이므로 3 j (m n) 2. 64. 자연수 n을주면1부터 n까지의역수의합 (1 + 1 2 + 1 3 + + 1 ) n 을계산해주는버튼이들어있는계산기가있다. 이계산기에허용된연산은이것과사칙연산뿐이다. 다음중이계산기에서 n이얼마나크든상관없이 20번이내의연산으로항상결과를얻을수있는식들을모두골라라. (IT꿈나무올림피아드 2006 1차 ) (1) 2 에서 2n 까지의짝수들의역수의합을구하는함수 (2) 1에서 2n 1 까지의홀수들의역수의합을구하는함수 (3) 1 1 2 + 1 3 1 4 + + 1 2n 1 1 을구하는함수 2n (4) 1 + p 1 + p 1 + + p 1 을구하는함수 2 3 n (5) 1 p 1 + p 1 1 1 p + + p p 1 을구하는함수 2 3 4 2n 1 2n 풀이 f(n) =1+ 1 2 + 1 3 + + 1 n 라하자.(1) 은 f(n)=2에해당한다.(2) 는 f(2n) (1). (3) 은 (2) (1) 이므로모두가능하다. 이계산기의연산으로는항상유리수만나오므로 (4), (5) 는일반적으로는얻을수없다. 65. 어떠한세점을뽑아도그중에단위거리만큼떨어진두점이항상존재하도록, 평면위에일곱개의점을배치할수있음을보여라. (IMTS R29-4) 증명 ( 이주호 ) 다음과같이하면된다.. 아래그림은단위길이인선분만연결한것이다.